Chuyên đề luyện thi đại học môn Toán - VipLam.Net

292
CHUYÊN Đ TOÁN CHN LC LN 1 DI™N ĐÀN TOÁN TRUNG H¯C PH˚ THÔNG Ngày 20 tháng 3 năm 2013 VipLam.Net

Transcript of Chuyên đề luyện thi đại học môn Toán - VipLam.Net

Page 1: Chuyên đề luyện thi đại học môn Toán - VipLam.Net

CHUYÊN ĐỀ TOÁN

CHỌN LỌC

LẦN 1

DIỄN ĐÀN TOÁN TRUNG HỌC PHỔ THÔNG

Ngày 20 tháng 3 năm 2013

VVViiipppLLLaaammm...NNNeeettt

Page 2: Chuyên đề luyện thi đại học môn Toán - VipLam.Net
Page 3: Chuyên đề luyện thi đại học môn Toán - VipLam.Net

Mục lục

I CÁC CHUYÊN ĐỀ 3

1 PHƯƠNG PHÁP GIẢI CÁC BÀI PHƯƠNG TRÌNH - BẤT PHƯƠNG

TRÌNH VÔ TỶ

(Nguyễn Thị Ngân) 4

1 Các phương pháp giải (Dạng cơ bản) . . . . . . . . . . . . . . . . . . . . . . . . 4

2 Phần riêng . . . . . . . . . . . . . . . . . . . . . . . . . . . . . . . . . . . . . . . 5

I Bất phương trình . . . . . . . . . . . . . . . . . . . . . . . . . . . . . . . 5

II Phương trình . . . . . . . . . . . . . . . . . . . . . . . . . . . . . . . . . 13

2 PHƯƠNG PHÁP GIẢI PHƯƠNG TRÌNH MŨ VÀ LOGARIT (Đỗ Đường

Hiếu) 17

1 PHƯƠNG TRÌNH MŨ . . . . . . . . . . . . . . . . . . . . . . . . . . . . . . . . 17

I Phương pháp đưa về cùng cơ số . . . . . . . . . . . . . . . . . . . . . . 17

II Phương pháp đặt ẩn phụ . . . . . . . . . . . . . . . . . . . . . . . . . . . 18

III Phương pháp sử dụng tính chất đồng biến và nghịch biến của hàm số . . 19

IV Phương pháp lôgarit hóa . . . . . . . . . . . . . . . . . . . . . . . . . . . 21

2 PHƯƠNG TRÌNH LÔGARIT . . . . . . . . . . . . . . . . . . . . . . . . . . . . 21

I Phương pháp đưa về cùng cơ số . . . . . . . . . . . . . . . . . . . . . . 21

II Phương pháp đặt ẩn phụ . . . . . . . . . . . . . . . . . . . . . . . . . . . 22

III Phương pháp sử dụng tính chất đồng biến và nghịch biến của hàm số . . 23

IV Phương pháp mũ hóa . . . . . . . . . . . . . . . . . . . . . . . . . . . . . 24

3 CÁC BÀI TẬP TỔNG HỢP . . . . . . . . . . . . . . . . . . . . . . . . . . . . . 24

3 CỰC TRỊ CỦA HÀM NHIỀU BIẾN

(Lê Trung Tín) 35

1 Sử dụng bất đẳng thức cổ điển: . . . . . . . . . . . . . . . . . . . . . . . . . . . 35

2 Sử dụng phương pháp miền giá trị

(Điều kiện có nghiệm) . . . . . . . . . . . . . . . . . . . . . . . . . . . . . . . . 40

3 Sử dụng phương pháp đưa về khảo sát hàm 1 biến . . . . . . . . . . . . . . . . 43

2

VVViiipppLLLaaammm...NNNeeettt

Page 4: Chuyên đề luyện thi đại học môn Toán - VipLam.Net

4 PHƯƠNG PHÁP SỬ DỤNG TÍNH ĐƠN ĐIỆU CỦA HÀM SỐ TRONG

CÁC BÀI TOÁN CHỨNG MINH BẤT ĐẲNG THỨC 52

1 MỘT SỐ VÍ DỤ TÌM GTLN, GTNN CỦA HÀM SỐ . . . . . . . . . . . . . . . 53

2 KỸ THUẬT GIẢM BIẾN . . . . . . . . . . . . . . . . . . . . . . . . . . . . . . 53

I Kỹ thuật tìm GTLN, GTNN bằng phương pháp thế . . . . . . . . . . . . 53

II Bài toán tìm GTLN, GTNN của biểu thức đối xứng. . . . . . . . . . . . 55

III Bài toán tìm GTLN, GTNN của biểu thức chứa 3 biến . . . . . . . . . . 56

3 BÀI TOÁN TỔNG HỢP . . . . . . . . . . . . . . . . . . . . . . . . . . . . . . . 63

5 SỬ DỤNG BẤT ĐẲNG THỨC ĐỂ GIẢI BẤT PHƯƠNG TRÌNH (hthtb22) 68

6 CHỨNG MINH BẤT ĐẲNG THỨC, TÌM GIÁ TRỊ LỚN NHẤT, GIÁ TRỊ

NHỎ NHẤT BẰNG PHƯƠNG PHÁP HÀM SỐ (Nguyễn Hữu Phương) 77

7 SỬ DỤNG CÁC BẤT ĐẲNG THỨC CƠ BẢN GIẢI HỆ PHƯƠNG TRÌNH

(Ngô Hoàng Toàn) 101

1 KIẾN THỨC CHUẨN BỊ . . . . . . . . . . . . . . . . . . . . . . . . . . . . . . 102

I MỘT SỐ BẤT ĐẲNG THỨC THƯỜNG DÙNG . . . . . . . . . . . . . 102

2 CON ĐƯỜNG ĐI TỪ BÀI TOÁN ĐẾN SUY NGẪM CỦA BẢN THÂN . . . . 104

I BẤT ĐẲNG THỨC & HỆ PHƯƠNG TRÌNH 2 ẨN . . . . . . . . . . . . 104

II BẤT ĐẲNG THỨC & HỆ PHƯƠNG TRÌNH 3 ẨN . . . . . . . . . . . 122

III TỔNG HỢP CÁC BÀI TOÁN HỆ PT GIẢI BẰNG PHƯƠNG PHÁP

BẤT ĐẲNG THỨC . . . . . . . . . . . . . . . . . . . . . . . . . . . . . . 125

IV SÁNG TẠO HỆ QUA CÁC BÀI TOÁN BẤT ĐẲNG THỨC . . . . . . . 148

8 THỂ TÍCH VÀ KHOẢNG CÁCH

(Nguyễn Trung Kiên) 151

9 THAM SỐ HÓA HÌNH GIẢI TÍCH TRONG MẶT PHẲNG

(Nguyễn Thị Thỏa) 161

1 KIẾN THỨC CƠ BẢN . . . . . . . . . . . . . . . . . . . . . . . . . . . . . . . 161

I ĐƯỜNG THẲNG . . . . . . . . . . . . . . . . . . . . . . . . . . . . . . 161

II ĐƯỜNG TRÒN . . . . . . . . . . . . . . . . . . . . . . . . . . . . . . . 162

III ELIP . . . . . . . . . . . . . . . . . . . . . . . . . . . . . . . . . . . . . 162

2 BÀI TẬP CƠ BẢN . . . . . . . . . . . . . . . . . . . . . . . . . . . . . . . . . . 162

3 BÀI TẬP NÂNG CAO RÈN LUYỆN KĨ NĂNG . . . . . . . . . . . . . . . . . . 166

II CÁC BÀI TOÁN HAY 172

10 BẤT PHƯƠNG TRÌNH (hthtb22) 173

3

Page 5: Chuyên đề luyện thi đại học môn Toán - VipLam.Net

11 HỆ PHƯƠNG TRÌNH (Lê Nhất Duy) 180

12 HỆ PHƯƠNG TRÌNH NÂNG CAO

(Lê Trung Tín) 187

13 PHƯƠNG TRÌNH & BẤT PHƯƠNG TRÌNH VÔ TỶ (Đinh Văn Trường) 195

14 SỬ DỤNG KĨ THUẬT ĐÁNH GIÁ ĐƯA VỀ CÙNG MẪU ĐỂ CHỨNG

MINH BẤT ĐẲNG THỨC

(Hoàng Trung Hiếu) 240

15 SỬ DỤNG CÁC BẤT ĐẲNG THỨC CỔ ĐIỂN ĐƯA BÀI TOÁN TÌM

GTLN, GTNN CỦA HÀM NHIỀU BIẾN VỀ HÀM MỘT BIẾN

(Lê Hoàng Hải) 249

1 Kiến thức cần nhớ vể các bất đẳng thức cổ điển thường dùng. . . . . . . . . . . 249

I Bất đẳng thức AM-GM. . . . . . . . . . . . . . . . . . . . . . . . . . . . 249

II Bất đẳng thức Cauchy-Schwarz . . . . . . . . . . . . . . . . . . . . . . . 249

2 Các ví dụ minh họa. . . . . . . . . . . . . . . . . . . . . . . . . . . . . . . . . . 250

I Bài tập tự luyện. . . . . . . . . . . . . . . . . . . . . . . . . . . . . . . . 257

16 SỬ DỤNG PHÉP BIẾN HÌNH TRONG GIẢI TOÁN HÌNH HỌC GIẢI

TÍCH PHẲNG (Lê Hoàng Hải) 260

1 Kiến thức cần nhớ. . . . . . . . . . . . . . . . . . . . . . . . . . . . . . . . . . . 260

I Phép dời hình. . . . . . . . . . . . . . . . . . . . . . . . . . . . . . . . . 260

II Phép đồng dạng. . . . . . . . . . . . . . . . . . . . . . . . . . . . . . . . 260

2 Các ví dụ minh họa. . . . . . . . . . . . . . . . . . . . . . . . . . . . . . . . . . 261

I Phép dời hình. . . . . . . . . . . . . . . . . . . . . . . . . . . . . . . . . 261

II Phép đồng dạng. . . . . . . . . . . . . . . . . . . . . . . . . . . . . . . . 263

3 Bài tập tự luyện. . . . . . . . . . . . . . . . . . . . . . . . . . . . . . . . . . . . 264

17 PHƯƠNG PHÁP TỌA ĐỘ HOÁ ĐỂ CHỨNG MINH HÌNH HỌC PHẲNG

Lưu Giang Nam - Hoàng Trung Hiếu 266

1 Các ví dụ . . . . . . . . . . . . . . . . . . . . . . . . . . . . . . . . . . . . . . . 266

I Bài tập tự luyện . . . . . . . . . . . . . . . . . . . . . . . . . . . . . . . 275

III CÁC ĐỀ THI TỰ LUYỆN 277

4

VVViiipppLLLaaammm...NNNeeettt

Page 6: Chuyên đề luyện thi đại học môn Toán - VipLam.Net

PHẦN THỨ I

CÁC CHUYÊN ĐỀ

5

Page 7: Chuyên đề luyện thi đại học môn Toán - VipLam.Net

CHUYÊN ĐỀ 1

PHƯƠNG PHÁP GIẢI CÁC BÀI PHƯƠNG

TRÌNH - BẤT PHƯƠNG TRÌNH VÔ TỶ

(Nguyễn Thị Ngân)

Lời nói đầu

Phương trình- bất phương trình là chuyên đề mà chúng ta thường gặp trong các kì thi các cấp

và đặc biệt là thi đại học. Phương trình-bất phương trình vô tỷ rất đa dạng và phong phú về cả

đề bài và lời giải. Một bài phương trình- bất phương trình có thể có nhiều cách xử lý bài toán

khác nhau.

Tuy nhiên, để tìm ra lời giải cho bài toán của mình thì rất khó đối với đa số các bạn. Đứng

trước 1 bài phương trình- bất phương trình chúng ta thường rất lúng túng và khó khăn. Như

các bạn đã biết phương trình- bất phương trình luôn luôn có trong đề thi đại học, thường thì

nó nằm ở câu II.

Bài viết này sẽ giúp các bạn phần nào đó về phương trình-bất phương trình.Những lời giải

dưới đây tuy không phải là những lời giải hay nhất nhưng nó sẽ giúp các bạn nắm rõ được

chuyên đề này.

Hy vọng chuyên đề này sẽ đồng hành với các bạn, giúp đỡ các bạn trên con dườngđi đến

thành công, đi đến mục đích cuối cùng của chúng ta là cổng trường đại học mơ ước và có thể

nó sẽ khiến cho các bạn đam mê với môn học này ( Khó- khổ- khô).

Mặc dù đã rất cố gắng trình bày cẩn thận nhưng sẽ không tránh khỏi nhiều sai sót trong

chuyên đề.Mong các bạn thông cảm.

Nếu có gì thắc mắc và những ý kiến về chuyên đề của mình thì các bạn liên lạc cho mình

với địa chỉ [email protected] nhé!

§ 1. Các phương pháp giải (Dạng cơ bản)

Một số phép toán biến đổi tương đương khi sử dụng để giải phương trình- bất phương trình.

1.√f (x) =

√g (x) ⇐⇒

{f (x) = g (x)

g (x) ≥ 0

2.√f (x) = g (x) ⇐⇒

{f (x) = [g (x)]2

g (x) ≥ 0

6

VVViiipppLLLaaammm...NNNeeettt

Page 8: Chuyên đề luyện thi đại học môn Toán - VipLam.Net

3.√f (x) >

√g (x) ⇐⇒

{f (x) > g (x)

g (x) ≥ 0

4.√f (x) > g (x) ⇐⇒

{

f (x) > [g (x)]2

g (x) ≥ 0{f (x) ≥ 0

g (x) < 0

5.√f (x) < g (x) ⇐⇒

f (x) < [g (x)]2

f (x) ≥ 0

g (x) ≥ 0

§ 2. Phần riêng

I. Bất phương trình

Ngoài những cách giải trên. Một số dạng giải bất phương trình

1.1√f (x)

>1

g (x)⇐⇒

{

g (x) < 0

f (x) > 0{g (x) > 0√f (x) < g (x)

2. f (x)√g (x) ≥ 0 ⇐⇒

{

g (x) > 0

f (x) ≥ 0

g (x) = 0

3. af (x) + bg (x) + c√f (x) g (x) < 0 (đẳng cấp)

Nếu g (x) =0 thì dễ dàng rồi nhé!

Giả sử g (x) > 0 chia cho g (x) ta được: af (x)

g (x)+ b+ c

√f (x)

g (x)< 0

Đặt

√f (x)

g (x)= t , khi đó ta có at2 + ct+ b < 0

Hệ thống phương pháp giải:

+ Biến đổi tương đương

+ Nhân liên hợp

+ Hàm số

+ Đánh giá ( AM-GM; Bunhiacopxki, vecto)

Đôi chút về bất đẳng thức vecto:∣∣−→u + −→v

∣∣ ≤ ∣∣−→u ∣∣ +∣∣−→v ∣∣. Đẳng thức xảy ra ⇐⇒ −→u ,−→v cùng

hướng

Các ví dụ

Bài 1. Giaỉ bất phương trình sau:

√x2 − x− 6 + 7

√x−

√6 (x2 + 5x− 2)

x+ 3−√

2 (x2 + 10)≤ 0

7

Page 9: Chuyên đề luyện thi đại học môn Toán - VipLam.Net

Lời giải:

Công việc đầu tiên của chúng ta không thể thiếu được đó chính là tìm điều kiện cho bài toán.

Đối với bài toán này thì : ĐK:

x2 − x− 6 ≥ 0

x ≥ 0

x2 + 5x− 2 ≥ 0

x+ 3 6=√

2 (x2 + 10)

⇐⇒ x ≥ 3

Khi đó,x+ 3 <

√2 (x2 + 10) ⇐⇒ x2 − 6x+ 11 > 0

⇐⇒ (x− 3)2 + 2 > 0(Luôn đúng)

Bất phương trình đã cho trở thành:√x2 − x− 6 + 7

√x ≥

√6 (x2 + 5x− 2)

Vì hai vế của bất phương trình này đều dương nên cho phép chúng ta bình phương 2 vế Nên:√x2 − x− 6 + 7

√x ≥

√6 (x2 + 5x− 2)

⇐⇒ x2 − x− 6 + 49x+ 14√x (x2 − x− 6) ≥ 6x2 + 5x− 2

⇐⇒ −5x2 + 18x+ 6 + 14√

(x2 − 3x) (x+ 2) ≥ 0

Đến đây thấy trong căn xuất hiện 2 nhân tử là x2 − 3x và x+ 2 ,

ta nghĩ ngay đến việc phân tích −5x2 + 18x+ 6 cũng xuất hiện 2 nhân tử đó.

Quả nhiên ông trời không phụ lòng người, ta phân tích được

−5x2 + 18x+ 6 = −5(x2 − 3x

)+ 3 (x+ 2)

Tuyệt vời!!! Công việc bây giờ là biến đổi phương trình trên thôi, ta được:

−5(x2 − 3x

)+ 14

√(x2 − 3x) (x+ 2) + 3 (x+ 2) ≥ 0

Vì x ≥ 3 nên x+ 2 > 0 Chia 2 vế của bất phương trình cho x+ 2 > 0, được:

−5.x2 − 3x

x+ 2+ 14

√x2 − 3x

x+ 2+ 3 ≥ 0(1)

Đặt

√x2 − 3x

x+ 2= a (a ≥ 0) , khi đó

(1) ⇐⇒ −5a2 + 14a+ 3 ≥ 3 ⇐⇒ −1

5≤ a ≤ 3

Mà a ≥ 0 =⇒ 0 ≤ a ≤ 3 Ta chỉ cần xét a ≤ 3 ,lúc đó:√x2−3xx+2

≤ 3 ⇐⇒ x2−3xx+2

≤ 9

⇐⇒ 6− 3√

6 ≤ x ≤ 6 + 3√

6

Hi, vậy bài toán được giải quyết trọn vẹn.Nhưng trước đó bạn đừng vội vàng kết luận mà nhớ

phải đối chiếu với điều kiện nhé!

Thật vậy, kết hợp với điều kiện ta có tập nghiệm của bất phương trình là: S =[3; 6 + 3

√6]

Bài 2. Giải bất phương trình sau:

(2x− 1)√x+ 3

2√x+ (2 +

√x)√

1− x+ 1− x≥ 1

8

Page 10: Chuyên đề luyện thi đại học môn Toán - VipLam.Net

Lời giải:

ĐK: 0 ≤ x ≤ 1 Với điều kiện đó thì,

2√x+

(2 +√x)√

1− x+ 1− x > 0

Bất phương trình đã cho trở thành:

(2x− 1)√x+ 3 ≥ 2

√x+ (2 +

√x)√

1− x+ 1− x⇐⇒ (2x− 1)

√x+ 3 ≥

(√x+√

1− x) (

2 +√

1− x)

Đặt√x = a;

√1− x = b (a, b ≥ 0) Ta có{

a2 + b2 = 1

2a2 + b2 − 1 = x

Bất phương trình trở thành:

(a2 − b2)√

2a2 + b2 + 2 ≥ (a+ b) (2 + b)

⇐⇒ (a− b)√

2a2 + b2 + 2 ≥ 2 + b

⇐⇒ (a− b)2 (2a2 + b2 + 2) ≥ (2 + b)2

⇐⇒ (1− 2ab) (a2 + 3) ≥ 4 + 4b+ b2 = 4 + 4b+ 1− a2

⇐⇒ (2a2 − 2)− 2b (a3 + 3a+ 2) ≥ 0

Mà a2 ≤ 1,∀0 ≤ a ≤ 1 và a3 + 3a+ 2 > 0,∀a ≥ 0 , nên:(2a2 − 2

)− 2b

(a3 + 3a+ 2

)≤ 0

Dấu ‘=’ xảy ra ⇐⇒

{a = 1

b = 0⇐⇒

{ √x = 1√1− x = 0

⇐⇒ x = 1

Đối chiếu lại với điều kiện đầu bài. Vậy bất phương trình đã cho có nghiệm duy nhất là x = 1

Bài 3. Giải bất phương trình sau:

6− 3x+√

2x2 + 5x+ 2

3x−√

2x2 + 5x+ 2≤ 1− x

x

Lời giải:

ĐK:

x 6= 0

3x 6=√

2x2 + 5x+ 2

2x2 + 5x+ 2 ≥ 0

⇐⇒ x ∈ (−∞;−2] ∪[−1

2; +∞

)\ {0; 1}

Bất phương trình đã cho tương đương

6− 3x+√

2x2 + 5x+ 2

3x−√

2x2 + 5x+ 2+ 1 ≤ 1 +

1− xx

⇐⇒ 6

3x−√

2x2 + 5x+ 2≤ 1

x

⇐⇒ 6x− 3x+√

2x2 + 5x+ 2

x(3x−

√2x2 + 5x+ 2

) ≤ 0

⇐⇒ 3x+√

2x2 + 5x+ 2

x(3x−

√2x2 + 5x+ 2

) ≤ 0 (1)

9

Page 11: Chuyên đề luyện thi đại học môn Toán - VipLam.Net

Đến đây ta chia thành 2 trường hợp

Trường hợp 1: √2x2 + 5x+ 2 + 3x = 0

⇐⇒ 2x2 + 5x+ 2 = 9x2

⇐⇒

x =−2

7x = 1

Đối chiếu với điều kiện đầu bài thì x =−2

7là nghiệm của bất phương trình (2)

Trường hợp 2:

3x+√

2x2 + 5x+ 2 6= 0

Khi đó, bất phương trình (2) trở thành:

3x+√

2x2 + 5x+ 2)2

x (7x2 − 5x− 2)≤ 0

⇐⇒ x (7x2 − 5x− 2) < 0

⇐⇒

0 < x < 1

x <−2

7

Đối chiếu với điều kiện đầu bài.

Vậy tập nghiệm của bất phương trình đã cho là: S = (−∞; 2) ∪ −12

; −27

]∪ (0; 1)

Bài 4. Giải bất phương trình sau:√(x+ 2) (2x− 1)− 3

√x+ 6 ≤ 4−

√(x+ 6) (2x− 1) + 3

√x+ 2

Lời giải:

ĐK: x ≥ 1

2Khi đó, Bất phương trình đã cho trở thành:(√

x+ 2 +√x+ 6

) (√2x− 1− 3

)≤ 4 (1)

Trường hợp 1:√

2x− 1− 3 ≤ 0 ⇐⇒ x ≤ 5. Nên (1) luôn đúng

Trường hợp 2: Với x > 5 Xét hàm số : f(x) =√x+ 2 +

√x+ 6

) (√2x− 1− 3

)=⇒ f ′(x) =

(1

2√x+ 2

+1

2√x+ 6

)(√2x− 1− 3

)+

√x+ 2 +

√x+ 6√

2x− 1> 0.

Nên f(x) đồng biến. Mặt khác: f(7) = 4 , nên (1) ⇐⇒ f(x) ≤ f(7) ⇐⇒ x ≤ 7.

Đối chiếu với điều kiện đầu bài tập nghiệm của bất phương trình đã cho là: S =[−1

2; 7]

Bài 5. Giải bất phương trình sau:x+ 2√

2 (x4 − x2 + 1)− 1≥ 1

x− 1

Lời giải:

ĐK: x 6= 1 Khi đó: √2 (x4 − x2 + 1)− 1 =

√2(x2 − 1)2 +

3

2− 1 > 0

10

VVViiipppLLLaaammm...NNNeeettt

Page 12: Chuyên đề luyện thi đại học môn Toán - VipLam.Net

Trường hợp 1: Nếu x > 1 Bất phương trình đã cho tương đương

x2 + x− 1 ≥√

2 (x4 − x2 + 1) (1)

Với ∀a, b ta luôn có a+ b ≤√

2 (a2 + b2) Dấu ’=’ xảy ra ⇐⇒ a = b

Áp dụng: Đặt a = x;x2 − 1 = b Khi đó,

x+ x2 − 1 ≤√

2[x2 + (x2 − 1)2] =

√2 (x4 − x2 + 1) (2)

Từ (1) và (2) =⇒ x2 − x+ 1 =√

2 (x4 − x2 + 1)

⇐⇒ x2 − 1 = x ⇐⇒

x =1−√

5

2

x =1 +√

5

2

⇐⇒ x =1 +√

5

2

Trường hợp 2: Nếu x < 1 Bất phương trình đã cho tương đương

x2 + x− 1 ≤√

2 (x4 − x2 + 1)

Luôn đúng ∀x < 1 (Vì đã chứng minh ở (2))

Vậy bất phương trình đã cho có tập nghiệm là S = (−∞; 1) ∪{

1+√

52

}Bài 6. Giải bất phương trình sau: √

3 + 3x+√

3− x√3 + 3x−

√3− x

≥ 4

x

Lời giải:

ĐK: x ∈ [−1; 3] \ {0}Trường hợp 1: x ∈ (0; 3]

Ta cần chứng minh√

3 + 3x−√

3− x > 0

Thật vậy,√

3 + 3x−√

3− x > 0 ⇐⇒ 3 + 3x > 3− x ⇐⇒ x > 0 (đúng)

Do đó, bất phương trình đã cho tương đương(√3 + 3x+

√3− x

)2

(3 + 3x)− (3− x)≥ 4

x

⇐⇒ 6 + 2x+ 2√

(3 + 3x) (3− x) ≥ 16

⇐⇒√

(3 + 3x) (3− x) ≥ 5− x(⊕)

Vì cả 2 vế đều dương nên

⊕ ⇐⇒ (3 + 3x) (3− x) ≥ (5− x)2 ⇐⇒ x2 − 4x+ 4 ≤ 0 ⇐⇒ (x− 2)2 ≤ 0

Mà (x− 2)2 ≥ 0 , nên =⇒ x− 2 = 0 ⇐⇒ x = 2

Trường hợp 2: x ∈ [−1; 0)

11

Page 13: Chuyên đề luyện thi đại học môn Toán - VipLam.Net

Dễ dàng chứng minh√

3 + 3x−√

3− x < 0

Thật vậy,√

3 + 3x−√

3− x < 0 ⇐⇒ x < 0 (đúng)

Do đó, bất phương trình đã cho tương đương

√3 + 3x+

√3− x√

3− x−√

3 + 3x≤ 4

−x

⇐⇒(√

3 + 3x+√

3− x)2

(3− x)− (3 + 3x)≤ 4

−x(−x > 0)

⇐⇒(√

3 + 3x+√

3− x)2

≤ 16

⇐⇒ 6 + 2x+ 2√

(3 + 3x) (3− x) ≤ 16

⇐⇒√

(3 + 3x) (3− x) ≤ 5− x

Vì 5− x > 0 nên ta có thể bình phương 2 vế lên để mất căn

Khi đó ta sẽ có điều ta mong muốn, (3− x) (3 + 3x) ≤ (5− x)2 ⇐⇒ (x− 2)2 ≥ 0 (Luôn đúng)

Vậy bất phương trình đã cho có tập nghiệm là : S = [−1; 0) ∪ {2}

Bài 7. Giải bất phương trình sau:√4− 1

x2+|2x+ 1|

x≥ 0

Lời giải:

ĐK: x ≤ −2 hay x ≥ 2

Bắt gặp dấu giá trị tuyệt đối chúng ta thường liên tưởng đến bình phương 2 vế và đánh giá

hoặc chỉ đánh giá, bình phương.. . .

Nhưng lần này đối với tôi, phép bình phương xuất hiện đầu tiên. Chúng ta cùng thử xem nhé!

Chuyển vế bất phương trình đã cho thành:√4− 1

x2≥ |2x+ 1|

−x

Trường hợp 1: Với x ≥ 2 thì bất phương trình luôn đúng

Trường hợp 2: x ≤ −2 Vì cả 2 vế đều dương nên bình phương 2 vế được:

4− 1

x2≥ (2x+ 1)2

x2⇐⇒ 4x2 − 1 ≥ (2x+ 1)2 ⇐⇒ −4x ≥ 2 ⇐⇒ x ≤ −1

2

Đối chiếu với điều kiện trong trường hợp x ≤ −2 thì nghiệm bất phương trình là x ≤ −2

Kết hợp 2 trường hợp lại với nhau tập nghiệm của bất phương trình là: S = [2; +∞)∪(−∞;−2]

Bài 8. Gỉai bất phương trình sau:√

2− x2 +

√2 +

1

x2< 4 + |x|+ 1

|x|Lời giải:

Ôi! Lại một bài toán có chứa dấu giá trị tuyệt đối nữa. Liệu lần này cách bình phương lên còn

có thể giúp được gì cho chúng ta không nhỉ??? Thử xem nhé!

12

Page 14: Chuyên đề luyện thi đại học môn Toán - VipLam.Net

ĐK:

{−√

2 ≤ x ≤√

2

x 6= 0

Bất phương trình đã cho trở thành:

4 +1

x2− x2 + 2

√(2− x2)

(2 +

1

x2

)< x2 +

1

x2+ 18 + 8 |x|+ 8

|x|

⇐⇒√

3 +2

x2− 2x2 < x2 + 7 + 4 |x|+ 4

|x|⇐⇒

√3x2 − 2x4 + 2 < (|x|)3 + 7 |x|+ 4x2 + 4 (1)

Bây giờ đến đây thì làm sao nhỉ?

Đừng nản chí vội, ở đây ta thử đánh giá biểu thức vế trái√

3x2 − 2x4 + 2 xem sao nhé! Mà vế

phải có 1 hằng số nên, ta thử xem nó có mối quan hệ gì đến biểu thức ta đang tìm hiểu.Thử

nhé!

Ta có:√

3x2 − 2x4 + 2 < 4

⇐⇒ 3x2 − 2x4 + 2 < 16 ⇐⇒ 2x4 − 3x2 + 14 > 0 ⇐⇒ 2(x2 − 1

)2+ x2 + 12 > 0

Đây là 1 diều hiển nhiên đúng với mọi x Do đó, (1) luôn đúng

Vậy tập xác định của bất phương trình chính là nghiệm của nó.

P/S: Một lần nữa cách bình phương 2 vế lại được phát huy 1 cách triệt để.

Bài 9. Gỉai bất phương trình sau:

(x2 + 4)√

2x+ 4 ≤ 3x2 + 6x− 4

Lời giải:

ĐK: 2x+ 4 ≥ 0 ⇐⇒ x ≥ −2

Khi đó, bất phương trình tương đương{(x2 + 4)

2(2x+ 4) ≤ (3x2 + 6x− 4)

2

3x2 + 6x− 4 ≥ 0

⇐⇒

{(2x+ 3) (x2 − 2x− 4)

2 ≤ 0

3x2 + 6x− 4 ≥ 0⇐⇒

[

x2 − 2x− 4 = 0

2x+ 3 ≤ 0

3x2 + 6x− 4 ≥ 0

⇐⇒

[x2 − 2x− 4 = 0

2x+ 3 ≤ 0 x ≥√

21− 3

3

x ≤ −3−√

21

3

⇐⇒

[

x = 1 +√

5

x = 1−√

5

x ≤ −3

2 x ≥√

21− 3

3

x ≤ −3−√

21

3

⇐⇒

x = 1 +√

5

x ≤ −3−√

21

3

Đối chiếu với điều kiện ta thấy, Bất phương trình đã cho có nghiệm duy nhất là x = 1 +√

5

13

Page 15: Chuyên đề luyện thi đại học môn Toán - VipLam.Net

Nhận xét: Chắc các bạn đang thắc mắc tại sao ở đoạn cuối tôi lại suy ra được kết quả là x = 1 +√

5

x ≤ −3−√

21

3

phải không? Lúc đầu khi làm bài toán này tôi cũng bó tay ở đoạn này đấy, nhưng sau khi thầy

tôi giải thích thì tôi đã hiểu ra.

Nhận thấy ở phía trên có 2 dấu ngoặc vuông cùng 1 lúc, đừng nghĩ nó phức tạp quá làm gì mà

thực ra thì đó cũng chính là 1 dấu ngoặc vuông mà thôi. Đối với những bài toán này thì đ xử

lý 1 cách nhanh gọn và đẹp thì cách vẽ trục số là biện pháp hữu hiệu nhất.Trong bài toán này

chúng ta cũng dùng cách vẽ trục số như vậy đấy.( Có thể vẽ 2 trục số ra để dễ nhìn hơn đó !)

Một điều nữa tôi muốn nói với các bạn chú ý khi giải những bài bất phương trình. Đó là, để

bình phương 2 vế của bất phương trình thì phải thoả mãn điều kiện 2 vế điều dương. Nhớ nhé,

tuy vấn đề này có nhiều bạn vẫn chủ quan, nhưng chỉ cần sơ suất 1 chút thôi là uổng phí công

sức toàn bài.

Bài 10. Gỉai bất phương trình sau:√x(x+√

1− x2)

x√x+ 1−

√x2 − x3

≥ 1

Lời giải:

ĐK: 0 ≤ x ≤ 1

Nhìn bài bất đẳng thức này sa tôi lại muốn nhân chéo nó lên nhỉ. Haizz, thử xem nào.Nhưng

trước tiên ta đi xét mẫu xem sao nhé!( Bật mí nhé: Muốn nhân chéo 1 bất phương trình nào

đó thì chúng ta phải đảm bảo rằng cả 2 vế đều dương). Mà bài toán này thì tử số luôn dương

rồi, vế phải chắc chắn 100% dương. Với điều kiện trên thì,

x√x+ 1−

√x2 − x3 ≥ x

√x+ 1−

√x2 = x

√x+ 1− x > 0,∀0 ≤ x ≤ 1

Woa, thật là may mắn.Chúng ta đã xử lý xong mẫu rồi, giờ chỉ việc nhân chéo lên.hì! Bất

phương trình đã cho trở thành√x(x+√

1− x2)≥ x√x+ 1−

√x2 − x3

⇐⇒√x2 − x3 ≥ 1−

√x (1− x2)

⇐⇒ x2 − x3 ≥ 1 + x (1− x2)− 2√x (1− x2)

⇐⇒ x+ 1− x2 − 2√x (1− x2) ≤ 0

⇐⇒(√

1− x2 −√x)2 ≤ 0

Mà(√

1− x2 −√x)2 ≥ 0,∀x Nên:

√1− x2 −

√x = 0

⇐⇒√

1− x2 =√x

⇐⇒ x2 + x− 1 = 0

⇐⇒

x =1−√

5

2

x =−1−

√5

2

14

Page 16: Chuyên đề luyện thi đại học môn Toán - VipLam.Net

Đối chiếu với điều kiện thì x =−1 +

√5

2thoả mãn bài toán

Vậy bất phương trình đã cho có ngiệm duy nhất là x =−1 +

√5

2Trên đây là 1 số ví dụ tôi đua ra cho các ban. Sau đây sẽ là 1 số bài tập để các bạn làm nhé!

Áp dụng: Gỉai các bất phương trình sau:

1. 2 (x2 + 2) < 3(2x+

√x3 + 8

)2.√

17x+ 53−√x+ 5− 4x < 12

3. x+2x√x2 − 4

> 3√

5

4.√

2x+ 4− 2√

2− x > 12x− 8√9x2 + 16

5.

√x+

1

x2+

√x− 1

x2>

2

x

6.√∣∣1

4− x∣∣ ≥ x+ 1

2

7.√

1 + x−√

1− x ≥ x√

5− 4√x+

√5 + 4

√x ≥ 4

8. (x2 − 3x)√

2x2 − 3x− 2 ≥ 0

9.1

1− x2+ 1 >

3x√1− x2

10. (x+ 1) (x− 3)√−x2 + 2x+ 3 < 2− (x− 1)2

Bất phương trình chúng ta đã đi được 1 quãng đường rồi nhỉ, bây giờ giành ít thời gian cho

phương trình nhé!

II. Phương trình

Về phương trình thì thế này. Nó có rất nhiều cách giải quyết. Đa số là

+ Đánh giá

+ Nhân liên hợp

+ Đặt ẩn phụ

+ Hàm số

Sau đây, tôi sẽ làm các ví dụ về phương trình nhé! Tuy nó chỉ là 1 phần nhỏ, chưa đi hết được

các dạng toán nhưng nó sẽ giúp ích cho các bạn phần nào.

Bài 1. Giải phương trình

x3 + 1 = 2 3√

2x− 1

Lời giải:

Đặt 3√

2x− 1 = a ⇐⇒ 2x = a3 + 1 Do đó, ta có hệ{x3 + 1 = 2a (1)

a3 + 1 = 2x (2)

Trừ vế với vế (1) cho (2) ta có:

x3 − a3 = 2 (a− x) ⇐⇒ (x− a)(x2+ax+a2 + 2

)= 0

15

Page 17: Chuyên đề luyện thi đại học môn Toán - VipLam.Net

Mặt khác: Do

x2+ax+a2 + 2 =(x+

a

2

)2

+3

4a2 + 2 > 0,∀x, a =⇒ x− a = 0 ⇐⇒ x = a

Thay vào (1) ta được:

x3 + 1 = 2x ⇐⇒ x3 + 1− 2x = 0 ⇐⇒ (x− 1)(x2 + x− 1

)= 0 ⇐⇒

x = 1

x =−1±

√5

2

Vậy phương trình đã cho có nghiệm là

x = 1

x =−1±

√5

2Chú ý: Ngoài cách đã làm ở trên, thì ta có thể làm như sau:

Phương trình đã cho được viết lại thành

x3 + 1

2= 3√

2x− 1

Ta thấy hàm số f(x) =x3 + 1

2và g(x) = 3

√2x− 1 là 2 hàm số ngược nhau, do đó đồ thị của

chúng đối xứng nhau qua đường y = x .

Mặt khác hai hàm số này không trùng nhau vì f(0) =1

2; g(0) = −1 , nên:

Nếu 2 đồ thị cắt nhau thì phải cắt nhau trên đường y = x . Do đó ta chuyển được việc giải

phương trình đã cho về việc giải phương trình

x3 + 1

2= x

Đến đây các bạn giúp mình giải tiếp nhé!

Bài 2. Giải phương trình

3(2 +√x− 2

)= 2x+

√x+ 6

Lời giải:

ĐK: x ≥ 2

Theo thói quen của tôi mỗi khi làm những bài bất phương trình hoặc phương trình là nhanh

chóng lấy chiếc máy tính ra nhẩm nghiệm của bài toán.Chỉ 1 lúc sau chúng ta đã thấy kết quả.

Thật là may mắn vì ài này nghiệm của nó có 1 nghiệm rất đẹp đó nha!!

Ta thấy x = 3 là một nghiệm của phương trình. Ta nghĩ ngay đến việc đưa bài toán về

dạng:(x− 3) f(x) = 0 , nên ta biến đổi phương trình như sau:

2 (x− 3) +(√

x+ 6− 3√x− 2

)= 0

Vấn đề còn lại là đi phân tích√x+ 6− 3

√x− 2 ra thừa số x− 3 .

Sao nhìn vào biểu thức này mà tôi lại liên tưởng ra hằng đẳng thức a2 − b2 = (a− b) (a+ b) .

Vậy ta thử đi theo hướng này xem sao, Ta biến đổi

√x+ 6− 3

√x− 2 =

−8 (x− 3)√x+ 6 + 3

√x− 2

16

Page 18: Chuyên đề luyện thi đại học môn Toán - VipLam.Net

Ồ, cách nhân liên hợp đây mà! Vậy thì phương trình đã cho trở thành

(x− 3)

(2− 8√

x+ 6 + 3√x− 2

)= 0 ⇐⇒

x = 38√

x+ 6 + 3√x− 2

= 2

Đến đây bài toán trở nên dễ dàng biết mấy. x = 3 là 1 nghiệm của phương trình đã cho nên ta

chỉ cần đi theo con đường giải quyết phương trình8√

x+ 6 + 3√x− 2

= 2 nữa thôi.

Thật dễ! ta viết lại phương trình đó thành

√x+ 6 + 3

√x− 2 = 4

Đến đây thì tiếp tục giải được rồi nhỉ. Các bạn trình bày tiếp cho tôi với nhé!

Từ đây suy ra được x =11− 3

√5

2

Vậy nghiệm của phương trình đã cho là

x = 3

x =11− 3

√5

2Thông thường nếu ta gặp phương trình dạng :

√A+√B =

√C +√D

ta thường bình phương 2 vế hoặc lập phương. . . nhưng đôi khi việc làm này sẽ dẫn đến cho ta

những vấn đề khó khăn hơn nhiều.

Ví dụ như: 3√A + 3√B = 3

√C =⇒ A + B + 3 3

√AB

(3√A+ 3√B)

= C Và ta lại sử dụng phép

thế. Thế 3√A+ 3√B = 3

√C vào để được A+B + 3 3

√ABC = C

Khó khăn thật nhỉ! Sau đây tôi đưa ra ví dụ để các bạn xem cách giải quyết vấn đề này như

thế này như thế nào nhé!

Bài 3. Giải phương trình: √x+ 3 +

√3x+ 1 = 2

√x+√

2x+ 2

Lời giải:

ĐK: x ≥ 0

Nếu theo cách làm như tôi nói trên thì. Bình phương 2 vế không âm của phương trình ta được:

1 +√

(x+ 3) (3x+ 1) = x+ 2√x (2x+ 1)

Để giải phương trình này không khó nhưng hơi phức tạp thôi. Cách giải quyết bài toán này rất

đơn giản nếu ta chuyển vế của phương trình đã cho về

√3x+ 1−

√2x+ 2 =

√4x−

√x+ 3

Khi đó bình hương 2 vế lên thật đơn giản ta đưa về được

√6x2 + 8x+ 2 =

√4x2 + 12x

17

VVViiipppLLLaaammm...NNNeeettt

Page 19: Chuyên đề luyện thi đại học môn Toán - VipLam.Net

Dễ dàng giải ra kết quả phải không, tôi tin các bạn sẽ làm được và kết quả là x = 1

Đối chiếu với điều kiện thấy thoả mãn, từ đây ta cứ thế kết luận được rồi!

Nhận xét: Nếu phương trình có dạng√f(x) +

√g(x) =

√h(x) +

√k(x)

Mà ta lại có được: f(x) + h(x) = g(x) + k(x) , lúc đó ta biến đổi phương trình về dạng√f(x)−

√h(x) =

√k(x)−

√g(x)

Sau đó bình phương lên thôi. Nhanh gọn.hì hì!!!

Chú ý: Ngoài ra nếu chúng ta bắt gặp những bài phương trình mà có dạng như trên nhưng

thay vì có f(x) + h(x) = g(x) + k(x) mà nó có f(x).h(x) = k(x).g(x) thì cũng biến đổi được√f(x)−

√h(x) =

√k(x)−

√g(x) Nhớ nhé!

Một số bài tập áp dụng:

1. (1− 4x)√

4x2 + 1 = 8x2 + 2x+ 1

2.√x−√x2 − 1 +

√x+√x2 + 1 =

√2 (x3 + 1)

3.√x2 + 1− 1√

x2− 53

= x

4.√x2 +

√x (x− 3) =

√x (2x+ 1)

5.√x+√x+ 11 +

√x−√x+ 11 = 4

Một lần nữa chân thành cảm ơn các bạn đã đón đọc tuyển tập này. Tuyển tập này còn chưa

đầy đủ lắm, nó mới chỉ đưa chúng ta đi một đoạn đường nhỏ trên chặng đường học tập nói

chung, trên con đường chinh phục tuyển tập phương trình - bất phương trình nói chung. Các

bạn nhớ đón đọc tuyển tập của mình lần sau nhé, hi vọng nó sẽ củng cố kiến thức đầy đủ hơn.

Chúc các bạn thành công, chinh phục ước mơ của mình.

18

Page 20: Chuyên đề luyện thi đại học môn Toán - VipLam.Net

CHUYÊN ĐỀ 2

PHƯƠNG PHÁP GIẢI PHƯƠNG TRÌNH

MŨ VÀ LOGARIT (Đỗ Đường Hiếu)

§ 1. PHƯƠNG TRÌNH MŨ

I. Phương pháp đưa về cùng cơ số

Sử dụng các phép biến đồi, ta đưa phương trình về một trong các dạng sau:

• af(x) = ag(x) ⇐⇒ f (x) = g (x) với 0 < a 6= 1

• af(x) = b ⇐⇒ f (x) = loga b nếu b > 0

Ví dụ 1. Giải phương trình: 3x2−4x+5=0 = 9

Lời giải: Phương trình tương đương với:

3x2−4x+5=0 = 32 ⇐⇒ x2 − 4x+ 5 = 2

x2 − 4x+ 3 = 0 ⇐⇒

[x = 1

x = 3

Vậy, phương trình có hai nghiệm x = 1 và x = 3.

Ví dụ 2. Giải phương trình: 0, 125.42x−3 =3√

2

8

)−xLời giải: Đưa hai vế về cùng cơ số 2, ta được:

2−3.24x−6 =(

2−83

)−x⇐⇒ 24x−9 = 2

83x

4x− 9 =8

3x ⇐⇒ x =

27

4

Vậy phương trình có nghiệm x =27

4.

Ví dụ 3. Giải phương trình: 5x+1 − 5x = 2x+1 + 2x+3

Lời giải: Phương trình đã cho tương đương với:

5.5x − 5x = 2.2x + 8.2x ⇐⇒ 4.5x = 10.2x

⇐⇒(

5

2

)x=

5

3⇐⇒ x = 1

19

VVViiipppLLLaaammm...NNNeeettt

Page 21: Chuyên đề luyện thi đại học môn Toán - VipLam.Net

II. Phương pháp đặt ẩn phụ

Mục đích của đặt ẩn phụ là để đưa phương trình về phương trình đại số quen thuộc. Khi đặt

t = af(x), với 0 < a 6= 1, để giải phương trình không có tham số đôi khi ta chỉ cần đưa ra điều

kiện t > 0, nhưng với phương trình chứa tham số ta cần phải chỉ ra điều kiện đúng của ẩn phụ.

Ví dị 1. Giải phương trình: 32x−1 = 2 + 3x−1

Lời giải: Phương trình đã cho tương đương với:

32x − 3x − 6 = 0

Đặt t = 3x, với t > 0, ta có phương trình:

t2 − t− 6 = 0 ⇐⇒

[t = 3

t = −2=⇒ t = 3

Từ đó: 3x = 3 ⇐⇒ x = 1.

Ví dụ 2. Giải phương trình:√x2−2+x − 5.2x−1+

√x2−2 = 6

Lời giải: Đặt 2x+√x2−2 = t, với t > 0, ta có phương trình:

t2 − 5

2t− 6 = 0 ⇐⇒

t = 4

t = −3

2

=⇒ t = 4

Từ đó:

2x+√x2−2 = 4 ⇐⇒ x+

√x2 − 2 = 2

√x2 − 2 = 2− x ⇐⇒

x2 − 2 = (2− x)2

2− x ≥ 0

⇐⇒

4x = 6

x ≤ 2⇐⇒ x =

3

2

Vậy phương trình có một nghiệm là : x =3

2Ví dụ 3. Giải phương trình: 6.4

1x − 13.6

1x + 6.9

1x

Lời giải: Điều kiện: x 6= 0. Chia cả hai vế của phương trình cho 91x > 0, ta có:

6.

(4

9

) 1x

− 13.

(2

3

) 1x

+ 6 = 0

Đặt t =

(2

3

) 1x

, (t > 0), phương trình trở thành:

6t2 − 13t+ 6 = 0 ⇐⇒

t =3

2

t =2

3

20

Page 22: Chuyên đề luyện thi đại học môn Toán - VipLam.Net

Với t =3

2thì

(2

3

) 1x

=3

2⇐⇒ 1

x= −1 ⇐⇒ x = −1

Với t =2

3thì

(2

3

) 1x

=2

3⇐⇒ 1

x= 1 ⇐⇒ x = 1

Phương trình có hai nghiệm: x = −1 và x = 1.

Ví dụ 4. Giải phương trình: 2x2−x − 22+x−x2 = 3

Lời giải: Phương trình đã cho tương đương với:

2x2−x − 4

2x2−x= 3

Đặt t = 2x2−x, (t > 0), ta có phương trình:

t− 4

t= 3 ⇐⇒ t2 − 3t− 4 = 0 ⇐⇒

[t = −1(loại)

t = 4

Với t = 4, ta có:

2x2−x = 4 ⇐⇒ x2 − x = 2 ⇐⇒

[x = −1

x = 2

Phương trình có hai nghiệm: x = −1 và x = 2.

Ví dụ 5. Giải phương trình:(3 +√

5)x

+ 16.(3−√

5)x

= 2x+3

Lời giải: Ta đưa phương trình đã cho tương đương với phương trình :(3−√

5

2

)x

+ 16

(3 +√

5

2

)x

= 8

Tới đây ta để ý rằng :

(3−√

5

2

)x

·

(3 +√

5

2

)x

= 1

Do đó đặt t =

(3 +√

5

2

)x

, t > 0 thì

(3−√

5

2

)x

=1

t

Khi đó phương trình trở thành :

1

t+ 16t = 8 ⇐⇒ 16t2 − 8t+ 1 = 0 ⇐⇒ t =

1

4

Từ đó, ta có: (3 +√

5

2

)x

=1

4⇐⇒ x = −2 log

( 3+√5

2)2

Phương trình có nghiệm duy nhất: x = −2 log( 3+√5

2)2.

III. Phương pháp sử dụng tính chất đồng biến và nghịch biến của

hàm số

Ví dụ 1. Giải phương trình 76−x = x+ 2

Lời giải: Phương trình đã cho tương đương với: 76−x − x− 2 = 0

21

Page 23: Chuyên đề luyện thi đại học môn Toán - VipLam.Net

Nhận thấy x = 5 là nghiệm của phương trình, ta chứng minh phương trình không còn nghiệm

nào khác.

Xét hàm số f (x) = 76−x − x− 2 trên R.

Ta có: f ′ (x) = −76−x ln 7− 1 < 0,∀x ∈ R Nên hàm số f(x) nghịch biến trên R. Từ đó:

- Với x < 5 thì f(x) > f(5) hay 76−x − x− 2 > 0, nên phương trình không có nghiệm x < 5.

- Với x > 5 thì f(x) < f(5) hay 76−x − x− 2 < 0, nên phương trình không có nghiệm x > 5.

Vậy, phương trình có nghiệm duy nhất x = 5.

Ví dụ 2. Giải phương trình:3x + 5x = 6x+ 2

Lời giải: Phương trình đã cho viết lại là:

3x + 5x − 6x− 2 = 0

Nhận thấy rằng x = 0 và x = 1 là các nghiệm của phương trình.

Xét hàm số f(x) = 3x + 5x − 6x− 2, ta có f ′(x) = 3x ln 3 + 5x ln 5− 6

Ta có f ′(0) = ln 15− 6 < 0 và f ′(1) = 3 ln 3 + 5 ln 5− 6 > 0 nên f ′(0).f ′(1) < 0, suy ra tồn tại

t ∈ (0; 1) sao cho f ′(t) = 0.

Ta lại có: f ′′(x) = 3x ln2 3 + 5x ln2 5 > 0∀x ∈ R nên hàm số f ′(x) đồng biến trên R.

Từ đó:

+ Nếu x < t thì f ′(x) < f ′(t) = 0

+ Nếu x < t thì f ′(x) < f ′(t) = 0

Do vậy, ta có bảng biến thiên:

x −∞ t +∞

f ′(x) − 0 +

f(x) +∞& f(t)%

+∞

Từ bảng biến thiên của hàm số suy ra phương trình f(x) = 0 nếu có nghiệm thì chỉ có tối đa

là 2 nghiệm.

Do đó phương trình có hai nghiệm là x = 0 và x = 1.

Ví dụ 3. Giải phương trình: 5x−2 = 5x2−x−1 + (x− 1)2

Lời giải: Phương trình tương đương với:

5x−2 + x− 1 = 5x2−x−1 + x2 − x ⇐⇒ 5x−1 + 5 (x− 1) = 5x

2−x + 5(x2 − x

)(∗)

Xét f(t) = 5t + t, (t ∈ R). Ta có f ′(t) = 5t ln 5 + 1 > 0∀t ∈ R, nên hàm số f(t) luôn đồng biến.

Do vậy:

(∗) ⇐⇒ f(x− 1) = f(x2 − x) ⇐⇒ x− 1 = x2 − x ⇐⇒ x = 1

Phương trình có nghiệm duy nhất x = 1.

22

Page 24: Chuyên đề luyện thi đại học môn Toán - VipLam.Net

IV. Phương pháp lôgarit hóa

Ví dụ 1. Giải phương trình: 3x.2x2

= 1

Lời giải: Lấy lôgarit cơ số 3 hai vế, ta có phương trình tương đương với:

log3 3x.2x2

= log3 1 ⇐⇒ log3 3x + log3 2x2

= 0 ⇐⇒ x+ x2 log3 2 = 0

⇐⇒

x = 0

x = − 1

log3 2

⇐⇒

[x = 0

x = − log2 3

Vậy, phương trình có hai nghiệm x = 0 và x = − log2 3

Ví dụ 2. Giải phương trình: 2x+2.3x = 4x.5x−1

Lời giải: Lôgarit cơ số 2 hai vế phương trình đã cho:

log2

(2x+2.3x

)= log2

(4x.5x−1

)⇐⇒ log2

(2x+2

)+ log2 (3x) = log2 (4x) + log2

(5x−1

)⇐⇒ x+ 2 + x log2 3 = 2x+ (x− 1) log2 5 ⇐⇒ (log2 3− log2 5− 1)x = −2− log2 5

⇐⇒ x =−2− log2 5

log2 3− log2 5− 1

Vậy, phương trình có nghiệm: x =−2− log2 5

log2 3− log2 5− 1.

§ 2. PHƯƠNG TRÌNH LÔGARIT

I. Phương pháp đưa về cùng cơ số

Ví dụ 1. Giải phương trình: log x+ log(x+ 9) = 1

Lời giải: Điều kiện:

x > 0

x+ 9 > 0⇐⇒ x > 0

Với điều kiện đó, phương trình tương đương với:

log [x (x+ 9)] = log 10 ⇐⇒ x (x+ 9) = 10 ⇐⇒ x2 + 9x− 10 = 0 ⇐⇒

[x = 1

x = −10

Vậy, phương trình có một nghiệm x = 1.

Ví dụ 2. Giải phương trình: log4 (log2 x) + log2 (log4 x) = 2

Lời giải: Phương trình đã cho tương đương với:

1

2log2 (log2 x) + log2

(1

2log2 x

)= 2 ⇐⇒ 1

2log2 (log2 x) + log2

(1

2

)+ log2 (log2 x) = 2

⇐⇒ 3

2log2 (log2 x) = 3 ⇐⇒ log2 (log2 x) = 2 ⇐⇒ log2 x = 4 ⇐⇒ x = 16

Vậy, phương trình có một nghiệm duy nhất x = 16.

Ví dụ 3. Giải phương trình:log4 (x+ 1)2 + 2 = log√2

√4− x+ log8 (4 + x)3

23

Page 25: Chuyên đề luyện thi đại học môn Toán - VipLam.Net

Lời giải: Điều kiện:

x 6= −1

−4 < x < 4

Với điều kiện đó, phương trình tương đương với:

log2 |x+ 1|+ log2 4 = log2 (4− x) + log2 (4 + x)

⇐⇒ log2 (4 |x+ 1|) = log2 [(4− x) (4 + x)]

⇐⇒ 4 |x+ 1| = (4− x) (4 + x) ⇐⇒ 4 |x+ 1| = 16− x2 (∗)

- Với −1 < x < 4, ta có:

(∗) ⇐⇒ 4(x+ 1) = 16− x2 ⇐⇒ x2 + 4x− 12 = 0 ⇐⇒

[x = 2

x = −6 (loại)

- Với −4 < x < −1, ta có:

(∗) ⇐⇒ −4(x+ 1) = 16− x2 ⇐⇒ x2 − 4x− 20 = 0 ⇐⇒

[x = 2− 2

√6

x = 2 + 2√

6 (loại)

Vậy, phương trình có hai nghiệm: x− 2 và x = 2− 2√

6

II. Phương pháp đặt ẩn phụ

Ví dụ 1. Giải phương trình: log22 x− 5. log2 x+ 6 = 0

Lời giải: Đặt t = log2 x, ta thu được phương trình:

t2 − 5t+ 6 = 0 ⇐⇒

[t = 2

t = 3

- Với t = 2, ta có: log2 x = 2 ⇐⇒ x = 4

- Với t = 3, ta có: log2 x = 3 ⇐⇒ x = 8

Vậy, phương trình đã cho có hai nghiệm là x = 4 và x = 8.

Ví dụ 2. Giải phương trình:√

3 + log2 (x2 − 4x+ 5) + 2√

5− log2 (x2 − 4x+ 5) = 6

Lời giải: Đặt

u =√

3 + log2 (x2 − 4x+ 5)

v =√

5− log2 (x2 − 4x+ 5); u ≥ 0, v ≥ 0.Từ phương trình đã cho ta

có hệ phương trình: u+ 2v = 6

u2 + v2 = 8⇐⇒

u = 6− 2v

(6− 2v)2 + v2 = 8u = 6− 2v

5v2 − 24v + 28 = 0⇐⇒

u =

2

5

v =14

5

hoặc

u = 2

v = 2

24

Page 26: Chuyên đề luyện thi đại học môn Toán - VipLam.Net

- Với

u =

2

5

v =14

5

, ta có:

3 + log2 (x2 − 4x+ 5) =2

5√5− log2 (x2 − 4x+ 5) =

14

5

⇐⇒ log2

(x2 − 4x+ 5

)= −71

25

x2 − 4x+ 5 = 2−7125 (vô nghiệm)

- Với

u = 2

v = 2, ta có:

3 + log2 (x2 − 4x+ 5) = 2√5− log2 (x2 − 4x+ 5) = 2

⇐⇒ log2

(x2 − 4x+ 5

)= 1

x2 − 4x+ 5 = 2 ⇐⇒

[x = 1

x = 3

Vậy, phương trình đã cho có hai nghiệm: x = 1 và x = 3.

III. Phương pháp sử dụng tính chất đồng biến và nghịch biến của

hàm số

Ví dụ 1. Giải phương trình log7 (x+ 2) = 6− xLời giải: Điều kiện: x > −2

Nhận thấy x = 5 là một nghiệm của phương trình, ta chứng minh phương trình không còn

nghiệm nào khác.

Xét hàm số f (x) = log7 (x+ 2) + x− 6 với x > −2.

Ta có: f ′ (x) =1

(x+ 2) ln 7+ 1 > 0,∀x > −2, nên hàm số đồng biến trên khoảng (−2; +∞). Từ

đó: - Nếu −2 < x < 5 thì f(x) < f(5) = 0 ⇐⇒ log7 (x+ 2) < 6− x nên phương trình không

có nghiệm x, với −2 < x < 5.

- Nếu x > 5 thì f(x) > f(5) = 0 ⇐⇒ log7 (x+ 2) > 6− x nên phương trình không có nghiệm

x, với x > 5.

Vậy. phương trình đã cho chỉ có nghiệm duy nhất x = 5.

Ví dụ 2. Giải phương trình:log3

(x2 + x+ 3

2x2 + 4x+ 5

)= x2 + 3x+ 2

Lời giải: Phương trình đã cho tương đương với:

log3

(x2 + x+ 3

)− log3

(2x2 + 4x+ 5

)=(2x2 + 4x+ 5

)−(x2 + x+ 3

)log3

(x2 + x+ 3

)+(x2 + x+ 3

)= log3

(2x2 + 4x+ 5

)+(2x2 + 4x+ 5

)

25

Page 27: Chuyên đề luyện thi đại học môn Toán - VipLam.Net

Xét hàm số: f(t) = log3 t+ t với t > 0

Ta có:f ′(t) =1

t ln 3+ 1 > 0,∀t > 0 nên hàm số đồng biến trên khoảng (0 : +∞)

Phương trình đã cho tương đương:

f(x2 + x+ 3

)= f

(2x2 + 4x+ 5

)⇐⇒ x2 + x+ 3 = 2x2 + 4x+ 5 ⇐⇒

[x = −1

x = −2

Vậy, phương trình có hai nghiệm x = −2 và x = −1.

IV. Phương pháp mũ hóa

Ví dụ . Giải phương trình: logx−2 (2x) = 3

Lời giải: Điều kiện:

0 < x− 2 6= 1

2x > 0⇐⇒ 2 < x 6= 3

Với điều kiện đó, phương trình tương đương với:

2x = (x− 2)3 ⇐⇒ x3 − 6x2 + 10x− 8 = 0

(x− 4)(x2 − 2x+ 2

)= 0 ⇐⇒ x = 4(thỏa mãn điều kiện)

Vậy, phương trình có nghiệm duy nhất x = 4.

§ 3. CÁC BÀI TẬP TỔNG HỢP

Bài 1. (Trích Đề thi ĐH-CĐ năm 2008 - Khối A) Giải phương trình:

log2x−1

(2x2 + x− 1

)+ logx+1 (2x− 1)2 = 4

Lời giải: Điều kiện:

0 < 2x− 1 6= 1

0 < x+ 1 6= 1⇐⇒

{1

2< x 6= 1

Với điều kiện đó, phương trình đã cho tương đương với:

log2x−1 [(2x− 1) (x+ 1)] + logx+1 (2x− 1)2 = 4

⇐⇒ log2x−1 (2x− 1) + log2x−1 (x+ 1) + 2 logx+1 (2x− 1) = 4

⇐⇒ log2x−1 (x+ 1) + 2 logx+1 (2x− 1) = 3

Đến đây, đặt t = log2x−1 (x+ 1), ta được phương trình:

t+2

t= 3 ⇐⇒ t2 − 3t+ 2 = 0 ⇐⇒

[t = 1

t = 2

- Với t = 1, ta có:

log2x−1 (x+ 1) = 1 ⇐⇒ x+ 1 = (2x− 1) ⇐⇒ x = 2

26

Page 28: Chuyên đề luyện thi đại học môn Toán - VipLam.Net

- Với t = 1, ta có:

log2x−1 (x+ 1) = 1 ⇐⇒ x+ 1 = (2x− 1)2

⇐⇒ 4x2 − 5x = 0 ⇐⇒

x = 0 (loại)

x =5

4(thỏa mãn)

Vậy, phương trình có hai nghiệm x = 2 và x =5

4.

Bài 2.Giải phương trình√9 log2

18x− 4 log2

√x+ 8 =

√2(4− log16 x

4)(x ∈ R)

Lời giải: Phương trình đã cho tương đương với:√9

(−1

3log2 x

)2

− 2 log2 x+ 8 =√

2(4− log2 x)

⇐⇒√

(log2 x)2 − 2 log2 x+ 8 =√

2(4− log2 x)

Đặt t = log2 x, ta được phương trình:

√t2x− 2t+ 8 =

√2(4− t)

⇐⇒

t2 − 2t+ 8 = 2(4− t)2

4− t ≥ 0

⇐⇒

t2 − 14t+ 24 = 0

t ≤ 4⇐⇒ t = 2

Với t = 2, ta có log2 x = 2 ⇐⇒ x = 4

Vậy, phương trình có nghiệm duy nhất x = 4.

Bài 3. Giải phương trình 2xlog4 x = 8log2

√x

Lời giải: Điều kiện xác định: x > 0.

Với điều kiện đó, lôgarit cơ số 2 hai vế phương trình, ta có:

log2 2xlog4 x)

= log2

(8log2

√x)

⇐⇒ 1 + log2 x. log4 x = 3 log2

√x

⇐⇒ 2 + log2 x. log2 x = 3 log2 x

Đặt t = log2 x, ta có phương trình:

t2 − 3t+ 2 = 0 ⇐⇒

[t = 1

t = 2

- Với t = 1, ta có: log2 x = 1 ⇐⇒ x = 2(thỏa mãn)

- Với t = 2, ta có: log2 x = 2 ⇐⇒ x = 4(thỏa mãn)

27

VVViiipppLLLaaammm...NNNeeettt

Page 29: Chuyên đề luyện thi đại học môn Toán - VipLam.Net

Vậy, phương trình có hai nghiệm x = 2 và x = 4.

Bài 4. Giải phương trình:

(9x − 2.3x − 3) log3 (x− 1) + log 13

27 =2

3.9

x+12 − 9x

Lời giải: Điều kiện: x > 1.

Với điều kiện đó, phương trình đã cho tương đương với:

(3x + 1) (3x − 3) log3 (x− 1)− 3 =2

3.3x+1 − 32x

⇐⇒ (3x + 1) (3x − 3) log3 (x− 1) + (3x + 1) (3x − 3) = 0

⇐⇒ (3x + 1) (3x − 3) [log3 (x− 1) + 1] = 0

⇐⇒

3x + 1 = 0

3x − 3 = 0

log3 (x− 1) + 1 = 0

⇐⇒

3x = −1(vô nghiệm)

x = 1 (loại)

x =4

3(thỏa mãn)

Vậy, phương trình có một nghiệm x =4

3Bài 5. Giải phương trình:

log212

(5− 2x) + log2 (5− 2x) . log2x+1 (5− 2x) = log2 (2x− 5)2 + log2 (2x+ 1) . log√2 (5− 2x)

Lời giải: Điều kiện:

5− 2x > 0

0 < 2x+ 1 6= 1⇐⇒

−1

2< x <

5

2x 6= 0

.

Với điều kiện đó, phương trình đã cho tương đương với:

log22 (5− 2x) +

log22 (5− 2x)

log2 (2x+ 1)= 2 log2 (5− 2x) + 2 log2 (2x+ 1) . log2 (5− 2x)

⇐⇒ log22 (5− 2x)

[log2 (2x+ 1) + 1

log2 (2x+ 1)

]= 2 log2 (5− 2x) [1 + log2 (2x+ 1)]

⇐⇒

log2 (5− 2x) = 0

log2 (2x+ 1) = −1

log2 (5− 2x) = 2 log2 (2x+ 1)

⇐⇒

x = 2

x = −1

4x = −2 ∨ x = 1

2

Kết hợp với điều kiện trên, phương trình có 3 nghiệm x = −1

4, x = 1

2và x = 2.

Bài 6. Giải phương trình : log2(x−√x2 − 1)log3(x+

√x2 − 1) = log6(x−

√x2 − 1)

Lời giải: Điều kiện x ≥ 1.

Ta nhận thấy rằng x = 1 là một nghiệm của phương trình đã cho. Do đó ta chỉ cần xét với điều

28

Page 30: Chuyên đề luyện thi đại học môn Toán - VipLam.Net

kiện x > 1. Kèm theo một lưu ý (x−√x2 − 1)(x+

√x2 − 1) = 1

Lúc này, phương trình ban đầu tương đương với

log3(x+√x2 − 1) = log6(x−

√x2 − 1) log(x−

√x2−1) 2

⇐⇒ log3(x+√x2 − 1) = log6 2

⇐⇒ x+√x2 − 1 =3log6 2 (1)

⇐⇒ x−√x2 − 1 =

1

3log6 2(2)

Từ (1) và (2) suy ra x =1

2

(3log6 2 +

1

3log6 2

).

Giá trị x này thoả mãn PT ban đầu!

Vậy, PT đã cho có tập nghiệm S =

{1;

1

2

(3log6 2 +

1

3log6 2

)}Bài 7. Giải phương trình:

√3x − x−

√x+ 1 + 2x.3x + 2x+ 1 = 9x

Lời giải: Điều kiện:

{3x − x ≥ 0

x ≥ −1

Biến đổi phương trình về dạng:

3x − 2x− 1√3x − x+

√x+ 1

= (3x − 2x− 1) (3x + 1)

⇐⇒

3x − 2x− 1 = 0

3x + 1 =1√

3x − x+√x+ 1

Xét phương trình:

3x + 1 =1√

3x − x+√x+ 1

⇐⇒(√

3x − x+√x+ 1

)(3x + 1) = 1

Đế ý là:

1 =(√

3x − x+√x+ 1

)(3x + 1)

=(√

3x − x+√x+ 1

)((√3x − x

)2+(√

x+ 1)2)

<(√

3x − x+√x+ 1

)3

=⇒√

3x − x+√x+ 1 > 1 =⇒ 1√

3x − x+√x+ 1

< 1

Mặt khác 3x + 1 > 1 nên phương trình này vô nghiệm

Xét phương trình: 3x − 2x− 1 = 0, là một dạng phương trình mũ hay gặp

Ta xét hàm số f(x) = 3x − 2x− 1 ta có f ′(x) = 3x ln 3− 2 = 0 ⇐⇒ x = log3

(ln 2

3

)Lập bảng biến thiên của hàm số ra suy ra phương trình nếu có nghiệm thì tối đa là 2 nghiệm.

Mặt khác nhận thấy x = 0;x = 1 thỏa mãn. Nên đó là hai nghiệm của phương trình trên.

29

Page 31: Chuyên đề luyện thi đại học môn Toán - VipLam.Net

Kết luận: Phương trình có hai nghiệm x = 0;x = 1

Bài 8. Giải phương trình sau

3√x2+1 + 2 |x| = 3x+1

Lời giải: Ta xét hai trường hợp:

Trường hợp 1: x < 0

Ta có V T > 3, V P < 3 =⇒ phương trình (1) vô nghiệm.

Trường hợp 2: x ≥ 0

Phương trình (1) ⇐⇒ 3x+1 − 3√x2+1 − 2x = 0

Đặt F (x) = 3x+1 − 3√x2+1 − 2x với x ≥ 0.

Ta có: F′(x) =

(3x+1 − x+ 1√

x2 + 1.3√x2+1

). ln 3 +

3√x2+1

√x2 + 1

. ln 3− 2.

Ta sẽ chứng minh:

(3x+1− x+ 1√

x2 + 1.3√x2+1

)≥ 0(2) với mọi x ≥ 0 ⇐⇒ 3x+1

x+ 1≥ 3

√x2+1

√x2 + 1

với

mọi x ≥ 0.

Thật vậy ta xét hàm số sau: G(a) =3a

avới a ≥ 1

Ta có: G′(a) =

3a(a ln 3− 1)

a2> 0 với mọi a ≥ 1

=⇒ hàm số G(a) đồng biến trên miền a ≥ 1.

Mặt khác với x ≥ 0 =⇒ x+ 1 ≥√x2 + 1 =⇒ G(x+ 1) ≥ G

(√x2 + 1

)=⇒ (2) được chứng

minh xong.

Tiếp tục ta lại có: G

(√x2 + 1

)≥ G(1) = 3 =⇒ 3

√x2+1

√x2 + 1

. ln 3− 2 ≥ 3. ln 3− 2 > 0

=⇒ F′(x) > 0 với mọi x ≥ 0 =⇒ F (x) là hàm số đồng biến trên miền x ≥ 0 =⇒ F (x) ≥

F (0) = 0

=⇒ phương trình F (x) = 0 ⇐⇒ x = 0.

Vậy phương trình (1) có nghiệm duy nhất x = 0.

Bài 9. Giải phương trình :

64log24 x = 3.2log2

2 x + 3xlog4 x + 4

Lời giải: Điều kiện: x > 0

Phương trình tương đương với:

64

1

4log2

2 x= 3.2log2

2 x + 3x

1

2log2 x

+ 4

Đặt t = xlog4 x > 0, ta có: log2 t =1

2log2

2 t

Thay vào phương trình ta có:

64

1

2log2 t

= 3.22 log2 t + 3t+ 4

⇐⇒(2log2 t

)3= 3.

(2log2 t

)2+ 3t+ 4

30

Page 32: Chuyên đề luyện thi đại học môn Toán - VipLam.Net

⇐⇒ t3 − 3t2 − 3t− 4 = 0

⇐⇒ (t− 4)(t2 + t+ 1

)= 0

⇐⇒ t = 4 ⇐⇒ xlog4 x = 4

⇐⇒ log4 x2 = 1 ⇐⇒ log4 x = ±1

⇐⇒

x = 4

x =1

4

Bài 10. Giải phương trình :

log(9x + 7) = 2 + log2(3x + 1)

Lời giải: Ta có: log(9x + 7) = 2 + log2(3x + 1) ⇐⇒ log(9x + 7) = log2(4.3x + 4)

Đặt t = log2(4.3x + 4) = log(9x + 7).

Ta thấy rằng cả hai vế đều là hàm tăng nên khi x =⇒ −∞ thì t =⇒ 2. Do đó t > 2.

Sau phép đặt ta chuyển PT về ẩn t như sau

16.10t + 8.2t − 4t = 128 (1)

Xét f(t) = 16.10t + 8.2t − 4t có f ′(t) = 8(10t ln 10− 2t ln 2) + 8.10t ln 10− 4t ln 4 > 0 ∀t > 2.

Suy ra hàm f(t) đồng biến trên (2; +∞).

Suy ra f(t) > f(2) = 1616 > 128.

Vậy PT (1) vô nghiệm, suy ra PT ban đầu không có nghiệm thực.

Bài 11.Giải phương trình : log27(x2 − 5x+ 6)3

= 12log√3

x−12

+ log9(x− 3)2

Lời giải: Bài toán này, nhìn hình thức thì chắc nhiều bạn thấy bài toán rất cơ bản. Tuy nhiên,

điều cơ bản đó rất dễ làm mất sai lầm nếu chúng ta không tỉnh táo.

Điều kiện :

x2 − 5x+ 6 > 0

x− 1 > 0

x 6= 3

⇐⇒

[1 < x < 2

x > 3

Với điều kiện này phương trình đã cho tương đương với phương trình :

log2(x− 2)(x− 3) = log2

(x− 1

2

)+ log3 |x− 3|

⇐⇒ log3(x− 2)(x− 3) = log3

(x− 1) |x− 3|2

⇐⇒ 2(x− 2)(x− 3) = (x− 1) |x− 3| (1)

Trường hợp 1: Với 1 < x < 2 thì x− 3 < 0. Do đó phương trình (1) tương đương:

2(x− 2)(x− 3) = (x− 1)(3− x) ⇐⇒ −2(x− 2) = x− 1 ⇐⇒ x =5

3

31

Page 33: Chuyên đề luyện thi đại học môn Toán - VipLam.Net

Đối chiếu điều kiện trong trường hợp này ta có x =5

3là nghiệm của phương trình.

Trường hợp 2: Với x > 3 thì x− 3 > 0.Do đó phương trình (1) tương đương :

2(x− 2)(x− 3) = (x− 1)(x− 3) ⇐⇒ 2(x− 2) = x− 1 ⇐⇒ x = 3

Đối chiếu điều kiện trong trường hợp này ta có x = 3 không là nghiệm của phương trình.

Do đó phương trình đã cho có nghiệm duy nhất x =5

3

Bài 12. Giải phương trình 4x + x = 5x

Lời giải: Chia cả hai vế cho 5x ta có :(

45

)x+ x

5x− 1 = 0

Xét hàm số : f (x) =(

45

)x+ x

5x− 1

ta có :

f ′ (x) =

(4

5

)xln

(4

5

)+

1− x. ln (5)

5x

=⇒ f ′ (x) = 0 ⇐⇒ 4x. ln

(4

5

)− x. ln (5) + 1 = 0

Xét : g (x) = 4x. ln(

45

)− x. ln (5) + 1

có : g′ (x) = 4x. ln(

45

). ln (4)− ln (5) < 0

hay f ′(x) = g(x) = 0 có duy nhất 1 nghiệm. Vậy f(x) = 0 có tối đa 2 nghiệm.

Và hai nghiệm đó là : x = 0;x = 1.

Bài 13.Giải phương trình : log2

(x+ 6log3x

)= log3x

Lời giải: Điều kiện:x > 0

Đặt: u = log3x =⇒ x = 3u phương trình đã cho trở thành

log2 (3u + 6u) = u

⇐⇒ 3u + 6u = 2u

⇐⇒(

32

)u+ 3u = 1

Ta thấy hàm số: f (u) =(

32

)u+ 3u đồng biến trên R, mặt khác ta thấy u = −1 là nghiệm của

phương trình.

Vậy pt có nghiệm duy nhất là u = -1 nên pt ban đầu có nghiệm là: x =1

3Bài 14.Giải phương trình log2(1 + 3

√x) = log7 x

Lời giải: Điều kiện x > 0.

Đặt : log7 x = t. Ta có phương trình :

1 +(

3√

7)t

= 2t

Chia cả 2 vế cho 2t, ta có : (1

2

)t+

(3√

7

2

)t

= 1

32

Page 34: Chuyên đề luyện thi đại học môn Toán - VipLam.Net

Hàm số bên VT nghịch biến, pt có nghiệm duy nhất : t = 3.

Từ đó: log7 x = 3 ⇐⇒ x = 343

Bài 15.Giải phương trình:(√

3 + 1)log2 x + x

(√3− 1

)log2 x = 1 + x2

Lời giải: Điều kiện: x > 0.

Đặt

(√

3 + 1)log2 x = u(√

3− 1)log2 x = v

. Khi đó:

uv =(√

3 + 1)log2 x

.(√

3− 1)log2 x

= 2log2 x = x

Nên, phương trình đã cho trở thành:

u+ uv2 = 1 + u2v2

⇐⇒ (u− 1) (uv2 − 1) = 0

⇐⇒

[u = 1

uv2 = 1

⇐⇒

[ (√3 + 1

)log2 x = 1

x(√

3− 1)log2 x = 1

⇐⇒

[log2 x = 0

(log2 x)2 log2

(√3− 1

)= 1

⇐⇒ x = 1

Vậy, phương trình có một nghiệm x = 1.

Bài 16. Giải phương trình :

x3 − 3√x+ 2 lnx− 2

3ln (x+ 2 lnx) = 0

Lời giải: Điều kiện:

x > 0

x+ 2 lnx > 0.

Đặt y = 3√x+ 2 lnx > 0 =⇒ x+ 2 lnx = y3. Từ phương trình đã cho ta có:

x3 − y − 2

3ln y3 = 0 ⇐⇒ x3 = y + 2 ln y

Từ đó, ta có hệ phương trình {x3 = y + 2 ln y

y3 = x+ 2 lnx(∗)

Trừ từng vế của hệ cho nhau ta được:

x3 + x+ 2 lnx = y3 + y + 2 ln y (∗∗)

Xét hàm số f(t) = t3 + t+ 2 ln t với t > 0.

Ta có: f ′(t) = 3t2 + 1 +2

t> 0,∀t > 0, nên hàm số f(t) đồng biến trên (0; +∞).

Do vậy:

(∗∗) ⇐⇒ f(x) = f(y) ⇐⇒ x = y

33

Page 35: Chuyên đề luyện thi đại học môn Toán - VipLam.Net

Thay x = y vào phương trình đầu của hệ (∗), ta được:

x3 = x+ 2 lnx ⇐⇒ x3 − x− 2 lnx = 0

Xét hàm số g(x) = x3 − x− 2 lnx với x > 0, ta có g(1) = 0.

Ta có: g′(x) = 3x2 − 2− 2

x⇐⇒ g′(x) =

(x− 1)(3x2 + 3x+ 2)

x= 0 ⇐⇒ x = 1

Bảng biến thiên của hàm số g(x):

x

f ′(x)

f(x)

0 1 +∞− 0 +

000

Từ bảng biến thiên suy ra phương trình x3 − x− 2 lnx = 0 có nghiệm duy nhất x = 1.

Bài 17. Tìm x để phương trình sau có nghiệm đúng với mọi a:

2 log2+a2

(4−√

7 + 2x)

= log2+a2x2 (4− 3x)

Lời giải: - Điều kiện cần: Giả sử phương trình đã cho nghiệm đúng với mọi a. Khi đó, thay

a = 0, ta được:

2 log2

(4−√

7 + 2x)

= log2 (4− 3x)

⇐⇒

4−√

7 + 2x > 0

4− 3x > 0

4−√

7 + 2x)2

= 4− 3x

⇐⇒

−7

2< x <

4

3(x− 1)

[(5−√

7 + 2x)

+ 3]

= 0

⇐⇒

−7

2< x <

4

3 x = 1

x = −87

25

⇐⇒

x = 1

x = −87

25

- Điều kiện đủ:

* Với x = 1, phương trình đã cho có dạng:

2 log2+a2 1 = log2+a2 1 luôn đúng ∀a)

34

VVViiipppLLLaaammm...NNNeeettt

Page 36: Chuyên đề luyện thi đại học môn Toán - VipLam.Net

* Với x = −87

25, phương trình đã cho có dạng:

log2+a219

5= log

2+a2.(− 8725)

2

19

5

Rõ ràng phương trình không thỏa mãn khi a = 1.

Vậy, giá trị cần tìm của x là x = 1.

Bài 18. Giải phương trình:

3√

5x − 4 +√

5x + 4 = 2√

3. 4√

25x − 16

Lời giải: Đặt: {4√

5x − 4 = u ≥ 04√

5x + 4 = v > 0

Phương trình đã cho trở thành:

3u2 + v2 = 2√

3.uv

⇐⇒ 3(uv

)2

− 2√

3.u

v+ 1 = 0

⇐⇒ u

v=

1√3

√3u = v ⇐⇒ 9u4 = v4

⇐⇒ 9 (5x − 4) = 5x + 4

⇐⇒ 5x = 5 ⇐⇒ x = 1

Vậy, phương trình có nghiệm duy nhất x = 1.

Bài 19. Giải phương trình:(√10 + 1

)log3 x

−(√

10− 1)log3 x

=2x

3

Lời giải: Điều kiện: x > 0. Đặt log3 x = t =⇒ x = 3t, thay vào phương trình ta được:(√10 + 1

)t − (√10− 1)t

=2

3.3t

⇐⇒

(√10 + 1

3

)t

(√10− 1

3

)t

=2

3

Đặt u =(√

10 + 1)t

với t > 0, phương trình trở thành:

u− 1

u=

2

3⇐⇒ 3u2 − 2u− 1 = 0

⇐⇒

u = 1

u = −1

3(loại)

35

Page 37: Chuyên đề luyện thi đại học môn Toán - VipLam.Net

Từ u = 1, suy ra: (√10 + 1

)t= 1 ⇐⇒ t = 0

⇐⇒ log3 x = 0 ⇐⇒ x = 1

Vậy, phương trình có nghiệm duy nhất x = 1.

Bài 20. Xác định k để phương trình sau có ba nghiệm:

4−|x−k| log√2

(x2 − 2x+ 3

)+ 2−x

2+2x log 12

(2 |x− k|+ 2) = 0

Lời giải: Phương trình đã cho tương đương với:

2x2−2x+1 log2

(x2 − 2x+ 3

)= 22|x−k| log2 (2 |x− k|+ 2)

Xét hàm số: f(t) = 2t log2 (t+ 2) với t ≥ 0. Ta có:

f ′(t) = 2t ln t+2t

t+ 2> 0,∀t ≥ 0

Do vậy, hàm số f(t) đồng biến trên [0 : +∞). Từ đó, phương trình đã cho tương đương với:

f(x2 − 2x+ 1

)= f (2 |x− k|) ⇐⇒ x2 − 2x+ 1 = 2 |x− k|

⇐⇒[(x− 1)2 − 2 (x− k)

] [(x− 1)2 + 2 (x− k)

]= 0

⇐⇒

[x2 − 4x+ (1 + 2k) = 0(1)

x2 − (2k − 1) = 0(2)

Phương trình đã cho có ba nghiệm thì phải xảy ra một trong các khả năng sau đây:

a) (1) có hai nghiệm phân biệt, (2) có nghiệm kép không phải là nghiệm của (1). Trường hợp

này tìm được k =1

2.

b) (2) có hai nghiệm phân biệt, (1) có nghiệm kép không phải là nghiệm của (2). Trường hợp

này tìm được k =3

2.

c) (1) và (2) đều có hai nghiệm phân biệt, và chúng có một nghiệm chung. Trường hợp này ta

tìm được k = 1.

Vậy, các giá trị cần tìm của k là: k =1

2, k = 1, k =

3

2.

36

Page 38: Chuyên đề luyện thi đại học môn Toán - VipLam.Net

CHUYÊN ĐỀ 3

CỰC TRỊ CỦA HÀM NHIỀU BIẾN

(Lê Trung Tín)

§ 1. Sử dụng bất đẳng thức cổ điển:

Bất đẳng thức cô-si:

(1) Cho a, b là các số thực không âm. Khi đó, ta luôn có

a+ b

2≥√ab

Dấu “=” xảy ra khi và chỉ khi a = b.

(2) Cho a, b, c là các số thực không âm. Khi đó, ta luôn có

a+ b+ c

3≥ 3√abc

Dấu “=” xảy ra khi và chỉ khi a = b = c.

Bất đẳng thức bunhiacốpxki và hệ quả

(1) Cho a, b, x, y là các số thực tùy ý. Khi đó ta luôn có

(ax+ by)2 ≤ (a2 + b2).(x2 + y2)

Dấu “=” xảy ra khi và chỉ khia

x=b

y(Qui ước: nếu mẫu bằng 0 thì tử bằng 0)

(2) Cho a, b, c, x, y, z là các số thực tùy ý. Khi đó ta luôn có

(ax+ by + cz)2 ≤ (a2 + b2 + c2).(x2 + y2 + z2)

Dấu “=” xảy ra khi và chỉ khia

x=b

y=c

z(Qui ước: nếu mẫu bằng 0 thì tử bằng 0)

Hệ quả: (Svacxơ) Nếu a, b, c là các số thực, x, y, z là các số thực dương thì

a2

x+b2

y+c2

z≥ (a+ b+ c)2

x+ y + z

37

Page 39: Chuyên đề luyện thi đại học môn Toán - VipLam.Net

Dấu “=” xảy ra khi và chỉ khia

x=b

y=c

z.

Chú ý: Khi sử dụng hệ quả này ta phải chứng minh.

Một số hằng đẳng thức đáng nhớ:

(1) (x+ y)2 = x2 + y2 + 2xy.

(2) x3 + y3 = (x+ y)(x2 − xy + y2).

(3) x3 − y3 = (x− y)(x2 + xy + y2).

(4) (x+ y + z)2 = x2 + y2 + z2 + 2(xy + yz + zx).

(5) x3 + y3 + z3 = (x+ y + z)(x2 + y2 + z2 − (xy + yz + zx)) + 3xyz.

Bài 1: Cho x ∈ [0; 3], y ∈ [0; 4]là số thực thay đổi. Tìm giá trị lớn nhất của biểu thức

P = (3− x)(4− y)(2x+ 3y)

Lời giải:

Vì x ∈ [0; 3], y ∈ [0; 4] nên

P =1

6.2(3− x).3(4− y).(2x+ 3y) ≤ 1

6

(2(3− x) + 3(4− y) + (2x+ 3y)

3

)3

= 36

Dấu “=” xảy ra khi và chỉ khi 2(3− x) = 3(4− y) = (2x+ 3y) ⇐⇒ x = 0, y = 2

Vậy giá trị lớn nhất của P là 36, đạt được khi và chỉ khi x = 0, y = 2.

Bài 2: Cho x, y, z là số thực dương thay đổi và thỏa mãn a+ b+ c = 1. Tìm giá trị lớn nhất

của biểu thức

P =3√a+ b+

3√b+ c+ 3

√c+ a

Lời giải:

Theo bất đẳng thức cô-si, ta có:

3√a+ b =

3

√9

4.

3

√(a+ b).

2

3.2

3≤ 3

√9

4.(a+ b) +

2

3+

2

33

3√b+ c =

3

√9

4.

3

√(b+ c).

2

3.2

3≤ 3

√9

4.(b+ c) +

2

3+

2

33

3√c+ a =

3

√9

4.

3

√(c+ a).

2

3.2

3≤ 3

√9

4.(a+ b) +

2

3+

2

33

Suy ra

P ≤ 3

√9

4.2(a+ b+ c) + 4

3=

3√

18

38

Page 40: Chuyên đề luyện thi đại học môn Toán - VipLam.Net

Dấu “=” xảy ra khi và chỉ khi a = b = c =2

3

Vậy giá trị lớn nhất của P là 3√

18, dấu “=” xảy ra khi và chỉ khi a = b = c =2

3.

Bài 3: Cho x, y, z là số thực không âm thay đổi và thỏa mãn xy + yz + zx = 5. Tìm giá trị

nhỏ nhất của biểu thức

P = 6x2 + 6y2 + 2z2

Lời giải:

Ta có:

4x2 + z2 ≥ 4xz

4y2 + z2 ≥ 4yz

2x2 + 2y2 ≥ 4xy

Do đó: P ≥ 4(xy + yz + zx) = 20, dấu “=” xảy ra khi và chỉ khi x = y = 1, z = 2

Vậy giá trị nhỏ nhất của P là 20, dấu “=” xảy ra khi và chỉ khi x = y = 1, z = 2

Bài 4: Cho a, b, c là ba số thực dương thay đổi và thỏa mãn a + b + c = 3. Tìm giá trị nhỏ

nhất của biểu thức

P =a

1 + b2+

b

1 + c2+

c

1 + a2

Lời giải:

Theo bất đẳng thức cô-si, ta có

a

1 + b2= a− ab2

1 + b2≥ a− ab

2b

1 + c2= b− bc2

1 + c2≥ b− bc

2c

1 + a2= c− ca2

1 + a2≥ c− ca

2

Suy ra

P ≥ (a+ b+ c)− ab+ bc+ ca

2= 3− ab+ bc+ ca

2

Mặt khác

(a+ b+ c)2 = a2 + b2 + c2 + 2(ab+ bc+ ca) ≥ 3(ab+ bc+ ca)

⇐⇒ − (ab+ bc+ ca) ≥ (a+ b+ c)2

3= 3

Do đó: P ≥ 3

2, dấu “=” xảy ra khi và chỉ khi a = b = c = 1.

Vậy giá trị nhỏ nhất của P là3

2, đạt được khi và chỉ khi a = b = c = 1.

39

Page 41: Chuyên đề luyện thi đại học môn Toán - VipLam.Net

Bài 5: Cho a, b, c là ba số thực dương thay đổi và thỏa mãn a + b + c = 3. Tìm giá trị nhỏ

nhất của biểu thức

P =a2

a+ 2b2+

b2

b+ 2c2+

c2

c+ 2a2

Lời giải:

Theo bất đẳng thức cô-si, ta có:

a2

a+ 2b2= a− 2ab2

a+ 2b2≥ a− 2ab2

33√ab4

= a− 2

3

3√a2b2

b2

b+ 2c2= b− 2bc2

b+ 2c2≥ b− 2bc2

33√bc4

= b− 2

3

3√b2c2

c2

c+ 2a2= c− 2ca2

c+ 2a2≥ c− 2ca2

33√ca4

= c− 2

3

3√c2a2

Suy ra

P ≥ (a+ b+ c)− 2

3

(3√a2b2 +

3√b2c2 +

3√c2a2

)Mặt khác, theo bất đẳng thức cô-si, ta có:

3√a2b2 =

3√

1.ab.ab ≤ 1 + 2ab

33√b2c2 =

3√

1.bc.bc ≤ 1 + 2bc

33√c2a2 =

3√

1.ca.ca ≤ 1 + 2ca

3

Suy ra

3√a2b2 +

3√b2c2 +

3√c2a2 ≤ 1 +

2(ab+ bc+ ca)

3≤ 1 +

2(a+ b+ c)2

9= 1 + 2 = 3

Do đó P ≥ 3− 2

3.3 = 1, dấu “=” xảy ra khi và chỉ khi a = b = c = 1

Vậy giá trị nhỏ nhất của P là 1, đạt được khi và chỉ khi a = b = c = 1

Bài 6: Cho a, b, c là các số thực dương thay đổi và thỏa mãn a + b + c ≥ 6. Tìm giá trị nhỏ

nhất của biểu thức

P =

√a2 +

1

b2+

√b2 +

1

c2+

√c2 +

1

a2

Lời giải:

Ta có: √a2 +

1

b2=

1√17

√√√√(a2 +

(1

b

)2)

(42 + 12) ≥ 1√17

(4a+

1

b

)√b2 +

1

c2=

1√17

√√√√(b2 +

(1

c

)2)

(42 + 12) ≥ 1√17

(4b+

1

c

)√c2 +

1

a2=

1√17

√√√√(c2 +

(1

a

)2)

(42 + 12) ≥ 1√17

(4c+

1

a

)

40

Page 42: Chuyên đề luyện thi đại học môn Toán - VipLam.Net

Suy ra:

P ≥ 1√17

(4a+ 4b+ 4c+

1

a+

1

b+

1

c

)≥ 1√

17

(15(a+ b+ c)

4+

(a+ b+ c)

4+

9

a+ b+ c

)≥ 1√

17

(15.6

4+ 2.

3

2

)=

3√

17

2

Dấu “=” xảy ra khi và chỉ khi a = b = c = 2

Vậy giá trị nhỏ nhất của P là3√

17

2, đạt được khi và chỉ khi a = b = c = 2.

Bài 7: Cho x, y, z là các số thực thay đổi và thỏa mãn x + y + z = 3. Tìm giá trị nhỏ nhất

của biểu thức

P = x2 + 2y2 + z2

Lời giải:

Ta có

(x2 + 2y2 + 3z2)

((18

11

)2

+ 2

(9

11

)2

+ 3

(6

11

)2)≥ 18

11(x+ y + z)2 =

18

11.9

Suy ra: S ≥ 3

Dấu “=” xảy ra khi và chỉ khi x =18

11, y =

9

11, z =

6

11

Vậy giá trị nhỏ nhất của P là 3, đạt được khi và chỉ khi x =18

11, y =

9

11, z =

6

11

Bài 8: Cho x, y là các số thực thay đổi và thỏa mãn 36x2 + 16y2 = 9. Tìm giá trị lớn nhất

và giá trị nhỏ nhất của biểu thức

P = y − 2x+ 5

Lời giải:

Ta có25

16=((6x)2 + (4y)2

)((−1

3

)2

+

(1

4

)2)≥ (−2x+ y)2

Suy ra −5

4≤ y − 2x ≤ 5

4⇐⇒ 15

4≤ y − 2x+ 5 ≤ 25

4Ta có:

• P =15

4khi và chỉ khi x =

2

5, y = − 9

20

• P =25

4khi và chỉ khi x = −2

5, y =

9

20

Vậy:

• Giá trị nhỏ nhất của P là15

4, đạt được khi và chỉ khi x =

2

5, y = − 9

20

41

Page 43: Chuyên đề luyện thi đại học môn Toán - VipLam.Net

• Giá trị lớn nhất P là25

4, đạt được khi và chỉ khi x = −2

5, y =

9

20

Bài 9: Cho x, y, z là các số thực dương thay đổi và thỏa mãn x2 + y2 + z2 = 3. Tìm giá trị

nhỏ nhất của biểu thức

P =1

xy + 2+

1

xy + 2+

1

yz + 2+

1

zx+ 2

Lời giải:

Theo bất đẳng thức bunhiacốpxki, ta có

xy + yz + zx ≤√x2 + y2 + z2

√y2 + z2 + x2 = 3

Theo hệ quả bất đẳng thức bunhiacốpxki, ta có:

P ≥ 9

xy + yz + zx+ 6≥ 1

Dấu “=” xảy ra khi và chỉ khi x = y = z = 1.

Vậy giá trị nhỏ nhất của P là 1, đạt được khi và chỉ khi x = y = z = 1.

§ 2. Sử dụng phương pháp miền giá trị

(Điều kiện có nghiệm)

Bài 1: Cho các số thực x, y thỏa mãn x2 − xy + y2 = 3. Tìm GTLN, GTNN của biểu thức

P = x2 + xy − 2y2

Lời giải:

Gọi T là tập giá trị của P . Khi đó, m ∈ T khi và chỉ khi hệ sau có nghiệmx2 − xy + y2 = 3

x2 + xy − 2y2 = m(I)

• Nếu y = 0 thì (I) trở thành

x2 = 3

m = 3

• Nếu y 6= 0 thì đặt x = ty, ta có hệy2(t2 − t+ 1) = 3

y2(t2 + t− 2) = m⇐⇒

y2 =

3

t2 − t+ 1

m =3(t2 + t− 2)

t2 − t+ 1

⇐⇒

y = ±√

3

t2 − t+ 1

(m− 3)t2 − (m+ 3)t+m+ 6 = 0

(II)

Trong trường hợp này, hệ (I) có nghiệm khi và chỉ khi hệ (II) có nghiệm y 6= 0, điều này

tương đương phương trình (m− 3)t2 − (m+ 3)t+m+ 6 = 0 có nghiệm.

+ Nếu m = 3 thì (2) có nghiệm t =3

2.

+ Nếu m 6= 3 thì (2) có nghiệm khi và chỉ khi ∆ = −3m2 − 6m + 81 ≥ 0 ⇐⇒ −1 − 2√

7 ≤m ≤ −1 + 2

√7

42

Page 44: Chuyên đề luyện thi đại học môn Toán - VipLam.Net

Ta có:

• m = −1− 2√

7 khi và chỉ khi t =3−√

7

2

⇐⇒

x = −(

√7− 3)

√1

2+

1√7

y =

√2 +

4√7

,

x = (

√7− 3)

√1

2+

1√7

y = −√

2 +4√7

• m = −1 + 2√

7 khi và chỉ khi t =3 +√

7

2

⇐⇒

x = (

√7 + 3)

√1

2− 1√

7

y =

√2− 4√

7

,

x = −(

√7 + 3)

√1

2− 1√

7

y = −√

2− 4√7

Vậy: minP = −1− 2√

7, đạt tại(−(√

7− 3)

√1

2+

1√7,

√2 +

4√7

)hoặc

((√

7− 3)

√1

2+

1√7,−√

2 +4√7

)maxP = −1 + 2

√7, đạt tại(

(√

7 + 3)

√1

2− 1√

7,

√2− 4√

7

), hoặc

(−(√

7 + 3)

√1

2− 1√

7,−√

2− 4√7

)Bài 2: Cho các số thực x 6= 0, y 6= 0 thỏa mãn (x + y)xy = x2 + y2 − xy. Tìm GTLN của

biểu thức

S =1

x3+

1

y3

Lời giải:

Gọi T là tập giá trị của P . Khi đó m ∈ T khi và chỉ khi hệ sau có nghiệm x 6= 0, y 6= 0(x+ y)xy = x2 + y2 − xy1

x3+

1

y3= m

⇐⇒

(x+ y)xy = (x+ y)2 − 3xy(x+ y

xy

)2

= m(I)

Đặt

S = x+ y

P = xy, điều kiện S2 − 4P ≥ 0

Ta có, hệ: SP = S2 = 3P(S

P

)2

= m(II)

Hệ (I) có nghiệm x 6= 0, y 6= 0 khi và chỉ khi hệ (II) có nghiệm (S, P ) thỏa mãn S2 ≥ 4P

Vì SP = x2 + y2 − xy =(x− y

2

)2

+3y2

4> 0 với mọi x 6= 0, y 6= 0. Do đó

S

P> 0 với mọi

x 6= 0, y 6= 0.

Từ đó:

• Nếu m ≤ 0 thì hệ (II) vô nghiệm.

• Nếu m > 0 thì từ phương trình thứ hai của hệ (II), ta cóS

P=√m ⇐⇒ S =

√mP thay

vào phương trình thứ nhất của hệ (II), ta có√mP 2 = mP 2 − 3P ⇐⇒ (m−

√m)P = 3 (Vì

43

Page 45: Chuyên đề luyện thi đại học môn Toán - VipLam.Net

SP > 0 nên P 6= 0).

Để có P thì√m −m 6= 0 ⇐⇒ m 6= 1 (do m > 0) và ta được P =

3√m(√m− 1)

=⇒ S =

3√m− 1

.

Trong trường hợp này, hệ (II) có nghiệm (S, P ) thỏa mãn S2 ≥ 4P khi và chỉ khi(3√m− 1

)2

≥ 43√

m(√m− 1)

⇐⇒√m ≤ 4 ⇐⇒ 0 < m ≤ 16(m 6= 1)

Do đó, Hệ (I) có nghiệm x 6= 0, y 6= 0 khi và chỉ khi 0 < m ≤ 16(m 6= 1)

Ta có m = 16 khi và chỉ khi P =1

4, S = 1 hay x = y =

1

2

Vậy maxP = 16, đạt tại x = y =1

2Sử dụng phương pháp tiếp tuyến:

Bài 3: Cho a, b, c là các số thực không âm thay đổi và thỏa mãn a + b + c = 6. Tìm giá trị

nhỏ nhất của biểu thức

P = a4 + b4 + c4 − 2(a3 + b3 + c3)− 6

Lời giải:

Ta có

P = (a4 − 2a3 − 2) + (b4 − 2b3 − 2) + (c4 − 2c3 − 2)

Xét hàm số f(x) = x4 − 2x3 − 2, với x ∈ [0; 6]

Ta có f ′(x) = 4x3 − 6x2

Phương trình tiếp tuyến của f(x) tại x = 2 là

y = f ′(2)(x− 2) + f(2) ⇐⇒ y = 8x− 18

Bây giờ ta chứng minh f(x) ≥ 8x− 18 (1). Thật vậy, ta có

f(x)− (8x− 18) = x4 − 2x3 − 8x+ 16

= (x− 2)2(x2 − 2x+ 4) ≥ 0

Dấu “=” xảy ra khi và chỉ khi x = 2. Theo (1), ta được

P ≥ (8a− 18) + (8b− 18) + (8c− 18) = 8(a+ b+ c)− 54 = 8.6− 54 = −6

Dấu “=” xảy khi và chỉ khi a = b = c = 2.

Vậy giá trị nhỏ nhất của P là −6, đạt được khi và chỉ khi a = b = c = 2.

Bài 4: Cho a, b, c là các số thực dương thay đổi và thỏa mãn a + b + c = 1. Tìm giá trị nhỏ

nhất của biểu thức

P =a

1 + bc+

b

1 + ca+

c

1 + ab

44

Page 46: Chuyên đề luyện thi đại học môn Toán - VipLam.Net

Lời giải:

Vì a, b, c > 0 và a+ b+ c = 1 nên ta có

a

1 + bc≥ a

1 +

(b+ c

2

)2 =4a

4 + (1− a)2

b

1 + ca≥ b

1 +

(c+ a

2

)2 =4b

4 + (1− b)2

c

1 + ab≥ c

1 +

(a+ b

2

)2 =4c

4 + (1− c)2

Suy ra:

P ≥ 4a

4 + (1− a)2+

4b

4 + (1− b)2+

4c

4 + (1− c)2

Xét hàm số f(x) =4x

4 + (1− x)2=

4x

x2 − 2x+ 5, với x ∈ (0; 1)

Ta có

f ′(x) =−4x2 + 20

(x2 − 2x+ 5)2

Phương trình tiếp tuyến của f(x) tại điểm x =1

3là y =

99x− 3

100

Bây giờ ta chứng minh f(x)− 99x− 3

100≥ 0 (1). Thật vậy, ta có

f(x)− 99x− 3

100=

4x

x2 − 2x+ 5− 99x− 3

100

=(3x− 1)2(15− 11x)

100(x2 − 2x+ 5)≥ 0,∀x ∈ (0; 1)

Dấu “=” xảy ra khi và chỉ khi x =1

3Theo (1), ta được

P ≥ 99a− 3

100+

99b− 3

100+

99c− 3

100=

9

10

Dấu “=” xảy ra khi và chỉ khi a = b = c =1

3

Vậy giá trị nhỏ nhất của P là9

10, đạt được khi và chỉ khi a = b = c =

1

3.

§ 3. Sử dụng phương pháp đưa về khảo sát hàm 1 biến

Bài 5: Cho x, y, z là ba số thực thuộc đoạn [1; 4] và x ≥ y, x ≥ z. Tìm giá trị nhỏ nhất của

biểu thức

P =x

2x+ 3y+

y

y + z+

z

z + x

Lời giải:

45

Page 47: Chuyên đề luyện thi đại học môn Toán - VipLam.Net

Trước hết, ta chứng minh với a, b là các số thực dương và thỏa mãn ab ≥ 1, ta luôn có

1

1 + a+

1

1 + b≥ 2

1 +√ab

(1)

Thật vậy, ta có

(1) ⇐⇒ (a+ b+ 2)(1 +√ab) ≥ 2(1 + a)(1 + b)

⇐⇒ (√ab− 1)(

√a−√b)2 ≥ 0 (luôn đúng với a, b > 0 và ab ≥ 1)

Dấu bằng xảy ra khi và chỉ khi a = b hoặc ab = 1.

Áp dụng (1), với x, y ∈ [1; 2] và x ≥ y, ta có

P ≥ 1

2 + 3y

x

+2

1 +

√x

y

Dấu bằng xảy ra khi và chỉ khiz

y=x

zhoặc

x

y= 1 (2).

Đặt t =

√x

y, điều kiện t ∈ [1; 2]. Khi đó:

P ≥ f(t) =t2

2t2 + 3+

2

1 + t, với t ∈ [1; 2]

Ta có

f ′(t) =−2 (t3(4t− 3) + 3t(2t− 1) + 9)

(2t2 + 3)2(1 + t)2< 0,∀t ∈ [1; 2]

Suy ra f(t) ≥ f(2) =34

33

Dấu “=” xảy ra khi và chi khi t = 2 ⇐⇒ x

y= 4 ⇐⇒

x = 4

y = 1(Do x ≥ y, x, y ∈ [1; 4]) (3)

Kết hợp (2), (3), ta được z = 2

Vậy giá trị nhỏ nhất của P bằng34

33, dấu “=” xảy ra khi và chỉ khi x = 4, y = 1, z = 2.

Bài 6: Cho a và b là các số thực dương thỏa mãn 2(a2 + b2) + ab = (a+ b)(ab+ 2). Tìm giá

trị nhỏ nhất của biểu thức

P = 4

(a3

b3+b3

a3

)− 9

(a2

b2+b2

a2

)Lời giải:

Với a, b > 0, ta có:

2(a2 + b2) + ab = (a+ b)(ab+ 2) ⇐⇒ 2(a2 + b2) + ab = a2b+ b2a+ 2(a+ b)

⇐⇒ 2

(a

b+b

a

)+ 1 = (a+ b) + 2

(1

a+

1

b

)

46

Page 48: Chuyên đề luyện thi đại học môn Toán - VipLam.Net

Theo bất đẳng thức cô-si, ta có

(a+ b) + 2

(1

a+

1

b

)≥ 2

√2

(a

b+b

a+ 2

)

Dấu “=” xảy ra khi và chỉ khi (a+ b) = 2

(1

a+

1

b

)Suy ra:

2

(a

b+b

a

)+ 1 ≥ 2

√2

(a

b+b

a+ 2

)=⇒ a

b+b

a≥ 5

2

Đặt t =a

b+b

a, điều kiện t ≥ 5

2Khi đó:

P = 4(t3 − 3t)− 9(t2 − 2) = 4t3 − 9t2 − 12t+ 18

Xét hàm số f(t) = 4t3 − 9t2 − 12t+ 18, với t ≥ 5

2Ta có:

f ′(t) = 12t2 − 18t− 12 = 6(2t2 − 3t− 2) > 0, ∀t ≥ 5

2

Suy ra: f(t) ≥ f

(5

2

)= −23

4

Dấu “=” xảy ra khi và chỉ khi t =5

2⇐⇒

a

b+b

a=

5

2

(a+ b) = 2

(1

a+

1

b

) ⇐⇒

[a = 2, b = 1

a = 1, b = 2

Vậy giá trị nhỏ nhất của P là −23

4đạt được khi a = 2, b = 1 hoặc a = 1, b = 2.

Bài 7: Cho các số thực không âm a, b, c thỏa mãn a + b + c = 1. Tìm giá trị nhỏ nhất của

biểu thức

P = 3(a2b2 + b2c2 + c2a2) + 3(ab+ bc+ ca) + 2√a2 + b2 + c2

Lời giải:

Ta có

P ≥ (ab+ bc+ ca)2 + 3(ab+ bc+ ca) + 2√

1− 2(ab+ bc+ ca)

Đặt t = ab+ bc+ ca, ta có 0 ≤ t ≤ (a+ b+ c)2

3=

1

3

Xét hàm số f(t) = t2 + 3t+ 2√

1− 2t trên

[0;

1

3

]Ta có:

f ′(t) = 2t+ 3− 2√1− 2t

f ′′(t) = 2− 2√(1− 2t)3

< 0,∀t ∈(

0;1

3

]

47

Page 49: Chuyên đề luyện thi đại học môn Toán - VipLam.Net

Suy ra f ′(t) ≥ f

(1

3

)=

11

3− 2√

3 > 0,∀t ∈[0;

1

3

]Do đó

P ≥ f(t) ≥ f(0) = 2, ∀t ∈[0;

1

3

]

Dấu “=” xảy ra khi và chỉ khi

ab+ bc+ ca = 0

ab = bc = ca

a+ b+ c = 1

⇐⇒

a = 1, b = 0, c = 0

a = 0, b = 1, c = 0

a = 0, b = 0, c = 1

Vậy giá trị nhỏ nhất của P là 2, đạt được khi a = 1, b = 0, c = 0 hoặc a = 0, b = 1, c = 0 hoặc

a = 0, b = 0, c = 1.

Bài 8: Cho các số thực a.b thay đổi và thỏa mãn (a + b)3 + 4ab ≥ 2. Tìm giá trị nhỏ nhất

của biểu thức

P = 3(a4 + b4 + a2b2)− 2(a2 + b2) + 1

Lời giải:

Ta có

(a+ b)3 + 4ab ≥ 2

=⇒ 2 ≤ (a+ b)3 + (a+ b)2 (Do 2√ab ≤ a+ b)

=⇒ (a+ b− 1)((a+ b)2 + 2(a+ b) + 2) ≥ 0

=⇒ a+ b ≥ 1

Mặt khác

P = 3(a4 + b4 + 2a2b2 − a2b2)− 2(a2 + b2) + 1

= 3(a2 + b2)2 − 3a2b2 − 2(a2 + b2) + 1

≥ 3(a2 + b2)2 − 3

4(a2 + b2)2 − 2(a2 + b2) + 1

=9

4(a2 + b2)2 − 2(a2 + b2) + 1

Đặt t = a2 + b2, ta có t = a2 + b2 ≥ (a+ b)2

2≥ 1

2

Xét hàm số f(t) =9

4t2 − 2t+ 1 với t ≥ 1

2

Ta có f ′(t) =9

2t− 2 > 0,∀t ≥ 1

2

Suy ra P ≥ f(t) ≥ f

(1

2

)=

9

16

Dấu “=” xảy ra khi và chỉ khi t =1

2⇐⇒

a+ b = 1

a2 + b2 =1

2

⇐⇒ a = b =1

2.

Vậy giá trị nhỏ nhất của P là9

16, đạt được khi a = b =

1

2.

48

Page 50: Chuyên đề luyện thi đại học môn Toán - VipLam.Net

Bài 9: Cho hai số thực a.b thay đổi và thỏa mãn a2 + b2 = 1. Tìm giá trị lớn nhất và giá trị

nhỏ nhất của biểu thức

P =2(a2 + 6ab)

1 + 2ab+ 2b2

Lời giải:

• Với b = 0, kết hợp điều kiện ta được P = 2.

• Với b 6= 0, kết hợp điều kiện ta được

P =2(ab

)2

+ 12(ab

)(ab

)2

+ 2(ab

)+ 3

Đặt t =a

b, ta được

P = f(t) =2t2 + 12t

t2 + 2t+ 3

Ta có

f ′(t) =−8t2 + 12t+ 36

(t2 + 2t+ 3)2

f ′(t) = 0 ⇐⇒ −8t2 + 12t+ 36 = 0 ⇐⇒

t = −3

2t = 3

Bảng biến thiên:

x

f ′(x)

f(x)

−∞ −32 3 +∞

− 0 + 0 −

22

−6−6

33

22

Theo bảng biến thiên ta được:

• P = f(t) ≥ −6, dấu “=” xảy ra khi và chỉ khi t = −3

2⇐⇒

a =3√13, b = − 2√

13

a = − 3√13, b =

2√13

• P = f(t) ≤ 3, dấu “=” xảy ra khi và chỉ khi t = 3 ⇐⇒

a =3√10, b =

1√10

a = − 3√10, b = − 1√

10

49

Page 51: Chuyên đề luyện thi đại học môn Toán - VipLam.Net

Vậy:

• Giá trị nhỏ nhất của P là -6, đạt được khi và chỉ khi a =3√13, b = − 2√

13, hoặc a =

− 3√13, b =

2√13

• Giá trị lớn nhất của P là 3, đạt được khi và chỉ khi a =3√10, b =

1√10, hoặc a =

− 3√10, b = − 1√

10

Bài 10: Cho x, y, z ∈ (0; 1) và xyz = (1 − x)(1 − y)(1 − z). Tìm giá trị nhỏ nhất của biểu

thức

P = x2 + y2 + z2

Lời giải:

Ta có

xyz = (1− x)(1− y)(1− z) ⇐⇒ 1− (x+ y + z) + xy + yz + zx− 2xyz = 0

⇐⇒ 2− 2(x+ y + z) + 2(xy + yz + zx)− 4xyz = 0

⇐⇒ x2 + y2 + z2 = (x+ y + z)2 − 2(x+ y + z) + 2− 4xyz

Theo bất đẳng thức côsi tacó (x+ y + z

3

)3

≥ xyz

Do đó: x2 + y2 + z2 ≥ (x+ y + z)2 − 2(x+ y + z) + 2− 4

(x+ y + z

3

)3

Đặt t = x+ y + z, điều kiện 0 < t < 3.

Xét hàm số g(t) = − 4

27t3 + t2 − 2t+ 2 với t ∈ (0; 3)

Ta có: g′(t) = −4

9t2 + 2t− 2 với t ∈ (0; 3).

Suy ra g′(t) = 0 ⇐⇒ −4

9t2 + 2t− 2 = 0 ⇐⇒

t =3

2t = 3

Bảng biến thiên:

t

g′(t)

g(t)

0 32 3

− 0 + 0

22

3

4

3

4

11

Dựa vào bảng biến thiên, ta được:

P ≥ g(t) ≥ g

(3

2

)=

3

4, dấu “=” xảy ra khi và chỉ khi t =

3

2⇐⇒ x = y = z =

1

2

50

Page 52: Chuyên đề luyện thi đại học môn Toán - VipLam.Net

Vậy giá trị nhỏ nhất của P là3

4, đạt được khi và chỉ khi x = y = z =

1

2

Bài 11: Cho các số thực x, y, z thoả mãn điều kiện x + y + z = 0.Tìm giá trị nhỏ nhất của

biểu thức:

P = 3|x−y| + 3|y−z| + 3|z−x| −√

6x2 + 6y2 + 6z2

Lời giải:

Ta chứng minh rằng với mọi t ≥ 0 ta luôn có: 3t ≥ t+ 1

Thật vậy,ta xét hàm số f(t) = 3t − t− 1,ta có f ′(t) = 3t. ln 3− 1 > 0 với mọi t ≥ 0.

Áp dụng nhận xét trên ta có :

3|x−y| + 3|y−z| + 3|z−x| ≥ 3 + |x− y|+ |y − z|+ |z − x|

Mặt khác áp dụng bất đẳng thức |a|+ |b| ≥ |a+ b| ta có :

(|x− y|+ |y − z|+ |z − x|)2 = |x− y|2 + |y − z|2 + |z − x|2 + 2 |x− y| (|y − z|+ |z − x|)++ 2 |y − z| (|z − x|+ |x− y|) + 2 |z − x| (|x− y|+ |y − z|)

=⇒ (|x− y|+ |y − z|+ |z − x|)2 ≥ 2(|x− y|2 + |y − z|2 + |z − x|2)

Do đó:

|x− y|+|y − z|+|z − x| ≥√

2(|x− y|2 + |y − z|2 + |z − x|2) =√

6x2 + 6y2 + 6z2 − 2(x+ y + z)2

Mà x+ y + z = 0 nên suy ra : |x− y|+ |y − z|+ |z − x| ≥√

6x2 + 6y2 + 6z2.

Do đó:

P = 3|x−y| + 3|y−z| + 3|z−x| −√

6x2 + 6y2 + 6z2 ≥ 3

Đẳng thức xảy ra khi x = y = z.

Vậy minP = 3, đạt tại x = y = z.

BÀI TẬP LUYỆN TẬP

1. Cho x, y là các số thực dương thay đổi và thỏa mãn x+ y ≥ 4. Tìm giá trị nhỏ nhất của

biểu thức

P = 2x+ 3y +6

x+

10

y

2. Cho x, y, z là ba số thực dương thay đổi. Tìm giá trị nhỏ nhất của biểu thức

P = x

(x

2+

1

yz

)+ y

(y

2+

1

zx

)+ z

(z

2+

1

xy

)3. Cho x, y, z là các số thực dương thay đổi và thỏa mãn x+ y+ z = 1. Tìm giá trị lớn nhất

của

P = 3√

2x+ y + 3√

2y + z + 3√

2z + x

4. Cho x, y, z là các số thực dương thay đổi và thỏa mãn xy + yz + zx = 1. Tìm giá trị nhỏ

nhất của

P = 10x2 + 10y2 + z2

51

Page 53: Chuyên đề luyện thi đại học môn Toán - VipLam.Net

5. Cho các số thực dương x, y, z thỏa mãn 4(x + y + z) = 3xyz. Tìm giá trị lớn nhất của

biểu thức:

P =1

2 + x+ yz+

1

2 + y + zx+

1

2 + z + xy

6. Cho a, b, c là ba số thực dương thay đổi và thỏa mãn a+ b+ c = 3. Tìm giá trị nhỏ nhất

của biểu thức

P =a2

a+ 2b3+

b2

b+ 2c3+

c2

c+ 2a3

7. Cho a, b, c là ba số thực dương thay đổi và thỏa mãn a+ b+ c = 3. Tìm giá trị nhỏ nhất

của biểu thức

P =a2

b2 + 1+

b2

c2 + 1+

c2

a2 + 1

8. Cho a, b, c là ba số thực dương thay đổi và thỏa mãn a+ b+ c = 3. Tìm giá trị nhỏ nhất

của biểu thức

P =a2

b2 + a+

b2

c2 + b+

c2

a2 + c

9. Cho a, b, c là các số thực dương thay đổi và thỏa mãn a+ b+ c ≤ 1. Tìm giá trị nhỏ nhất

của biểu thức

P =

√a2 +

1

a2+

√b+

1

b2+

√c+

1

c2

10. Cho x, y là các số thực thay đổi và thỏa mãn 3x− 4y = 7. Tìm giá trị nhỏ nhất của biểu

thức

P = 3x2 + 4y2

11. Cho x, y, z là các số thực thay đổi và thỏa mãn x2 + y2 + z2 = 1. Tìm giá trị nhỏ nhất

của biểu thức

P = x+ 3y + 5z

12. Cho a, b, c là các số thực dương thay đổi và thỏa mãn a+ b+ c = 1. Tìm giá trị nhỏ nhất

của biểu thức

P =1

a+

1

b+

1

c−(

1

a+ b+

1

b+ c+

1

c+ a

)

13. Cho a, b, c ≥ −3

4là các số thực thay đổi và thỏa mãn a+ b+ c = 1. Tìm giá trị lớn nhất

của biểu thức

P =a

a2 + 1+

b

b2 + 1+

c

c2 + 1

14. Cho a, b, c là các số thực không âm thay đổi và thỏa mãn a + b + c = 1. Tìm giá trị nhỏ

nhất của biểu thức

P =(b+ c− a)2

(b+ c)2 + a2+

(c+ a− b)2

(c+ a)2 + b2+

(a+ b− c)2

(a+ b)2 + c2

52

Page 54: Chuyên đề luyện thi đại học môn Toán - VipLam.Net

15. Cho a, b, c là các số thực dương thay đổi và thỏa mãn a2 + b2 + c2 = 1. Tìm giá trị lớn

nhất của biểu thức

P =1

1− ab+

1

1− bc+

1

1− ca

16. Cho x, y là hai số thực thay đổi và thỏa mãn x2 + xy + y2 = 3. Tìm giá trị lớn nhất và

giá trị nhỏ nhất của biểu thức

P = x2 − xy − 3y2

17. Cho x, y là các số thực dương thỏa mãn x + y + 1 = 3xy. Tìm giá trị lớn nhất của biểu

thức

P =3x

y(x+ 1)+

3y

x(y + 1)− 1

x2− 1

y2

18. Cho x, y, z là các số thực dương thỏa mãn x2 + y2 + z2 + 2xy = 3(x+ y + z). Tìm giá trị

nhỏ nhất của biểu thức

P = x+ y + z +20√x+ z

+20√y + 2

19. Cho hai số dương x, y thỏa mãn 12x2 + 2y2 = 5. Chứng minh rằng:

x+ y +1

xy≥ 7

2

20. Cho các số thực không âm x, y, z thỏa mãn x2 + y2 + z2 = 3. Tìm giá trị lớn nhất của

biểu thức

P =√

3x2 + 7y +√

5y + 5z +√

7z + 3x2

53

Page 55: Chuyên đề luyện thi đại học môn Toán - VipLam.Net

CHUYÊN ĐỀ 4

PHƯƠNG PHÁP SỬ DỤNG TÍNH ĐƠN

ĐIỆU CỦA HÀM SỐ TRONG CÁC BÀI

TOÁN CHỨNG MINH BẤT ĐẲNG THỨC

Lời nói đầu

Phương pháp sử dụng tính đơn điệu của hàm số trong các bài toán chứng minh

bất đẳng thức (BĐT) có lẽ đã trở thành một trong những phương pháp “kinh điển”,

chính vì thế bạn đọc có thể tìm thấy phương pháp này trong tất cả các quyển sách

về BĐT. Tuy nhiên, trong hầu hết các quyển sách viết về BĐT nói chung và phương

pháp sử dụng tính đơn điệu của hàm số nói riêng chỉ mới dừng lại ở việc nêu ra một

BĐT và chứng minh, mà không nói từ đâu lại có BĐT đó.

Trong chuyên đề này các BĐT cũng là những BĐT quen thuộc và đều được

chứng minh bằng phương pháp sử dụng tính đơn điệu của hàm số, tuy nhiên vấn đề

“trọng tâm” ở đây là từ các BĐT đó ta có thể sáng tạo ra các BĐT khác.

Hy vọng rằng qua đó, học sinh có thể biết “quy lạ về quen” khi đứng trước một

BĐT lạ - đây cũng là phương pháp quan trọng thường xuất hiện trong các kì thi

tuyển sinh đại học cao đẳng trong nhưng năm gần đây.

Bài toán chung: Chứng minh BĐT f(x) ≥ g(x) (1)

Cách giải:

Chuyển BĐT(1) thành: h(x) ≥ 0 (2)

Tìm TXĐ của hàm số y = h(x), giả sử là [a; b]

Tính y′ = h′(x) và giải phương trình h′(x) = 0 suy ra nghiệm.

Lập bảng biến thiên suy ra BĐT cần chứng minh.

Chú ý:

1. Nếu phương trình h′(x) = 0 không giải được thì ta tiếp tục tính đạo hàm cấp 2, 3,. . . đến

khi nào có thể xét dấu được thì dừng.

2. Trong một số trường hợp ta phải đặt ẩn phụ cho gọn, chẳng hạn đặt t = ϕ(x) thì ta phải

dựa vào tập xác định của x để tìm tập giá trị của t, từ đó xét hàm số mới có ẩn là t.

3. Trong một số bài toán ta có thể áp dụng các BĐT Côsi, Bunhiacopski, sau đó mới xét hsố.

54

VVViiipppLLLaaammm...NNNeeettt

Page 56: Chuyên đề luyện thi đại học môn Toán - VipLam.Net

§ 1. MỘT SỐ VÍ DỤ TÌM GTLN, GTNN CỦA HÀM SỐ

Trong mục này tôi trình bày một số ví dụ tìm GTLN, GTNN của hàm số.

Ví dụ 1: Tìm GTLN, GTNN của hàm số

f(x) = x+√

4− x2

Lời giải: TXĐ: D = [−2; 2]; f ′(x) = 1− x√4− x2

; f ′(x) = 0 ⇐⇒ x =√

2

Từ bảng biến thiên minx∈[−2;2]

f(x) = f(−2) = −2; maxx∈[−2;2]

f(x) = f(√

2) = 2√

2

Ví dụ 2: Cmrsin3 x

x3> cosx ∀x ∈ (0; ) (1)

Lời giải: BĐT (1)sinx

x> 3√

cosx ∀x > 0

Xét hàm số f(x) =sinx

x− 3√

cosx ∀x ∈ (0; ) có f ′(x) = +− 1

f ′′(x) => 0x(0; )f ′(x) > f ′(0) = 0x(0; )

f(x) > f(0) = 0x(0; ) > cosxx(0; )

Bài tập:

1. Tìm GTNN, GTLN của hàm số f(x) =√

1− x2 + 2 3

√(1− x2)2

2. f(x) = 5 cosx− cos 5x với x ∈[−π4

4

]3. Tìm GTNN, GTLN của hàm số f(x) =

2x+ 3√x2 + 1

4. Tìm GTNN, GTLN của hàm số f(x) = x6 + 4(1− x2)3với x ∈ [−1; 1]

5. Tìm GTNN, GTLN của hàm sốx(√

1− x2 + x)

§ 2. KỸ THUẬT GIẢM BIẾN

I. Kỹ thuật tìm GTLN, GTNN bằng phương pháp thế

Ví dụ 1: Cho x,y>0 thỏa x+ y =5

4. Tìm giá trị nhỏ nhất của biểu thức P =

4

x+

1

4y

Lời giải: Từ giả thiết ta có y =5

4− x khi đó P =

4

x+

1

5− 4x.

Xét hàm số f(x) =4

x+

1

5− 4x;x ∈ (0;

5

4).

Ta có: f ′(x) = − 4

x2+

4

(5− 4x)2;f ′(x) = 0 ⇐⇒ x = 1 ∨ x =

5

3

Từ bảng biến thiên ta có min f(x) = f(1) = 5. Đạt được khi x=1 và x=1

4

55

Page 57: Chuyên đề luyện thi đại học môn Toán - VipLam.Net

Nhận xét: Bài toán này được giải bằng cách thế một biến qua một biến còn lại rồi đưa về

khảo sát hàm số

Ví dụ 2: Cho x, y ∈ R thỏa mãn y ≤ 0;x2 + x = y + 12. Tìm GTLN, GTNN của biểu thức

P = xy + x+ 2y + 17

Lời giải: Từ giả thiết ta có y = x2 + x− 12 ≤ 0 hay −4 ≤ x ≤ 3. Khi đó x3 + 3x2− 9x− 7.

Xét hàm số f(x) = x3 + 3x2 − 9x− 7;x ∈ [−4; 3]

Ta có f ′(x) = 3(x2 − 3 + 2x); f ′(x) = 0 ⇐⇒ x = −3 ∨ x = 1

Từ bảng biến thiên ta có minx∈[−4;3]

f(x) = f(1) = −12; m axx∈[−4;3]

f(x) = f(−3) = f(3) = 20

Vậy minP = −12 đạt được khi x = 1; y = −10 và maxP = 20 đạt được khi x = −3; y = −6

hay x = 3; y = 0

Nhận xét: Bài toán này được giải bằng cách thế một biến qua biến còn lại nhưng phải đánh

giá biến còn lại. Từ đó tìm GTLN, GTNN của hàm số chứa biến bị chặn.

Ví dụ 3: Cho x,y>0 thỏa x+y=1. Tìm GTNN của biểu thức K =x√

1− x+

y√1− y

Lời giải: Từ giả thiết ta có y=1-x, 0<x<1. Khi đó K viết lại thành K =x√

1− x+

1− x√x

Xét hàm số f(x)=x√

1− x+

1− x√x

f ′(x) =2− x

2(1− x)√

1− x− x+ 1

2x√x. f ′(x) = 0 ⇐⇒ x =

1

2

Vậy min K=√

2 đạt được khi x = y =1

2Nhận xét: Bài toán này được giải bằng cách thế một biến qua một biến còn lại và sử dụng

các giả thiết để đánh giá biến còn lại. Từ đó tìm được giá trị nhỏ nhất của hàm số chứa một

biến bị chặn. Ngoài ra ta có thể dùng BĐT Côsi, Bernouli, Jensen,. . . để chứng minh BĐT này.

Bài tập:

1. Cho x, y > 0 thỏa mãn x+ y ≥ 4. Tìm GTLNN của biểu thức P =3x2 + 4

4x+

2 + y3

y2

2. Cho x, y ∈ [−3; 2] thỏa mãn x3 + y3 = 2. Tìm GTNN của P = x2 + y2

3. Cho x+ y = 1. Tìm GTNN của biểu thức P = x3 + y3 + 3(x2 − y2) + 3(x+ y)

4. Cho x, y, a, b ∈ R thỏa mãn 0 < a, b ≤ 4; a+ b ≤ 7 và 2 ≤ x ≤ 3 ≤ y tìm GTNN của biểu

thức P =2x2 + y2 + 2x+ y

xy(a2 + b2)HD: Tìm GTLN của Q = a2 + b2, xét hàm số g(y) = f(x; y với ẩn y; x là tham số, tìm

GTNN của g(y) là h(x). Sau đó tìm GTNN của h(x) với x ∈ [2; 3]

5. Cho x, y ∈ R thỏa x3 ≤ y. Tìm GTLNN của P = x2 + y2 − 8x+ 16

HD: Nếu x>0 thì x6 ≤ y2. Từ đó xét hàm số f(x) = x6 + x2 − 8x + 16 Nếu x ≤ 0 thì

x2 + y2 − 8x+ 16 ≥ 16 với mọi x ≤ 0;x3 ≤ y

56

Page 58: Chuyên đề luyện thi đại học môn Toán - VipLam.Net

6. Cho x, y ∈ (0; 1) thỏa mãn x+ y = 1. Tìm GTNN của biểu thức P = xx + yy.

HD: Xét hàm số f(x) = xx;x ∈ (0; 1). Ta sẽ chứng minhf(x) + f(y)

2≥ f(

x+ y

2) Ta có

P = xx + (1− x)1−x = f(x) + f(1− x) ≥ 2f(1

2) =√

2

II. Bài toán tìm GTLN, GTNN của biểu thức đối xứng.

Trong phần này tôi xin trình bày một số bài toán tìm GTLN, GTNN của biểu thức hai biến

mà giả thiết hoặc biểu thức đó thể hiện tính đối xứng. Từ đó xét hàm số và tìm GTLN, GTNN

của hàm số

Ví dụ 1: Đề thi thử ĐH chuyên Nguyễn Tất Thành – Yên Bái 2012

Cho x, y, z thoả mãn là các số thực: x2 − xy + y2 = 1.Tìm giá trị lớn nhất ,nhỏ nhất của biểu

thức P =x4 + y4 + 1

x2 + y2 + 1

Lời giải: Từ giả thiết suy ra

1 = x2 − xy + y2 ≥ 2xy − xy = xy; 1 = (x+ y)2 − 3xy ≥ −3xy

do đó −1

3≤ xy ≤ 1 . Mặt khác x2 − xy + y2 = 1 ⇐⇒ x2 + y2 = 1 + xy nên

x4 + y4 = −x2y2 + 2xy + 1

Đặt t = xy Bài toán quy về tìm GTLN,GTNN của: P = f(t) =−t2 + 2t+ 2

t+ 2;−1

3≤ t ≤ 1

Ta có f ′(t) = 0 ⇐⇒ −1 +6

(t+ 2)2 = 0 ⇐⇒

[t =√

6− 2

t = −√

6− 2(l)

Do hàm số liên tục trên [ − 1

3; 1] nên so sánh các giá trị của f(

−1

3), f(√

6− 2), f(1)

cho ra kết quả maxP = f(√

6− 2) = 6− 2√

6,minP = f(−1

3) =

11

15Nhận xét: Giả thiết và biểu thức P ở dạng đối xứng với 2 biến x,y. Vì vậy ta tìm cách đổi

biến t=xy. Nhưng để giải trọn vẹn ta phải tìm điều kiện của biến t.

Sau đây là một số bài toán tương tự.

Ví dụ 2: Cho x2 + y2 = x+ y. Tìm GTNN,GTLN của biểu thức P = x3 + y3 + x2y + y2x

Lời giải: Đặt t = x+ y từ giả thiết ta có 2xy = (x+ y)2− (x+ y) = t2− t hay xy =t2 − t

2.

Áp dụng BĐT (x+ y)2 ≤ 2(x2 + y2) = 2(x+ y) hay t2 ≤ 2t suy ra 0 ≤ t ≤ 2.

Khi đó biểu thức P = (x+ y)3 − 2xy(x+ y) = t2

Do đó Max P=4 đạt dược khi t = 2 hay x+ y = 2 và xy = 1 suy ra x = 1; y = 1

Ta có min P=0 khi t = 0 hay x = 0; y = 0

Ví dụ 3: Cho x, y > 0 thỏa mãn x2 + y2 + xy = 1. Tìm GTLN của biểu thức K =xy

x+ y + 1

Lời giải: Đặt t = x+ y. Từ giả thiết ta có xy = t2 − 1.

Áp dụng BĐT (x+ y)2 ≥ 4xy ta có 0 < t ≤ 1√3.

57

Page 59: Chuyên đề luyện thi đại học môn Toán - VipLam.Net

Khi đó K = t− 1 ≤√

3− 3

3

Vậy maxK = t− 1 ≤√

3− 3

3đạt được khi (x; y) = (

2√3

);− 1√3

) và hoán vị của chúng.

Ví dụ 4: Cho xy 6= 0 thỏa mãn x+ y = 1. Tìm GTNN của P =1

x2 + y2+

x2

y2 + 1+

y2

x2 + 1

Lời giải: Đặt t = x2 + y2 ta có (x+ y)2 = 1 nên xy =1− t

2

Áp dụng BĐT (x+ y)2 ≤ 2(x2 + y2) suy ra t ≥ 1

2.

Khi đó P =1

x2 + y2+

(x4 + y4) + (x2 + y2)

x2y2 + (x2 + y2) + 1=

1

t+

2t2 + 8t− 2

t2 + 2t+ 5

Xét hàm số f(t) = P =1

x2 + y2+

(x4 + y4) + (x2 + y2)

x2y2 + (x2 + y2) + 1=

1

t+

2t2 + 8t− 2

t2 + 2t+ 5

f ′(t) = − 1

t2+−4t2 + 24t+ 44

(t2 + 2t+ 5)2f ′(t) = 0 ⇐⇒ (t− 1)(t+ 1)2(t− 5) = 0

Từ BTT ta có maxP = maxt∈[ 1

2;+∞]

f(t) = f(1

2) = f(5) =

12

5đạt được khi (x;y)=(

1

2;1

2) hoặc

x = 2; y = −1

minP = mint∈[ 1

2;+∞]

f(t) = f(1) = 2 đạt được khi (x; y) = (1; 0) hoặc (0; 1)

Bài tập:

1. Cho x, y ∈ R thỏa mãn 0 < x, y ≤ 1 và x + y = 4xy. Tìm GTNN, GTLN của biểu thức

P = x2 + y2 − xy

2. Cho x2 + y2 = 1. Tìm GTLN, GTNN của biểu thức P =x√

1 + y+

y√1 + x

3. Cho xy 6= 0 thỏa (x+ y)xy = x2 + y2 − xy. Tìm GTLN của biểu thức P =1

x3+

1

y3

4. Cho x, y ∈ R thỏa mãn x2 + y2 + xy ≤ 2. Tìm GTLN của A = x2 − xy + y2

5. Cho x, y > 0 thỏa mãn x+y ≤ 1. Tìm GTNN của biểu thức C =x2 + y2 + xy

x+ y+

1

x+

1

y+

3

xy

III. Bài toán tìm GTLN, GTNN của biểu thức chứa 3 biến

Trong phần này tôi trình bày một số dạng bài tìm GTNN- GTLN của biểu thức bằng cách đặt

ẩn phụ kết hợp các BĐT cơ bản hoặc thế hai biến qua một biến còn lại. Từ đó chuyến được về

bài toán tìm GTLN, GTNN hàm số.

Ví dụ 1: Cho x, y, z > 0 thỏa mãn x+ y + 1 =. Chứng minh A =1

xz+

1

yz≥ 16

Lời giải: Đặt t = x+ y từ giả thiết ta có z = 1− t và 0 < t < 1.

Áp dụng BĐT (x+ y)2 ≥ 4xy hay xy ≤ t2

4

Khi đó A =1

xz+

1

yz=

t

xy(1− t)≥ 4

−t2 + t

58

Page 60: Chuyên đề luyện thi đại học môn Toán - VipLam.Net

Xét hàm số f(t) =4

−t2 + t; f ′(t) =

4(2t− 1)

(t2 − t)2; f ′(t) = 0 ⇐⇒ t =

1

2

Từ BBT ta có minA = maxt∈(0;1)

f(t) = f(1

2) = 16 đạt được khi x = y =

1

t; z =

1

2Nhận xét: Bài toán này khá đơn giản ta chỉ cần thế z theo x,y và đổi biến t = x + y chúng

ta đã chuyển được bài toán về một biến.

Ví dụ 2: (khối B–2007)

Cho các số thực không âm thỏa mãn a+ b+ c = 1. Tìm giá trị nhỏ nhất của biểu thức:

A = 3(a2b2 + b2c2 + c2a2) + 3(ab+ bc+ ca) + 2√a2 + b2 + c2

Lời giải: Đặt ab+ bc+ ca = x =⇒ x ≤ (a+ b+ c)2

3=

1

3=⇒ x ∈ [0,

1

3]

A = 3(a2b2 + b2c2 + c2a2) + 3(ab+ bc+ ca) + 2√a2 + b2 + c2

≥ 3(ab+ bc+ ca) + 2√a2 + b2 + c2

= 3(ab+ bc+ ca) + 2√

(a+ b+ c)2 − 2(ab+ bc+ ca) = 3x+ 2√

1− 2x

Xét hàm số f(x) = 3x+ 2√

1− 2x, ∀x ∈ [0,1

3] Đạo hàm f ′(x) = 3− 2√

1− 2x

f ′(x) = 0 ⇐⇒ 3√

1− 2x = 2 ⇐⇒ 9(1− 2x) = 4 ⇐⇒ x =5

18

Dựa vào bảng biến thiên, ta được f(x) ≥ f(0),∀x ∈ [0,1

3] =⇒ A ≥ f(x) ≥ f(0) = 2

Đẳng thức xảy ra khi x = 0 hay hoán vị của bộ (0, 0, 1)

Ví dụ 3: [Dự bị khối A-2 -2009]

Cho x, y, z ∈ [1; 3] và x+ y + z = 6. Tìm GTLN của

P = x3 + 2y3 + z3P = (x+ y)3− 3xy(x+ y) + 5z3 = (8− 2z)3− 3xy(8− 2z) + 5z3

Lời giải: ⇐⇒ P = −3z3 + 96z2 − 384z + 512− 3xy(8− 2z)

Ta có (x− 1)(y − 1) ≥ 0 =⇒ xy ≥ x+ y − 1 = 8− 2z − 1 = 7− 2z

xy(8− 2z) ≥ (7− 2z)(8− 2z) =⇒ −3xy(8− 2z) ≤ −3(7− 2z)(8− 2z)

Do z < 4 =⇒ 8− 2z > 0

P = −3z3 + 96z2 − 384z + 512− 3xy(8− 2z) ≤ −3z3 + 96z2 − 384z + 512− 3(7− 2z)(8− 2z)

=⇒ P ≤ −3z3 + 84z2 − 294z + 344

Từ giả thiết suy ra 2z ≤ 6 =⇒ z ≤ 3 =⇒ 1 ≤ z ≤ 3

Xét hàm số f(z) = −3z3 + 84z2 − 294z + 344 với z ∈ [1; 3] f ′(z) = −9c2 + 168z − 294;

f ′(z) = 0 ⇐⇒ −9z2+168z−294 = 0 ⇐⇒ 3z2−56z+98 = 0 ⇐⇒

z =28 + 7

√10

3/∈ [1; 3]

z =28− 7

√10

3Từ đây ta dễ dàng kết luận bài toán.

Ví dụ 4: (Khối A–2011)

Cho x,y,z là ba số thực thuộc đoạn [1;4] và x ≥ y;x ≥ z. Tìm giá trị nhỏ nhất của biểu thức

P =x

2x+ 3y+

y

y + z+

z

z + x

59

Page 61: Chuyên đề luyện thi đại học môn Toán - VipLam.Net

Lời giải: Đặty

x= a;

z

y= b;

x

z= c. Khi đó abc = 1 và 2 ≥

√bc ≥ 1.

Ta có P =1

2 + 3a+

1

1 + b+

1

1 + c.

Xét bài toán mới này có các biến b và c bình đẳng nên ta dự đoán đẳng thức xảy ra khi

b = c =1√a.

Khi đó P =1

2 + 3a+

2√a

1 +√a

:= f(a) với a ∈[

1

4; 1

].

So sánh f

(1

4

)với f(1) ta dự đoán được P đạt giá trị nhỏ nhất khi a =

1

4.

Khi đó b=c=2 và ta tìm được các giá trị của (x, y, z) tương ứng là (4, 1, 2) .

Ví dụ 5: Cho a, b, c ≥ 0 thỏa mãn a+ b+ c = 3. Chứng minh a2 + b2 + c2 + abc ≥ 4

Lời giải: Ta có:

a2 + b2 + c2 + abc = (a+ b)2− 2ab+ c2 + abc = (c− 2)ab+ (3− c)2 + c2 = (c− 2)ab+ 2c2− 6c+ 9

Đặt: t = ab; (0 ≤ t ≤ (a+ b)2

4=

(3− c)2

4)

Ta có: f(t) = (c− 2)t+ 2c2 − 6c+ 9 Dễ thấy: f(t) là một hàm bậc nhất với biến t. Ta lại có

f(0) = 2c2 − 6c+ 9 = 2(c− 3

2)2 +

9

2≥ 9

2>

f((3− c)2

4) = (c− 2)

((3− c)2)

4+ 2c2 − 6c+ 9 =

(c− 1)2(c+ 2)

4+ 4 ≥ 4

Suy ra: f(t) ≥ 4; t ∈ [0;(3− c)2

4]. Điều phải chứng minh.

Dấu bằng xảy ra khi: a = b = c = 1.

Ví dụ 6: Cho a,b,c là các số thực dương thỏa a ≥ 2b. Chứng minh rằng:

14(a2 + b2 + c2) ≥ 5(a+ b+ c)2

Lời giải:

14(a2 + b2 + c2

)≥ 5(a+ b+ c)2 ⇐⇒ 9

(a2 + b2 + c2

)− 10 (ab+ bc+ ca) ≥ 0

Cố định a và c. Xét hàm: f (b) = 9 (a2 + b2 + c2)− 10 (ab+ bc+ ca) với b ∈(

0;a

2

]f ′ (b) = 18b− 10a− 10c < 0 =⇒ f (b) ≥ 25

4a2 + 9c2 − 15ac =

(5

2a− 3c

)2

≥ 0 Tư đây ta có

đpcm

Ví dụ 7: (Khối A –2012)

Cho x,y,z thực không âm thỏa x+y+z=0. Tìm GTNN của chiều thức thực x,y,z thỏa mãn

x+y+z=0. Tìm giá trị nhỏ nhất của biểu thức:

P = 3|x−y| + 3|y−z| + 3|z−x| −√

6x2 + 6y2 + 6z2

Lời giải:

Cách 1: Từ giả thiết ta có z = −(x+ y) (1) trong 3 số x,y,z luôn có 2 số cùng dấu,

60

Page 62: Chuyên đề luyện thi đại học môn Toán - VipLam.Net

không mất tính tổng quát giả sử hai số đó là x,y ta có xy ≥ 0 Thay (1) vào P ta có

P = 3|x−y| + 3|2y+x| + 3|2x+y| −√

[12(x+ y)2 − xy)]

≥ 3|x−y| + 2.3

|2y + x|+ |2x+ y|2 − 2

√3|x+ y|

≥ 3|x−y| + 2.3

3|x+ y|2 − 2

√3|x+ y|

Đặt t = |x+ y|, (t ≥ 0) xét f(t) = 2.(√

3)3t − 2√

3t

f ′(x) = 2.3(√

3)3tln√

3− 2√

3 = 2√

3.(√

3(√

3)3tln√

3− 1) > 0

Nên suy ra hàm f đồng biến trên [0; +∞) nên f(t) ≥ f(0) = 2. Ta có: 3|x−y| ≥ 30 = 1

Vậy P ≥ 30 + 2 = 3. Dấu bằng xảy ra khi và chỉ khi x = y = z = 0

Cách 2: Ta chứng minh 3t ≥ t+ 1 (∗)Xét hàm f(t) = 3t − t− 1 Có f ′(t) = 3t ln 3− 1 > 0,∀t ≥ 0 và f(t) = 0 nên (∗) đúng

Áp dụng (∗), ta có:

3|x−y| + 3|y−z| + 3|z−x| ≥ 3 + |x− y|+ |y − z|+ |z − x|

Áp dụng bất đẳng thức: |a|+ |b| ≥ |a+ b| ta có:

(|x− y|+ |y − z|+ |z − x|)2 =

= |x−y|2+|y−z|2+|z−x|2+|x−y|(|y−z|+|z−x|)+|y−z|(|x−y|+|z−x|)+|z−x|(|x−y|+|y−z|) ≥≥ 2(|x− y|2 + |y − z|2 + |z − x|2)

Do đó:

|x− y|+|y − z|+|z − x| ≥√

2(|x− y|2 + |y − z|2 + |z − x|2

)=√

6x2 + 6y2 + 6z2 − 2(x+ y + z)2

Mà x+ y + z = 0 suy ra

|x− y|+ |y − z|+ |z − x| ≥√

6x2 + 6y2 + 6z2

Suy ra

P = 3|x−y| + 3|y−z| + 3|z−x| −√

6x2 + 6y2 + 6z2 ≥ 3

Khi x = y = z = 0 thì dấu bằng xảy ra, vậy giá trị nhỏ nhất của P bằng 3.

Ví dụ 8: (Khối B–2012)

Cho các số thực x,y,z thỏa mãn x+ y + z = 0 và x2 + y2 + z2 = 1. Tìm GTLN của biểu thức

P = x5 + y5 + z5

Lời giải: Nếu x+ y + z = 0 thì 2(x5 + y5 + z5) = 5xyz(x2 + y2 + z2)

Chứng minh: Vì x+ y + z = 0 nên dễ có được

x3 + y3 + z3 = 3xyz ⇐⇒ (x3 + y3 + z3)(x2 + y2 + z2) = 3xyz(x2 + y2 + z2)

61

Page 63: Chuyên đề luyện thi đại học môn Toán - VipLam.Net

⇐⇒ x5+y5+z5−xyz(xy+yz+xz) = 3xyz(x2+y2+z2) ⇐⇒ 2(x5+y5+z5) = 5xyz(x2+y2+z2)

Mà x2 + y2 + z2 = 1 nên ta chỉ cần tìm GTLN của:

P =5

2xyz =

5

2x.

(y + z)2 − (y2 + z2)

2=

5

2x3 − 5

4x

Đặt f(x) =5

2x3 − 5

4x với x ∈ [−1; 1]. f ′(x) =

15

2x2 − 5

4. f ′(x) = 0 ⇐⇒ x = ±−

√6

6

dựa vào bảng biến thiên ta tìm được GTLN =5√

6

36khi x,y,z là bộ hoán vị của

(−1√

6;−1√

6;

√6

3

)

Ví dụ 9: Cho a,b,c là các số thực không âm,khác nhau từng đôi một thỏa mãn ab+bc+ca = 4.

Chứng minh rằng :1

(a− b)2 +1

(b− c)2 +1

(c− a)2 ≥ 1.

Lời giải: Không mất tính tổng quát giả sử c = mina,b,c khi đó ta có

1

(a− b)2+

1

(b− c)2+

1

(c− a)2≥ 1

(a− b)2+

1

b2+

1

a2

Khi đó ta chỉ cần chứng minh:

1

(a− b)2+

1

b2+

1

a2≥ 1 ⇐⇒ (ab+ bc+ ca)

(1

(a− b)2+

1

b2+

1

a2

)≥ 4

Mà lại có:

⇐⇒ (ab+ bc+ ca)

(1

(a− b)2+

1

b2+

1

a2

)≥ ab

(1

(a− b)2+

1

b2+

1

a2

)Vì thế ta sẽ đi chứng minh:

ab

(1

(a− b)2+

1

b2+

1

a2

)≥ 4

Ta có:

ab

(1

(a− b)2+

1

b2+

1

a2

)=

1

a

b+b

a− 2

+a

b+b

a.

Khảo sát hàm số f(t) =1

t− 2+ t với t > 2 ; f ′(t) =

−1

(t− 2)2 + 1; f ′(t) = 0 ⇐⇒ t = 3

Dựa vào bảng biến thiên ta sẽ tìm được min f(t) = f(3) = 4. Vậy bài toán đã giải quyết xong.

Ví dụ 10: Cho a,b,c số thực dương và(a+ b+ c)2 = 2(a2 + b2 + c2) > 0 Tìm max ;min

P =a3 + b3 + c3

(a+ b+ c)(ab+ bc+ ca)

Lời giải: Từ giả thiết ta có : ab+ bc+ ac =1

4(a+ b+ c)2.

Do đó P =4(a3 + b3 + c3)

(a+ b+ c)3=

1

16

[(

4a

a+ b+ c)3 + (

4b

a+ b+ c)3 + (

4c

a+ b+ c)3

]62

Page 64: Chuyên đề luyện thi đại học môn Toán - VipLam.Net

Đặt x =4a

a+ b+ c; y =

4b

a+ b+ c; z =

4c

a+ b+ cTa có

{x+ y + z = 4

xy + xz + yz = 4⇐⇒

{y + z = 4− xyz = x2 − 4x+ 4

Do (y + z)2 ≥ 4yz =⇒ 0 ≤ x ≤ 8

3

P =1

16(x3 + y3 + z3) =

1

16

[x3 + (y + z)3 − 3zy(y + z)

]=⇒ P =

1

16(3x3 − 12x2 + 12x+ 16)

Xét hàm số f(x) = 3x3 − 12x2 + 12x+ 16 với x ∈ [0;8

3] =⇒ f ′(x) = 9x2 − 24x+ 12

=⇒ f ′(x) = 0 ⇐⇒ x = v∨x =2

3thỏa mãn đk f(0) = 16; f(2) = 16; f(

2

3) =

176

9; f(

8

3) =

176

9

Vậy MinP = 1 chẳng hạn khi a = 0; b = c 6= 0 MaxP =11

9chẳng hạn khi a = b; c = 4a; a 6= 0

Ví dụ 11: Cho a,b,c>0 thỏa mãn :21ab+ 2bc+ 8ac ≤ 12 Chứng minh rằng:

1

a+

2

b+

3

c≥ 15

2

Lời giải: Đặt x =1

a, y =

1

b, z =

1

cbài toán trở thành: Cho các số thực dương x, y, z thỏa

mãn 2x+ 8y + 21z ≤ 12xyz. Chứng minh rằng: x+ 2y + 3z ≥ 15

2(1)

Từ giả thiết z(12xy − 21) ≥ 2x + 8y > 0, từ đó z ≥ 2x+ 8y

12xy − 21với x >

7

4y(2). Suy ra

VT(1)≥ x+ 2y +2x+ 8y

4xy − 7(3).

Xét hàm số f(x) = x +2x+ 8y

4xy − 7=

4x2y − 5x+ 8y

4xy − 7với biến x >

7

4yvà y là tham số thực

dương =⇒ f ′(x) =16x2y2 − 56xy − 32y2 + 35

(4xy − 7)2

Trên

(7

4y; +∞

)thì f ′(x) = 0 ⇐⇒ x = x0 =

7

4y+

√32y2 + 14

4yvà qua x0 thì f ′(x) đổi dấu

từ âm sang dương nên f(x) đạt cự tiểu tại x0. Suy ra f(x) ≥ f(x0) = 2x0 −5

4y=⇒ V T (1) ≥

f(x) + 2y ≥ f(x0) + 2y = g(y) (4).

Xét hàm số g(y) = 2y+9

4y+

1

2

√32y2 + 14 =⇒ g′(y) = 0 ⇐⇒ (8y2−9)

√32y2 + 14−28 =

0.

Đặt t =√

32y2 + 14 > 0 thì pt trên trở thành t3 − 50t− 112 = 0.

Phương trình này có duy nhất một nghiệm dương t = 8 ⇐⇒ y = y0 =5

4.

Vậy g′(

5

4

)= 0. Với y>0 và qua y0 thì g′(y) đổi dấu từ âm sang dương nên g(y) đạt cực tiểu

tại y0. Lúc đó g(y0) = g

(5

4

)=

15

2.

63

Page 65: Chuyên đề luyện thi đại học môn Toán - VipLam.Net

Từ đó kết hợp với (4) suy ra VT(1) ≥ g(y) ≥ g(y0) =15

2.

Dấu "=" xảy ra với x = 3, y =5

4, z =

2

3hay a =

1

3, b =

4

5, c =

3

2.

Ví dụ 12: Cho x, y, z ∈ (0; 1) thỏa mãn xy + zx+ xz = 1. Tìm GTLNN của

P =x

1− x2+

y

1− y2+

1

1− z2

Lời giải: Ta có P =x2

x(1− x2)+

y2

y(1− y2)+

z2

(1− z2)

Xét hàm số f(x) =1

(1− x2)với 0 < x < 1

f ′(x) =3x2 − 1

x2(1− x2)2 ; f ′(x) = 0 ⇐⇒ x =1√3

(n) ∨ x = − 1√3

(l) Ta có bbt

Từ btt ta có1

(1− x2)≥ 3√

3

2∀x ∈ (0; 1)

Vì vậy P ≥ 3√

3

2(x2 + y2 + z2) ≥ 3

√3

2(xy + xz + yz) =

3√

3

2

Dấu"=" xảy ra khi x = y = z =1√3

Bài tập:

1. Cho x,y,z là ba số thực dương có tổng bằng 3.Tìm giá trị nhỏ nhất của biểu thức P =

3(x2 + y2 + z2)− 2xyz

HD. Biến đổi P = 3[(x+ y + z)2 − 2(xy + yz + zx)

]− 2xyz = 3 [9− 2(xy + yz + zx)]−

2xyz = 27− 6x(y+ z)− 2yz(x+ 3) ≥ 27− 6x(3− x)− (y + z)2

2(x+ 3) =

1

2(−x3 + 15x2−

27x+ 27) Xét hàm số f(x) = −x3 + 15x2 − 27x+ 27 , với 0 < x < 3.

2. (Khối A – 2009) Cho x, y, z là các số thực dương thỏa mãn x(x + y + z) = 3yz. Chứng

minh rằng (x+ y)3 + (x+ z)3 + 3(x+ y)(x+ z)(y + z) ≤ 5(y + z)3

3. (B-09) Cho các số thực x, y thay đổi và thoả mãn (x+ y)3 + 4xy ≥ 2. Tìm GTNN của

biểu thức A = 3(x4 + y4 + x2y2)− 2(x2 + y2) + 1.

4. (B-2006) Cho x, y là các số thực dương không đổi. Tìm GTNN của biểu thức:

A =√

(x− 1)2 + y2 +√

(x+ 1)2 + y2 + |y − 2|.

HD. Áp dụng:√a2 + b2 +

√c2 + d2 ≥

√(a+ c)2 + (b+ d)2, (dấu bằng xảy ra khi ad-bc

= 0), A =√

(1− x)2 + y2 +√

(x+ 1)2 + y2 + |y − 2| ≥ 2√

4 + 4y2 + |y − 2|.

5. Các số thực không âm a, b, c thoả mãn ddk: a+ b+ c = 3.Tìm GTLN của biểu thức

P = (a2 − ab+ b2)(b2 − bc+ c2)(c2 − ca+ a2).

6. Cho x, y, z > 0 thỏa mãn x+ y + z = 3. Tìm GTNN của biểu thức

K =1

4 + 2 ln(1 + x)− y+

1

4 + 2 ln(1 + y)− z+

1

4 + 2 ln(1 + z)− x

64

Page 66: Chuyên đề luyện thi đại học môn Toán - VipLam.Net

HD. Áp dụng BĐT Bunhiacopski và xét hàm số f(t) = 2 ln(1 + t)− t

7. Cho x, y, z≥ 0thoả mãn x+y+z > 0. Tìm giá trị nhỏ nhất của biểu thức P =x3 + y3 + 16z3

(x+ y + z)3

§ 3. BÀI TOÁN TỔNG HỢP

Bài 1: (Đề thi ĐH khối B năm 2011 )

Cho a, b là 2 số thực dương thỏa 2(a2 + b2) + ab = (a+ b)(ab+ 2) Tìm GTNN của biểu thức

P = 4

(a3

b3+b3

a3

)− 9

(a2

b2+b2

a2

)Lời giải: Ta có: a, b > 0

=⇒ 2(a2 + b2

)+ ab = (a+ b) (ab+ 2)

⇐⇒ 2(a2 + b2

)+ ab = a2b+ ab2 + 2 (a+ b)

⇐⇒ 2

(a

b+b

a

)+ 1 = (a+ b) + 2

(1

a+

1

b

)Theo BĐT Cauchy ta có

(a+ b) + 2

(1

a+

1

b

)> 2

√2 (a+ b)

(1

a+

1

b

)= 2

√2

(a

b+b

a+ 2

)

=⇒ 2

(1

a+

1

b

)+ 1 > 2

√2

(a

b+b

a+ 2

)⇐⇒ a

b+b

a>

5

2

Đặt t =a

b+b

a, t >

5

2

=⇒ P = 4(t3 − 3t

)− 9

(t2 − 2

)= 4t3 − 9t2 − 12t+ 18 = f (t)

f ′(x) = 12t2 − 18t− 12; f ′(x) = 0 ⇐⇒

x = 2

x =1

2

Khảo sát hàm số trên ta được: minP = −23

4⇐⇒

a

b+b

a=

5

2

a+ b = 2

(1

a+

1

b

) ⇐⇒

[(a, b) = (1, 2)

(a, b) = (2, 1)

Bài 2: (Đề dự bị khối D năm 2008 )

Cho số nguyên n (n ≥ 2) và 2 số thực không âm x, y. Chứng minh rằngn√xn + yn ≥ n+1

√xn+1 + yn+1. Dấu bằng xảy ra khi nào?

Lời giải: Xét

[x = 0

y = 0, ta có: n

√xn + yn = n+1

√xn+1 + yn+1

Xét x 6= 0, y 6= 0 =⇒ x > 0, y > 0

65

Page 67: Chuyên đề luyện thi đại học môn Toán - VipLam.Net

Ta chứng minh: n√xn + yn > n+1

√xn+1 + yn+1

Ta có:

n√xn + yn > n+1

√xn+1 + yn+1 ⇐⇒ n

√(x

y

)n+ 1 >

n+1

√(x

y

)n+1

+ 1

Không mất tính tổng quát, giả sử x ≤ y. Đặt t =x

y=⇒ 0 < t ≤ 1

Khi đó, bất đẳng thức cần chứng minh tương đương với:

n√tn + 1 >

n+1√tn+1 + 1 ⇐⇒ 1

nln (tn + 1) >

1

n+ 1ln(tn+1 + 1

)Xét hàm số: f (u) =

1

uln (tu + 1) , u ∈ [2,+∞). Ta có: f ′ (u) = − 1

u2ln (tu + 1) +

tu ln t

u (tu + 1)<

0 ∀u ∈ [2,+∞)

Suy ra f (u) giảm trên [2,+∞) =⇒ f (n) > f (n+ 1) ⇐⇒ 1

nln (tn + 1) >

1

n+ 1ln (tn+1 + 1)

Vậy với x, y ≥ 0;n ∈ {Z : n ≥ 2} thì: n√xn + yn ≥ n+1

√xn+1 + yn+1

Đẳng thức xảy ra khi x = 0, y ∈ R ∨ x ∈ R, y = 0.

Bài 3: y =√−x2 + 4x+ 21−

√−x2 + 3x+ 10 = f(x)

Lời giải: Điều kiện : 2 ≤ x ≤ 5

y′ =−x+ 2√

−x2 + 4x+ 21− 3− 2x

2√−x2 + 3x+ 10

. y′ = 0 ⇐⇒ −x+ 2√−x2 + 4x+ 21

− 3− 2x

2√−x2 + 3x+ 10

= 0

Tới đây , chỉ cần bình phương 2 vế lên , và đặt điều kiện (2− x)(3− 2x) ≥ 0 là xong.

⇐⇒

(2− x)(3− 2x) ≥ 0x2 − 4x+ 4

−x2 + 4x+ 21=

4x2 − 12x+ 9

−4x2 + 12x+ 40(2)

Cộng hai vế của (2) ta đượcx ≥ 2 ∨ x ≤ 3

225

−x2 + 4x+ 21=

49

−4x2 + 12x+ 40

⇐⇒ x =1

3∨ x =

29

17(loại)

f(1

3) =√

2. f(2) = 5− 2√

3. f(5) = 4 minY =√

2, đạt được tại x =1

3

Bài 4: Cho x,y thực dương thỏa x2 + y2 = 1. Tìm giá trị nhỏ nhất của biểu thức P =x2 + xy + 2y2

y2 + 1

Lời giải:

Với y = 0, ta được P = 1.

Với y 6= 0, ta đặt t =x

yKhi đó: P =

t2 + t+ 2

t2 + 2= f(t) Ta có

f ′(t) =−t2 + 2

(t2 + 2)2 ; f ′(t) = 0 ⇐⇒ −t2 + 2 = 0 ⇐⇒ t = ±√

2

66

Page 68: Chuyên đề luyện thi đại học môn Toán - VipLam.Net

Lập bảng biến thiên rồi suy ra kết quả sau:

maxP =4 +√

2

4, đạt được khi và chỉ khi x =

√6

3, y =

√3

3hoặc x = −

√6

3, y = −

√3

3

minP =4−√

2

4, đạt được khi và chỉ khi x = −

√6

3, y =

√3

3hoặc x =

√6

3, y = −

√3

3

Bài 5: Cho : a2 + b2 + c2 = 65. Tìm giá trị lớn nhất và nhỏ nhất của hàm số :

y = a + b√

2. sinx + c. sin 2x(x ∈ (0 ,

π

2))

Lời giải:

y2 ≤(a2 + b2 + c2

) (1 + 2sin2x+ sin22x

)= 65

(1 + 2sin2x+ sin22x

)Đặt f(x) = 1 + 2sin2x+ sin22x = 1 + 2sin2x+ 4sin2x.(1− sin2x) = −4sin4x+ 6sin2x+ 1

Đặt sin2x = t , t ∈ (0 , 1) được g(t) = −4t2 + 6t + 1 =⇒ g/(t) = −8t + 6 ; g/(t) =

0 ⇐⇒ t =3

4

y2 ≤ 65.13

4=⇒ −13

√5

2≤ y ≤ 13

√5

2

Dấu ”=” xảy ra khi x =π

3và

1

a=

√2 sinx

b=

sin 2x

chay

1

a=

√6

2b=

√3

2c

Thay vào : a2 + b2 + c2 = 65 =⇒

a = 2

√5

b =√

30

c =√

15

a = −2

√5

b = −√

30

c = −√

15

Bài 6: Tìm giá trị lớn nhất và giá trị nhỏ nhất của hàm số : y = f(x) = x(2010 +

√2012− x2

)Lời giải: Xét hàm số y = f(x) = 2010x+ x

√2012− x2

TXĐ: D =[−√

2012;√

2012].

Ta có y′ =2010√

2012− x2 − 2x2 + 2012√2012− x2

. y′ = 0 ⇐⇒ 2010√

2012− x2 − 2x2 + 2012 = 0

Đặt t =√

2012− x2, t ≥ 0 suy ra x2 = 2012− t2

ta có 2t2 + 2010t− 2012 = 0 ⇐⇒ t = 1 Hay x = ±√

2011.

Ta có f(√

2011) = 2011√

2011; f(−√

2011) = −2011√

2011;

f(√

2012) = 2010√

2012; f(−√

2012) = −2010√

2012.

GTLN là 2011√

2011 đạt được khi x =√

2011.

GTNN là −2011√

2011 đạt được khi x = −√

2011.

Bài 7: Cho x, y, z là các số dương thỏa mãn x+ y + z = 4 và xyz = 2.Tìm GTLN và GTNN

của biểu thức P = x4 + y4 + z4

67

Page 69: Chuyên đề luyện thi đại học môn Toán - VipLam.Net

Lời giải: Đặt

a = xy + yz + zx; x+ y + z = 4;xyz = 2 =⇒ x+ y +2

xy= 4

=⇒ 1 ≤ xy ≤ 3 +√

5

2; 1 ≤ x+ y ≤ 1 +

√5

a = xy +2(x+ y)

xy= xy +

8xy − 4

x2y2; f(xy) = xy +

8xy − 4

x2y2(1 ≤ xy ≤ 3 +

√5

2)

=⇒ 5 ≤ f(xy) = a ≤ −1 + 5√

5

2

P = x4 + y4 + z4 = 2(xy + yz + zx)2 − 64(xy + yz + zx) + 288 = 2a2 − 64a+ 288

f(a) = 2a2 − 64a+ 288(5 ≤ a ≤ −1 + 5√

5

2) =⇒ 383− 165

√5 ≤ f(a) = P ≤ 18

MinP = 383− 165√

5 khi x = 3−√

5; y = z =1 +√

5

2và các hoán vị

MaxP = 18 khi x = 2; y = z = 1 và các hoán vị

Bài 8: Cho 0 < a ≤ b ≤ c. Chứng minh

2a

b+ c+

2b

c+ a+

2c

a+ b≤ 3 +

(c− a)2

a(c+ a)

Lời giải: Đặtb

a= α;

c

a= x.ĐK 1 ≤ α ≤ x. Khi đó bất đẳng thức cần chứng minh trở

thành

2

α + x+

1 + x+

2x

1 + α≤ x2 + x+ 1

x+ 1⇐⇒ x2 + x+ 1 ≥ 2(

x+ 1

α + x+ α +

x(x+ 1)

1 + α)

Xét hàm số f(x) = x2 + x+ 1− 2(x+ 1

α + x+ α +

x(x+ 1)

1 + α) với 1 ≤ α ≤ x

f ′(x) = 2x+ 1− 2(2x+ 1)

α + 1− 2(α− 1)

(x+ α)2 = (α− 1)[2x+ 1

α + 1− 2

(x+ α)2 ] ≥ 0

Vậy f(x) là hàm đồng biến nên f(x) ≥ f(α) = α2 − 3α + 3− 1

α

Nhưng f ′(α) = 2α− 3 +1

α2≥ 3 (theo Cauchy) =⇒ f(x) ≥ f(α) ≥ f(1) = 0

Suy ra điều phải chứng minh

Bài 9: Cho a > b > c > 0. Chứng minh

a3b2 + b3c2 + c3a2 ≥ a2b3 + b2c3 + c2a3

Lời giải: Xét hàm số f(a) = a3b2 + b3c2 + c3a2 − (a2b3 + b2c3 + c2a3)

Có f ′(a) = 3a2b2 + 2ac3 − 2ab3 − 3a2c2

f ′′(a) = 6ab2 − 6ac2 + 2c3 − 2b3 = 2(b− c)[3a(b+c)-b2 − c2 − bc] > 0(do a>b>c>0)

Suy ra f’(a) là hàm đồng biến =⇒ f ′(a) ≥ f ′(b) = b4 + 2bc3 − 3b2c2 > 0(theo Cauchy)

Nên f(a) là hàm đồng biến do đó f(a) > f(b) = 0 Điều phải chứng minh

68

Page 70: Chuyên đề luyện thi đại học môn Toán - VipLam.Net

Bài 10: Cho 0 < α < β <√

6. Chứng minh rằng:

sin β

sinα>β − β3

6

α− α3

6

Lời giải: Bất đẳng thức đã cho tương đương với:

sin β

β − β3

6

>sinα

α− α3

6

Xét : f(x) =x− x3

6sinx

BĐT tương đương: f(β) < f(α)

Ta có: f ′(x) =sinx− x2

2. sinx− x cosx+

x3

3. cosx

sin2xTiếp tục xét:

g(x) = sin x− x2

2. sinx− x cosx+

x3

3. cosx.g′(x) =

−x3 sinx

6< 0∀x ∈ (0; π)

Suy ra g(x) nghịch biến trên (0;π) =⇒ g(x) < g(0) = 0 Suy ra f ′(x) < 0∀x ∈ (0;π) =⇒f ′(x) < 0∀x ∈ (0;

√6) Như vậy f(x) nghịch biến trên (0; π) Với 0 < α < β <

√6 =⇒ f(β) <

f(α) điều cần chứng minh

69

VVViiipppLLLaaammm...NNNeeettt

Page 71: Chuyên đề luyện thi đại học môn Toán - VipLam.Net

CHUYÊN ĐỀ 5

SỬ DỤNG BẤT ĐẲNG THỨC ĐỂ GIẢI BẤT

PHƯƠNG TRÌNH (hthtb22)

NẾU NHƯ PHƯƠNG TRÌNH VÀ BẤT ĐẲNG THỨC ĐÃ ĐẸP LÀ VẬY THÌ CÓ THỂ NÓI

BẤT PHƯƠNG TRÌNH LÀ SỰ KẾT GIỮA 2 NÉT ĐẸP TRÊN .

Qua phương pháp này chúng ta sẽ thấy vẻ đẹp của bất phương trình qua 2 phương diện này

Các đẳng thức thường sử dụng+ Bất đẳng thức AM-GM: Cho x1;x2; ...;xn là các số thực dương. Ta có:

x1 + x2 + ..+ xnn

≥ n√x1.x2.....xn

+ Bất đẳng thức Cauchy - swchwarz

(a21 + a2

2 + ..+ a2n)(b2

1 + b22 + ..+ b2

n) ≥ (a1b1 + a2b2 + ...+ anbn)2

Bài 1. Giải bất phương trình:

√x− 1 +

√3− x+ 4x

√2x ≤ x3 + 10

Lời giải

ĐKXĐ: 1 ≤ x ≤ 3

Áp dụng bất đẳng thức Cauchy − Schwarz ta có:

(√x− 1 +

√3− x)2 ≤ 2(x− 1 + 3− x) = 4

=⇒√x− 1 +

√3− x ≤ 2(1)

Mặt khác :

Áp dụng bất đẳng thức AM −GM ta có:

x3 + 8 ≥ 4x√

2x(2)

70

Page 72: Chuyên đề luyện thi đại học môn Toán - VipLam.Net

Từ (1) và (2) =⇒√x− 1 +

√3− x+ 4x

√2x ≤ x3 + 10

Vậy bất phương trình có nghiệm: 1 ≤ x ≤ 3.

Bài 2. Giải bất phương trình:

x−√x

1−√

2(x2 − x+ 1)≥ 1

<Trích đề thi đại học khối A năm 2010>

Lời giải

ĐK: x ≥ 0

Ta có: x2 − x+ 1 = (x− 1

2)2 +

3

4≥ 3

4∀x ∈ R

Do đó : 1−√

2(x2 − x+ 1) < 0

Khi đó bất phương trình trở thành:√2(x2 − x+ 1) ≤ −x+

√x+ 1(1)

Đây là điểm mấu chốt của bài toán

Nếu bình phương lên chúng ta phải bình phương 2 lần và kết quả là 1 phương trình bậc 4. Với

các quá trình biến đổi tương đương rất dễ gây sai lầm . Và đến phương trình bậc 4 chúng ta

cần có thủ thuật mới tìm được nghiệm. Khi đó ta cần chú ý quan sát:

Sử dụng bất đẳng thức Cauchy-Schwarz ta có:√2(x2 − x+ 1) =

√2[(x− 1)2 + (

√x)2] ≥ 1 +

√x− x (2)

Từ (1) và (2) =⇒ x− 1 =√x ⇐⇒

√x =−1 +

√5

2(do√x ≥ 0)

⇐⇒ x =3−√

5

2

Bài 3.Giải bất phương trình :

√2x− 11 + x√

2 (x2 − 8x+ 14) + 5≥ 1

.

Lời giải

Điều kiện x ≥ 11

2.

Với điều kiện đó, bất phương trình tương đương với

71

Page 73: Chuyên đề luyện thi đại học môn Toán - VipLam.Net

√2x− 11 + x−

√2x2 − 16x+ 28− 5 ≥ 0

⇐⇒(√

2x− 11− (x− 5))

+((2x− 10)−

√2x2 − 16x+ 28

)≥ 0

⇐⇒ −(x− 6)2

√2x− 11 + x− 5

+2(x− 6)2

2x− 10 +√

2x2 − 16x+ 28≥ 0

⇐⇒ (x− 6)2

(2

2x− 10 +√

2x2 − 16x+ 28− 1√

2x− 11 + x− 5

)≥ 0

⇐⇒ (x− 6)2 (2√2x− 11−√

2x2 − 16x+ 28)≥ 0

⇐⇒ (x− 6)2

(−2(x− 6)2

2√

2x− 11 + 2x2 − 16x+ 28

)≥ 0⇐⇒ x = 6.

Vậy tập nghiệm bất phương trình S = {6}.

Bài 4.Giải bất phương trình

√x4 + x2 + 1−

√x(x2 − x+ 1) ≤

√(x2 + 1)3

x

Lời giải

ĐK x > 0.

BPT ⇐⇒√x2 − x+ 1

(√x2 + x+ 1−

√x)≤ (x2 + 1)

√x2 + 1

x

⇐⇒√x2 − x+ 1 (x2 + 1) ≤ (x2 + 1)

√x2 + 1

x

(√x2 + x+ 1 +

√x)

⇐⇒√

1− x

x2 + 1≤√

1 +x2 + 1

x+ 1 luôn đúng với mọi x > 0

Vậy nghiệm BPT là:x > 0.

Bài 5.Giải bất phương trình :

√x− 2 +

√4− x ≤ x2 − 6x+ 11

Lời giải

ĐK: 2 ≤ x ≤ 4

Áp dụng bất đẳng thức Cauchy − Schwarz ta có:

(√x− 2 +

√4− x)2 ≤ 2(x− 2 + 4− x) = 4

=⇒√x− 2 +

√4− x ≤ 2

Lại có; x2 − 6x+ 11 = (x− 3)2 + 2 ≥ 2

72

Page 74: Chuyên đề luyện thi đại học môn Toán - VipLam.Net

Nên√x− 2 +

√4− x ≤ x2 − 6x+ 11

Vậy bất phương trình có nghiệm 2 ≤ x ≤ 4

Bài 6. Giải bất phương trình

1−4√x+ 4√x− 1

3≥ 3√x (x− 1)

(3√x− 3√x− 1

)+

2x

3Lời giải

ĐK:x ≥ 1

BPT ⇐⇒(

3√x− 3√x− 1

)3 ≥ 4√x+ 4√x− 1 + 2x− 2(1)

Do x ≥ 1 =⇒ V P (1) ≥ 1 =⇒ V T (1) ≥ 1 =⇒(

3√x− 3√x− 1

)3 ≥ 1

⇐⇒ 3√x− 1 ≥ 3

√x− 1 ⇐⇒ x− 1− 3 3

√x ( 3√x− 1) ≥ x− 1 ⇐⇒ 0 ≤ x ≤ 1

So với đk suy ra nghiệm BPT là :x = 1.

Bài 7. Giải bất phương trình sau:

√1− x2 +

√1− x+

√1 + x ≥ 3

Lời giải

ĐK: |x| ≤ 1 Áp dụng bất đẳng thức Cauchy − Schwarz ta có:

(√

1− x+√

1 + x)2 ≤ 2(1− x+ 1 + x) = 4

=⇒√

1− x+√

1 + x ≤ 2

Mà√

1− x2 ≤ 1

=⇒√

1− x2 +√

1− x+√

1 + x ≤ 3

Dấu = xảy ra khi x = 1.

Tổng quát:k√

1− x2 + 2n√

1− x+ 2n√

1 + x ≥ 3

Bài 8. Giải bất phương trình sau:

4 + 4x− x2 ≥ |x− 1|+ |x− 2|+ |2x− 3|+ |4x− 14|

Lời giải

Áp dụng bất đẳng thức về dấu giá trị tuyệt đối ta có:

|x− 1|+ |x− 2|+ |2x− 3|+ |4x− 14|= |x− 1|+ |x− 2|+ |2x− 3|+ |14− 4x|≥ |x− 1 + x− 2 + 2x− 3 + 14− 4x| = 8

Mà 4 + 4x− x2 = 8− (x− 2)≤8

Như vâyL 4 + 4x− x2 ≥ |x− 1|+ |x− 2|+ |2x− 3|+ |4x− 14| ⇐⇒ x = 2

73

Page 75: Chuyên đề luyện thi đại học môn Toán - VipLam.Net

Bài 9. Giải bất phương trình sau:

2x2 − 11x− 3 3√

4x− 4 ≤ −21

Lời giải

BPT ⇐⇒ 2x2 − 11x+ 21 ≤ 3 3√

4x− 4

Vì 2x2 − 11x+ 21 = (√

2x− 11

2√

2)2 +

47

8> 0

=⇒ 4x− 4 > 0 =⇒ x > 4

Áp dụng bất đẳng thức AM-GM ta có:

3 3√

4x− 4 = 3 3√

2.2(x− 1) ≤ 2 + 2 + x− 1 = x+ 3

=⇒ 2x2 − 11x+ 21 ≤ x+ 3 ⇐⇒ 2(x− 3)2 ≤ 0 ⇐⇒ x = 3

Thử lại thoả mãn

Vậy bất phương trình có nghiệm x = 3

Bài 10. Giải bất phương trình sau:

√13x2 − 6x+ 10 +

√5x2 − 13x+

17

2+√

17x2 − 48x+ 36 ≤ 1

2(36x− 8x2 − 21)

Lời giải

Ta có:√

13x2 − 6x+ 10 +

√5x2 − 13x+

17

2+√

17x2 − 48x+ 36

=√

(3x+ 1)2 + (2x− 3)2 +

√(2x− 5

2)2 + (x− 3

2)2 +

√x2 + (4x− 6)2

≥ |3x+ 1|+ |2x− 5

2|+ |x|

≥ |6x− 3

2| ≥ 6x− 3

2

Mà1

2(36x− 8x2 − 21)

= 6x− 3

2− (2x− 3)2 ≤ 6x− 3

2Như vậy:√

13x2 − 6x+ 10 +

√5x2 − 13x+

17

2+√

17x2 − 48x+ 36 ≥ 1

2(36x− 8x2 − 21)∀x

√13x2 − 6x+ 10 +

√5x2 − 13x+

17

2+√

17x2 − 48x+ 36 ≤ 1

2(36x− 8x2 − 21)

⇐⇒ x =3

2

Bài 11. Giải bất phương trình sau:

3x(2 +√

9x2 + 3) + (4x+ 2)(1 +√

1 + x+ x2) ≥ 0

74

Page 76: Chuyên đề luyện thi đại học môn Toán - VipLam.Net

Lời giải

BPT ⇐⇒ 3x(2 +√

(3x)2 + 3) ≥ −(2x+ 1)(2 +√

(2x+ 1)2 + 3)

Nếu x ≥ 0 thoả mãn

Nếu x ≤ −1

2Thấy V T < 0;V T ≥ 0 - loại

Nếu −−1

2< x ≤ −1

5=⇒ 3x ≤ −2x− 1 < 0

=⇒ 3x(2 +√

(3x)2 + 3) ≥ −(2x+ 1)(2 +√

(2x+ 1)2 + 3) -thoả mãn

Nếu −−1

5< x < 0. Suy luận tuơng tự để suy ra không thoả mãn.

Kết luận: x ≥ 0;−−1

2< x ≤ −1

5

Bài 12. Giải bất phương trình sau:√1 +√

2x− x2 +

√1−√

2x− x2 ≤ 2(x− 1)4(2x2 − 4x+ 1)

Lời giải

ĐKXĐ: 0 ≤ x ≤ 2

Đặt t = (x− 1)2(0 ≤ t ≤ 1)

BPT trở thành:√1 +√

1− t+√

1−√

1− t = 2t2(2t− 1)

⇐⇒ 1 + t = 2t4(2t− 1)2

⇐⇒ 1

t4+

1

t√t

= 2(2t− 1)2

Vì 0 ≤ t ≤ 1 =⇒ 1

t4+

1

t√t≥ 2 ≥ 2(2t− 1)2

Nên t = 1 ⇐⇒ x = 2

Vậy x=2

Bài 13.Giải bất phương trình sau:

2 4√x+ 4√

2− x ≥ 3 + 4√

1− xLời giải

BPT ⇐⇒ ( 4√x− 4√

1− x) + ( 4√x+ 4√

2− x) ≥ 3

Mà 4√x− 4√

1− x ≤ 1

Vì 4√x− 4√

1− x ≤ 1 là hàm tăng trên [0; 1]

Kết hợp [ 4√x+ 4√

2− x]4 ≤ 4(√x+ 4√

2− x)2 ≤ 8(x+ 2− x) = 16

=⇒ 4√x+ 4√

2− x ≤ 2

Nên ( 4√x− 4√

1− x) + ( 4√x+ 4√

2− x) ≤ 3

Vậy bất phương trình có nghiệm x = 1.

75

Page 77: Chuyên đề luyện thi đại học môn Toán - VipLam.Net

Bài 14.Giải bất phương trình sau:

(1 + x+ x3)(1 +√x.√

1− x) ≥ 2(1 + x2)

Lời giải

ĐK: 0 ≤ x ≤ 1

BPT ⇐⇒ x+ x3 + 1

1 + x2≤ 2

1 +√x√

1− x⇐⇒ x+

1

1 + x2≤ 2

1 +√x√

1− xĐặt x =

1

1 + y2

=⇒ y =

√1− xx

Như vậy BPT ⇐⇒ 1

1 + x2+

1

1 + y2≤ 2

1 + xy⇐⇒ xy ≤ 1

⇐⇒√x√

1− x ≤ 1 Vậy bất phương trình có nghiệm 0 ≤ x ≤ 1

Bài 15.Giải bất phương trình sau:

4√x+ 4√

2x− 1 ≥ 4

√4x− 1

3+

4

√5x− 2

3

Lời giải

Trước hết ta sẽ chứng minh bất đẳng thức sau:

4√a+ 4√b+ 4√c ≤

∑4

√a+ 2b

3(?)

Thật vậy:

Ta có:4√a+ 4√b

2≤

√√a+√b

2≤ 4

√a+ b

2

Lại có: 4√a+ 4√b+ 4√c+ 4

√a+ b+ c

3

≤ 2

4

√a+ b

2+

4

√√√√c+a+ b+ c

32

= 4 4

√a+ b+ c

3

Áp dụng ta có:4√a+ 4√b+ 4√b

3≤ a+ b+ b

3

76

Page 78: Chuyên đề luyện thi đại học môn Toán - VipLam.Net

Tương tự với các biến khác và cộng lại ta chứng minh đươc bất đẳng thức (?)

Trở lại bài toán

Ta có:

4√x+ 4√x+ 4√

2x− 1 ≤ 4√x+

4

√4x− 1

3+

4

√5x− 2

3

Dấu "=" xảy ra khi x = 1. Vậy bất phương trình có nghiệm x = 1

BÀI TẬP TỰ LUYỆN

Giải các bất phương trình sau

1 √6

2− x+

√10

3− x≤ 4

22013√

1− x2 + 14√

1− x+ 2√

1 + x ≤ 3

3 √4x− 1 +

√8x3 − 1 ≤ 1

4 (1 +

9

x

)√x2 − 9x+ 22 +

(1− 9

x

)√x2 − 10x+ 22 ≤ 6

5 √6

3− x+

√8

2− x≤ 6

6 √4x− 1 +

√4x2 − 1 ≥ 1

7√

3x2 − 1 +√x2 − x− x

√x2 + 1 ≤ 1

2√

2(7x2 − x+ 4)

8 √x2 + 2x+

√2x− 1 ≤

√3x2 + 4x+ 1

9

32x2 − 4x+ 1 ≤√

4x(8x+ 1)

10 √(x+ 2)(2x− 1)− 3

√x+ 6 = 4−

√(x+ 6)(2x− 1) + 3

√x+ 2

11 √9 + x2 − 3x

√2 +

√16 + x2 − 4x

√2 ≤ 5

77

Page 79: Chuyên đề luyện thi đại học môn Toán - VipLam.Net

12 √6 + 2

√5 + 4x2 − x(1 +

√5)2 +

√4 + 4x2 − 2x(1 +

√5) ≤ 1 +

√5

13 √2x− 1 + 4

√4x− 3 + 6

√6x− 5 ≥ 3x

14 √x+ 4√x+ 6√x ≥ 3 +

√1− x+ 4

√1− x+ 6

√1− x

15

2(x−√x)2 − 4(x−

√x) + 4 ≤ 3x+ 3

√x− 1

(x+ 1)√√

x+ 1

16 √x− 1 ≥ x3 − 2x2 + 2x

17

(1 + x+ x3)(1 +√x.√

1− x) ≤ 2(1 + x2)

18 √2 +

√2 +

√2 + ...+

√x+ 2 ≤ 2

191√x

+1√

2x− 1≤√

3

(1√

4x− 1+

1√5x− 2

)20

(1 +√x)(1 +

√2x− 1) ≤ (1 + 6

√x 3√

2x− 1)(1 + 3√x. 6√

2x− 1)

21

x+ 4√x(1− 2x)(

√x+√

1− 2x) ≥ 1

22

(√x+ 2

√2x− 1)(

√2x− 1 + 2

√x) < 9

√x(2x− 1)

23

x√x+ (2x− 1) ≥ 1

3√

3[(5x− 2)

√5x− 2 + (4x− 1)

√4x− 1]

24

2 4√x+ 4√

3− 2x < 3

Và bây giờ các bạn có thể đặt hết các dấu ≥;≤.Như vậy các bạn có thể giải thêm độ khoảng gần 50 bài phương trình. Ngoài ra chúng ta cũng

biết thêm nhiều bất đằng thức và áp dụng của nó.

78

Page 80: Chuyên đề luyện thi đại học môn Toán - VipLam.Net

CHUYÊN ĐỀ 6

CHỨNG MINH BẤT ĐẲNG THỨC, TÌM

GIÁ TRỊ LỚN NHẤT, GIÁ TRỊ NHỎ NHẤT

BẰNG PHƯƠNG PHÁP HÀM SỐ (Nguyễn

Hữu Phương)

Bài 1: Cho các số dương a, b, c đôi một khác nhau thỏa mãn đồng thời các điều kiện ab+ bc =

2c2 và 2a ≤ c. Tìm giá trị lớn nhất của biểu thức:

P =a

a− b+

b

b− c+

c

c− aLời giải

Đặt

a = x.c

b = y.c=⇒

0 < x ≤ 1

2xy + y = 2

=⇒

0 < x ≤ 1

2

y =2

x+ 1Ta có

P =x

x− y+

y

y − 1+

1

1− x=

x (x+ 1)

x2 + x− 2+

2

1− x+

1

1− x=x2 − 2x− 6

x2 + x− 2

Ta có

P ′ =3x2 + 8x+ 10

(x2 + x− 2)2 > 0 ∀x ∈(

0;1

2

]=⇒ P (x) ≤ P

(1

2

)=

27

5

=⇒ MaxP =27

5⇐⇒

x =

1

2

y =4

3

⇐⇒

a =

1

2c

b =4

3c

Bài 2: Cho x, y, z thỏa mãn: x+ y + z = 0. Tìm GTNN của biểu thức:

P = 3|x−y| + 3|y−z| + 3|z−x| −√

6x2 + 6y2 + 6z2

Lời giải

Đặt a = |x− y| ≥ 0, b = |y − z| ≥ 0, c = |z − x| ≥ 0 =⇒

a2 = x2 + y2 − 2xy

b2 = y2 + z2 − 2yz

c2 = z2 + x2 − 2zx

79

Page 81: Chuyên đề luyện thi đại học môn Toán - VipLam.Net

=⇒ a2 + b2 + c2 = 2(x2 + y2 + z2

)− 2 (xy + yz + zx)

= 2(x2 + y2 + z2

)−[(x+ y + z)2 −

(x2 + y2 + z2

)]= 3

(x2 + y2 + z2

)=⇒ x2 + y2 + z2 =

a2 + b2 + c2

3

Thay vào P ta có;

P = 3a + 3b + 3c −√

2.√a2 + b2 + c2

Ta có:

|a− b| ≤ c, |b− c| ≤ a, |c− a| ≤ b

=⇒ 2 (ab+ bc+ ca) ≥ a2 + b2 + c2

⇐⇒ (a+ b+ c)2 ≥ 2(a2 + b2 + c2

)=⇒ a2 + b2 + c2 ≤ (a+ b+ c)2

2

=⇒ P ≥ 3a + 3b + 3c − (a+ b+ c)

Áp dụng BĐT Cauchy ta có:

P ≥ 33√

3a+b+c − (a+ b+ c)

Đặt t = a+ b+ c ≥ 0 =⇒ P ≥ 3.3

t

3 − t Xét hàm số f (t) = 3.3

t

3 − t (t ≥ 0)

Ta có: f ′ (t) = 3.3

t

3 .1

3. ln 3− 1 = 3

t

3 . ln 3− 1 > 0 ∀t ≥ 0 =⇒ f (t) đồng biến trên [0; +∞)

=⇒ f (t) ≥ f (0) = 3 =⇒ MinP = 3 ⇐⇒ a = b = c = 0 ⇐⇒ x = y = z = 0.

Bài 3: Cho a, b, c dương. CMR:√(a2b+ b2c+ c2a) (ab2 + bc2 + ca2) ≥ abc+ 3

√(a3 + abc) (b3 + abc) (c3 + abc) (1)

Lời giải

Chia cả 2 vế của (1) cho abc, BĐT

⇐⇒

√(a

c+b

a+c

b

)(b

c+c

a+a

b

)≥ 1 + 3

√(a2

bc+ 1

)(b2

ca+ 1

)(c2

ab+ 1

)

Đặt x =a

b, y =

b

c, z =

c

a. BĐT

⇐⇒

√(x+ y + z)

(1

x+

1

y+

1

z

)≥ 1 + 3

√(xz

+ 1)(y

x+ 1)(z

y+ 1

)80

Page 82: Chuyên đề luyện thi đại học môn Toán - VipLam.Net

⇐⇒√

3 +x

y+y

z+z

x+y

x+z

y+x

z≥ 1 + 3

√2 +

x

y+y

z+z

x+y

x+z

y+x

z

Đặt t =x

y+y

z+z

x+y

x+z

y+x

z=⇒ t ≥ 6. Đặt f (t) =

√t+ 3− 3

√t+ 2− 1 (t ≥ 6)

Ta có

f ′ (t) =1

2√t+ 3

− 1

3 3

√(t+ 2)2

, f ′ (t) = 0 =⇒ 729t4 + 5768t3 + 16920t2 + 21600t+ 9936 = 0

Vì t ≥ 6 =⇒ f ′ (t) > 0 =⇒ f (t) đồng biến trên [6; +∞).

Lập bảng biến thiên ta có Min f (t) = 0 tại t = 6 =⇒ đpcm.

BĐT ban đầu được chứng minh. Dấu “=” xảy ra khi x = y = z ⇐⇒ a = b = c.

Bài 4: Cho a, b, c là 3 số thực không đồng thời bằng 0, thỏa mãn:

(a+ b+ c)2 = 2 (a2 + b2 + c2). Tìm GTLN, GTNN của biểu thức:

P =a3 + b3 + c3

(a+ b+ c) (ab+ bc+ ca)

Lời giải

Từ giả thiết:

ab+ bc+ ca =1

2

[(a+ b+ c)2 − a2 − b2 − c2

]Suy ra:

ab+ bc+ ca =1

4(a+ b+ c)2

Do đó:

P =4 (a3 + b3 + c3)

(a+ b+ c)3 =1

16

[(4a

a+ b+ c

)3

+

(4b

a+ b+ c

)3

+

(4c

a+ b+ c

)3]

Đặt

x =

4a

a+ b+ c

y =4b

a+ b+ c

z =4c

a+ b+ c

=⇒

{y + z = 4− x

yz = x2 − 4x+ 4

Vì (y + z)2 ≥ 4yz =⇒ 0 ≤ x ≤ 8

3Ta có

P =1

16

(x3 + y3 + z3

)=

1

16

[x3 + (y + z)3 − 3yz (y + z)

]=⇒ P =

1

16

(3x3 − 12x2 + 12x+ 16

)Xét hàm số f (x) = 3x3 − 12x2 + 12x+ 16 với x ∈

[0;

8

3

]=⇒ f ′ (x) = 9x2 − 24x+ 12 =⇒ f ′ (x) = 0 ⇐⇒

x = 2

x =2

3Ta tính được

f (0) = f (2) = 16, f

(2

3

)= f

(8

3

)=

176

9

81

Page 83: Chuyên đề luyện thi đại học môn Toán - VipLam.Net

Suy ra Max[0; 8

3 ]f (x) =

176

9, Min

[0; 83 ]f (x) = 16 =⇒

MaxP =11

9⇐⇒ a = b =

c

4, a 6= 0

MinP = 1 ⇐⇒ a = 0, b = c 6= 0

Bài 5: Cho x, y, z là các số thực thỏa mãn: 1 ≤ x, y, z ≤ 4. Tìm GTNN của biểu thức:

P =x

2x+ 3y+

y

y + z+

z

z + x

Lời giải

Đặt y = ax, z = bx =⇒ a, b ∈[

14; 1]. Biểu thức P được viết lại như sau:

P =1

2 + 3a+

a

a+ b+

b

b+ 1=⇒ P ′ (b) = − a

(a+ b)2 +1

(b+ 1)2 ;

P ′ (b) = 0 =⇒ (a+ b)2 − a(b+ 1)2 = 0 ⇐⇒ (1− a)(b2 − a

)= 0 =⇒ b =

√a

Vì a, b ∈[

14; 1]

=⇒ MinP (b) = P (a) =1

2 + 3a− 2

1 +√a

+ 2.

Đặt t =√a =⇒ t ∈

[12; 1]. Xét hàm số F (t) =

1

2 + 3t2− 2

1 + t+ 2.

Ta có: F ′ (t) = − 6t

(2 + 3t2)2 +2

(1 + t)2 ;

F ′ (t) = 0 ⇐⇒ 2(2 + 3t2

)2 − 6t(1 + t)2 = 0 ⇐⇒ 18t4 − 6t3 + 12t2 − 6t+ 8 = 0

Vì t ∈[

12; 1]

=⇒ 18t4 − 6t3 + 12t2 − 6t+ 8 > 0 =⇒ F ′ (t) > 0 ∀t ∈[

12; 1]

=⇒ F (t) ≥ F

(1

2

)=

34

33

Vậy MinF (t) = F

(1

2

)=

34

33=⇒ MinP =

34

33tại a =

1

4, b =

1

2=⇒ x = 4, y = 1, z = 2.

Bài 6: Cho a, b, c ≥ 0 thỏa mãn: a+ b+ c = 3. Tìm GTLN, GTNN của biểu thức

P = a2 + b2 + c2 + abc

Lời giải

Từ giả thiết ta có: 0 ≤ ab ≤ (3− c)2

4và c ∈ [0; 3].

Ta có :

P = (3− c)2 + c2 − ab (2− c)

Với 0 ≤ ab ≤ (3− c)2

4=⇒ 1

4(c3 − 3c+ 18) ≤ P ≤ 2c2 − 6c+ 9.

Đặt f (c) =1

4(c3 − 3c+ 18) , g (c) = 2c2 − 6c+ 9. Khảo sát hàm số f (c) , g (c) , c ∈ [0; 3].

Ta có Min f (c) = 4 tại c = 1,Max g (c) = 9 tại c = 0 ∨ c = 3.

Vậy MinP = 4 tại a = b = c = 1. MaxP = 9 tại (0; 0; 3) và các hoán vị.

Bài 7: Cho x, y, z ∈ [1; 3] : x+ y + 2z = 6. Tìm GTLN, GTNN của biểu thức :

P = x3 + y3 + 5z3

82

Page 84: Chuyên đề luyện thi đại học môn Toán - VipLam.Net

Lời giải

Từ giả thiết ta có : 4xy = 4(3− z)2 − (x− y)2 ≤ 4(3− z)2 =⇒ xy ≤ (3− z)2

Ta lại có :

(x− 1) (y − 1) ≥ 0 =⇒ xy ≥ x+ y − 1 = 5− 2z =⇒ 5− 2z ≤ xy ≤ (3− z)2

Mặt khác: 2z = 6− x− y ≤ 4 =⇒ z ∈ [1; 2] . Ta có :

P = (x+ y)[(x+ y)2 − 3xy

]+ 5z3 = 2 (3− z)

[4(3− z)2 − 3xy

]+ 5z3.

Vì 5− 2z ≤ xy ≤ (3− z)2

=⇒ 2(3− z)3 + 5z3 ≤ P ≤ 2 (3− z)[4(3− z)2 + 6z − 15

]+ 5z3, (z ∈ [1; 2])

⇐⇒ 2(3− z)3 + 5z3 ≤ P ≤ −3z3 + 60z2 − 150z + 126, (z ∈ [1; 2])

Đặt f (z) = 2(3− z)3 + 5z3, g (z) = −3z3 + 60z2 − 150z + 126.

Xét hàm số f (z) , g (z) trên miền z ∈ [1; 2]

Đạo hàm và lập bảng biến thiên hàm số f (z) , g (z) ta có :

Min f (z) = 210− 60√

10 tại z = −2 +√

10 =⇒ x = y = 5−√

10.

Max g (z) = 42 tại z = 2 =⇒ x = y = 1.

Vậy MinP = 210− 60√

10 khi x = y = 5−√

10, z = −2 +√

10.

MaxP = 42 khi x = y = 1, z = 2.

Bài 8: Cho ba số thực dương a, b, c thỏa :√a− c+

√b− c =

√ab

c. Tìm GTNN của biểu thức

P =a

b+ c+

b

c+ a+

c

a+ b+

c2

a2 + b2

Lời giải

Đặt a = xc, b = yc (x, y ≥ 1)

Thay x = 1 vào giả thiết ta có :√b− c =

√b =⇒ c = 0 không thỏa vì c > 0.

Thay y = 1 vào giả thiết ta có :√a− c =

√a =⇒ c = 0 không thỏa vì c > 0.

Xét x, y > 1.Thay vào giả thiết ta có :

√x− 1 +

√y − 1 =

√xy ⇐⇒ x+ y − 2 + 2

√(x− 1) (y − 1) = xy

⇐⇒ xy − x− y + 1− 2√

(x− 1) (y − 1) + 1 = 0 ⇐⇒(√

(x− 1) (y − 1)− 1)2

= 0

=⇒√

(x− 1) (y − 1) = 1 ⇐⇒ xy = x+ y ≥ 2√xy =⇒ xy ≥ 4.

Biểu thức P được viết lại như sau :

P =x

y + 1+

y

x+ 1+

1

x+ y+

1

x2 + y2=

x2

xy + x+

y2

xy + y+

1

x+ y+

1

(x+ y)2 − 2xy

83

Page 85: Chuyên đề luyện thi đại học môn Toán - VipLam.Net

P ≥ (x+ y)2

2xy + x+ y+

1

x+ y+

1

(x+ y)2 − 2xy=xy

3+

1

xy+

1

x2y2 − 2xy=x3y3 − 2x2y2 + 3xy − 3

3 (x2y2 − 2xy)

Đặt t = xy, t ≥ 4 Xét hàm số f (t) =t3 − 2t2 + 3t− 3

t2 − 2t; (t ≥ 4)

Ta có : f ′ (t) =t4 − 4t3 + t2 + 6t− 6

(t2 − 2t)2 =t3 (t− 4) + t2 + 6 (t− 4) + 18

(t2 − 2t)2 > 0 ∀t ≥ 4.

Lập bảng biến thiên ta có : Min f (t) = f (4) =41

8

Vậy MinP =41

24khi x = y = 2 hay a = b = 2c.

Bài 9: Tìm GTNN của biểu thức :

A =2

|a− b|+

2

|b− c|+

2

|c− a|+

5√ab+ bc+ ca

Trong đó a, b, c là các số thực thỏa mãn :a+ b+ c = 1; ab+ bc+ ca > 0

Lời giải

Không mất tính tổng quát ta giả sử a > b > c.

Biểu thức A được viết lại như sau :

A =2

a− b+

2

b− c+

2

a− c+

5√ab+ bc+ ca

Đặt

x = a− by = b− cz = a− c

=⇒

{x, y, z > 0

x+ y = z. Ta có :

x2 + y2 + z2 = (a− b)2 + (b− c)2 + (c− a)2 = 2(a2 + b2 + c2

)− 2 (ab+ bc+ ca)

Mặt khác

a2 + b2 + c2 = 1− 2 (ab+ bc+ ca) =⇒ ab+ bc+ ca =2− (x2 + y2 + z2)

6

=⇒ A =2

x+

2

y+

2

z+

5√

6√2− (x2 + y2 + z2)

Áp dụng BĐT Cauchy- Schwarz ta có :

1

x+

1

y≥ 4

x+ y=

4

z

x2 + y2 ≥ (x+ y)2

2=z2

2

=⇒ A ≥ 10

z+

10√

3√4− 3z2

= 10

(1

z+

√3√

4− 3z2

),

(0 < z <

2√3

)

84

Page 86: Chuyên đề luyện thi đại học môn Toán - VipLam.Net

Đặt F (z) =1

z+

√3√

4− 3z2,

(0 < z <

2√3

)Ta có : F ′ (z) = − 1

z2+

3√

3z√(4− 3z2)3

F ′ (z) = 0 ⇐⇒ − 1

z2+

3√

3z√(4− 3z2)3

= 0 ⇐⇒ 3√

3z√(4− 3z2)3

=1

z2=⇒ 3

√3z3 =

√(4− 3z2)3

⇐⇒ 27z6 =(4− 3z2

)3 ⇐⇒ 3z2 = 4− 3z2 =⇒ z = ±√

6

3

Lập bảng biến thiên ta có :

MinF (z) = F

(√6

3

)=√

6 =⇒ MinA = 10√

6

⇐⇒

x = y

x+ y = z =

√6

3

=⇒

x = y =

√6

6

z =

√6

3

=⇒

a =

1

3+

√6

6

b =1

3

c =1

3−√

6

6

Vậy MinA = 10√

6 tại (a; b; c) =

(1

3+

√6

6;1

3;1

3−√

6

6

)và các hoán vị.

Bài 10: Cho a, b, c > 0. Tìm GTNN của biểu thức :

P =

√a

b+ c+

√b

c+ a+

√c

a+ b+

3√

2 (a+ b+ c)√ab+

√bc+

√ca

+9√

2 (ab+ bc+ ca)

4(a+ b+ c)2

Lời giải

P = 2√

2

[a

2√

2a (b+ c)+

b

2√

2b (c+ a)+

c

2√

2c (a+ b)

]+

3√

2 (a+ b+ c)√ab+

√bc+

√ca

+9√

2 (ab+ bc+ ca)

4(a+ b+ c)2

≥√

2

a

a+b+ c

2

+b

b+c+ a

2

+c

c+a+ b

2

+3√

2 (a+ b+ c)√ab+

√bc+

√ca

+3√

2

4

(√ab+

√bc+

√ca

a+ b+ c

)2

Áp dụng BĐT Cauchy- Schwarz ta có :

P ≥√

2

(√

a+√b+√c)2

2 (a+ b+ c)+

3 (a+ b+ c)√ab+

√bc+

√ca

+3

4

(√ab+

√bc+

√ca

a+ b+ c

)2

≥√

2

3(√

ab+√bc+

√ca)

2 (a+ b+ c)+

3 (a+ b+ c)√ab+

√bc+

√ca

+3

4

(√ab+

√bc+

√ca

a+ b+ c

)2

85

Page 87: Chuyên đề luyện thi đại học môn Toán - VipLam.Net

Đặt t =

√ab+

√bc+

√ca

a+ b+ c=⇒ t ∈ (0; 1]. Xét hàm số F (t) =

3t

2+

3

t+

3t2

4(t ∈ (0; 1]) Ta có

F ′ (t) =3

2− 3

t2+

3t

2=

3t3 + 3t2 − 6

2t2< 0

F ′ (t) = 0 =⇒ 3t3 + 3t2 − 6 = 0 ⇐⇒ (t− 1)(3t2 + 6t+ 6

)= 0 =⇒ t = 1

Lập bảng biến thiên ta có : MinF (t) = F (1) =21

4.

Vậy MinP =21√

2

4khi a = b = c.

Bài 11: Cho x, y > 0. Tìm GTNN của biểu thức

Q =√

2 (x2 + y2) +4

4√

4xy − 4√xy + 2 + 8

√xy − 1

Lời giải

Áp dụng BĐT Cauchy ta có :

Q ≥√

4xy +4

4√

4xy − 2√

4xy + 2 + 4√

4xy − 1

Đặt a =√

4xy > 0

=⇒ Q ≥ a+4

4

√(a− 1)2 + 1 + 4a− 1

Ta lại có :[(a− 1)2 + 1

]+ 1 + 1 + 1 ≥ 4

4

√(a− 1)2 + 1 =⇒ Q ≥ a+

16

(a− 1)2 + 16a

Xét hàm số F (a) = a+16

(a− 1)2 + 16a(a > 0).

Ta có :

F ′ (a) =(a− 1) (a3 + 29a2 + 227a+ 223)

(a2 + 14a+ 1)2 = 0 =⇒ a = 1

Lập bảng biến thiên ta có: MinF (a) = F (1) = 2.

Vậy MinQ = 2 khi x = y =1

2.

Bài 12: Cho a, b, c > 0. Chứng minh rằng

a

b+b

c+c

a+

9abc

(a+ b+ c) (ab+ bc+ ca)≥ 4

Lời giải

V T =a

b+b

c+c

a+

9

a

b+b

c+c

a+a

c+c

b+b

a+ 3

86

Page 88: Chuyên đề luyện thi đại học môn Toán - VipLam.Net

Đặt x =a

b, y =

b

c, z =

c

a=⇒

{x, y, z > 0

xyz = 1

V T = x+ y + z +9

x+ y + z + xy + yz + zx+ 3≥ x+ y + z +

27

(x+ y + z)2 + 3 (x+ y + z) + 9

Đặt t = x+ y + z =⇒ t ≥ 3. Xét hàm số F (t) = t+27

t2 + 3t+ 9, t ≥ 3

F ′ (t) =t4 + 6t3 + 27t2

(t2 + 3t+ 9)2 > 0∀t ≥ 3

Lập bảng biến thiên ta có : MinF (t) = F (3) = 4 =⇒ V T ≥ 4 =⇒ đpcm.

Dấu "=" xảy ra khi a = b = c.

Bài 13: Cho các số thực a, b, c ∈ (0; 1) : abc = (1− a) (1− b) (1− c). Chứng minh rằng :

a2 + b2 + c2 ≥ 3

4

Lời giải

Ta có:

abc = 1− (a+ b+ c) + ab+ bc+ ca− abc⇐⇒ 1− (a+ b+ c) + ab+ bc+ ca = 2abc

⇐⇒ 1− (a+ b+ c) +(a+ b+ c)2 − (a2 + b2 + c2)

2= 2abc

⇐⇒ a2 + b2 + c2 = (a+ b+ c− 1)2 + 1− 4abc ≥ (a+ b+ c− 1)2 + 1− 4

27(a+ b+ c)3

Đặt t = a+ b+ c =⇒ t ∈ (0; 3). Xét hàm số F (t) = − 4

27t3 + t2 − 2t+ 2

Ta có: F ′ (t) = −4

9t2 + 2t− 2 = 0 =⇒

t =3

2t = 3

.

Lập bảng biến thiên ta có: MinF (t) = F

(3

2

)=

3

4

Vậy a2 + b2 + c2 ≥ 3

4=⇒ đpcm.

Dấu " = " xảy ra khi a = b = c =1

2.

Bài 14: Cho a, b, c dương thỏa mãn : a ≥ b ≥ c. Tìm giá trị nhỏ nhất của biểu thức:

T =(a2 + c2)

√ab+ bc+ ca

ac (a+ b+ c)

Lời giải

Đặt a = xb, c = yb =⇒ 0 < y ≤ 1 ≤ x.

87

Page 89: Chuyên đề luyện thi đại học môn Toán - VipLam.Net

Biểu thức T được viết lại như sau:

T =(x2 + y2)

√x+ y + xy

xy (x+ y + 1)≥ (x+ y)2√x+ y + xy

2xy (x+ y + 1)

Đặt S = x+ y, P = xy Ta có:

(x− 1) (y − 1) ≤ 0 =⇒ xy ≤ x+ y − 1 =⇒

{S > 1

0 < P ≤ S − 1

=⇒ T ≥ S2√S + P

2P (S + 1)⇐⇒ T 2 ≥ S4 (S + P )

4P 2(S + 1)2

Xét hàm số F (P ) =S4 (S + P )

4P 2(S + 1)2 , (0 < P ≤ S − 1). Ta có:

F ′ (P ) = −S4 (2S + P )

4P 3(S + 1)2 < 0 ∀ 0 < P ≤ S − 1

=⇒ hàm số F (P ) nghịch biến trên (0;S − 1] =⇒ F (P ) ≥ F (S − 1) =S4 (2S − 1)

4(S2 − 1)2 .

Tiế tục ta đặt G (S) =S4 (2S − 1)

4(S2 − 1)2 , (S > 1).

Ta có : G′ (S) =(S − 2) (S2 + 2S − 1)S3

2(S2 − 1)3 . Với S > 1 =⇒ G′ (S) = 0 ⇐⇒ S = 2.

Lập bảng biến thiên ta có : MinG (S) = G (2) =4

3.

Vậy MinT =2√3⇐⇒ x = y = 1 ⇐⇒ a = b = c.

Bài 15: Cho a, b, c là các số thực không âm thỏa mãn : a+ b+ c = 1. Tìm giá trị lớn nhất và

giá trị nhỏ nhất của biểu thức :

P = 5a2

(a− 3

√5

10

)+ 3b2 + 6c

Lời giải

Ta có : P = 5a3 − 3√

5

2a2 − 6a+ 3b2 − 6b+ 6.

Ta khảo sát hàm số P (b) với b ∈ [0; 1− a] , a ∈ [0; 1].

Ta có : P ′ (b) = 6b − 6 ≤ 0 ∀b ∈ [0; 1− a] =⇒ hàm số P (b) nghịch biến trên [0; 1− a]

P (1− a) ≤ P (b) ≤ P (0).

? Tìm Max của biểu thức P :

Đặt F (a) = P (0) = 5a3 − 3√

5

2a2 − 6a+ 6, a ∈ [0; 1]

Ta có : F ′ (a) = 15a2 − 3√

5a2 − 6 = 0 =⇒

a =2√

5

5

a = −√

5

5

.

88

Page 90: Chuyên đề luyện thi đại học môn Toán - VipLam.Net

Lập bảng biến thiên ta được : MaxF (a) = F (0) = 6.

Vậy MaxP = 6 ⇐⇒ a = b = 0, c = 1.

? Tìm Min của biểu thức P :

Đặt G (a) = P (1− a) = 5a3 +

(3− 3

√5

2

)a2 − 6a+ 3, a ∈ [0; 1].

Ta có : G′ (a) = 15a2 +(6− 3

√5)a− 6 = 0 =⇒

a =−6 + 3

√5 +

√441− 36

√5

30

a =−6 + 3

√5−

√441− 36

√5

30

Lập bảng biến thiên ta có : MinG (a) = G

(−6 + 3

√5 +

√441− 36

√5

30

).

Vậy MinP = MinG

(−6 + 3

√5 +

√441− 36

√5

30

)

tại a =−6 + 3

√5 +

√441− 36

√5

30, b =

36− 3√

5−√

441− 36√

5

30, c = 0.

Bài 16: Cho x, y, z là các số thực dương thỏa mãn : x+ y + 1 = z. Tìm giá trị nhỏ nhất của

biểu thức :

P =x3

x+ yz+

y3

y + zx+

z3

z + xy+

14

(z + 1)√

(x+ 1) (y + 1)

Lời giải

Biểu thức P được viết lại như sau :

P =x4

x2 + xyz+

y4

y2 + xyz+

z3

(x+ 1) (y + 1)+

14

(z + 1)√

(x+ 1) (y + 1)

Áp dụng BĐT Cauchy ta có :

(x+ 1) (y + 1) ≤ (x+ y + 2)2

4=

(z + 1)2

4

=⇒ ≥ (x2 + y2)2

x2 + y2 + xyz+

4z3

(z + 1)2 +28

(z + 1)2 ≥x2 + y2

z + 1+

4z3 + 28

(z + 1)2 ≥(x+ y)2

2 (z + 1)+

4z3 + 28

(z + 1)2

=(z − 1)2

2 (z + 1)+

4z3 + 28

(z + 1)2 =9z3 − z2 − z + 57

2(z + 1)2 , (z > 1)

Xét hàm số P (z) =9z3 − z2 − z + 57

2(z + 1)2 , (z > 1).

Ta có : P ′ (z) =(3z − 5) (3z2 + 14z + 23)

2(z + 1)3 = 0 =⇒ z =5

3.

Lập bảng biến thiên ta có : MinP (z) = P

(5

3

)=

53

8.

Vậy MinP =53

8⇐⇒ x = y =

1

3, z =

5

3.

89

Page 91: Chuyên đề luyện thi đại học môn Toán - VipLam.Net

Bài 17: Cho các số thực dương x, y, z thỏa mãn :x2 + 8y2 + 9z2 ≤ 4xyz. Tìm giá trị nhỏ nhất

của biểu thức

P =4x+ 2y2 + z3√

6(36y − 11

√2z)− 11x

Lời giải

Từ giả thiết ta có :

(x− 3z)2+8y2+6zx ≤ 4xyz =⇒ 0 < 8y2 ≤ 2zx (2y − 3)−(x− 3z)2 ≤ 2zx (2y − 3) =⇒ y >3

2

Ta lại có :

4zx ≥ x2

y+ 4y +

9z2

y+ 4y ≥ 4x+ 12z ≥ 8

√3zx =⇒ zx ≥ 12

Ta có :

P =4x+ 2y2 + z3 + 8 + 8− 16√216y − 11

(3√

2z + 3√

2z + x) ≥ 4x+ 12z + 2y2 − 16√

216y − 33 3√

18zx≥ 2y2 + 8

√3zx− 16√

216y − 198≥ 2y2 + 32√

216y − 198

Xét hàm số F (y) =2y2 + 32√216y − 198

,

(y >

3

2

).

Ta có : F ′ (y) =8 (81y2 − 99y − 432)

(216y − 198)√

216y − 198= 0 =⇒ y = 3.

Lập bảng biến thiên ta có : MinF (y) = F (3) =5√

2

3.

Vậy MinP =5√

2

3khi x = 6, y = 3, z = 2.

Bài 18: Cho a, b, c là các số thực không âm có tổng bằng 1. Chứng minh rằng :√a+ (b− c)2 +

√b+ (c− a)2 +

√c+ (a− b)2 ≥

√3

Lời giải

Do a, b, c có vai trò như nhau nên không mất tính tổng quát ta giả sử a ≥ b ≥ c =⇒ a ∈[1

3; 1

], c ∈

[0;

1

3

]. Ta có:

√a+ (b− c)2 +

√b+ (a− c)2 ≥

√2 (a+ b) + (a+ b− 2c)2 =

√9c2 − 8c+ 3.

=⇒ V T ≥√

9c2 − 8c+ 3 +√c

Xét hàm số F (c) =√

9c2 − 8c+ 3 +√c

(0 ≤ c ≤ 1

3

).

Ta có :

F ′ (c) =(18c− 8)

√c+√

9c2 − 8c+ 3

2√c√

9c2 − 8c+ 3= 0 =⇒

√9c2 − 8c+ 3 =

√c (8− 18c)

(c ≤ 4

9

)(1)

90

Page 92: Chuyên đề luyện thi đại học môn Toán - VipLam.Net

Giải phương trình (1) và so sánh điều kiện ta được c =1

3, c =

7−√

33

24.

Lập bảng biến thiên ta có : MinF (c) = F (0) = F

(1

3

)=√

3 =⇒ V T ≥√

3 =⇒ đpcm.

Dấu " = " xảy ra khi (a, b, c) là

(1

2;1

2; 0

),

(1

3;1

3;1

3

)và các hoán vị.

Bài 19: Cho a, b, c ≥ 0 thỏa mãn : a2 + b2 + c2 = 1. Tìm giá trị lớn nhất, giá trị nhỏ nhất của

biểu thức :

P = (a− b) (b− c) (c− a) (a+ b+ c)

Lời giải

Để biểu thức P ≥ 0 ta sẽ có các trường hợp sau : a ≥ c ≥ b; b ≥ a ≥ c; c ≥ b ≥ a. Ta chỉ cần

xét một trường hợp sẽ đúng cho hai trường hợp còn lại.

Ta xét trường hợp : a ≥ c ≥ b.

Từ giả thiết ta có :

a2 = 1− b2 − c2 ≥ 1− 2a2 =⇒ 1√3≤ a ≤ 1

Tương tự ta có :

b2 = 1− a2 − c2 ≤ 1− 2b2 =⇒ 0 ≤ b ≤ 1√3

c2 ≤ a2 = 1− b2 − c2 ≤ 1− c2 =⇒ c2 ≤ 1− c2 =⇒ c2 ≤ 1

2=⇒ 0 ≤ c ≤

√2

2

a2 + c2 = 1− b2 ≤ 1 =⇒ a2 ≤ 1− c2 =⇒ a ≤√

1− c2

Biểu thức P được viết lại như sau :

P = (a− c)[2b3 − (1 + ca) b+ a2c+ ac2

]Ta xét hàm số P (b) = (a− c) [2b3 − (1 + ca) b+ a2c+ ac2] với b ∈

[0;

1√3

]

Ta có : P ′ (b) = (a− c) (6b2 − 1− ca) = 0 =⇒

b = −√

1 + ca

6

b =

√1 + ca

6

Vì a ∈[

1√3

; 1

], c ∈

[0;

√2

2

]=⇒ b =

√1 + ca

6∈[0;

1√3

]Ta lại có :

P (0) = ac(a2 − c2

), P

(1√3

)= a2c+ ac2 − ca√

3− 1

3√

3=⇒ P (0) > P

(1√3

)Lập bảng biến thiên ta có : MaxP = P (0) = ac (a2 − c2) ≤ c

√1− c2 (1− 2c2)

Đặt F (c) = c√

1− c2 (1− 2c2) , c ∈

[0;

√2

2

].

91

Page 93: Chuyên đề luyện thi đại học môn Toán - VipLam.Net

Ta có :

F ′ (c) =8c4 − 8c2 + 1√

1− c2= 0 =⇒ 8c4 − 8c2 + 1 = 0 =⇒

c = −√

2 +√

2

2

c = −√

2−√

2

2

c =

√2−√

2

2

c =

√2 +√

2

2

Lập bảng biến thiên ta có : MaxF = F

(√2−√

2

2

)=

1

4.

Vậy ta có : MaxP =1

4tại a =

√2 +√

2

2, c =

√2−√

2

2, b = 0.

Tìm GTNN bạn đọc tự chứng minh bằng phương pháp trên cho ba trường hợp còn lại.

Bài 20: Cho các số thực thay đổi x, y, z thỏa mãn : x2 + y2 + z2 +16

25xy = 3. Tìm giá trị nhỏ

nhất của biểu thức :

P =3

5

(x2 + y2

)+

5

6z2 + xy −

√10 (xy + yz + zx)

Lời giải

Áp dụng BĐT Cauchy-Schwazr ta có : x2 + y2 ≥ (x+ y)2

2

=⇒ P ≥ 3

10(x+ y)2 +

5

6z2 + xy −

√10 (xy + yz + zx)

≥ |(x+ y) z|+ xy −√

10 (xy + yz + zx)

≥ xy + yz + zx−√

10 (xy + yz + zx)

Đặt t =√xy + yz + zx , t ≥ 0 Xét hàm số F (t) = t2 − t

√10 , t ≥ 0

Ta có : F ′ (t) = 2t−√

10 = 0 =⇒ t =

√10

2

Lập bảng biến thiên ta có : MinF = F

(√10

2

)= −5

2.

Vậy MinP = −5

2khi x = y =

5√34, z =

6√34

hoặc x = y = − 5√34, z = − 6√

34.

Bài 21: Cho a, b, c > 0. Chứng minh rằng :√a

a+ b+

√b

b+ c+

√c

c+ a≤ 3√

2(1)

Lời giải

Đặt x =

√b

a, y =

√c

b, z =

√a

c=⇒

{x, y, z > 0

xyz = 1=⇒ V T (1) =

√1

1 + x2+

√1

1 + y2+

92

Page 94: Chuyên đề luyện thi đại học môn Toán - VipLam.Net

√1

1 + z2

Giả sử xy ≤ 1 =⇒ z ≥ 1 Vì xy ≤ 1

=⇒

(√1

1 + x2+

√1

1 + y2

)2

≤ 2

(1

1 + x2+

1

1 + y2

)≤ 4

1 + xy=

4z

1 + z

=⇒√

1

1 + x2+

√1

1 + y2≤ 2

√z

1 + z

Mặt khác ta lại có: √1

1 + z2≤√

2

1 + z

Như vậy ta có :

V T (1) ≤ 2

√z

1 + z+

√2

1 + z

Xét hàm số F (z) = 2

√z

1 + z+

√2

1 + z, z ≥ 1

Ta có : F ′ (z) =

√1 + z −

√2z

√z(1 + z)2 = 0 =⇒

√1 + z −

√2z = 0 =⇒ z = 1.

Lập bảng biến thiên ta có : MaxF = F (1) =3√2

=⇒ V T (1) ≤ 3√2

=⇒ đpcm.

Dấu " = " xảy ra khi x = y = z = 1 ⇐⇒ a = b = c.

Bài 22: Cho a, b, c dương thỏa mãn : a2 + b2 + c2 = 3. Tìm giá trị nhỏ nhất của biểu thức:

P = 5 (a+ b+ c) +3

abc

Lời giải

Đặt x = a+ b+ c, y = ab+ bc+ ca =⇒

{x >√

3, y > 0

x2 = 2y + 3

Mặt khác ta có:

(ab+ bc+ ca)2 ≥ 3abc (a+ b+ c) =⇒ 1

abc≥ 3x

y2

=⇒ P ≥ 5x+9x

y2= 5x+

36x

(x2 − 3)2

Xét hàm số F (x) = 5x+36x

(x2 − 3)2 với x >√

3

Ta có: F ′ (x) =(x− 3) (5x5 + 15x4 + 27x+ 81)

(x2 − 3)3

Vì x >√

3 =⇒ 5x5 + 15x4 + 27x+ 81 > 0 =⇒ F ′ (x) = 0 ⇐⇒ x = 3.

Lập bảng biến thiên ta có: MinF = F (3) = 18.

Vậy MinP = 18 khi

{x = 3

y = 3⇐⇒ a = b = c = 1.

93

Page 95: Chuyên đề luyện thi đại học môn Toán - VipLam.Net

Bài 23: Cho a, b là các số thực thuộc [0; 2]. Tìm giá trị nhỏ nhất của biểu thức:

P =1

(a− b)2 +1

(b− c)2 +1

(c− a)2

Lời giải

Không mất tính tổng quát ta giả sử 0 ≤ a < b < c ≤ 2. Từ 0 < c− a ≤ 2 =⇒ 1

(c− a)2 ≥1

4.

Tiếp tục ta có: 0 < c− b ≤ 2− b =⇒ 1

(b− c)2 ≥1

(2− b)2 và 0 < b− a ≤ b =⇒ 1

(a− b)2 ≥1

b2

Suy ra P ≥ 1

b2+

1

(2− b)2 +1

4,∀b ∈ (0; 2).

Xét hàm số F (b) =1

b2+

1

(2− b)2 +1

4,∀b ∈ (0; 2) Ta có : F ′ (b) = − 2

b3+

2

(2− b)3 = 0 =⇒ b = 1.

Lập bảng biến thiên ta có : MinF (b) = F (1) =9

4=⇒ MinP =

9

4Dấu " = " xảy ra tại (a; b; c) = (0; 1; 2) .

Bài 24: Cho a, b, c là độ dài ba cạnh của tam giác có chu vi bằng 3. Tìm giá trị nhỏ nhất của

biểu thức

Q = 3(a2 + b2 + c2

)+ 4abc

Lời giải

Không mất tính tổng quát ta giả sử 0 < a ≤ b ≤ c.

Ta có : a+ b+ c = 3 =⇒ a+ b = 3− c và c ≥ 1.

Mặt khác a+ b > c =⇒ 3− c > c =⇒ c <3

2=⇒ c ∈

[1; 3

2

).

Ta có : Q = 3 (a2 + b2) + 3c2 + 4abc = 3(3− c)2 + 3c2 − 2 (3− 2c) ab.

Vì c <3

2=⇒ 3− 2c > 0. Ta có :

(a+ b

2

)2

≥ ab =⇒(

3− c2

)2

≥ ab =⇒ Q ≥ c3 − 3

2c2 +

27

2

Xét hàm số F (c) = c3 − 3

2c2 +

27

2, c ∈

[1; 3

2

)Ta có : F ′ (c) = 3c2 − 3c = 0 =⇒

[c = 0

c = 1.

Lập bảng biến thiên ta có : MinF (c) = F (1) = 13.

Vậy MinQ = 13 tại (a; b; c) = (1; 1; 1).

Bài 25: Cho các số thực không âm a, b, c thỏa mãn : a + b + c = 3. Tìm giá trị lớn nhất của

biểu thức

P =(a2 − ab+ b2

) (b2 − bc+ c2

) (c2 − ca+ a2

)Lời giải

Không mất tính tổng quát ta giả sử 0 ≤ a ≤ b ≤ c ≤ 3

=⇒

{a (a− b) ≤ 0

a (a− c) ≤ 0=⇒

{a2 − ab+ b2 ≤ b2

a2 − ca+ c2 ≤ c2=⇒ P ≤ b2c2 (b2 − bc+ c2) = b2c2

[(b+ c)2 − 3bc

]94

Page 96: Chuyên đề luyện thi đại học môn Toán - VipLam.Net

Mặt khác ta có : b+ c ≤ a+ b+ c = 3 =⇒ 2√bc ≤ b+ c ≤ 3 =⇒ 0 ≤ bc ≤ 9

4Do đó P ≤ b2c2 (9− 3bc) = −3b3c3 + 9b2c2.

Xét hàm số F (x) = −3x3 + 9x2, x ∈[0;

9

4

]=⇒ F ′ (x) = −9x2 + 18x = 0 =⇒

[x = 0

x = 2

Lập bảng biến thiên ta có : MaxF (x) = F (2) = 12 =⇒ MaxP = 12 tại (a; b; c) = (0; 1; 2)

Bài 26: Cho a, b, c dương thỏa mãn : a2 + b2 + c2 = 3. Tìm giá trị nhỏ nhất của biểu thức:

P =1

2− a+

1

2− b+

1

2− cLời giải

Dự đoán dấu bằng xảy ra tại a = b = c = 1 và giá trị nhỏ nhất bằng 3.

Ta xét hàm số f (x) =1

2− x− 1− 1

2x2, x ∈ 0;

√3)

Ta có: f ′ (x) =1

(2− x)2 − x = 0 =⇒

x =

3−√

5

2∈ 0;

√3)

x = 1 ∈ 0;√

3)

x =3 +√

5

2/∈ 0;

√3) .

Lập bảng biến thiên ta có f (x) ≥ f (1) = −1

2=⇒ 1

2− x≥ 1

2+

1

2x2.

Thay x bởi a, b, c vào ta được : P =1

2− a+

1

2− b+

1

2− c≥ 3

2+

1

2(a2 + b2 + c2) = 3.

Vậy MinP = 3 tại a = b = c = 1.

Bài 27: Cho a, b, c là 3 số dương thỏa mãn: a+b−c ≥ 0, b+c−a ≥ 0, c+a−b ≥ 0, (a+ b+ c)2 =

4 (ab+ bc+ ca− 1). Tìm giá trị nhỏ nhất của biểu thức:

S =

√a+ b

c− 1 +

√b+ c

a− 1 +

√c+ a

b− 1 +

2√

2√a2 + b2 + c2 − 2

Lời giải

Đặt

x = a+ b− cy = b+ c− az = c+ a− b

=⇒

{x, y, z ≥ 0

xy + yz + zx = 4. Biểu thức S được viết lại như sau:

S =√

2

(√x

y + z+

√y

z + x+

√z

x+ y

)+

4√x2 + y2 + z2

Cách 1:

Ý tưởng giải quyết bài toán: Rõ ràng chúng ta cần đưa

√x

y + z+

√y

z + x+

√z

x+ yvề biểu

thức có chứa x2 + y2 + z2 dựa vào giả thiết

{x, y, z ≥ 0

xy + yz + zx = 4.

Đặt Q =

√x

y + z+

√y

z + x+

√z

x+ y

95

VVViiipppLLLaaammm...NNNeeettt

Page 97: Chuyên đề luyện thi đại học môn Toán - VipLam.Net

=⇒ Q2 =x

y + z+

y

z + x+

z

x+ y+2

(√xy

(y + z) (z + x)+

√yz

(z + x) (x+ y)+

√zx

(x+ y) (y + z)

)Ta có:

x

y + z+

y

z + x+

z

x+ y=

1

4

(x

y + z+

y

z + x+

z

x+ y

)(xy + yz + zx) =

=x2 + y2 + z2

4+xyz

4

(1

x+ y+

1

y + z+

1

z + x

)≥ x2 + y2 + z2

4

Tiếp tục, áp dụng BĐT Cauchy ta có:√xy

(y + z) (z + x)=

2xy

2√

(xy + xz) (yz + yx)≥ 2xy

2xy + yz + zx≥

≥ 2xy

2 (xy + yz + zx)=

xy

xy + yz + zx

Tương tự ta có:√yz

(z + x) (x+ y)≥ yz

xy + yz + zx;

√zx

(x+ y) (y + z)≥ zx

xy + yz + zx

=⇒√

xy

(y + z) (z + x)+

√yz

(z + x) (x+ y)+

√zx

(x+ y) (y + z)≥ 1

=⇒ Q ≥√x2 + y2 + z2

4+ 2 =⇒ P ≥

√x2 + y2 + z2

2+ 4 +

4√x2 + y2 + z2

Đặt t =√x2 + y2 + z2 =⇒ t ≥

√xy + yz + zx = 2 Xét hàm số F (t) =

√t2

2+ 4 +

4

t, t ≥ 2

Ta có:

F ′ (t) =t

2

√t2

2+ 4

− 4

t2= 0 =⇒ t6 − 32t2 − 256 = 0

⇐⇒(t− 2

√2)(

t5 + 2√

2t4 + 8t3 + 16√

2t2 + 32t+ 64√

2)

= 0 =⇒ t = 2√

2

Lập bảng biến thiên ta có:

MinF (t) = F(2√

2)

= 3√

2 =⇒ MinP = 3√

2 khi (x; y; z) = (0; 2; 2) và các hoán vị hay

(a; b; c) = (1; 1; 2) và các hoán vị.

Cách 2: S =√

2

(√x

y + z+

√y

z + x+

√z

x+ y

)+

4√x2 + y2 + z2

Giả sử x ≥ y ≥ z ≥ 0. Ta chứng minh

√y

z + x+

√z

x+ y≥√y + z

x(1)

Thật vậy ta có BĐT (1) ⇐⇒√xy (x+ y) +

√zx (z + x) ≥

√(x+ y) (y + z) (z + x)

⇐⇒ 2x√yz (x+ y) (x+ z) ≥ y2z + yz2 + 2xyz (2)

96

Page 98: Chuyên đề luyện thi đại học môn Toán - VipLam.Net

Vì x ≥ y ≥ z ≥ 0 nên với z = 0 thì dấu đẳng thức xảy ra.

Bây giờ ta xét x ≥ y ≥ z > 0. Chia cả hai vế của (2) cho xyz ta được:

2

√(x+ y

y

)(x+ z

z

)≥ y

x+z

x+ 2 (3)

Rõ ràng x ≥ y ≥ z > 0 nên ta có:

x+ y

y≥ 2;

x+ z

z≥ 2;

y

x≤ 1;

z

x≤ 1 =⇒ V T (3) ≥ 4 ≥ V P (3) =⇒ đpcm.

Dấu đẳng thức xảy ra khi

[z = 0

x = y = z > 0.

Trở lại bài toán ban đầu ta có :

S ≥√

2

(√x

y + z+

√y + z

x

)+

4√x2 + y2 + z2

=√

2

(√x

y + z+

√y + z

x

)+ 2

√xy + yz + zx

x2 + y2 + z2

=√

2

(√x

y + z+

√y + z

x

)+ 2

√x (y + z) + yz

x2 + (y + z)2 − 2yz

≥√

2

(√x

y + z+

√y + z

x

)+ 2

√x (y + z)

x2 + (y + z)2

=√

2

(√x

y + z+

√y + z

x

)+ 2

√√√√ 1x

y + z+y + z

x

Đặt t =√

2

(√x

y + z+

√y + z

x

)=⇒ t ≥ 2

√2 Xét hàm số F (t) = t+

2√

2√t2 − 4

, t ≥ 4

Ta có :

F ′ (t) = 1− 2√

2.t√(t2 − 4)3

= 0 =⇒√

(t2 − 4)3 − 2√

2.t = 0

⇐⇒(t2 − 8

) (t4 − 4t2 + 8

)= 0 =⇒

[t = −2

√2

t = 2√

2

Lập bảng biến thiên ta có : MinF (t) = F(2√

2)

= 3√

2.

Vậy MinS = 3√

2 khi

{z = 0

x = y = 2⇐⇒ (a; b; c) = (1; 2; 1) và các hoán vị.

Bài 28: Cho x, y, z là các số thực không âm thỏa mãn: x2 + y2 + z2 = 1. Tìm giá trị lớn nhất,

giá trị nhỏ nhất của biểu thức:

P = x+ y + z − xyz.Lời giải

97

Page 99: Chuyên đề luyện thi đại học môn Toán - VipLam.Net

Ta có: x2 + y2 + z2 = 1 =⇒ (y + z)2 = 1− x2 + 2yz ≥ 1− x2.

Tiếp tục ta có: (y + z)2 ≤ 2 (y2 + z2) = 2 (1− x2)

Từ đó ta suy ra:√

1− x2 ≤ y + z ≤√

2 (1− x2), x ∈ [0; 1].

Từ giả thiết ta có : x2 = 1− (y + z)2 + 2yz =⇒ −2xyz = −x3 + x− x(y + z)2

Suy ra : 2P = Q = −x3 + 3x− x(y + z)2 + 2 (y + z).

Xét hàm số Q(y+z) = −x(y + z)2 + 2 (y + z)− x3 + 3x với√

1− x2 ≤ y + z ≤√

2 (1− x2).

Ta có : Q′(y+z) = −2x (y + z) + 2 = (x− y)2 + (x− z)2 + y2 + z2 > 0

=⇒ Hàm số Q(y+z) đồng biến trên miền[√

1− x2;√

2 (1− x2)]

=⇒ Q(√

1− x2)≤ Q(y+z) ≤ Q

(√2 (1− x2)

).

Đặt F (x) = Q(√

1− x2)

= 2x+ 2√

1− x2, x ∈ [0; 1]

Ta có : F ′ (x) = 2− 2x√1− x2

= 0 =⇒

x = − 1√2

x =1√2

Lập bảng biến thiên ta có :

MinF (x) = F (0) = F (1) = 2 =⇒ MinP = 1 khi (x; y; z) = (0; 0; 1) và các hoán vị.

Đặt H (x) = Q(√

2 (1− x2))

= x3 + x+ 2√

2 (1− x2), x ∈ [0; 1].

Ta có : H ′ (x) = 3x2 + 1− 2√

2x√1− x2

= 0 =⇒ (3x2 + 1)√

1− x2 = 2√

2x

⇐⇒ (3x2 + 1)2

(1− x2) = 8x2 ⇐⇒ (x2 − 1)3

= −8x6 =⇒

x = − 1√3

x =1√3

Lập bảng biến thiên ta có :

MaxH (x) = H

(1√3

)=

16

3√

3=⇒ MaxP =

8

3√

3khi x = y = z =

1√3.

Vậy MinP = 1 khi (x; y; z) = (0; 0; 1) và các hoán vị.

MaxP =8

3√

3khi (x; y; z) =

(1√3

;1√3

;1√3

).

Bài 29: Cho a, b, c không âm thỏa mãn : a+ b+ c = 3. Tìm giá trị lớn nhất của biểu thức:

P =(a2 + a+ 1

) (b2 + b+ 1

) (c2 + c+ 1

)Lời giải

Lấy logarit cơ số e hai vế của biểu thức P ta được :

LnP = ln(a2 + a+ 1

)+ ln

(b2 + b+ 1

)+ ln

(c2 + c+ 1

)Xét hàm số F (t) = ln (t2 + t+ 1)− t, t ∈ [0; 3] Ta có :

F ′ (t) =2t+ 1

t2 + t+ 1− 1 = 0 =⇒ −t2 + t = 0 =⇒

[t = 0

t = 1

Lập bảng biến thiên ta có :

MaxF (t) = F (1) = −1+ln 3 =⇒ ln(t2 + t+ 1

)−t ≤ −1+ln 3 =⇒ ln

(t2 + t+ 1

)≤ t−1+ln 3

98

Page 100: Chuyên đề luyện thi đại học môn Toán - VipLam.Net

Thay biến t bởi a, b, c ta được:

ln (a2 + a+ 1) ≤ a− 1 + ln 3

ln (b2 + b+ 1) ≤ b− 1 + ln 3

ln (c2 + c+ 1) ≤ c− 1 + ln 3

=⇒ LnP = ln (a2 + a+ 1) + ln (b2 + b+ 1) + ln (c2 + c+ 1) ≤ (a+ b+ c)− 3 + 3 ln 3 = 3 ln 3.

=⇒ P ≤ e3 ln 3 = 27. Vậy MaxP = 27 ⇐⇒ a = b = c = 1.

Bài 30: Cho x, y, z là các số thực dương thỏa mãn : x2 + y2 + z2 = 1. Tìm giá trị nhỏ nhất

của biểu thức :

P = (x+ y + z)2 +1

2

(x3 + y3 + z3

xyz− 1

xy + yz + zx

)Lời giải

Từ giả thiết ta có : (x+ y + z)2 = 1 + 2 (xy + yz + zx)

Ta có :

x3 + y3 + z3 − 3xyz = (x+ y + z)[x2 + y2 + z2 − xy − yz − zx

]=

= 3xyz + (x+ y + z) [1− (xy + yz + zx)]

=⇒ x3 + y3 + z3

xyz= 3 +

(1

xy+

1

yz+

1

zx

)[1− (xy + yz + zx)] ≥

≥ 3 +9

xy + yz + zx[1− (xy + yz + zx)]

Đặt t = xy + yz + zx ≤ x2 + y2 + z2 = 1 =⇒ 0 < t ≤ 1.

=⇒ P ≥ 1 + 2t+1

2

[3 +

9

t(1− t)− 1

t

]= −2 + 2t+

4

t.

Xét hàm số F (t) = 2t− 2 +4

t, t ∈ (0; 1].

Ta có : F ′ (t) = 2− 4

t2= 0 =⇒ 2t2 − 4 = 0 =⇒ t = ±

√2.

Lập bảng biến thiên ta có : MinF (t) = F (1) = 4

Vậy MinP = 4 khi x = y = z =1√3.

BÀI TẬP TỰ LUYỆN

Bài 1: Cho x, y là số thực thỏa mãn : x2 + xy + y2 ≤ 3. Tìm giá trị lớn nhất và giá trị nhỏ

nhất của biểu thức :

M = x2 − xy − 3y2

Bài 2: Cho các số thực x, y thay đổi thỏa mãn: (x+ y)3 + 4xy ≥ 2. Tìm giá trị nhỏ nhất của

biểu thức:

P = 3(x4 + y4 + x2y2

)− 2

(x2 + y2

)+ 1

Bài 3: Cho a, b > 0 : 2 (a2 + b2) + ab = (a+ b) (ab+ 2). Tìm giá trị nhỏ nhất của biểu thức:

Q = 4

(a3

b3+b3

a3

)− 9

(a2

b2+b2

a2

)

99

Page 101: Chuyên đề luyện thi đại học môn Toán - VipLam.Net

Bài 4: Cho x, y là hai số thực không âm thay đổi. Tìm giá trị lớn nhất và giá trị nhỏ nhất của

biểu thức:

P =(x− y) (1− xy)

(1 + x)2(1 + y)2

Bài 5: Cho x, y là các số thực thay đổi. Tìm giá trị nhỏ nhất của biểu thức:

M =

√(x− 1)2 + y2 +

√(x+ 1)2 + y2 + |y − 2|

Bài 6: Cho a, b, c ≥ 0 : a+ b+ c = 2. Tìm giá trị nhỏ nhất của :

P =a

3 + b2 + c2+

b

3 + c2 + a2+

c

3 + a2 + b2

Bài 7: Cho a, b, c dương thỏa :

2 ≤ c ≤ 3b

2+

3

c≥ 2

a+b

2+

3

c≥ 3

. Tìm giá trị lớn nhất của biểu thức:

P = c2 − a2 − b2

Bài 8: Cho tam giác ABC không nhọn với a, b, c là ba cạnh của tam giác. Tìm giá trị nhỏ nhất

của biểu thức:

Q = a

(1

b+

1

c

)+ b

(1

c+

1

a

)+ c

(1

a+

1

b

)Bài 9: Cho x, y, z > 0 : xyz = 1. Chứng minh rằng:

1√1 + x2

+1√

1 + y2+

1√1 + z2

≤ 3√

2

2

Bài 10: Chứng minh rằng với mọi số thực dương x, y, z thỏa mãn: x (x+ y + z) = yz, ta có:

(x+ y)3 + (x+ z)3 + 3 (x+ y) (x+ z) (y + z) ≤ 5(y + z)3

Bài 11: Cho a, b, c ≥ 0 : a+ b+ c = 1. Tìm giá trị nhỏ nhất của biểu thức:

P = 3(a2b2 + b2c2 + c2a2

)+ 3 (ab+ bc+ ca) + 2

√a2 + b2 + c2

Bài 12: Cho x, y, z là ba số thực thuộc [1; 9] và x ≥ y, x ≥ z. Tìm giá trị lớn nhất và giá trị

nhỏ nhất của biểu thức :

P =x

x+ 2y+

y

y + z+

z

z + x

Bài 13: Cho các số thực x, y, z thỏa mãn : x+ y+ z = 0. Tìm giá trị nhỏ nhất của biểu thức :

M = |2x− y|+ |2y − z|+ |2z − x| − ln(√

14 (x2 + y2 + z2) + 1)

100

Page 102: Chuyên đề luyện thi đại học môn Toán - VipLam.Net

Bài 14: Cho các số thực dương x, y, z thỏa mãn: xy + yz + zx = 3. Tìm giá trị nhỏ nhất của

biểu thức :

P = 10|x−y| + 10|y−z| + 10|z−x| − 4

√x2y2 + y2z2 + z2x2

3− x2 + y2 + z2

3 (x+ y + z)

Bài 15: Cho x, y, z > 0 : x2 + 2x (y + z) = 5yz. Chứng minh :

(x+ y)3 + (x+ z)3 + (x+ y) (y + z) (z + x) ≤ 3(y + z)3

Bài 16: Cho x, y, z là các số thực không âm x, y, z thỏa mãn : x + y + z = 1. Tìm giá trị lớn

nhất của biểu thức:

Q = x2y + y2z + z2x

Bài 17: Cho a, b, c dương thỏa mãn: a+ b+ c = 1. Tìm giá trị nhỏ nhất của biểu thức:

P = a2 + b2 + c2 +ab+ bc+ ca

a2b+ b2c+ c2a

Bài 18: Cho a, b, c dương thỏa mãn: a+ b+ c = 1. Chứng minh rằng:

9

1− 2 (ab+ bc+ ca)+

2

abc≥ 81

Bài 19: Cho x, y, z là các số thực thỏa mãn: x2 + y2 + z2 = 12. Tìm giá trị lớn nhất và giá trị

nhỏ nhất của biểu thức:

P = (x+ 4) (y + 4) (z + 4)

Bài 20: Cho x, y, z dương thỏa mãn: x2 + y2 + z2 =1− 16xyz

4. Tìm giá trị nhỏ nhất của biểu

thức:

S =x+ y + z + 4xyz

1 + 4 (xy + yz + zx)

Bài 21: Cho a, b, c dương thỏa mãn: a2 + b2 + c2 = 1. Tìm giá trị nhỏ nhất của biểu thức:

P =a

b2 + c2+

b

c2 + a2+

c

a2 + b2

Bài 22: Cho a, b, c ≥ 1. Chứng minh rằng:

a4

a2 + 1+

b4

b2 + 1+

c4

c2 + 1≥ 1

Bài 23: Cho x, y, z là các số thực không âm thỏa: x2 + y2 + z2 = 1. Tìm GTLN, GTNN của

biểu thức:

P = x+ y + z + xy + yz + zx− xyz.

101

Page 103: Chuyên đề luyện thi đại học môn Toán - VipLam.Net

Bài 24: Cho x, y, z dương thỏa mãn: x2 + y2 + z2 = 3. Tìm giá trị lớn nhất của biểu thức:

P =

√xy

4−√xy+

√yz

4−√yz+

√zx

4−√zx

Bài 25: Cho a, b, c dương thỏa mãn: Min {a, b, c} ≥ 1

4Max {a, b, c}. Tìm giá trị lớn nhất và

giá trị nhỏ nhất của biểu thức:

P =a− bc

+b− ca

+c− ab

.

102

Page 104: Chuyên đề luyện thi đại học môn Toán - VipLam.Net

CHUYÊN ĐỀ 7

SỬ DỤNG CÁC BẤT ĐẲNG THỨC CƠ BẢN

GIẢI HỆ PHƯƠNG TRÌNH (Ngô Hoàng

Toàn)

LỜI NÓI ĐẦU

Trong thế giới toán học,các mảng kiến thức luôn có mối quan hệ hữu cơ với nhau. Nhà toán

học René Descartes đã đại số hoá hình học khi tạo ra thế giới hình giải tích . Có thể nói khi ta

quan tâm đến một vấn đề nào đó trong toán mà lại lãng quên đi các lĩnh vực khác thì thật là

điều đầy tiếc nối,muốn thành công phải biết chiêm nghiệm và học hỏi nhiều điều,giữa những

khoảng không gian bao la luôn tồn tại tình yêu đẹp.

Bất đẳng thức được xem như là đề tài hấp dẫn thu hút sự quan tâm của các toán thủ trên

các diễn đàn,sự huyền bí của hai cặp dấu ≥ ≤ luôn thách thức trí óc và độ tư duy của người

giải toán. Chẳng những thế,phân môn này luôn là câu đánh đố cao trong các đề thi học sinh

giỏi,Olympic và tuyển sinh đại học. Chợt nhớ đến đề thi đại học khối A năm 2012 vừa qua là

câu khống chế điểm của hầu hết thí sinh,như thế cho ta thấy mức độ khó của lớp bài toán này

luôn cao.

Nếu xem bất đẳng thức như "Ông hoàng" trong toán phổ thông thì hệ phương trình như

một cô gái chân quê hút hồn bao nhiêu gã si tình,những chàng thợ săn mãi mê tìm vẻ đẹp của

nàng,khẽ gõ cửa trái tim nàng để mong đến lúc nào đó tìm ra chân lí (x; y) ở đâu ? Ta lại chợt

nhận ra nếu câu bất đẳng thức trong đề đại học không có hay không quá khó thì nàng lại vững

bước kiêu sa làm cho bao sỉ tử đau đầu đi chinh phục. Đó có lẽ là tình yêu đẹp có chút nhẹ

nhàng nhưng lắm phong ba.

Sự xa cách của nơi đô thành tấp nập và chốn thôn quê bình dị ấy,có lúc nào lại gặp nhau

nơi dòng sông hò hẹn này,nơi tình cảm nồng ấm của những đôi trai gái yêu nhau.Tác giả xin

làm con đò nhỏ đưa lữ khách sang sông,nối nhịp đôi bờ lại với nhau qua chuyên đề

SỬ DỤNG CÁC BẤT ĐẲNG THỨC CƠ BẢN GIẢI HỆ PHƯƠNG TRÌNH

Cho bài biết tham gia vào "Tuyển tập các chuyên đề ôn thi đại học của diễn đàn

www.k2pi.net " để góp phần đưa các thí sinh đang có chút phân vân về cách học và ôn tập

103

Page 105: Chuyên đề luyện thi đại học môn Toán - VipLam.Net

toán như thế nào là hiệu quả để thẳng tiến vào cánh cổng đại học phía xa kia có một tài liệu

ôn tập bổ trợ cả hai mảng hệ phương trình và bất đẳng thức,đồng thời cũng muốn gởi đến đọc

giả yêu toán chút gia vị yêu thương dù nhỏ bé này.

Điều đặc biệt tác giả muốn gởi tặng chuyên đề này các thầy cô,các em học sinh của trường

chuyên Vị Thanh tỉnh Hậu Giang như món quà và lời cám ơn cho những ngày tháng mà tác giả

học ở mái trường này.Em luôn nhớ về những lúc bên các bạn lớp chuyên Toán khoá 2009-2012

đầy kỉ niệm,nhớ các thầy cô tổ Toán -Tin luôn vui tính và dạy dỗ tận tình.Những kỉ niệm đó

và tình yêu Toán từ các thầy cô sẽ truyền mãi ngọn lửa cho em bước tiếp vào đời

Do không phải theo nghiệp cầm phấn và kiến thức toán còn hạn hẹp nên chắc hẳn sai sót

là điều không thể tránh khỏi được,rất mong nhận được sự góp ý của quý bạn đọc qua địa chỉ

Ngô Hoàng Toàn Lớp YD1 khoá 38 Trường Đại học Y Dược Cần Thơ hoặc

email:[email protected].

Rất mong nhận được sự quan tâm của các bạn để lần viết chuyên đề sau được hoàn thiện hơn.

Thân mến!

§ 1. KIẾN THỨC CHUẨN BỊ

I. MỘT SỐ BẤT ĐẲNG THỨC THƯỜNG DÙNG

• Bất đẳng thức AM-GM:

Cho a1, a2, ..., an là các số thực không âm thì ta có:

a1 + a2 + ...+ an ≥ n n√a1a2...an

Đẳng thức xảy ra khi và chỉ khi a1 = a2 = ... = an .

Tuy nhiên,khi giải toán ta hay quan tâm nhiều đến trường hợp và .Mà ta thường được

biết đến dưới phát biểu:

1. Cho a, b ≥ 0 .Khi đó ta có:a+ b ≥ 2√ab .Đẳng thức xảy ra khi và chỉ khi: a = b.

Bất đẳng thức này còn được viết dưới dạng khác tương đương là:

(a)

(a+ b

2

)2

≥ ab

(b) (a+ b)2 ≥ 4ab

(c) a2 + b2 ≥ 2ab

(d) a2 + b2 ≥ (a+ b)2

2

2. Cho a, b, c ≥ 0 Khi đó ta có: a + b + c ≥ 3 3√abc .Đẳng thức xảy ra khi và chỉ khi

a = b = c.

Bất đẳng thức này còn có một số ứng dụng khác khá phổ biến như sau: Với mọi số

thực a, b, c ta luôn có:

(a) a2 + b2 + c2 ≥ ab+ bc+ ca

104

Page 106: Chuyên đề luyện thi đại học môn Toán - VipLam.Net

(b) a2 + b2 + c2 ≥ (a+ b+ c)2

3(c) (a+ b+ c)2 ≥ 3 (ab+ bc+ ca)

(d) a2b2 + b2c2 + c2a2 ≥ abc (a+ b+ c)

(e) (ab+ bc+ ca)2 ≥ 3abc (a+ b+ c)

• Bất đẳng thức Cauchy-Schwarz:

Với hai bộ số thực tùy ý a1, a2, ..., an vàb1, b2, ..., bn ta có :

(a21 + a2

2 + ...+ a2n)(b2

1 + b22 + ...+ b2

n) ≥ (a1b1 + a2b2 + ...+ anbn)2

Đẳng thức xảy ra khi và chỉ khia1

b1

=a2

b2

= ... =anbn

.

Bất đẳng thức Cauchy-Schwarz dạng Engel

Giả sử a1, a2, ..., an là các số thực bất kì và b1, b2, ..., bn là các số thực dương . Khi đó ta

luôn có :a1

2

b1

+a2

2

b2

+ ...+an

2

bn≥ (a1 + a2 + ...+ an)2

b1 + b2 + ...+ b

Đẳng thức xảy ra khi và chỉ khia1

b1

=a2

b2

= ... =anbn

Tuy nhiên,khi giải toán ta hay quan tâm nhiều đến trường hợp n = 2 và n = 3 .

Khi đó ta gặp một số đánh giá quen thuộc sau:

Cho a, b, c > 0 ta có:

1. a2 + b2 + c2 ≥ (a+ b+ c)2

3

2. (a+ b+ c)

(1

a+

1

b+

1

c

)≥ 9

• Bất đẳng thức Minkowski

Cho

{a1, a2, ..., an ∈ +

b1, b2, ..., bn ∈ +và 1 < p ∈ + .Khi đó

(n∑k=1

apk

)1

p+

(n∑k=1

bpk

)1

p ≥[

n∑k=1

(ak + bk)p

]1

p

Nhưng ta quan tâm nhiều nhất là các bất đẳng thức quen thuộc sau:

1.√a2 + b2 +

√c2 + d2 ≥

√(a+ c)2 + (b+ d)2

2.√a2 + b2 + c2 +

√m2 + n2 + p2 ≥

√(a+m)2 + (b+ n)2 + (c+ p)2

3.√a1

2 + b12+√a2

2 + b22+...+

√an2 + bn

2≥√

(a1 + a2 + ...+ an)2 + (b1 + b2 + ...+ bn)2

b)Giao điểm của PQ và RS của chúng nằm trên một trong 2 đường chéo của hình chữ nhật.

105

Page 107: Chuyên đề luyện thi đại học môn Toán - VipLam.Net

§ 2. CON ĐƯỜNG ĐI TỪ BÀI TOÁN ĐẾN SUY NGẪM

CỦA BẢN THÂN

I. BẤT ĐẲNG THỨC & HỆ PHƯƠNG TRÌNH 2 ẨN

Bài toán 1 Giải hệ phương trình:x4 + y4

(x+ y)4 =

√xy

x+ y− 3

81√x

+3√y

= 4(1)

Phân tích bài toán

Câu hỏi đặt ra lúc này là khi ta nhìn vào hệ này,tại sao ta lại nghĩ rằng đây là hệ giải bằng

phương pháp Bất đẳng thức.Thật ra, điều ta quan tâm đến giả thiết bài toán đó chính là

phương trình thứ nhất.Sự đối xứng hai biến x, y và có sự xuất hiện của đại lượngx4 + y4

(x+ y)4và

√xy

x+ y.Đây là điều quen thuộc trong các bước đánh giá bất đẳng thức.Đại lượng (x + y)4 yếu

hơn x4 + y4 nên ta nhận ra được V T ≥ a còn x + y thì mạnh hơn√xy nên ta nghĩ đến việc

đánh giá V P ≤ a để đưa về V T ≥ a ≥ V P từ đó đưa ra dấu đẳng thức.Việc còn lại là giải

phương trình thứ hai không quá khó.

Lời Giải

Điều kiện x, y > 0

Áp dụng bất đẳng thức Cauchy − Schwarz ta có:

x4 + y4 ≥ (x2 + y2)2

2≥ (x+ y)4

8

Do đó vế trái hệ (1) ≥ 1

8Áp dụng bất đẳng thức AM −GM cho vế phải hệ (1) ta có

√xy

x+ y− 3

8≤√xy

2√xy− 3

8=

1

8

Dấu "=" xảy ra khi và chỉ khi x = y

Thay vào hệ (2) ta có4√x

= 4 ⇐⇒ x = 1.

Vậy nghiệm của hệ là x = y = 1 .

Nhận xét

106

Page 108: Chuyên đề luyện thi đại học môn Toán - VipLam.Net

Với việc bắt đầu bước vào giải các lớp bài toán hệ bằng bất đẳng thức,đây có lẽ là ví dụ dễ tiếp

cận với các bạn làm quen phương pháp này.Ý tưởng trong sáng cho hệ trên và phương pháp

kết hợp chặn V P ≤ 1

8≤ V T là một trong những bước khởi đầu cho con đường chinh phục các

dạng toán như thế này.Với việc đánh giá như thế ta có thể đưa bài toán khó hơn chút nửa như

sau.

Giải hệ phương trình : 4x4 + y4 + 6x2y2 = x3√x2 − y2

x4y + y4√x+ 7

√x = 0

Bài tập tương tự

1.Giải hệ phương trình sau: 2x + 4y = 32

xy = 8

Đề thi học sinh giỏi Hà Tĩnh năm 2008-2009.

2.Giải hệ phương trình sau:2(x+ y)2 + 4xy − 3 = 0

(x+ y)4 − 2x2 − 4xy + 2y2 + x− 3y + 1 = 0

Bài toán 2 Tìm tất cả các cặp số (x; y) không âm thỏa mãn hệ:(2x+√

4x2 + 1)(√y2 + 1− y) = 1

1

1 + 3x+

1

1 + 2y+

1

1 + 5x=

3

1 + 4x

Nguồn gốc

Bài toán được đưa lên trang www.k2pi.net bởi anh Nguyễn Trung Kiên,bài toán chuẩn mực và

đầy lí thú ,những tư duy chặt chẽ trong đó cho ta nhiều điều suy ngẫm để hướng tới những bài

toán và cách sáng tạo hệ mới.Lời giải đến hiện nay cho bài toàn này là từ anh Con Phố Quen,

một lời giải đẹp mang đậm chất nghệ thuật.

Phân tích bài toán

Điều đầu tiên khi ta nhìn vào bài toán này chính là các đại lượng1

1 + 3x+

1

1 + 2y+

1

1 + 5xđó

là các đại lượng mũ và điều kì thú là 3x.4x.5x = 60x ≤ 64x với 3√

64x = 4x.Vậy chắc rằng tồn tại

bất đẳng thức có dạng1

1 + a+

1

1 + b+

1

1 + c≥ 3

1 + 3√abc

để hệ thứ hai là một bất đẳng thức

107

Page 109: Chuyên đề luyện thi đại học môn Toán - VipLam.Net

dưới dạng bổ đề toán.Chính điều này đã định hướng phần nào cho ta cách tiếp cận bài toán

dưới cách nhìn bất đẳng thức.

Lời giải

Trước tiên ta cần để ý rằng :√y2 + 1− y 6= 0;

(√y2 + 1− y

)(√y2 + 1 + y

)= 1

Tiếp đến là một bất thức quen thuộc được dùng trong bài toán này như một bổ đề :

1

1 + a+

1

1 + b+

1

1 + c≥ 3

1 + 3√abc

với a, b, c ≥ 1

Với đánh giá thứ nhất ta đưa phương trình thứ nhất trong hệ về phương trình :

2x+√

4x2 + 1 = y +√y2 + 1

Tới đây xét hàm số f(t) = t+√t2 + 1, ∀t ≥ 0. Ta có f ′(t) = 1 +

t√t2 + 1

> 0,∀t ≥ 0.

Từ đó ta có :f(2x) = f(y) ⇐⇒ y = 2x.

Với kết quả này cùng với đánh giá thử hai tức là bổ đề nêu ra ta có :

1

1 + 3x+

1

1 + 4x+

1

1 + 5x≥ 3

1 + 3√

60x≥ 3

1 + 3√

64x

Dấu đẳng thức xảy ra khi x = y = 0

Nhận xét

Lời giải trên giúp ta có lối tư duy đẹp cho việc tạo ra các bài toán hay,cái khó của bài toán

còn nằm ở chổ đánh giá 60x ≤ 64x với x ≥ 0.Từ bài toán trên ta cũng có thể tạo ra những bài

toán ấn tượng,ví dụ như bài toán sau.

Giải hệ phương trình sau:(2x+√

4x2 + 1)(√y2 + 1− y) = 1

(1 + 2x)(1 + 2y)(1 + 5x) = (1 + 4x)

Bài tập tương tự

1.Tìm nghiệm dương của hệ :3x

x+ 1+

4y

y + 1+

2z

z + 1= 1

89.x3y4z2 = 1.

2.Giải hệ phương trìnhx+ y + z = 1

x4 + y4 + z4 = xyz

108

Page 110: Chuyên đề luyện thi đại học môn Toán - VipLam.Net

Bài toán 3 Tìm tất cả các cặp số (x; y) dương thỏa mãn hệ:9

√41

2

(x2 +

1

2x+ y

)= 3 + 40x (1)

x2 + 5xy + 6y = 4y2 + 9x+ 9 (2)

Nguồn gốc

Bài toán này được bạn Hải với nick hoanghai1195 đưa lên diễn đàn www.K2pi.net trong thời

gian dài dù nhận được nhiều sự quan tâm nhưng chưa có lời giải nào.Sau đó bạn đã đưa lời giải

của mình lên,một lời giải đẹp và đầy tính đánh đố.

Phân tích bài toán

Đại lượng căn làm ta có cảm giác thấy khó xử lí,công việc ta cần làm là phá các căn thức

đó đi,để ý rằng 41.2 = 82 = 92 + 12 vậy theo bất đẳng thức Cauchy Schwarz ta có :

(92 + 12)

(x2 +

1

2x+ y

)≥ |9x+

1√(2x+ y)

|

Mà ta dự đoán được x = y = 3 là nghiệm của hệ nên để ý đến việc chọn điểm rơi để phá căn

thức còn lại như sau:

|9x+1√

2x+ y)| = |9x+

1.3√9(2x+ y)

| ≥ |9x+6

2x+ y + 9| ≥ 9x+

6

2x+ y + 9

Việc phá căn hoàn tất vấn đề còn lại là sử dụng giả thiết còn lại bài toán để giải quyết vấn đề trên.

Lời giải

9

√41

2

(x2 +

1

2x+ y

)= 3 + 40x ⇐⇒

√82

(x2 +

1

2x+ y

)=

6 + 80x

9(a)

Theo bất đẳng thức Cauchy Schwarz ta có :√82

(x2 +

1

2x+ y

)=

√(92 + 12)

(x2 +

1

(√

2x+ y)2

)≥ |9x+

1√(2x+ y)

|

Theo bất đẳng thức AM −GM ta lại có :

|9x+1√

(2x+ y)| ≥ 9x+

1.3√9(2x+ y)

≥ 9x+6

2x+ y + 9

109

Page 111: Chuyên đề luyện thi đại học môn Toán - VipLam.Net

Để phương trình (a) có nghiệm thì

6 + 80x

9≥ 18x2 + 9xy + 81x+ 6

2x+ y + 9⇐⇒ 3x− 2x2 − xy + 6y ≥ 0 (3)

Cộng phương trình (2) với phương trình (3) ta được:

−x2 + 4xy − 4y2 + 12y − 6x− 9 ≥ 0 ⇐⇒ −(x− 2y + 3)2 ≥ 0 ⇐⇒ x+ 3− 2y = 0

Vậy dấu bằng ở các bất đẳng thức trên xảy ra hay: x = y = 3.

Thử lại ta thấy thoả mãn hệ ban đầu.

Vậy nghiệm của hệ :

{x = 3

y = 3

Nhận xét

Xét về tính thực tế bài này rất khó đòi hỏi người giải phải thuần thục kỉ năng sử dụng cả hai

bất đẳng thức AM − GM và Cauchy Schwarz.Điều ta quan tâm là cách tác giả đi từ những

đánh giá cơ bản đi đến bài toán của mình,khi các bạn đọc lời giải trên có lẽ các bạn thấy được

rằng điểm mấu chốt giải quyết bài toán nằm ở các đánh giá thông qua việc chọn điểm rơi trong

bất đẳng thức AM −GM và Cauchy Schwarz.Xin được nói thêm cách chọn được điểm rơi như

thế.

• Thứ nhất

Việc có đánh giá

√(92 + 12)

(x2 +

1

(√

2x+ y)2

)≥ |9x+

1√(2x+ y)

| là do ta đoán được

hệ có nghiệm x = y = 3 nên ta quan tâm dấu đẳng thức xảy ra là9

x=

11√

2x+ y

. khi ta

thay x = y = 3 vào thì điều này đúng,vậy ta lí giải được phần đáng giá này.

• Thứ hai

Với đánh giá này 9x+1.3√

9(2x+ y)≥ 9x+

6

2x+ y + 9ta dựa vào điểm rơi của AM −GM

qua việc phá bỏ căn thức.Thấy biểu thức trong căn có giá trị là 9 muốn phá bỏ căn thức

này ta cần thêm vào số 3 dưới mẫu thì thêm 3 trên tử.Như thế khi áp dụng AM − GMdấu đẳng thức vẫn bảo toàn.

Bài tập tương tự

Giải hệ phương trình: x+ 6

√xy − y = 6

x+ 6x3 + y3

x2 + xy + y2−√

2(x2 + y2) = 3

Bài toán 4 Tìm tất cả các cặp số (x; y) dương thỏa mãn hệ:2x2(4x+ 1) + 2y2(2y + 1) = y + 32

x2 + y2 − x+ y =1

2

110

Page 112: Chuyên đề luyện thi đại học môn Toán - VipLam.Net

Phân tích bài toán

Ta thấy rõ việc đánh giá bất đẳng thức qua sự đối xứng các biến x, y.Nhưng điều quan trọng là

ta nên khai thác giả thiết này như thế nào ? Để ý rằng phương trình thứ hai của hệ được viết

thành (x− 1

2)2 + (y +

1

2)2 = 1 vậy nếu đặt a = x− 1

2; b = y +

1

2thì ta có ngay chặn của biến

a, b ∈ [−1; 1].Việc còn lại là biến đổi phương trình thứ nhất về các đại lượng đánh giá thích

hợp.

Lời giải

Từ phương trình 2 ta có: (x− 1

2)2 + (y +

1

2)2 = 1

Vậy nếu ta đặt x− 1

2= a; y +

1

2= b thì x = a+

1

2; y = b− 1

2và a, b ∈ [−1; 1]

Lúc này thay vào phương trình 1 ta có được:

8a3 + 14a2 + 8a+ 4b3 − 4b2 = 30

Hay

(4a2 + 11a+ 15)(a− 1) + 2b2(b− 1) = 0(1)

Vì a, b ∈ [−1; 1] nên ta có (4a2 + 11a+ 15)(a− 1) ≤ 0 và b2(b− 1) ≤ 0

Kết hợp với (1) ta suy ra

a = 1

b = 0hoặc

a = 1

b = 1

* Nếu

a = 1

b = 0thì

x =

3

2

y =−1

2

*Nếu

a = 1

b = 1thì

x =

3

2

y =1

2

Vậy nghiệm (x; y) của hệ là (3

2;−1

2), (

3

2;1

2) .

Nhận xét

Thật trùng hợp khi bài toán trên có một dạng tương tự khá khó nằm trong đề thi thử lần 1

của diễn đàn K2pi.net như sau:

Giải hệ phương trình : (x+ y) (25− 4xy) =105

4+ 4x2 + 17y2

4x2 + 4y2 + 4x− 4y = 7

Lời giải Đặt x =3a− 1

2; y =

3b+ 1

2.Lúc đó hệ trở thành:−6b3 + 9b2 = 6a3 + 14a− 20 (1)

a2 + b2 = 1

111

Page 113: Chuyên đề luyện thi đại học môn Toán - VipLam.Net

Ta có (1) ⇐⇒ 3b2(3− 2b) = (a− 1)(6a2 + 6a+ 20)

⇐⇒ 3(1− a2)(3− 2b) = (a− 1)(6a2 + 6a+ 20)

⇐⇒ (a− 1)(6a2 + 6a+ 20 + 9− 6b+ 9a− 6ab) = 0

+) Với a = 1 =⇒ b = 0 =⇒ x = 1; y =1

2+) Với 6a2 + 29 + 15a− 6b− 6ab = 0 (2) ta có: V T (2) ≥ 6a2 + 29− 15− 6− 3 = 6a2 + 5 > 0

nên trường hợp này phương trình vô nghiệm.

Vậy phương trình có nghiệm duy nhất (x; y) = (1;1

2)

Bài toán tương tự

Giải hệ phương trình : x4 + y4 = 2

x3 − 2x2 + 2x = y2

.

Bài toán 5 Giải hệ bất phương trình:x+ 2(y −√x− 1) ≤ 19

5+

1

y2 + 1√2x+ y − 2 +

√y − x+ 1 = 3

• Lời giải 1

Phân tích :

Nhận thấy hệ này có ba biểu thức chứa căn, ta suy nghĩ đến việc đặt ẩn phụ để bỏ căn.

Nhưng bài toán đặt ra là đặt ẩn phụ như thế nào?

Rõ ràng hai biểu thức√

2x+ y − 2;√y − x+ 1 có mối liên hệ với nhau nên ta chỉ cần đặt

ẩn phụ cho một trong hai biểu thức này và đặt ẩn còn lại là√x− 1.

Lời giải dưới đây lựa chọn√y − x+ 1 làm một ẩn, bạn hoàn toàn có thể đặt u =

√2x+ y − 2

Lời giải

Điều kiện:

x ≥ 1

y − x+ 1 ≥ 0

2x+ y − 2 ≥ 0

Đặt

{u =√y − x+ 1 ≥ 0

v =√x− 1 ≥ 0

=⇒

{x = v2 + 1

y = u2 + v2

Khi đó đưa về hệ bất phương trình: v2 + 1 + 2 (u2 + v2 − v) ≤ 19

5+

1

(u2 + v2)2 + 1

u+√

2 (v2 + 1) + u2 + v2 − 2 = 3

112

Page 114: Chuyên đề luyện thi đại học môn Toán - VipLam.Net

⇐⇒

(v − 1)2 + 2 (u2 + v2) ≤ 19

5+

1

(u2 + v2)2 + 1

u+√u2 + 3v2 = 3

Để ý bất phương trình đầu của hệ có chung nhân tử (u2 + v2) nên ta nghĩ đến việc loại

bỏ (v − 1)2 ≥ 0, từ bất phương trình này ta suy ra được:

2(u2 + v2

)≤ 19

5+

1

(u2 + v2)2 + 1

⇐⇒(u2 + v2 − 2

) (10(u2 + v2

)2+ u2 + v2 + 12

)≤ 0

⇐⇒ u2 + v2 ≤ 2

Mặt khác sử dụng bất đẳng thức AM −GM cho

3 = u+√u2 + 3v2 ≤ u2 + 1

2+

4 + u2 + 3v2

4=

3 (u2 + v2) + 6

4≤ 3.2 + 6

4= 3

Do vậy các dấu đẳng thức xảy ra, tức(v − 1)2 = 0

u = 1

2 =√u2 + 3v2

⇐⇒

{u = 1

v = 1⇐⇒

{ √y − x+ 1 = 1√x− 1 = 1

⇐⇒

{x = 2

y = 2

thỏa mãn điều kiện.

Vậy hệ bất phương trình có nghiệm duy nhất (x; y) = (2; 2) .

• Lời giải 2

Điều kiện:

2x+ y − 2 ≥ 0

2y − 2 + 2 ≥ 0

x ≥ 1

Ta có:

x+ 2y − 2√x− 1 =

(x− 2)2

x+ 2√x− 1

+ 2y ≥ 2y (∗)

Do đó bất phương trình (1) đúng khi và chỉ khi bất phương trình sau phải đúng

19

5+

1

y2 + 1≥ 2y

⇐⇒ (y − 2)(10y2 + y + 12) ≤ 0 ⇐⇒ y ≤ 2 (∗∗)

Từ phương trình (2):

x+ 2y − 1 + 2√

(2x+ y − 2)(y − x+ 1) = 9

113

Page 115: Chuyên đề luyện thi đại học môn Toán - VipLam.Net

Ta có

V T ≤ x+ 2y − 1 +2x+ y − 2 + 4(y − x+ 1)

2=

9y

2Từ đó suy ra9y

2≥ 9 ⇐⇒ y ≥ 2 (? ? ?)

Từ (?); (??); (? ? ?) hệ bất phương trình có nghiệm duy nhất (x; y) = (2; 2)

Nhận xét

Các đánh giá trong bài toán đều rất khéo tuy nhiên lời giải 1 là tường minh và cho ta suy nghĩ

đẹp hơn.Những hệ bất phương trình dạng này là một ví dụ điển hình cho lối giải toán bất đẳng

thức trong hệ.

Bài toán tương tự

1. Giải hệ phương trình: 2√xy +

√1− 2y ≤

√2y

2005√

2xy − y + 2006y = 1003

2. Giải hệ phương trình : x6 + y8 + z10 ≤ 1

x2007 + y2009 + z2011 ≥ 1

Bài toán 6 Giải hệ phương trình:(y + 1)2 + y√y2 + 1 = x+

3

2x+√x2 − 2x+ 5 = 1 + 2

√2x− 4y + 2

Phân tích bài toán

Bài toán này không dễ nhận ra việc sử dụng bất đẳng thức như thế nào,lớp bài toán này

thường hay phổ biến trong các đề.Khi ta biến đổi qua một số bước sẽ đưa đến việc dùng bất

đẳng thức để chứng minh các phương trình là có nghiệm hay vô nghiệm.Qua biến đổi ta đưa

về một phương trình là : { √(x− 1)2 + 4 ≥ |x− 1| ≥ (x− 1)

2√y2 + 1 > 2 |y| ≥ 2y

.Việc phát hiện ra dùng đánh giá dạng A2 ≥ 0 là cách đưa bài toán dễ chứng minh hơn bởi nếu

ta tìm cách giải phương trình (x− 1) +√

(x− 1)2 + 4 = 2y +√

4y2 + 4 là điều rất khó bởi hệ

trên có cả căn thức và hai biến x, y.Vì thế ta đi đến lời giải sau

114

Page 116: Chuyên đề luyện thi đại học môn Toán - VipLam.Net

Điều kiện : x− 2y + 1 ≥ 0

Từ phương trình (1) của hệ ta có :

(y2 + 1) + 2y√y2 + 1 + y2 = 2x− 4y + 2

⇐⇒[y +

√y2 + 1

]2

= 2x− 4y + 2 (a)

Từ phương trình thứ (2) ta lại có :[(x− 1) +

√(x− 1)2 + 4

]2

= 4(2x− 4y + 2) (b)

Từ (a) và (b) cho ta :[(x− 1) +

√(x− 1)2 + 4

]2

= 4[y +

√y2 + 1

]2

⇐⇒

x+ 2y − 1 +√

(x− 1)2 + 4 + 2√y2 + 1 = 0 (3)

(x− 1) +√

(x− 1)2 + 4 = 2y +√

4y2 + 4 (4)

-Với phương trình (3) để ý là :{ √(x− 1)2 + 4 ≥ |x− 1| ≥ (x− 1)

2√y2 + 1 > 2 |y| ≥ 2y

=⇒ x+ 2y − 1 +

√(x− 1)2 + 4 + 2

√y2 + 1 > 0(?)

- Với phương trình (4) ta có :

[(x− 1)− 2y] +[(x− 1)− 2y] (x+ 2y − 1)√

(x− 1)2 + 4 +√

4y2 + 4= 0 ⇐⇒ x− 1 = 2y

( Do chứng minh (?) làm cho biểu thức trong ngoặc của nhân tử liên hợp >0 )

-Với x− 1 = 2y ta thay vào phương trình thứ hai của hệ ban đầu ta được phương trình:

x+√x2 − 2x+ 5 = 1 + 2

√4x+ 1

Phần việc còn lại không khó xin dành cho bạn đọc.

Bài toán 7 Giải hệ phương trình:{(x+ y + 1)(x+ y + 1 + xy) = 12xy

y√

3x− 2x2 − 1 + x√

1 + y − 2y2 + xy = 1

115

Page 117: Chuyên đề luyện thi đại học môn Toán - VipLam.Net

Phân tích bài toán

Bản chất của bài toán nằm ở đánh giá lượng của phương trình thứ nhất,khi quan sát ta nhận

thấy lượng xy của vế phải yếu hơn x + y của vế trái nên ta nghĩ đến việc dùng AM −GM để

đưa về tìm các chặn của xy.Tương tự như bài toán 1 ta đưa về dạng đánh giá 1 ≤ xy ≤ 1

(Lời giải 1).

Về lời giải thứ 2,sự khéo léo nằm ở việc biến đổi1

x+ y + 1− 1

x+ y + 1 + xy=

1

12ở phương trình

đầu,vấn đề còn lại là dùng bất đẳng thức AM − GM để chuyển về phương trình1

x+ y + 1−

4

(x+ y + 2)2 sau đó dùng đạo hàm chứng minh f (t) =1

t− 4

(t+ 1)2 ≤1

12.Từ những định tính

ban đầu ta đi đến lời giải sau.

Lời giải

Lời giải 1 Điều kiện:

3x− 2x2 − 1 ≥ 0

1 + y − 2y2 ≥ 0⇐⇒

1

2≤ x ≤ 1

−1

2≤ y ≤ 1

Lúc này ta có: x+ y + 1 ≥ 1

2+−1

2+ 1 = 1 > 0

x+ y + 1 + xy ≥ 1

2+−1

2+ 1 + 1.

−1

2=

1

2> 0

Suy ra 12xy = (x+ y + 1)(x+ y + 1 + xy) > 0 =⇒ xy > 0

Mà x > 0 nên suy ra y > 0

Do đó từ phương trình 2 ta dễ suy ra được xy ≤ 1

Mặt khác, theo bất đẳng thức AM −GM thì từ phương trình 1 ta có:

12xy = (x+ y + 1)(x+ y + 1 + xy) ≥ 3 3√x.y.1.(2

√x.y + 2

√1.xy) = 12(xy)

5

6

Suy ra xy ≥ 1

Do đó xy = 1 ⇐⇒ x = y = 1

Thử lại thấy x = 1; y = 1 là nghiệm của hệ.

Vậy

x = 1

y = 1.

Lời giải 2 Điều kiện:

−1

2≤ x ≤ 1

−1

2≤ y ≤ 1

+) Với : (x+ y + 1) (x+ y + 1 + xy) = 0 hệ vô nghiệm.

116

Page 118: Chuyên đề luyện thi đại học môn Toán - VipLam.Net

+) Với : (x+ y + 1) (x+ y + 1 + xy) 6= 0

Từ phương trình (1) cho ta :1

x+ y + 1− 1

x+ y + 1 + xy=

1

12

Lại có :1

x+ y + 1− 1

x+ y + 1 + xy≤ 1

x+ y + 1− 4

(x+ y + 2)2

Xét hàm số : f (t) =1

t− 4

(t+ 1)2 , t ∈ [1; 3] , trong đó : t = x+ y + 1

Lập bảng biến thiên cho ta : f (t) ≤ f (3) =1

12

Vậy (1) ⇐⇒

{x+ 1 = y + 1

x+ y + 1 = 3⇐⇒

{x = 1

y = 1

Thử lại cho ta nghiệm của hệ :

{x = 1

y = 1

Vậy nghiệm của hệ là (x, y) = (1; 1)

Nhận xét

• Lời giải 1 tự nhiên hơn hẳn,chỉ thuần sử dụng bất đẳng thức để đánh giá,cái tinh tế ở đây

là việc ta sử dụng bất đẳng thức AM −GM tại

12xy = (x+ y + 1)(x+ y + 1 + xy) ≥ 3 3√x.y.1.(2

√x.y + 2

√1.xy) = 12(xy)

5

6

Tại sao ta không áp dụng AM −GM trực tiếp cho các số mà phải ghép cặp lại,chỉ vì khi

ta xét dấu đẳng thức thì việc ghép cặp giúp ta bảo toàn dấu đẳng thức bài toán.

• Sự tinh ý của lời giải 2 nằm ở vị trí đánh giá

(x+ y + 2)2 ≥ 4(x+ y + 1 + xy) với điều kiện x, y ≤ 1.

Điều này đúng theo bất đẳng thức AM −GM vì ta có

x+y+ 1 +xy ≤ x+y+ 1 +(x+ y)2

4+x+y+ 1 =

(x+ y)2 + 4(x+ y) + 4

4=

(x+ y + 2)2

4

Bài toán 8 Giải hệ phương trình:(x+ 6y + 3)√xy + 3y = (8y + 3x+ 9)y√

−x2 + 8x− 24y + 417 = (y + 3)√y − 1 + 3y + 17

Phân tích bài toán

117

Page 119: Chuyên đề luyện thi đại học môn Toán - VipLam.Net

Hệ đã cho gồm các phương trình căn thức và đa thức,việc ta nên làm là giải quyết các căn

thức khó chịu trên.Và phương pháp thương dùng nhất là đặt ẩn phụ,nhưng đặt ẩn phụ như thế

nào là ổn.Đó là điều ta quan tâm ?

Lời giải

Ta viết lại hệ phương trình đã cho như sau:{(x+ 6y + 3)

√xy + 3y = (8y + 3x+ 9)y (1)√

−x2 + 8x− 24y + 417 = (y + 3)√y − 1 + 3y + 17 (2)

Ta đặt a =√x+ 3; b =

√y với a, b ≥ 0

Tư đó ta viết lại phương trình (1) thành :

(a2 + 6b2)ab = b2(8b2 + 3a2)

Vậy ta có : b = 0 hay a3 + 6ab2 = 8b3 + 3a2b

Vậy ta có :

• b = 0 Suy ra y = 0 không thoả phương trình (2).

• (a− 2b)(a2 − ab+ 4b2) = 0 =⇒ a = 2b

Với a = 2b =⇒ x+ 3 = 4y

Thay vào (2) ta có :

4√

(y + 4)(6− y) = (y + 3)√y − 1 + 3y + 17

Theo bất đẳng thức AM −GM ta có :

4√

(y + 4)(6− y) ≤ 4(y + 4 + 6− y)

2= 20

Và ta có :

(y + 3)√y − 1 + 3y + 17 ≥ 3y + 17 ≥ 3 + 17 = 20

Vậy đẳng thức xảy ra khi y = 1 thay vào ta có x = 1.

Vậy nghiệm của hệ là (x, y) = (1; 1)

Bài toán 9 Giải hệ phương trình: x3 − y3 +5

3(x+ y)2 + 5x2 − 8

3xy + 13x =

100

3x2 + y2 + xy − 3x− 4y + 4 = 0

Phân tích bài toán

118

Page 120: Chuyên đề luyện thi đại học môn Toán - VipLam.Net

Đây là dạng toán hay và quen thuộc với việc kết hợp hai công cụ mạnh là đánh giá bất đẳng

thức và đạo hàm.Với những dạng toán như thế này,công việc ban đầu ta làm là tìm các chặn

của các biến x, y dựa vào việc đánh giá Delta của một trong hai phương trình của hệ.Khi đó

ta sẽ có tiếp hai hướng để giải quyết

1. Sử dụng đạo hàm để đánh giá dạng f(x) + g(y) = a với min hoặc max của f(x) , g(y) có

tổng bằng a.

2. Sử dụng bất đẳng thức để đánh giá dạng A+ B ≥ a với A,B là hai biểu thức nào đó và

a là hằng số.

Lời giải:

Ta viết lại hệ đã cho như sau x3 − y3 +5

3(x+ y)2 + 5x2 − 8

3xy + 13x =

100

3(1)

x2 + y2 + xy − 3x− 4y + 4 = 0(2)

Từ (2) ta suy ra điều kiện của x; y là

4

3≥ x ≥ 0

7

3≥ y ≥ 1

Rút xy từ (2) thay vào (1) ta được

(3x3 + 18x2 + 45x) + (3y2 − 3y3 + 8y) = 108 (3)

Xét hàm số

f(x) = 3x3 + 18x2 + 45x;xε

[0;

4

3

]Suy ra f ′(x) ≥ 0 nênf(x) là hàm đồng biến với xε

[0;

4

3

]⇐⇒ f(x) ≤ f4

3

=892

9(4)

Xét hàm số

g(y) = 3y2 − 3y + 8y; yε

[1;

7

3

]Dựa vào bảng biến thiên:

g(y) ≤ g4

3

=80

9(5)

Từ (3), (4), (5) suy ra hệ có nghiệm ⇐⇒

x =

4

3

y =4

3Thử lại thấy nghiệm thỏa mãn phương trình thứ hai.

Hệ phương trình có nghiệm (x; y) = (4

3;4

3)

Bài toán tương tự

119

Page 121: Chuyên đề luyện thi đại học môn Toán - VipLam.Net

1. Giải hệ phương trình

x4 + y2 =698

81x2 + y2 + xy − 3x− 4y + 4 = 0

2. Giải hệ phương trình

x3 − y3 − 15(x− y)− (x+ y)2 = x2 − 9y2 − 15y + 94

4x2 + 4y2 + 6x+ 6y − 2xy − 9 = 0

Bài toán 10 Giải hệ phương trình :

√x+√y + 2(x2 + y2) = 4 + 2xy

x√

3x2 + 6xy + y√

3y2 + 6xy = 6

Lời giải:

Phân tích bài toán

Bài toán này rất thú vị,nếu ta không nhận ra rằng nếu sử dụng các bất đẳng thức quá mạnh

sẽ dẫn tới làm khó bài toán và gần như là đưa kết quả về con số 0.Bài toán được anh Nguyễn

Trung Kiên đưa lên trang www.k2pi.net trong một thời gian không nhận được lời giải,sau đây

là lời giải của chúng tôi,một lời giải áp dụng chỉ bất đẳng thức cơ bản nhưng khá tinh tế.Mời

các bạn cùng thưởng thức.

Lời giải

Ta viết lại đề bài: √x+√y + 2(x2 + y2) = 4 + 2xy

x√

3x2 + 6xy + y√

3y2 + 6xy = 6

Từ phương trình thứ hai,áp dụng bất đẳng thức AM −GM ta có :

x√

3x2 + 6xy + y√

3y2 + 6xy ≥ 2

√xy.(√

3x2 + 6xy.√

3y2 + 6xy)

≥ 2.

√xy√

(9x2y2 + 18xy(x2 + y2) + 36x2y2) ≥ 2.

√xy√

(9(xy)2 + 36(xy)2 + 36(xy)2) = 6xy

Suy ra xy ≤ 1 (1).

Để ý rằng

x√

3x2 + 6xy + y√

3y2 + 6xy =x.√

9x.√

3x+ 6y

3+y.√

9y.√

3y + 6x

3

Ta lại có theo bất đẳng thức AM −GM thì :

x.√

9x.√

3x+ 6y

3+y.√

9y.√

3y + 6x

3≤ 12x2 + 6xy

6+

12y2 + 6xy

6= 2(x2 + y2 + xy)

120

Page 122: Chuyên đề luyện thi đại học môn Toán - VipLam.Net

Vậy ta suy ra:x2 + y2 + xy ≥ 3

Mà xy ≤ 1 nên x2 + y2 ≥ 2 .

Từ phương trình thứ nhất ta có:

4 + 2xy ≥√x+√y + 4 ≥ 2. 4

√xy + 4.

Vậy suy ra xy ≥ 4√xy ⇐⇒ 4

√xy ≥ 1

Hay xy ≥ 1 (2).

Từ (1); (2) ta suy ra xy = 1.

Và từ các dấu bằng bất đẳng thức ta có x = y = 1.

Vậy tập nghiệm của hệ phương trình là S = (x; y) = (1; 1).

Bài toán 11Tìm tất cả các nghiệm dương của hệ phương trình :√x+ y + 2(2x2 − 5x− 3) = y(1− y − 5x) +

√3

1

16x4 + 2x(12x+ 1) + 2(y + 2)+

1

16y4 + 2y(12y + 1) + 2(x+ 2)=

2

145

Nguồn gốc

Bài toán được đưa lên www.k2pi.net bởi anh Con Phố Quen trong topic hệ sáng tạo từ các

thành viên K2pi.netvà lời giải đến hiện nay cho bài toán này là của chúng tôi,một lời giải thân

quen với những công cụ bất đẳng thức đưa bài toán trở nên đơn giản hơn nhiều.Thay vì phân

tích hướng làm,chúng tôi mời các bạn đọc bài toán và tự rút ra hướng làm cho bản thân mình.Vì

như thế các bạn sẽ hiểu rõ hơn cách giải và có những sáng tạo nhất định.Chúng tôi chờ đợi lời

giải từ các bạn.

Lời giải

Ta viết lại hệ phương trình√x+ y + 2(2x2 − 5x− 3) = y(1− y − 5x) +

√3 (1)

1

16x4 + 2x(12x+ 1) + 2(y + 2)+

1

16y4 + 2y(12y + 1) + 2(x+ 2)=

2

145(2)

-Xét (1). Đặt t =√x+ y;t > 0. Ta viết phương trình (1) về dạng:

t+ 4x2 − 10x− 6 = y − y2 − 5xy +√

3

Theo cách "chân quê" ta cứ rút y theo x, t nên ta có :y = t2 − x Vậy phương trình trên biến

đổi lại thành:

t4 − t2 + t− 6−√

3 = 3(3− t2)x

⇐⇒ (t−√

3)(t3 +√

3t2 + 2t+ 2√

3 + 1) = 3(√

3− t)(√

3 + t).x

121

Page 123: Chuyên đề luyện thi đại học môn Toán - VipLam.Net

Đến đây các bạn có thắc mắc tại sao tôi biết được cách phân tích như thế không ? Thật ra,khi

nhìn vào phương trình thứ hai,ta dễ nhận thấy phương trình trên là một bất đẳng thức nào đó

,nên điều ta thiết nghĩ lúc này có chăng là x = y. Thật vậy,bằng kiểm tra đơn giản,ta có ngay

một nghiệm bài toán là x = y =3

2.Đến đây với cách "chân quê" ta lại có biến đổi như trên là

điều hoàn toàn giải thích được. Phương trình trên cho ta :[t =√

3 (3)

t3 +√

3t2 + 2t+ 2√

3 + 1 = −3(√

3 + t).x (4)

Chúng ta hãy tạm giải quyết trường hợp t =√

3 trước. Quay trở lại phương trình (2) ta có:

1

16x4 + 2x(12x+ 1) + 2(y + 2)+

1

16y4 + 2y(12y + 1) + 2(x+ 2)=

2

145

Theo bất đẳng thức Cauchy Schwarz ta có :

1

16x4 + 2x(12x+ 1) + 2(y + 2)+

1

16y4 + 2y(12y + 1) + 2(x+ 2)≥

≥ 4

16(x4 + y4) + 24(x2 + y2) + 4(x+ y + 2)

Ta biến đổi bất đẳng thức về :

16(x4 + y4) + 24(x2 + y2) + 4(x+ y + 2) ≤ 290

Đặt t = x+ y . Theo bất đẳng thức AM −GM ta có các đánh giá :

x+ y ≤√

2(x2 + y2)

x2 + y2 ≥√

2(x4 + y4)

Đặt u =√

2√

2. 4√x4 + y4.

Vậy ta có :

16(x4 + y4) + 24(x2 + y2) + 4(x+ y + 2) ≤ 2u4 + 12u2 + 4(u+ 2) ≤ 290

Mà 2u4 + 12u2 + 4(u+ 2)− 290 = (u− 3)(2u3 + 6u2 + 30u+ 94) = 0 do u = 3.

Vậy đây chỉ là một đẳng thức.

Vậy dấu bằng xảy ra khi x = y =3

2-Ta giải quyết (4).

t3 +√

3t2 + 2t+ 2√

3 + 1 = −3(√

3 + t).x

Dễ nhận thấy với điều kiện x, t > 0 ta có t3 +√

3t2 + 2t+ 2√

3 + 1 + 3(√

3 + t).x > 0 nên phương

trình này vô nghiệm.

Vậy tóm lại hệ phương trình có nghiêm duy nhất (x; y) = (3

2;3

2).

Nhận xét Cách tạo hệ từ bất đẳng thức như thế này giúp ta có thể có được những bài toán

122

Page 124: Chuyên đề luyện thi đại học môn Toán - VipLam.Net

hay,có thể lấy một bài bất đẳng thức trong đề thi thử chuyên Khoa học tự nhiên năm 2012 làm

bài toán hệ như sau:

Tìm nghiệm dương của hệ saux+ y = 21

2 + 6x2 + 9x4+

1

2 + 6y2 + 9y4=

2

17

Bài tập tương tự

Giải hệ phương trình sau:√x+√y +√z = 2013

1

3x+ 2y+

1

3y + 2z+

1

3z + 2x=

1

x+ 2y + 2z+

1

y + 2z + 2x+

1

z + 2x+ 2y

BÀI TẬP RÈN LUYỆN CHƯƠNG I

1. Giải hệ phương trình

x+

2xy3√x2 − 2x+ 9

= x2 + y

y +2xy

3√y2 − 2y + 9

= y2 + x.

2. Giải hệ phương trình

(x+ y)3 + 4xy = 3

(x+ y)4 − 2x2 − 4xy + 2y2 + x− 3y + 1 = 0.

3. Giải hệ phương trình

(2x+ 3)√

4x− 1 + (2y + 3)√

4y − 1 = 2√

(2x+ 3)(2y + 3)

x+ y = 4xy.

4. Giải hệ phương trình

1 + 2x2 +√

1 + 2y2 = 2√

1 + 2xy√x(1− 2y) +

√x(1− 2y) =

2

3.

5. Giải hệ phương trình

x2 + y2 +

8xy

x+ y= 16

x2

8y+

2x

3=

√x3

3y+y2

4− y

2.

6. Cho hệ phương trình

x+ y + 4 = 2xy

2x+y = m(√x2 + y2 + x+ y + 5 + x+ y).

Tìm m để hệ có nghiệm (x, y) thoả x, y ≥ 1.

123

Page 125: Chuyên đề luyện thi đại học môn Toán - VipLam.Net

7. Giải hệ phương trình

√x+

√x2 + y + 3 = 2

2√x+ 4 + 3

√y + 8 = 13.

8. Giải hệ phương trình

3(x+ y) = 2 |xy + 1|

9(x3 + y3) = |x3y3 + 1|

9. Giải hệ phương trình

x+ y = 3

√24

(√x+√y)

(1√

x+ 3y+

1√y + 3x

)= 2

10. Giải hệ phương trình

√x+ 4√

32− x− y2 = −3

4√x+√

32− y + 6y = 24

II. BẤT ĐẲNG THỨC & HỆ PHƯƠNG TRÌNH 3 ẨN

Bài toán 1 Giải hệ phương trình:

1√x

+1√y

+1√z

= 3√

3 (1)

x+ y + z = 1 (2)

x+ y + z = 1 (2)

xy + yz + zx =7

27+ 2xyz (3)

Lời giải:

Điều kiện:x > 0, y > 0, z > 0

Kết hợp với (2): x+ y + z = 1 ta thấy trong các số x; y; z phải có ít nhất 1 số không lớn hơn1

3

, không mất tính tổng quát ta giả sử z ≤ 1

3

Do đó: z ∈(

0;1

3

]Đặt:

S = xy + yz + zx− 2xyz = xy (1− 2z) + z (x+ y) = xy (1− 2z) + z (1− z)

Do:

xy ≤(x+ y

2

)2

=

(1− z

2

)2

Vậy:

S ≤(

1− z2

)2

(1− 2z) + z (1− z) =1

4

(−2z3 + z2 + 1

)

124

Page 126: Chuyên đề luyện thi đại học môn Toán - VipLam.Net

Xét hàm số

f (z) =1

4

(−2z3 + z2 + 1

)Ta có:

f ′ (z) =1

4

(−6z2 + 2z

)=

1

2z (−3z + 1) ≥ 0,∀z ∈

(0;

1

3

]Vậy:

f (z) ≤ f

(1

3

)=

7

27, ∀z ∈

(0;

1

3

]Do đó:

S ≤ 7

27

Dấu xảy ra khi và chỉ khi:

x = y, z =1

3

Thay vào (2) ta được:

x = y = z =1

3

Thử lại ta thấy thỏa mãn hệ phương trình

Vậy hệ phương trình có nghiệm duy nhất (x; y; z) =

(1

3;1

3;1

3

)

Bài toán 2 Giải hệ phương trình:

1

xy=x

z+ 1

1

yz=y

x+ 1

1

zx=x

y+ 1

Lời giải:

Điều kiện xyz 6= 0 .Nhận thấy nếu một trong ba số x, y, z có một số âm,chẳng hạn x < 0

thì phương trình thứ ba vô nghiệm.Nếu hai trong ba số x, y, z là số âm,chẳng hạn x, y < 0 thì

phương trình thứ hai vô nghiệm.Vậy ba số x, y, z cùng dấu.

1. Xét trường hợp x, y, z > 0 thì ta viết lại hệ như sauz = x2y + xy

x = y2z + yz

y = z2x+ zx

125

Page 127: Chuyên đề luyện thi đại học môn Toán - VipLam.Net

Cộng ba phương trình lại ta được

x+ y + z = (x2y + y2z + z2x) + (xy + yz + zx) ≥ 6xyz (?)

Mặt khác ta biến đổi hệ về dạng z

xy= x+ z

x

yz= y + x

y

zx= y + z

=⇒ z

xy+

x

yz+

y

zx= 2(x+ y + z)

Thì ta có

2(x+ y + z) =x2 + y2 + z2

xyz≥ (x+ y + z)2

3xyz=⇒ 6xyz ≥ x+ y + z (?)(?)

Từ (?) và (?)(?) ta có x = y = z,từ đó ta có nghiệm của hệ là trong trường hợp này là

(x, y, z) =

(√2

2,

√2

2,

√2

2

)2. Trường hợp x, y, z < 0 ta đặt a = −x; b = −y; c = −z ta chuyển về trường hợp số dương

và làm như trường hợp 1.

Vậy nghiệm của hệ là (x, y, z) =

(√2

2,

√2

2,

√2

2

)Bài toán 3 Cải biên đề thi đại học khối B năm 2012

Giải hệ phương trình: x+ y + z = 0

x2 + y2 + z2 = 1

x5 + y5 + z5 =5√

6

36

Lời giải:

Với x+ y + z = 0 và x2 + y2 + z2 = 1 ta có

0 = (x+ y + z)2 = x2 + y2 + z2 + 2x (y + z) + 2yz = 1− 2x2 + 2yz

Nên yz = x2 − 1

2

Mặt khác yz ≤ y2 + z2

2=

1− x2

2suy ra x2 − 1

2≤ 1− x2

2do đó −

√6

3≤ x ≤

√6

3(∗)

Khi đó

P = x5 + (y2 + z2) (y3 + z3)− y2z2 (y + z)

= x5 + (1− x2) [(y2 + z2) (y + z)− yz (y + z)] +(x2 − 1

2

)2x

= x5 + (1− x2)[−x (1− x2) + x

(x2 − 1

2

)]+(x2 − 1

2

)2x = 5

4(2x3 − x)

126

Page 128: Chuyên đề luyện thi đại học môn Toán - VipLam.Net

Xét hàm số f (x) = 2x3 − x với −√

6

3≤ x ≤

√6

3suy ra f ′ (x) = 6x2 − 1; f ′ (x) = 0 ⇐⇒ x =

±√

6

6

Ta có f

(−√

6

6

)= f

(√6

6

)= −√

6

9, f

(√6

3

)= f

(−√

6

6

)=

√6

9

Do đó f (x) ≤√

6

9suy ra P ≤ 5

√6

36.

Vậy nghiệm của hệ là x =

√6

3; y = z = −

√6

6và các hoán vị.

BÀI TẬP RÈN LUYỆN CHƯƠNG II

1. Giải hệ phương trình

1√x

+1√y

+1√z

= 3√

3

x+ y + z = 1

2. Giải hệ phương trình

xy3 = 9

x+ 3y = 6

3. Giải hệ phương trình

x5 + y5 + z5 = 3

x6 + y6 + z6 = 3

4. Giải hệ phương trình

3(x2 + y2 + z2) = 1

x2y2 + y2z2 + z2x2 = xyz(x+ y + z)3

5. Giải hệ phương trình

36x2y − 60x2 + 25y = 0

36y2z − 60y2 + 25z = 0

36z2z − 60z2 + 25x = 0

III. TỔNG HỢP CÁC BÀI TOÁN HỆ PT GIẢI BẰNG PHƯƠNG

PHÁP BẤT ĐẲNG THỨC

Bài toán 1 Giải hệ phương trình:1√

1 + 2x2+

1√1 + 2y2

=1√

1 + 2xy√x(1− 2x) +

√y(1− 2y) =

2

9

Đề thi học sinh giỏi quốc gia năm 2009

Lời giải:

127

Page 129: Chuyên đề luyện thi đại học môn Toán - VipLam.Net

Điều kiện:

1 + 2xy ≥ 0

x(1− 2x) ≥ 0

y(1− 2y) ≥ 0

⇐⇒

0 ≤ x ≤ 1

2

0 ≤ y ≤ 1

2

Với điều kiện trên ta được :x2 ≤ 1

4; y2 ≤ 1

4=⇒ 1√

1 + 2xy=

1√1 + 2x2

+1√

1 + 2y2≥ 2√

2

3>

√2 =⇒ 2xy < 1

Mặt khác với mọi a, b ∈ [0;1√2

] và ab < 1 ta luôn có bất đẳng thức sau:

1√1 + a2

+1√

1 + b2≤ 2√

1 + ab(∗)

Thật vậy bất đẳng thức (∗) tương đương với :

1

1 + a2+

1

1 + b2+

2√1 + a2.

√1 + b2

− 4

1 + ab≤ 0

Theo bất đẳng thức Cauchy Schwarz ta có :

√(1 + a2)(1 + b2) ≥ 1 + ab =⇒ 2√

1 + a2.√

1 + b2≤ 2

1 + ab

Mặt khác ta có :

1

1 + a2+

1

1 + b2− 2

1 + ab=

(a− b)2(ab− 1)

(1 + ab)(1 + a2)(1 + b2)≤ 0

Áp dụng bất đẳng thức trên với a =√

2x; b =√

2y ta được

1√1 + 2x2

+1√

1 + 2y2≤ 1√

1 + 2xy

Đẳng thức xảy ra khi x = y.

Với x = y thay vào hệ thứ hai ta được phương trình:

162x2 − 81x+ 1 = 0

Giải phương trình trên ta được: x =81 +

√5913

324;x =

81−√

5913

324

Vậy nghiệm của hệ là : (x; y) = (81 +

√5913

324;81 +

√5913

324); (

81−√

5913

324;81−

√5913

324) .

Bài toán 2 Giải hệ phương trình:(2x2 − 1)(2y2 − 1) =7

2xy

x2 + y2 + xy − 7x− 6y + 14 = 0

128

Page 130: Chuyên đề luyện thi đại học môn Toán - VipLam.Net

Lời giải:

Dể thấy xy = 0 không là nghiệm của hệ

Với xy 6= 0 ta viết hệ dưới dạng(2x− 1

x)(2y − 1

y) =

7

2

x2 + y2 + xy − 7x− 6y + 14 = 0

Điều kiện để phương trình x2 + y2 + xy − 7x− 6y + 14 = 0 có nghiệm theo ẩn x là :

∆ = (y − 7)2 − 4y2 + 24y − 56 ≥ 0 ⇐⇒ y ∈ [1;7

3]

Điều kiện để phương trình x2 + y2 + xy − 7x− 6y + 14 = 0 có nghiệm theo ẩn y là :

∆ = (x− 6)2 − 4x2 + 28x− 56 ≥ 0 ⇐⇒ x ∈ [2;10

3]

Xét hàm số : f(t) = 2t− 1

tlà hàm đồng biến trên (0; +∞) nên:

f(x).f(y) ≥ f(1)f(2) =7

2.

Kết hợp với phương trình thứ nhất ta được (x; y) = (2; 1) .

Bài toán 3 Giải hệ phương trình:x4

y4+y4

x4− (

x2

y2+y2

x2) +

y

x+x

y= −2

x2 + y6 − 8x+ 6 = 0

Lời giải:

Điều kiện: x, y 6= 0

Ta viết lại hệ như sau: x4

y4+y4

x4− (

x2

y2+y2

x2) +

y

x+x

y= −2 (1)

x2 + y6 − 8x+ 6 = 0 (2)

Đặt t =x

y+y

xsuy ra t2 =

x2

y2+y2

x2+ 2

Mặt khác

(x2

y2+y2

x2)2 = (t2 − 2)2 ⇐⇒ x4

y4+y4

x4+ 2 = t4 − 4t2 + 4

Theo bất đẳng thức AM −GM ta có :x2

y2+y2

x2≥ 2 hay t2 ≥ 4 suy ra |t| ≥ 2

Ta có vế trái phương trình (1) được biến đổi thành g(t) = t4 − 5t2 + t+ 4 , |t| ≥ 2.

Có g′(t) = 4t3 − 10t+ 1 = 2t(2t2 − 5) + 1

Nhận xét:

1. Nếu t ≥ 2 suy ra 2t(2t2 − 5) ≥ 4(8− 5) > 0 =⇒ g′(t) > 0

129

Page 131: Chuyên đề luyện thi đại học môn Toán - VipLam.Net

2. Nếu t ≤ −2 suy ra 2t ≤ −4 ;2t2 − 5 ≥ 3 ;−2t(2t2 − 5) ≥ 12 =⇒ 2t(2t2 − 5) ≤ −12 hay

g′(t) < 0

Lập bảng biến thiên ta suy ra giá trị nhỏ nhất của g(t) = −2 đạt được khi t = −2.

Vậy từ phương trình (1) ta cóx

y+y

x= −2 từ đây suy ra x = −y.

Thay x = −y vào phương trình thứ hai ta có :x6 + x2 − 8x + 6 = 0 ⇐⇒ (x− 1)2(x4 + 2x3 +

3x2 + 4x+ 6) = 0 ⇐⇒ (x− 1)2[x2(x+ 1)2 + 2(x+ 1)2 + 4] = 0 .

Vậy ta có nghiệm của hệ là (x; y) = (1;−1) .

Bài toán 4 Giải hệ phương trình:x3 + 3xy2 = x2 + y2 + 2 (1)

x4 + y4 + 6x2y2 = 8 (2)

Lời giải:

Từ phương trình thứ hai ta có :

x(x2 + 3y2) = x2 + y2 + 2 =⇒ x > 0

Nếu y = 0 thì hệ trở thành :

x4 = 8

x3 = x2 + 2Vô nghiệm

Từ đó suy ra y 6= 0 .

Viết lại hệ dưới dạng: (x2 + y2)2 + (2xy)2 = 8 (3)

x2 + y2 + 2 = x(x2 + y2) + y(2xy) (4)

Từ (4) ta có :

(x2 + y2 + 2)2 = [x(x2 + y2) + y(2xy)]2 ≤ (x2 + y2)[(x2 + y2)2 + (2xy)2] = 8(x2 + y2) (?)do (3)

=⇒ (x2 + y2)2 − 4(x2 + y2) + 4 ≤ 0

=⇒ x2 + y2 = 2

Dấu đẳng thức trong (?) xảy ra khix2 + y2

x=

2xy

y⇐⇒ x2 = 1 ⇐⇒ x = 1.

Thế vào hệ ta được y = 1.

130

Page 132: Chuyên đề luyện thi đại học môn Toán - VipLam.Net

Vậy nghiệm của hệ là (x; y) = (1; 1) .

Bài toán 5 Giải hệ phương trình:√

1 +√

1− x2 = x(1 + 2√

1− y2) (1)1√

1 + x+

1√1 + y

=2√

1 +√xy

(2)

Lời giải:

Điều kiện:|x| , |y| ≤ 1 , xy > 0.

Từ (1) suy ra 0 ≤ x ≤ 1.Do đó 0 ≤ y ≤ 1.

Theo bất đẳng thức Cauchy Schwarz ta có :(1√

1 + x+

1√1 + y

)2

≤ 2

(1

1 + x+

1

1 + y

)(3)

Ta sẽ chứng minh

1

1 + x+

1

1 + y≤ 2√

1 +√xy

(4)

Thật vậy:

(4) ⇐⇒ (1−√xy)(√x−√y)2 ≥ 0

Điều trên đúng với mọi x, y ∈ [0; 1].

Từ đó suy ra :

1√1 + x

+1√

1 + y≤ 2√

1 +√xy

.

Đẳng thức xảy ra khi và chỉ khi x = y.

Thế x = y vào (2) ta được :√1 +√

1− x2 = x(1 + 2√

1− x2) (5)

Đặt x = sint, t ∈ [0;π

2] thì

(5) =⇒√

1 + cost = sint(1 + 2cost)

=⇒√

2cost

2= 2sin

t

2cos

t

2[1 + 2(1− 2sin2 t

2] (do t ∈ [0;

π

2] =⇒ cos

t

2> 0)

131

Page 133: Chuyên đề luyện thi đại học môn Toán - VipLam.Net

=⇒ 3sint

2− 4sin3 t

2=

√2

2

sin3t

2= sin

π

4

=⇒

t =π

6+k4π

3

t =π

2+k4π

3

k ∈ Z

Với t ∈ [0;π

2],ta được : t =

π

6+k4π

3

t =π

2+k4π

3

=⇒

x =1

2x = 1

Vậy hệ đã cho có nghiệm (x; y) = (1

2;1

2); (1; 1) .

Bài toán 6 Giải hệ phương trình:y2 + (4x− 1)2 = 3√

4x(8x+ 1)

40x2 + x = y√

14x− 1

Lời giải:

Điều kiện:x ≥ 1

14

Đặt t = 4x (t ≥ 2

7).Hệ phương trình trở thành:

y2 + (t− 1)2 = 3√t(2t+ 1) (1)

5

2t2 +

t

4= y

√7

2t− 1 (2)

Từ (2) ta có y > 0 nên áp dụng bất đẳng thức AM −GM ta có :

3√t(2t+ 1) =

3

√2t.

2t+ 1

2.1 ≥

2t+2t+ 1

2+ 1

3= t+

1

2

Do đó từ phương trình thứ nhất ta có :

y2 + (4x− 1)2 ≤ t+1

2=⇒ y2 ≤ −t2 + 3t− 1

2(3)

132

Page 134: Chuyên đề luyện thi đại học môn Toán - VipLam.Net

Mặt khác theo bất đẳng thức AM −GM ta lại có :

y

√7

2t− 1 ≤

y2 +7

2t− 1

2

Suy ra

y2 ≥ 5t2 − 3t+ 1 (4)

VTừ (3) và (4) ta có :

5t2 − 3t+ 1 ≤ −t2 + 3t− 1

2⇐⇒ (2t− 1)2 ≤ 0

Suy ra t =1

2⇐⇒ x =

1

8thay vào hệ ta tìm được y =

√3

2.

Vậy nghiệm của hệ là (x; y) = (1

8;

√3

2) .

Bài toán 7 Giải hệ phương trình:x2y2 − 2x+ y2 = 0 (1)

2x3 + 3x2 + 6y − 12x+ 13 = 0 (2)

Lời giải:

Từ phương trình (1) ta có :

y2 =2x

x2 + 1suy ra x ≥ 0.

Do x ≥ 0 nên áp dụng bất đẳng thức AM −GM ta có :

y2 ≤ 1 =⇒ −1 ≤ y ≤ 1 (?)

Từ (2) ta suy ra :

y =−2x3 − 3x2 + 12x− 13

6=

(−2x− 7)(x− 1)2

6− 1 (3)

Do x ≥ 0 nên từ (3) suy ra y ≤ −1 (??).

Từ (?) và (??) suy ra y = −1.

Thay y = −1 vào ta tìm được x = 1.

Vậy nghiệm của hệ là (x; y) = (1;−1) .

133

Page 135: Chuyên đề luyện thi đại học môn Toán - VipLam.Net

Bài toán 8 Giải hệ phương trình:x

y + 1+

y

x+ 1=

2√xy

√xy + 1

5√x− 1

+3√y − 1

= 4

Lời giải:

Điều kiện :x, y > 1 =⇒ xy > 1

Ta chứng minh

1

x+ 1+

1

y + 1≥ 2√xy + 1

(?)

⇐⇒ (x+ 1)(√xy + 1) + (y + 1)(

√xy + 1) ≥ 2(x+ 1)(y + 1)

⇐⇒ (√xy − 1)(

√x−√y)2 ≥ 0

Luôn đúng với mọi xy > 1.

Ta có :

x

y + 1+

y

x+ 1=

2√xy

√xy + 1

⇐⇒ x

y + 1+ 1 +

y

x+ 1+ 1 =

2√xy

√xy + 1

+ 2

⇐⇒ (x+ y + 1)(1

x+ 1+

1

y + 1) = 2

2√xy + 1

√xy + 1

Mặt khác theo bất đẳng thức AM −GM và (?) ta cóx+ y + 1 ≥ 2

√xy + 1

1

x+ 1+

1

y + 1≥ 2√xy + 1

=⇒ (x+ y + 1)(1

x+ 1+

1

y + 1) ≥ 2

2√xy + 1

√xy + 1

134

Page 136: Chuyên đề luyện thi đại học môn Toán - VipLam.Net

Đẳng thức xảy ra khi x = y.Thay vào hệ tìm được x = y = 5.

Vậy nghiệm của hệ là x = y = 5 .

Bài toán 9 Giải hệ phương trình:√x2 + y2

2+

√x2 + xy + y2

3= x+ y

x√

2xy + 5x+ 3 = 4xy − 5x− 3

Lời giải:

Ta có :

x2 + xy + y2 =1

2(x+ y)2 +

1

2(x2 + y2) ≥ 1

2(x+ y)2 +

1

4(x+ y)2 =

3

4(x+ y)2

Vậy suy ra

√x2 + y2

2+

√x2 + xy + y2

3≥√

(x+ y)2

4+

√√√√ 3

4(x+ y)2

3≥ x+ y

Từ đó suy ra x = y ≥ 0

Thay vào phương trình thứ hai của hệ ta được:

x√

2x2 + 5x+ 3 = 4x2 − 5x− 3

⇐⇒ 2x2 + 5x+ 3− x√

2x2 + 5x+ 3− 6x2 = 0

⇐⇒

[√2x2 + 5x+ 3 = −3x√2x2 + 5x+ 3 = 2x

Do x ≥ 0 nên ta nhận√

2x2 + 5x+ 3 = 2x

Suy ra :

−2x2 + 5x+ 3 = 0 ⇐⇒

x = −1

2x = 3

Thử lại nhận nghiệm x = 3.

Vậy nghiệm của hệ là: (x; y) = (3; 3) .

135

Page 137: Chuyên đề luyện thi đại học môn Toán - VipLam.Net

Bài toán 10 Giải hệ phương trình:x+ y + xy = z22003 + 2z22002 (1)

x4 + y4 = 2z22004 (2)

(x+ y)z−1 = (z + 2004)x−y (3)

Lời giải:

Từ phương trình thứ hai ta có :

2z22004 = x4 + y4 ≥ 2x2y2 =⇒ xy ≤ z22003(?)

Ta lại có

(x+ y)2 ≤ 2(x2 + y2)

=⇒ (x+ y)4 ≤ 2(x2 + y2)2 ≤ 8(x4 + y4) = 16z22004

=⇒ x+ y ≤ 2z22002(? ?)

Từ (?) và (? ?) ta có :

x+ y + xy ≤ z22003 + 2z22002

Đẳng thức xảy ra khi x = y = z22002 .

Hệ phương trình tương đương với :x = y = z22002

(2x)z−1 = 1⇐⇒

x = y = z = 1

x = y =1

2; z = ±2

−1

22002

Bài toán 11 Giải hệ phương trình:x2 + 2x− 2 =√−y2 − 4y − 2

6x− y + 11 +√

10− 4x− 2x2 = 0

136

Page 138: Chuyên đề luyện thi đại học môn Toán - VipLam.Net

Lời giải:

Từ phương trình thứ hai áp dụng bất đẳng thức AM −GM ta có :

y − 6x+ 11 =√

10− 4x− 2x2 =

√4(10− 4x− 2x2)

2≤ 4 + 10− 4x− 2x2

4

Thu gọn phương trình trên ta được

x2 − 10x+ 2y − 15 ≤ 0 (1)

Tiếp tục áp dụng bất đẳng thức AM −GM cho phương trình thứ nhất của hệ ta được

x2 + 2x− 2 =√−y2 − 4y − 2 =

√1(−y2 − 4y − 2) ≤ −y

2 − 4y − 2 + 1

2

Thu gọn ta có

2x2 + 4x+ y2 + 4y − 3 ≤ 0 (2)

Lấy (1) cộng (2) vế theo vế ta có :

3x2 − 6x+ y2 + 6y + 12 ≤ 0 ⇐⇒ 3(x− 1)2 + (y + 3)2 ≤ 0

Từ đó suy ra nghiệm của hệ là (x; y) = (1;−3) .

Bài toán 12 Giải hệ phương trình:{ √2x+ 1−

√2y + 1 = y − x

16x2y2 + 5 = 6 3√

4x2y + x

Lời giải:

Điều kiện: x, y ≥ −1

2.

Từ phương trình thứ nhất ta có

√2x+ 1 + x =

√2y + 1 + y,

hay

f(x) = f(y) với f(t) =√

2t+ 1 + t.

137

Page 139: Chuyên đề luyện thi đại học môn Toán - VipLam.Net

Dễ dàng nhận thấy rằng f(t) là hàm liên tục và tăng nghiêm ngặt trên

[−1

2, +∞

)nên f(x) =

f(y) sẽ tương đương với x = y.

Thay vào phương trình thứ hai ta được

16x4 + 5 = 63√

4x3 + x. (1)

Do 16x4 + 5 > 0 nên từ đây ta suy ra 4x3 + x > 0, tức x > 0.

Bây giờ, sử dụng bất đẳng thức AM-GM, ta có

3√

4x3 + x = 3√x(4x2 + 1) =

3√

4x · (4x2 + 1) · 22

≤ 4x+ (4x2 + 1) + 2

6.

Kết hợp với (1), ta suy ra

16x4 + 5 ≤ 4x2 + 4x+ 3,

hay tương đương

2(2x2 + 2x+ 1)(2x− 1)2 ≤ 0.

Nhưng điều này chỉ xảy ra khi x = 12.

Mặt khác, ta dễ thấy x = 12thỏa mãn phương trình (1).

Từ đây, ta đi đến kết luận hệ đã cho có nghiệm duy nhất (x, y) =

(1

2,

1

2

).

Bài toán 13 Giải hệ phương trình:x2 + 3 =√

3 |xy|

(2x−√

3y)2 + y2 = 4

Lời giải:

Ta viết lại hệ phương trình như sau:

⇐⇒

x2 + 3 =√

3 |xy|

x2 + y2 = 1 +√

3xy

Mặt khác ta có :

|xy| ≥ xy

Nên

x2 + 3 ≥ x2 + y2 − 1

138

Page 140: Chuyên đề luyện thi đại học môn Toán - VipLam.Net

Vậy suy ra :

y2 ≤ 4 =⇒ |y| ≤ 2

. Sử dụng bất đẳng thức AM −GM ta được:

x2 + 3 ≥ 2√

3 |x| ≥√

3 |xy|

Như vậy ta có đẳng thức xảy ra khi x = ±√

3; y = ±2

Vậy nghiệm của hệ là: (x, y) = ±(√

3, 2) .

Bài toán 14 Giải hệ phương trình: x2 = y + 2

y2 = z + 2

z2 = x+ 2

Lời giải:

Dòng tâm sự:

Bài toán trên được một bạn có nick luong_qt đưa lên diễn đàn toanphothong.vn và tại đây đã

nhận được hai lời giải đầy tính nghệ thuật từ anh Võ Quốc Bá Cẩn.Thật sự khi đọc hai lời giải

này,tác giả thật sự chiêm nghiệm được nhiều điều,mỗi bài toán khi đi ta điều nên có hướng dẫn

dắt,khi đặt bút lên trang giấy suy nghĩ,đòi hỏi ta phải có những cảm nhận thật xác đáng và có

hướng tư duy thật gọn.

• Về lời giải thứ nhất,nét nghệ thuật nằm ở những biến đổi lượng giác kết hợp với đánh giá

bất đẳng thức để đưa ta đến kết quả.Cách làm gọn và quen thuộc,nhưng mấu chốt nằm ở

việc ta quan sát được miền nghiệm bài toán là x, y, z ≥ −2 để ta nghĩ đến việc thêm một

chặn nữa x, y, z ≤ 2 với việc làm trên ta đã đưa khéo x, y, z ∈ [−2; 2] và như một thói

quen dân dã,ta nghĩ đến lượng giác hoá để được về bài toán lượng dễ xử lí hơn.Vậy việc

đặt x = 2 cosα là điều dễ hiểu.

• Lời giải thứ hai là nét tinh ý của người làm toán,với việc nhận ra các mối liên hệ đẳng

thức (x2 − 1) = (x+ 1)(x− 1) x2 − 4 = (x− 2)(x+ 2) với việc nhận ra từng mối liên hệ

như thế và việc hệ ban đầu là đối xứng nên khi ta nhân vào thì chắc hẳn các mối liên hệ

của (x− 1)(y − 1)(z − 1) và (x+ 2)(y + 2)(z + 2) sẽ giúp ta định hướng và giải bài toán

trên đơn giản hơn .

Cách 1 Do x2, y2, z2 ≥ 0 nên ta có y+ 2, z + 2, x+ 2 ≥ 0 (dựa vào các phương trình của hệ),

suy ra x, y, z ≥ −2.

Bây giờ, ta xét hai trường hợp:

1. Nếu −2 ≤ x, y, z ≤ 2

Đặt x = 2 cosα với α ∈ [0, π]. .

139

Page 141: Chuyên đề luyện thi đại học môn Toán - VipLam.Net

Ta có

y = x2 − 2 = 4 cos2 α− 2 = 2 cos 2α.

Suy ra

z = y2 − 2 = 4 cos2 2α− 2 = 2 cos 4α.

Từ đây đưa đến

2 cosα = x = z2 − 2 = 4 cos2 4α− 2 = 2 cos 8α.

Từ đó ta suy ra được:

[8α = α + k2π

8α = −α + k2π⇐⇒

α =k2π

7

α =k2π

9

(k ∈ Z)

Vậy ta suy ra các nghiệm của bài toán là :

(x; y; z) = (2 cosα; 2 cos 2α; 2 cos 4α) với

α =k2π

7

α =k2π

9

k ∈ Z

2. Nếu max{x, y, z} > 2:

Không mất tính tổng quát, giả sử x = max{x, y, z}..Khi đó, ta có x > 2, suy ra

x2 > 2x = x+ x > x+ 2 ≥ y + 2.

Mâu thuẫn với những gì có được từ hệ. Vậy trường hợp này không thể xảy ra.

Cách 2 Ta xét ba trường hợp:

1. Nếu (x+ 1)(y + 1)(z + 1) = 0:

Không mất tính tổng quát, giả sử x+ 1 = 0.

Khi đó ta có x = −1.

Từ phương trình thứ nhất suy ra y = −1.

Từ đây và từ phương trình thứ hai, ta được x = y = z = −1.

2. Nếu (x− 2)(y − 2)(z − 2) = 0:

Không mất tính tổng quát, giả sử x− 2 = 0.

Khi đó ta có x = 2. Từ phương trình thứ nhất suy ra y = 2.

Từ đây và từ phương trình thứ hai, ta được x = y = z = 2.

140

Page 142: Chuyên đề luyện thi đại học môn Toán - VipLam.Net

3. Nếu (x+ 1)(y + 1)(z + 1) 6= 0 và (x− 2)(y − 2)(z − 2) 6= 0:

Chú ý rằng hệ đã cho tương đương với hai hệ phương trình saux2 − 1 = y + 1

y2 − 1 = z + 1

z2 − 1 = x+ 1

x2 − 4 = y − 2

y2 − 4 = z − 2

z2 − 4 = x− 2

Từ hệ thứ nhất, ta suy ra (x2 − 1)(y2 − 1)(z2 − 1) = (x + 1)(y + 1)(z + 1), và do

(x+ 1)(y + 1)(z + 1) 6= 0 nên suy ra

(x− 1)(y − 1)(z − 1) = 1.

Từ hệ thứ hai, ta thu được (x2 − 4)(y2 − 4)(z2 − 4) = (x − 2)(y − 2)(z − 2), và do

(x− 2)(y − 2)(z − 2) 6= 0 nên ta có

(x+ 2)(y + 2)(z + 2) = 1.

Như vậy, trong trường hợp này, ta thu được

(x+ 2)(y + 2)(z + 2)− (x− 1)(y − 1)(z − 1) = 0,

Hay tương đương

xy + yz + zx+ x+ y + z + 3 = 0

Nếu x+ y + z = 0: Do xy + yz + zx+ x+ y + z + 3 = 0 nên xy + yz + zx = −3.

Từ đây và từ đẳng thức

1 = (x− 1)(y − 1)(z − 1) = xyz − (xy + yz + zx) + (x+ y + z)− 1,

ta thu được xyz = −1. Vậy ta cóx+ y + z = 0

xy + yz + zx = −3

xyz = −1

Suy ra x, y, z là ba nghiệm của phương trình

X3 − 3X + 1 = 0.

Dễ dàng giải phương trình này, từ đó tìm được x, y, z. Thử lại vào hệ ban đầu để loại

nghiệm. Nếu x+y+z = −1: Do xy+yz+zx+x+y+z+3 = 0 nên xy+yz+zx = −2.

Từ đây và từ đẳng thức

1 = (x− 1)(y − 1)(z − 1) = xyz − (xy + yz + zx) + (x+ y + z)− 1,

141

Page 143: Chuyên đề luyện thi đại học môn Toán - VipLam.Net

ta thu được xyz = 1. Vậy ta cóx+ y + z = −1

xy + yz + zx = −2

xyz = 1

Suy ra x, y, z là ba nghiệm của phương trình

X3 +X2 − 2X − 1 = 0.

Dễ dàng giải phương trình này, từ đó tìm được x, y, z. Thử lại vào hệ ban đầu để

loại nghiệm.

Bài toán 15 Giải hệ phương trình:√x2 + 4 +

√x2 − 2xy + y2 + 1 +

√y2 − 6y + 10 = 5

log3 8xyz3 = 10 log9 z2 −

(log3

3x2z

y

)2

Lời giải:

Xét phương trình:√x2 + 4 +

√x2 − 2xy + y2 + 1 +

√y2 − 6y + 10 = 5

Ta áp dụng bất đẳng thức Minkowski, ta có:{√x2 + 4 +

√(y − x)2 + 1 ≥

√y2 + 9√

y2 + 9 +√

(3− y)2 + 1 ≥ 5

=⇒√x2 + 4 +

√x2 − 2xy + y2 + 1 +

√y2 − 6y + 10 ≥ 5

Dấu bằng xảy ra khi và chỉ khi:x

y − x= 2

y

3− y= 3

=⇒

x =

3

2

y =9

4

Vậy hệ có nghiệm là (x; y) = (3

2;9

4)

Bài toán 16 Giải hệ phương trình:x2 − yz

2x2 + y2 + z2+

y2 − xz2y2 + x2 + z2

+z2 − xy

2z2 + y2 + x2= 0

x2y + y2z −(x√y

√z

+ 2√yz + 4

√xz

)= 5

Lời giải:

142

Page 144: Chuyên đề luyện thi đại học môn Toán - VipLam.Net

Ta sẽ chứng minh

x2 − yz2x2 + y2 + z2

+y2 − xz

2y2 + x2 + z2+

z2 − xy2z2 + y2 + x2

≥ 0

Để ý rằng:2(x2 − yz)

2x2 + y2 + z2= 1− (y + z)2

2x2 + y2 + z2

Thiết lập các biểu thức còn lại tương tự. Ta quy về chứng minh

3 ≥ (y + z)2

2x2 + y2 + z2+

(x+ z)2

2y2 + z2 + x2+

(x+ y)2

2z2 + x2 + y2

Áp dụng bất đẳng thức Cauchy Schwarz ta có∑ (y + z)2

(x2 + y2) + (y2 + z2)≤∑

(y2

x2 + y2+

z2

z2 + y2) = 3

Vậy bất đẳng thức được chứng minh. Đẳng thức xảy ra khi và chỉ khi x = y = z

Thay vào hệ (2) ta có 2y3 = 12 ⇐⇒ y = 3√

6

Vậy hệ có nghiệm x = y = z =3√

6 .

Bài toán 16 Giải hệ phương trình:x2 + y2 = 1

125y5 − 125y3 + 6√

15 = 0

Lời giải:

Ta biến đổi phương trình thứ hai về dạng

y3(1− y2) =6√

15

125⇐⇒

y > 0

y6x4 =4.32

55

Áp dụng bất đẳng thức AM −GM ta được:

3 = 3(x2 + y2) = y2 + y2 + y3 +3

2x2 +

3

2x2 ≥ 5

5

√y6

9

4x4 ⇐⇒ y6x4 ≤ 4.32

55

Từ đó ta có y2 =3

2x2.Do đó ta tìm được nghiệm của hệ là : (x; y) = ((

√10

5;

√10

5); (−

√10

5;−√

10

5))

143

Page 145: Chuyên đề luyện thi đại học môn Toán - VipLam.Net

Bài toán 17 Giải hệ phương trình:xyz = 32

x2 + 4xy + 4y2 + 2z2 = 96

x, y, z > 0

(?)

Lời giải:

Theo bất đẳng thức AM −GM ta có :

x2+4xy+4y2+2z2 = x2+z2+z2+4y2+4xy ≥ 2xz+4yz+4xy ≥ 3 3√

2.4.4x2.y2.z2 = 33√

322 = 96

Từ đó suy ra

x2 + 4xy + 4y2 + 2z2 ≥ 96

Đẳng thức xảy ra khi và chỉ khi

x = z

z = 2y

2xz = 4yz = 4xy

⇐⇒

x = 2y

z = 2y

Thế vào hệ (?) ta được:

2y.y.2y = 32 =⇒ y = 2 =⇒

x = 4

z = 4

Vậy nghiệm của hệ là (4, 2, 4) .

Bài toán 18 Giải hệ phương trình:x2 + y2 = 2 (1)

z2 + 2z(x+ y) = 8 (2)

z(x− y) = 4√

3 (3)

Lời giải:

Từ (1) và (2) ta có :

2x2 + 2y2 + z2 + 2xz + 2xy = 12 ⇐⇒(x+

z

2

)= 6.

Áp dụng bất đẳng thức Cauchy Schwarz ta có :

(zy − zx)2 ≤[(2y)

(x+

z

2

)+ (−2x)

(y +

z

2

)]2

≤ (4x2 + 4y2)[

(x+

z

2

)2

+

(y +

z

2

)2

]

144

Page 146: Chuyên đề luyện thi đại học môn Toán - VipLam.Net

Suy ra (zy − zx)2 ≤ 8.6 =⇒ zy − zx ≤ 4√

3

Do (3) nên ta có

zy − zx ≥ 0 (4)

x+z

22y

=y +

z

2−2x

(5)

(5) ⇐⇒ 2x2 + 2y2 = −z(x+ y) ⇐⇒ z(x+ y) = −4 (6)

Thay vào (2) ta có z2 = 16 ⇐⇒ z = ±4

• Nếu z = 4

Ta có :

x+ y = −1

x2 + y2 = 2⇐⇒

x =−1 +

√3

2

y =−1−

√3

2

(loại) hoặc

x =−1−

√3

2

y =−1 +

√3

2

• Nếu z = −4

Ta có :

x+ y = 1

x2 + y2 = 2⇐⇒

x =

1 +√

3

2

y =1−√

3

2

hoặc

x =

1−√

3

2

y =1 +√

3

2

(loại)

Vậy hệ có hai nghiệm:

x =−1−

√3

2

y =−1 +

√3

2z = 4

hoặc

x =

1 +√

3

2

y =1−√

3

2z = −4

Bài toán 19 Giải hệ phương trình:√x− x2 +

√y − y2 +

√z − z2

(x− y)(y − z)(z − x)(x+ y + z) =2√

3

9

Lời giải:

Điều kiện:

x− x2 ≥ 0

y − y2 ≥ 0

z − z2 ≥ 0

⇐⇒

0 ≤ x ≤ 1

0 ≤ y ≤ 1

0 ≤ z ≤ 1

Giả sử x = Maxx, y, z =⇒

x ≥ y

x ≥ z,có hai trường hợp:

• Nếu x ≥ y ≥ z

(x− y)(y − z)(z − x)(x+ y + z) ≤ 0 suy ra hệ vô nghiệm

• Nếu x ≥ z ≥ y

Ta chứng minh: P = (x− y)(y − z)(z − x)(x+ y + z) ≤ 2√

3

9

145

Page 147: Chuyên đề luyện thi đại học môn Toán - VipLam.Net

Thật vậy ta có :

4P = 4(x− y)(y − z)(z − x)(x+ y + z) ≤ 4(z − y)(x− z)(x+ y + z)

= 2(z − y)[(√

3 + 1)(x− z)][(√

3− 1)(x+ y + z)]

≤ 1

27

[2(z − y) + (

√3 + 1)(x− z) + (

√3− 1)(x+ y + z)

]3

=1

27

[2√

3x− (3−√

3)y

]3

≤ 1

27(2√

3x)3 ≤ (2√

3)3

27=

8√

3

9⇐⇒ P ≤ 2

√3

9

Dấu đẳng thức tức xảy ra khi

x = 1

y = 0

z =1√3

Do đó:(2) thế vào (1) ta thấy nghiệm này thoả(1).

Tương tự cho các trường hợp còn lại .

Vậy hệ có ba nghiệm là

x = 1

y = 0

z =1√3

;

x = 0

y =1√3

z = 1

;

x =

1√3

y = 1

z = 0

Bài toán 20 Giải hệ phương trình:2x2 = y(x2 + 1)

3y3 = z(y4 + y2 + 1)

4z4 = x(z6 + z4 + z2 + 1)

Lời giải:

• Trường hợp 1

Với x = 0 thì hệ có nghiệm x = y = z = 0

• Trường hợp 2

Với x 6= 0 để hệ có nghiệm thì x > 0, y > 0, z > 0 Gỉa sử (x, y, z) là nghiệm của hệ ta có :2x2 = y(1 + x2) ≥ 2xy ⇐⇒ x ≥ y (vì x2 + 1 ≥ 2x)

3y3 = z(y4 + y2 + 1) ≥ 3zy2 ⇐⇒ y ≥ z (vì y4 + y2 + 1 ≥ 3y2)

4z4 = x(z6 + z4 + z2 + 1) ≥ x4z3 ⇐⇒ z ≥ x (vì z6 + z4 + z2 + 1)

Từ hệ suy ra x = y = z

146

Page 148: Chuyên đề luyện thi đại học môn Toán - VipLam.Net

Thay vào hệ ban đầu ta được hai nghiệm của hệ là x = y = z = 0 ; x = y = z = 1 .

Bài toán 21 Giải hệ phương trình:xy +√

2(x4 + y4) = 1 (1)

xn+4yn + xnyn+4 =2

3n+2(2)

n là số nguyên dương lẻ,khác 1

Lời giải:

Từ (2) biến đổi ta được :(xy)n(x4 + y4) =2

3n+2=⇒ xy > 0

Khi đó từ (1) với lưu ý :2(x4 + y4) ≥ 4x2y2,suy ra :

0 < xy ≤ 1

3

x4 + y4 =(1− xy)2

2Kết hợp (1) với (2),ta có :

(xy)n(x4 + y4) = (xy)n−1xy.(1− xy)2

2

Áp dụng bất đẳng thức AM −GM ta có

(xy)n−1xy.(1− xy)2

2≤ 2.

(1

3

)n−1(xy +

(1

2− xy

2

)+

(1

2− xy

2

)3

)3

=2

3n+2

Đẳng thức xảy ra khi và chỉ khi

xy =1

3x = y

⇐⇒

x = y =

√3

3

x = y = −√

3

3

Vậy hệ đã cho có các nghiệm là x = y =

√3

3x = y = −

√3

3.

Bài toán 22 Giải hệ phương trình:x1 + x2 + ....x2009 = 2009

x81 + x8

2 + ...+ x82009 = x6

1 + x62 + ...+ x6

2009

Lời giải:

Giả sử(x1, x2, ..., x2009) là một nghiệm của hệ.

Không giảm tính tổng quát giả sử x21 ≥ x2

2 ≥ .... ≥ x22009

Áp dụng bất đẳng thức Cauchy Schwarz ta có :

2009

(x2

1+x22+....+x2

2009

)≥(x1+x2+....x2009

)2

= 20092 ⇐⇒ x21+x2

2+....+x22009 ≥ 2009 (1)

147

Page 149: Chuyên đề luyện thi đại học môn Toán - VipLam.Net

Áp dụng bất đẳng thức Chebyshev cho các bộ số (x21, x

22, ..., x

22009) và (x6

1, x62, ..., x

62009) ta có(

x21 + x2

2 + ....+ x22009

)(x6

1 + x62 + ....+ x6

2009

)≤ 2009

(x8

1 + x82 + ....+ x8

2009

)(2)

Từ (1) và (2) suy ra

x81 + x8

2 + ...+ x82009 ≥ x6

1 + x62 + ...+ x6

2009

Đẳng thức xảy ra tại x1 = x2 = ... = x2009 = 1.

Vậy nghiệm của hệ là x1 = x2 = ... = x2009 = 1 .

Bài toán 23 Giải hệ phương trình:xy(x+ y) = x2 − xy + y2

1

x3+

1

y3= 16

Lời giải:

Điều kiện:x, y 6= 0

Đặt a =1

x; b =

1

yTừ giả thiết ta suy ra

1

ab

(1

a+

1

b

)=

1

a2− 1

ab+

1

b2⇐⇒ a+ b = a2 − ab+ b2

1

x3+

1

y3= a3 + b3 = (a+ b)(a2 − ab+ b2) = (a+ b)2

Ta có bất đẳng thức cơ bản sau:

a2 − ab+ b2 ≥ (a+ b)2

4=⇒ a+ b ≥ (a+ b)2

4⇐⇒ 0 ≤ a+ b ≤ 4

=⇒ 1

x3+

1

y3= (a+ b)2 ≤ 16

Đẳng thức xảy ra khi a = b hay x = y =1

2

Vậy nghiệm của hệ là x = y =1

2.

Bài toán 24 Giải hệ phương trình:1√x

+1√y

+1√z

= 3√

3

x+ y + z = 1

xy + yz + zx =7

27+ 2xyz (?)

148

Page 150: Chuyên đề luyện thi đại học môn Toán - VipLam.Net

Lời giải:

Ta có

(?) ⇐⇒ xy + yz + zx− 2xyz =7

27

Lại có

xyz ≥ (x+ y − z)(y + z − x)(z + x− y) = (1− 2x)(1− 2y)(1− 2z)

=⇒ 9xyz ≥ 4(xy + yz + zx)− 1 ⇐⇒ 2xyz ≥ 8(xy + yz + zx)− 2

9

Ta phải chứng minh xy + yz + zx ≤ 1

3=

(x+ y + z)2

3đúng.

Đẳng thức xảy ra khi x = y = z =1

3.

Vậy nghiệm của hệ là x = y = z =1

3.

Bài toán 25 Giải hệ phương trình:√x+ a+

√y + a+

√z + a = 3

√a2 + 1

a√a− x+

√a− y +

√a− z = 3

√a2 − 1

a

a>1 và a là hằng số

Lời giải:

Đặt A =√x+ a+

√y + a+

√z + a,B =

√a− x+

√a− y +

√a− z

Áp dụng bất đẳng thức Cauchy Schwarz ta có :A2 ≤ 3(3a+ x+ y + z)

B2 ≤ 3(3a− x− y − z)

Cộng hai vế của hai bất đẳng thức trên lại ta có:

A2 +B2 ≤ 18a (?)

Mặt khác theo giả thiết ta lại có:

A2 +B2 = 9

(a2 + 1

a+a2 − 1

a

)= 18a

Dấu đẳng thức trong hệ (?) xảy ra khi và chỉ khi x = y = z =1

a.

Vậy nghiệm của hệ là x = y = z =1

a

149

Page 151: Chuyên đề luyện thi đại học môn Toán - VipLam.Net

Bài toán 26 Giải hệ phương trình:√

1 + x1 +√

1 + x2 + ....+√

1 + x2013 = 2013

√2014

2013√

1− x1 +√

1− x2 + ....+√

1− x2013 = 2013

√2014

2013

Điều kiện:−1 ≤ xi ≤ 1 (i = 1, 2, 3, ..., 2013).Ta có

20132 2014

2013=

(√1 + x1+

√1 + x2+

√1 + x3+....+

√1 + x2013

)2

≤ 2013(2013+x1+x2+...+x2013)

Suy ra x1 + x2 + ...+ x2013 ≥ 1 (1)

Lại có

20132 2014

2013=

(√1− x1+

√1− x2+

√1− x3+....+

√1− x2013

)2

≤ 2013(2013−x1−x2−...−x2013)

Suy ra x1 + x2 + ...+ x2013 ≤ 1 (2)

Từ (1) và (2) suy ra x1 + x2 + ...+ x2013 = 1

Do đó hệ phương trình trở thành1 + x1 = 1 + x2 = .... = 1 + x2013

1− x1 = 1− x2 = .... = 1− x2013

x1 + x2 + ...+ x2013 = 1

⇐⇒ x1 = x2 = ... = x2013 =1

2013

Vậy nghiệm của hệ là x1 = x2 = ... = x2013 =1

2013.

IV. SÁNG TẠO HỆ QUA CÁC BÀI TOÁN BẤT ĐẲNG THỨC

Bài toán Tìm nghiệm dương của hệ sau:x+ y +

√2x2 + 2xy + 3y2 = 4

x2y = 32

(√5

3− 1

)3

Bài toán trên được xuất phát từ bài toán sau đây:

Cho x, y là các số thực dương thoả mãn x+ y +√

2x2 + 2xy + 3y2 = 4.

Tìm giá trị lớn nhất của của biểu thức x2y.

Bài này được Michael Rozenberg đưa lên trang www.mathlink.ro và được đề cập đến trong sách

"Phân loại & phương pháp giải toán bất đẳng thức" của các tác giả Vasile Cirtoaje,Võ

150

Page 152: Chuyên đề luyện thi đại học môn Toán - VipLam.Net

Quốc Bá Cẩn,Trần Quốc Anh.Bài toán hay và có nhiều điều lí thú xung quanh bài này,tuy

nhiên khuôn khổ bài viết không cho phép chúng tôi bàn đến vẫn đề này.Quay trở lại hệ đã

cho,khi quan sát chắc hẳn ta có thể nghĩ ngay đến việc sử dụng bất đẳng thức để giải quyết bài

toán này.Nhưng đây không có ý gì là hay khi sáng tạo hệ kiểu này,ta tìm một bất đẳng thức có

điều kiện sau đó dùng cực trị của nó làm một phương trình trong hệ thì chắc hẳn ta có thể tạo

ra hằng trăm bài như thế.Việc quan trọng là ta khéo léo che đi cái giá trị lớn nhất nhìn không

đẹp mắt này.

Ta nghĩ đến việc thêm vào một ẩn số z =

√5

3− 1 vậy bài toán nhìn sẽ đẹp hơn và đầy thú

vị.Để ý rằng đẳng thức xảy ra tại x = 2y = 4z vậy ta có thể tìm thêm các mối liên hệ các biến

x, y, z với nhau.

Từ đây ta có thể tạo ra những hệ phương trình khó từ bài toán đặc biệt trên

Bài toán 1 Giải hệ phương trình sau trên tập số dương :x+ y +

√2x2 + 2xy + 3y2 = 4 (1)

x2y = 32z3 (2)

3xz2 + 6xz + 3z3 + 15z2 + 16z = 2x+ 6 (3)

Lời giải:

1. Từ giả thiết đã cho ta dễ dàng nhận thấy x + y < 4.Do đó phương trình (1) có thể viết

thành

2x2 + 2xy + 3y2 = (4− x− y)2

Từ đây bằng một số biến đổi ,ta có

(x+ y)2 + 2(y + 2)2 = 40,

hay

2

(2 +

x

2

)2

+ (y + 2)2 = 20

Tới đây sử dụng lần lượt các bất đẳng thức AM −GM và Holder ,ta có

(2 +

x

2

)2

+

(2 +

x

2

)2

+ (y + 2)2 ≥ 3

[(2 +

x

2

)2

(2 + y)

]2

3 ≥(

2 +3

√x2y

4

)2

Từ đó suy ra

20 ≥ 3

(2 +

3

√x2y

4

)2

,

hay

x2y ≤ 32

(√5

3− 1

)3

151

Page 153: Chuyên đề luyện thi đại học môn Toán - VipLam.Net

Từ (3) ta biến đổi về

[3(z + 1)2 − 5][x+ z + 3] = 0

Vậy ta có z =

(√5

3− 1

)Suy ra x2 ≤ 32z3.

Từ đó suy ra nghiệm của hệ là x = 2y = 4z = 4

(√5

3− 1

)2. Bằng cách đó ta tiếp tục biến đổi phương trình thứ (3) cho khó hơn,ta có thể đưa về việc

xét hàm như sau: f (3z2 + 5z) = f (2− z)

Vậy ta sẽ đưa về 3√

3z2 + 5z + 3z2 + 5z = (2− z)3 + 2− z

⇐⇒ 3√

3z2 + 5z+3z2+5z = 8−12z+6z2−z3+2−z ⇐⇒ z3−3z2+18z+3√

3z2 + 5z−10 = 0

Vậy là ta có được một hệ mới làx+ y +

√2x2 + 2xy + 3y2 = 4

x2y = 32z3

z3 − 3z2 + 18z + 3√

3z2 + 5z − 10 = 0

Ghi chú

Với những hướng như trên các bạn có thể tạo ra nhiều hệ hay hơn và khó hơn nửa,làm cho bài

toán càng đẹp hơn.

Đi từ các bài toán có vẻ không hay lắm,thế ta đi từ các bổ đề vậy.

Trong sách "Sử dụng phương pháp Cauchy Schwarz để chứng minh bất đẳng thức "

của các anh Võ Quốc Bá Cẩn và Trần Quốc Anh trang 194 có bổ đề nhỏ là

Nếu x ≥ 4y thì√x+√y ≥√x+ 5y

Vậy ta có ý tưởng lại tạo ra một phương trình có điều kiện là x ≥ 4y phương trình còn lại

của hệ sẽ sử dụng bất đẳng thức trên.Nhưng nếu để như thế thì lộ quá,ta sẽ biến đổi.

• Thay x bởi x2 − xy + y2

• Thay y bởi y2 + 2x

Vậy ta có hệ phương trình là:

Bài toán 2 Giải hệ phương trình sau :√x2 − x(8 + y)− 3y2 +

√x2 − 3x+ y2 + 9 = 7√

x2 − xy + y2 +√y2 + 2x =

√x2 − x(10 + y) + 6y2

152

Page 154: Chuyên đề luyện thi đại học môn Toán - VipLam.Net

CHUYÊN ĐỀ 8

THỂ TÍCH VÀ KHOẢNG CÁCH

(Nguyễn Trung Kiên)

Trong kỳ thi TSĐH hình không gian không phải là bài toán khó nhưng nó luôn làm cho không

ít học sinh bối rối, đặc biệt là các câu hỏi liên quan đến tính khoảng cách. Qua những bài toán

đề cập trong chuyên đề nhỏ này hy vọng sẽ giúp học sinh tháo gỡ những khó khăn trên để tự

tin hơn trong kỳ thi TSĐH sắp tới.

? Để tính thể tích một khối đa diện ta có thể dựa vào các cách sau:

Cách 1: Xác định đường cao của khối chóp . Sau đó dùng các công thức V =1

3B.h Đối

với khối chóp. Hoặc V = B.h đối với khối lăng trụ.

Điểm mấu chốt trong bài toán này là học sinh phải biết linh hoạt trong việc chọn ”đỉnh ”

và ”đáy” của khối đa diện để tạo ra khối chóp, lăng trụ mà việc nhận diện đường cao được dễ

nhất.

Cách 2: Dùng công thức tỷ số thể tích, hoặc chia khối đa diện thành các khối nhỏ có hình dạng

đặc biệt để tính thể tích.

? Để tính khoảng cách ta có thể dựa vào các cách sau:

Cách 1 Quy về bài toán cơ bản :” Tính khoảng cách từ chân đường cao đến một mặt bên

của khối chóp”. Để làm tốt phần này học sinh cần nắm chắc bài toán cơ bản và các tính chất:

- Bài toán cơ bản: Cho hình chóp SABC có SA⊥(ABC). Tính khoảng cách từ A đến mặt

phẳng (SBC)

Ta sẽ xác định chân đường cao hạ từ A lên mặt phẳng (SBC) theo cách:

Hạ AM⊥BC, AH⊥SM =⇒ AH⊥(SBC) =⇒ dA/(SBC) = AH =SA.AM√SA2 + AM2

153

Page 155: Chuyên đề luyện thi đại học môn Toán - VipLam.Net

- Các tính chất hình học cần nắm chắc khi tính khoảng cách:

+ Nếu đường thẳng (∆) song song với mặt phẳng (P ) thì khoảng cách từ mọi điểm thuộc (∆)

đến mặt phẳng (P ) là bằng nhau và bằng khoảng cách từ (∆) đến mặt phẳng (P ).

+ Nếu−−→MA = k

−−→MB, M ∈ (P ) thì dA/(P ) = |k|dB/(P )

+ Nếu hai đường thẳng a, b chéo nhau thì khoảng cách từ a đến b bằng khoảng cách từ đường

thẳng a đến mặt phẳng (P ) . Trong đó (P ) chứa đường thẳng b và (P )//a. Sau đó ta quy

khoảng cách giữa đường thẳng a và mặt phẳng (P ) về bài toán cơ bản.

Cách 2 . Dựa vào công thức: V =1

3B.h =⇒ h =

3V

B

Cách 3 . Dùng phương pháp tọa độ hóa.

Trong phạm vi bài viết nhỏ này tôi sẽ trình bày cách 1: Tư tưởng xuyên suốt

là quy về bài toán cơ bản để làm rõ bản chất các câu hỏi liên quan đến khoảng

cách.

Ví dụ 1. Cho hình chóp SABC có đáy ABC là tam giác đều cạnh bằng a . Chân đường

cao hạ từ S lên mặt phẳng (ABC) là điểm H thuộc AB sao cho−−→HA = −2

−−→HB . Góc tạo bởi

SC và mặt phẳng (ABC) bằng 600 . Tính thể tích khối chóp SABC và khoảng cách giữa hai

đường thẳng SA,BC theo a

Lời giải

Tính thể tích:

Vì SH⊥(ABCD) nên HC là hình chiếu vuông góc của SC lên mặt phẳng (ABCD) .

Góc tạo bởi SC và mặt phẳng (ABCD) là SCH = 600.

Xét tam giác BHC theo định lý hàm số cosin ta có:

HC2 = HB2 +BC2 − 2HB.BC. cos HBC = HB2 +BC2 − 2HB.BC. cos 600

⇐⇒ HC2 =a2

9+ a2 − 2.

a

3.a.

1

2=

7a2

9

Suy ra HC =a√

7

3=⇒ SH = HC. tan SCH =

a√

7

3.√

3 =a√

21

3.

154

Page 156: Chuyên đề luyện thi đại học môn Toán - VipLam.Net

Ta suy ra VSABC =1

3SH.S∆ABC =

1

3

a√

21

3.1

2a.a. sin 600 =

√7a3

12( ĐVTT)

Tính khoảng cách:

Gọi E là trung điểm của BC , D là đỉnh thứ tư của hình bình hành ABCD.

Ta có AD//BC nên dSA/BC = dBC/(SAD) = dB/(SAD)

”Trong ví dụ này để tính khoảng cách ta cần dựa vào các tính chất để chuyển bài toán thành

khoảng cách từ chân đường cao H đến mặt phẳng (SAD) (Bài toán cơ bản)”

Mặt khác ta có BA =3

2HA,A ∈ (SAD) ta suy ra:

dSA/BC = dBC/(SAD) = dB/(SAD) =3

2dH/(SAD)

Kẻ

{HF⊥ADHK⊥SF

=⇒ HK⊥(SAD) =⇒ dH/(SAD) = HK

Trong tam giác vuông SHF ta có1

HK2=

1

HF 2+

1

HS2=⇒ HK =

HF.HS√HS2 +HF 2

Mặt khác ta có HF =2

3AE =

2

3

a√

3

2=

√3a

3.

Suy ra HK =HF.HS√HS2 +HF 2

=

√3a3.a√

213√

39a2 + 21

9a2

=

√42

12a. Vậy dSA/BC =

3

2.

√42

12a =

√42

8a

Ví dụ 2. Cho hình chóp SABCD có đáy ABCD là hình chữ nhật với AB = 2a . Mặt bên

SAB là tam giác đều và nằm trong mặt phẳng vuông góc với đáy . Biết AC vuông góc với

SD tính thể tích khối chóp SABCD và khoảng cách giữa hai đường thẳng BD và SC

Lời giải

155

Page 157: Chuyên đề luyện thi đại học môn Toán - VipLam.Net

Tính thể tích:

Gọi H là trung điểm AB,O là giao điểm của hai đường chéo hình chữ nhật ABCD ;

SAB là tam giác đều và nằm trong mặt phẳng vuông góc với đáy

=⇒ SH⊥(ABCD) và SH =AB√

3

2= a√

3.

Gọi M là trung điểm của SB thì góc tạo bởi OM và AC cũng là góc tạo bởi SD và AC . Suy

ra MOC = 900

Ta có BC⊥(SAB) =⇒ BC⊥SB =⇒ MC2 = BC2 +MB2 = BC2 + a2.

OM2 = 14SD2 = 1

4SC2 = 1

4(BC2 + 4a2), OC2 = 1

4AC2 = 1

4(BC2 + 4a2)

Như vậy tam giác MOC vuông cân tại O

=⇒ MC2 = 2OC2 ⇐⇒ BC2 + a2 = 12(BC2 + 4a2) ⇐⇒ BC2 = 2a2 =⇒ BC = a

√2.

Thể tích hình chóp SABCD là

VS.ABCD =1

3SABCD.SH =

1

3AB.AD.SH =

1

32a.a√

2.a√

3 =2a3√

6

3Tính khoảng cách.

Gọi N là trung điểm của N thì SC//(BDN) =⇒ dSC/BD = dSC/(BDN) = dC/(BDN) = dA/(BDN)

Ta thấy rằng mối liên hệ giữa mặt phẳng (BMN) và chân đường cao H của khối chóp không

có dấu hiệu của bài toán cơ bản. Để tạo ra bài toán cơ ta sẽ dựng một đường cao liên quan trực

tiếp đến mặt phẳng (BMN) như sau.

Kẻ NK//SH =⇒ NK⊥(ABCD) =⇒ dA/(BDN) = 2dK/(BDN)

Kẻ

{KE⊥BDKF⊥NE

=⇒ KF⊥(BDN) =⇒ dK/(BDN) = KF =KE.KN√KE2 +KN2

Có KN =a√

3

2, KE =

3

4AQ =

3

4.

AB.AD√AB2 + AD2

=

√6a

4

Thay số ta tính được KF =a√

6

6

Vậy d (BD,SC) = 2KF =a√

6

3

156

Page 158: Chuyên đề luyện thi đại học môn Toán - VipLam.Net

Ví dụ 3 Cho hình chóp S.ABCD có đáy ABCD là hình bình hành, mặt phẳng (SBD)

vuông góc với đáy, các đường thẳng SA, SD hợp với đáy một góc 30o. Biết AD = a√

6,

BD = 2a và góc ADB = 45o. Tính thể tích khối chóp S.ABCD và khoảng cách từ đỉnh C

đến mặt phẳng (SAD) theo a.( Trích đề thi thử đại học số 2 của diễn đàn k2pi.net)

Lời giải

Tính thể tích:

Gọi O là tâm khối chóp. Hạ SH ⊥ BD =⇒ SH ⊥ (ABCD). suy ra SAH = SDH = 30o suy

ra HA = HD

nên tam giác AHD vuông cân tại H. =⇒ HA = HD =AD√

2= a√

3. =⇒ SH =

HD. tan(30o) = a.

Ta có . Diện tích đáy S = AD.BD. sin(45o) = 2a2.√

3 nên VS.ABCD =2a3.√

3

3Tính khoảng cách:

Để tính khoảng cách từ điểm C đến mặt phẳng (SAD) ta phải tìm cách quy về khoảng cách

từ chân đường cao H đến mặt phẳng (SAD). (Bài toán cơ bản)

Ta có: d(C/(SAD)) = 2d(O/(SAD)) =2√3d(H/(SAD)) Gọi K là trung điểm của AD.

suy ra HK ⊥ AD =⇒ AD ⊥ (SHK) Hạ HT ⊥ SK suy ra HT = d(H; (SAD)). Ta có :

HK =AD

2=a√

6

2.

Áp dụng hệ thức lượng trong tam giác SHK ta có :1

SH2+

1

HK2=

1

HT 2=⇒ HT =

a√

15

5

Vây d(C; (SAD)) =2√3.a√

15

5=

2a√

5

5

157

Page 159: Chuyên đề luyện thi đại học môn Toán - VipLam.Net

Ví dụ 4. Cho hình lăng trụ ABCA′B′C ′ có đáy ABC là tam giác vuông cân tại A cạnh

huyền BC = a√

2 cạnh bên AA′ = 2a, biết A′ cách đều các đỉnh A,B,C . Gọi M,N lần lượt

là trung điểm của AA′, AC. Tính thể tích khối chóp C ′MNB và khoảng cách từ C ′ đến mặt

phẳng (MNB)

Lời giải

Tính thể tích:

Vì A′ cách đều A,B,C nên chân đường cao hạ từ A′ lên mặt phẳng (ABC) là tâm vòng tròn

ngoại tiếp tam giác ABC. Gọi H là trung điểm của BC suy ra A′H⊥(ABC)

Gọi K = MN ∩ AC ′ =⇒ AK = 13C ′K =⇒ VC′MNB = 3VAMNB

Gọi E là trung điểm của AH =⇒ ME⊥(ABC) =⇒ VMANB = 13ME.dt(ANB)

Tính được: ME =1

2A′H =

1

2

a√

14

2=a√

14

4Suy ra: VMANB =

1

3.a√

14

4.a2

4=

√14a3

48.

Vậy VC′MNB =

√14a3

16Tính khoảng cách: Ta thấy rằng việc tính trực tiếp khoảng cách từ điểm C ′ đến mặt phẳng

(BMN) là tương đối khó. Để khắc phục khó khăn này ta sẽ tạo ra bài toán cơ bản tính khoảng

cách từ chân đường cao đến mặt phẳng (BMN) bằng cách dựng đường cao ME của khối chóp

ABMN . Ta có : dC′/(BMN) = 3dA/(BMN) .

Gọi F là trọng tâm tam giác ABC.

Ta có: AF =2

3AH;EH =

1

2AH =⇒ EF +

1

3AH =

1

2AH =⇒ EF =

1

6AH =⇒ dA/(BMN) =

4dE/(BMN)

Như vậy dC′/(BMN) = 3dA/(BMN) = 12dE/(BMN)

Hạ

{EP⊥BNEQ⊥MP

=⇒ EQ⊥(MNB) =⇒ dE/(MNB) = EQ = EP.EM√EP 2+EM2

Ta có ∆EPF đồng dạng với ∆BHF =⇒ EP

BH=EF

BF=⇒ EP =

BH.EF

BF

Tính được BH =a√

2

2; EF = 1

4AF = 1

4.23AH = 1

6AH = a

√2

12; BF = a

√5

3

158

Page 160: Chuyên đề luyện thi đại học môn Toán - VipLam.Net

Suy ra: EP =a√

5

20=⇒ EQ =

EP.EM√EP 2 + EM2

=

√994a

284

Vậy dC′/(BMN) = 12dE/(BMN)

= 12.

√994a

284=

3√

994a

71

Ví dụ 5. Cho hình lăng trụ ABCA′B′C ′ có đáy ABC là tam giác đều cạnh bằng a, cạnh

bên AA′ =√

3a, Gọi O,M lần lượt là trung điểm của BC và A′C ′, biết tam giác AA′O cân

tại A′ và nằm trong mặt phẳng vuông góc với đáy. Hãy tính thể tích khối chóp MBCC ′B′ và

khoảng cách giữa hai đường thẳng CM,AB′ theo a

Lời giải

Tính thể tích

GọiH là trung điểm củaAO từ giả thiết ta suy raA′H⊥(ABC) . Ta có VMBCC′B′ =1

2VA′BCC′B′ =

1

3VLT

Mặt khác VLT = A′H.S4ABC =√AA′2 − AH2.

√3a2

4=

√3a2 − 3a2

16

√3a2

4=

3√

15a3

16(đvtt.)

Vậy VMBCC′B′ =

√15a3

16(đvtt)

Tính khoảng cách:

Đây là câu hỏi tương đối khó. Đầu tiên ta sẽ dựng mặt phẳng chứa AB′ song song với CM .

Ta dựng khối hộp ABDCA′B′D′C ′ như hình vẽ và gọi N là trung điểm của BD thì B′N//CM

do đó mặt phẳng (AB′N) song song với CM .

159

Page 161: Chuyên đề luyện thi đại học môn Toán - VipLam.Net

Như vậy dCM/AB′ = dCM/(AB′N) = dC/(AB′N). Để tạo ra bài toán cơ bản ta sẽ dựng đường cao

B′K vuông góc với đáy (ABC) . Dễ thấy A′HBK là hình chữ nhật.

Bây giờ ta cần quy khoảng cách từ C đến (AB′N) thành khoảng cách từ điểm K đến mặt

phẳng (AB′N).

Ta có dC/(AB′N) = 2dB/(AB′N) = 2dD/(AB′N) = 8dH/(AB′N)

Vì HJ =1

3KJ =⇒ dH/(AB′N) =

1

3dK/(AB′N) =⇒ dC/(AB′N) =

8

3dK/(AB′N)

Kẻ KF⊥AN, KP⊥B′F =⇒ KP⊥(AB′N) =⇒ dK/(AB′N) = KP =KF.KB′√KF 2 +KB′2

Ta cần tính KF . Dựa vào hình vẽ ta thấy KF = 3HT .

Mà HT =1

2dO/AI =

1

2

AO.OI√AO2 +OI2

=a√

21

56

Suy ra KF =3a√

21

56. Thay số vào KP với chú ý KB′ = A′H =

3√

5a

4

ta thu được:KP =9a

224

√135135

448

Vậy dCM/AB′ = dCM/(AB′N) = dC/(AB′N) =3a

28

√135135

448

Ví dụ 6. Cho hình lăng trụ ABC.A′B′C ′ có đáy ABC là tam giác vuông tại A, AB =

a, AC = a√

3. Gọi H, M lần lượt là trung điểm của BC,CC ′. Biết A′ cách đều các đỉnh

A, B, C. Góc tạo bởi đường thẳng A′B và mặt phẳng (A′AH) bằng 300. Tính thể tích lăng

trụ ABCA′B′C ′ và khoảng cách giữa hai đường thẳng A′B và AM . ( Trích đề thi thử số 5

của diễn đàn k2pi.net)

Lời giải

Tính thể tích.

Ta chứng minh được A′H⊥(ABC). Từ B kẻ BI⊥AH =⇒ BI⊥(A′AH) =⇒ A′B, (A′AH) =

BA′I = 300

160

Page 162: Chuyên đề luyện thi đại học môn Toán - VipLam.Net

Tam giác ABH đều cạnh bằng a nên ta có BI =a√

3

2, =⇒ A′I = BI cot 300 =

3a

2=⇒

A′H =√A′I2 −HI2 = a

√2.

Thể tích khối lăng trụ là: V = A′H.dt(ABC) =√

2a.

√3a2

2=

√6a3

2(đvtt)

Tính khoảng cách:

Gọi O = A′C ∩ AM . Thì O là trọng tâm của tam giác ACC ′. Ta dựng mặt phẳng chứa AM

song song với A′B bằng cách: Qua O kẻ đường thẳng ON//A′B thì A′B//(OAN) .

Như vậy d(A′B/AM) = dAB′/(OAN) = dB/(OAN). Để tạo ra bài toán cơ bản ta sẽ dựng OK⊥AC =⇒

OK⊥(ABC) ( Chú ý rằng//OK// =1

3A′H)

Dễ thấyBN = 4KN =⇒ dB/(OAN) = 4dK/(OAN). Ta kẻKP⊥AN,KT⊥OP =⇒ KT⊥(OAN) =⇒dK/(OAN) = KT

Trong tam giác BAN . AN2 = BN2 +BA2 − 2BN.BA.cos(ABN) =⇒ AN =a√

13

3

Ta có: cos(BNA) =BN2 + AN2 − AB2

2.BN.AN=

5

2√

13=⇒ sin(BNA) =

3√

3√52.

Suy ra: sin(BNA) =KP

NK=⇒ PK =

a√

3√52.

Áp dụng hệ thức lượng trong tam giác vuông OPK ta có.1

KT 2=

1

PK2+

1

OK2=⇒ KT =

PK.OK√PK2 +OK2

= a

√6

131

Vậy d (B; (AMN)) = 4a.

√6

131=

2a

13

√4056

131

Ví dụ 7. Cho hình lăng trụ ABC.A′B′C ′ có đáy ABC là tam giác đều cạnh bằng a, cạnh

bên AA′ = a. Gọi I là trung điểm của AB. Biết B′I⊥(ABC) . Tính thể tích lăng trụ và

khoảng cách từ điểm B đến mặt phẳng (A′ACC ′).

Lời giải

161

Page 163: Chuyên đề luyện thi đại học môn Toán - VipLam.Net

Tính thể tích.

Vì BI ′⊥(ABC) gọi H là trung điểm của A′B′ thì AH⊥(A′B′C ′).

Thể tích lăng trụ là: V = B′I.dt(ABC).

Tính được B′I =√B′B2 −BI2 =

a√

3

2, dt(ABC) =

a2√

3

4=⇒ V =

a√

3

2.a2√

3

4=

3a3

8Tính khoảng cách:

Để tính khoảng từ điểm B đến mặt phẳng (ACC ′A′) ta sẽ tìm cách quy về khảng cách từ chân

đường cao H đến mặt phẳng (ACC ′A′).

Ta có BB′//(ACC ′A′) =⇒ dB/(ACC′A′) = dB′/(ACC′A′). Vì B′A′ = 2HA′ =⇒ dB′/(ACC′A′) =

2dH/(ACC′A′)

Kẻ HK⊥A′C ′, HT⊥A′K =⇒ HT⊥(ACC ′A′) =⇒ dH/(ACC′A′) = HT .

Trong tam giác vuông AHK ta có1

HT 2=

1

HK2+

1

AH2=⇒ HT =

HK.HA√HK2 +HA2

Tính được HK =B′M

2=a√

3

4với M là trung điểm của AC, AH =

a√

3

2thay số ta thu được:

HT =

√15a

10=⇒ dB′/(ACC′A′) = 2dH/(ACC′A′) = 2HT =

√15a

5

Một số bài tập rèn luyệnBài 1 Cho hình chóp SABCD có đáy ABCD là hình thoi tâm O cạnh bằng a, ABC = 600.

Hình chiếu của S lên mặt đáy là trung điểm của OB. SC tạo với đáy một góc 600 . Gọi M là

trung điểm của cạnh CD . Tính thể tích khối chóp SABCD và khoảng cách giữa hai đường

thẳng SB và AE.

Bài 2 Cho hình chóp SABCD có đáy ABCD là hình bình hành có AB = 2AD = 2a, góc

BAD = 600. Gọi M là trung điểm của AB. Biết SA = SD = SM , góc tạo bởi (SAD) và đáy

(ABCD) bằng 600. Tính thể tích khối chóp SABCD và khoảng cách giữa hai đường thẳng SD

và CM .

Bài 3 Cho hình chóp SABCD có đáy ABCD là hình vuông cạnh bằng 2a. Mặt bên (SAB)

là tam giác vuông và vuông góc đáy ABCD, biết SA = a . Gọi M, N lần lượt là trung điểm

của AD,SD. Tính thể tích khối chóp SACM và khoảng cách từ trọng tâm G của tam giác

SAC đến mặt phẳng (SAD)

Bài 4 Cho lăng trụ ABCA′B′C ′ có đáy ABC là tam giác cân AB = AC = a, BAC = 1200.

Biết đỉnh A′ cách đều các đỉnh A,B,C. Biết góc tạo bởi (A′AC) và đáy bằng 600. Tính thể tích

khối chóp A′ABC và khoảng cách từ điểm A đến mặt phẳng (A′AC).

Bài 5 Cho lăng trụ đứng ABCA′B′C ′ có tất cả các cạnh đều bằng a. Gọi M là trung điểm

của CC ′. Tính thể tích khối chóp AB′MC và khoảng cách từ điểm A đến mặt phẳng (B′MC).

Bài 6 Cho lăng trụ ABCA′B′C ′ có đáy là tam giác vuông cân AB = AC = a. Biết mặt

bên (ABB′A′) vuông góc với đáy và tam giác A′AB vuông cân tại A′. Tính thể tích khối lăng

trụ ABCA′B′C ′ và khoảng cách giữa hai đường thẳng A′C và BC ′.

162

Page 164: Chuyên đề luyện thi đại học môn Toán - VipLam.Net

CHUYÊN ĐỀ 9

THAM SỐ HÓA HÌNH GIẢI TÍCH TRONG

MẶT PHẲNG

(Nguyễn Thị Thỏa)

§ 1. KIẾN THỨC CƠ BẢN

Các vấn đề nêu sau đây đều xét trong mặt phẳng với hệ trục tọa độ Đề các vuông góc Oxy

I. ĐƯỜNG THẲNG

* Phương trình tham số:

Đường thẳng (∆) đi qua điểm M0 (x0; y0) và nhận −→v (a; b)(a2 + b2 6= 0) làm một véc tơ chỉ

phương có phương trình:

{x = x0 + at

y = y0 + bt(t ∈ R)

1 Tham số hóa điểm M thuộc đường thẳng (∆) :

{x = x0 + at

y = y0 + bt(t ∈ R)

là M (x0 + at; y0 + bt) ∈ (∆)

Chẳng hạn: Tham số hóa điểm M thuộc đường thẳng (∆) :

{x = 1 + 3t

y = 2− 4t(t ∈ R)

là M (1 + 3t; 2− 4t) ∈ (∆)với t ∈ R* Phương trình tổng quát:

Đường thẳng (∆) đi qua điểm M0 (x0; y0) và nhận −→n (A;B)(A2 +B2 6= 0) làm một véc tơ pháp

tuyến có phương trình: A(x− x0) +B (y − y0) = 0

Dạng tổng quát: Mọi đường thẳng đều có phương trình dạng:Ax+By+C = 0 với(A2 +B2 6= 0)

2 Tham số hóa điểm thuộc đường thẳng (∆):Ax+By + C = 0 với(A2 +B2 6= 0)

là: M(m; −Am−C

B

)∈ ∆

Chẳng hạn: Tham số hóa điểm M thuộc đường thẳng (∆): 2x−y+3 = 0 là: M(m; 2m+3) ∈ ∆

Tham số hóa điểm N thuộc đường thẳng d : x− 3y + 4 = 0 là: N (3n+ 4;n) ∈ (d)

3 Khoảng cách từ điểm đến đường thẳng: Khoảng cách từ điểm M0 (x0; y0) đến đường thẳng

(∆):Ax+By + C = 0 với(A2 +B2 6= 0)là:

d(M0;∆) =|Ax0 +By0 + C|√

A2 +B2

163

Page 165: Chuyên đề luyện thi đại học môn Toán - VipLam.Net

4 Góc giữa hai đường thẳng:

Tính theo tọa độ véc tơ pháp tuyến: Góc ϕ giữa(∆1) có một véc tơ pháp tuyến −→n1 (A1;B1) với

(A21 +B2

1 6= 0) và (∆2) có một véc tơ pháp tuyến −→n2 (A2;B2) với (A22 +B2

2 6= 0) được xác định

bởi:

cosϕ =|A1A2 +B1B2|√A2

1 +B21 .√A2

2 +B22

Tính theo tọa độ véc tơ chỉ phương: Góc ϕ giữa(∆1) có một véc tơ pháp tuyến −→u1 (a1; b1) với

(a21 + b2

1 6= 0) và (∆2) có một véc tơ pháp tuyến −→u2 (a2; b2) với (a22 + b2

2 6= 0) được xác định bởi:

cosϕ =|a1a2 + b1b2|√a2

1 + b21.√a2

2 + b22

Tính theo hệ số góc: Góc ϕ giữa(∆1) có hệ số góc k1 và (∆2) có hệ số góc k2 với được xác định

bởi:

tanϕ =

∣∣∣∣ k1 − k2

1 + k1k2

∣∣∣∣ hoặc cosϕ =|1 + k1k2|√

k21 + 1.

√k2

2 + 1

II. ĐƯỜNG TRÒN

1 Tham số hóa điểm thuộc đường tròn:

Điểm thuộc đường tròn tâm I(a; b) bán kính R có tọa độ dạng: M(a+R sin t; b+R cos t) với

t ∈ [0; 2π]

2 Điều kiện đường thẳng tiếp xúc với đường tròn:

Đường thẳng ∆ tiếp xúc với đường tròn I(a; b) bán kính R khi và chỉ khi d(I;∆) = R

III. ELIP

1 Tham số hóa điểm thuộc elip:

Điểm thuộc elipx2

a2+y2

b2= 1 (a > b > 0) có tọa độ dạng M(a sin t; b cos t) với t ∈ [0; 2π]

2 Các yếu tố liên quan elip:

Tiêu điểm: F1(−c; 0), F2(c; 0); Tiêu cự: F1F2 = 2c Đỉnh: A1(−a; 0), A2(a; 0), B1(0;−b), B2(0; b)

Độ dài trục lớn:2a Độ dài trục bé: 2b Hệ thức: a2 = b2 + c2 Tâm sai: e =c

aHình chữ nhật cơ sở cạnh 2a, 2b Phương trình đường chuẩn: ∆1 : x = −a

e; ∆2 : x =

a

e

§ 2. BÀI TẬP CƠ BẢN

Bài 1. Trong mặt phẳng Oxy cho điểm A(2,−3). Hai đường thẳng (d1); (d2) lần lượt có phương

trình là:

x = 7− 2m

y = m− 3;

x = −5 + 4t

y = −7 + 3tViết phương trình tham số đường thẳng ∆ đi qua

A và cắt 2 đường thẳng (d1); (d2) lần lượt tại B,C sao cho A là trung điểm của BC.

Lời giải:

Do B ∈ d1 ; C ∈ d2 nên ta có B(7− 2m ; −3 +m) ; C(5 + 4t ; −7 + 3t).

164

Page 166: Chuyên đề luyện thi đại học môn Toán - VipLam.Net

Từ đó vì là trung điểm của nên ta có hệ :xB + xC = 2xA

yB + yC = 2yA⇐⇒

−m+ 2t = 1

m+ 3t = 4⇐⇒

m = 1

t = 1

Từ đó ta có : B(5 ; −2) ; C(−1 ; −4). Đường thẳng cần tìm có phương trình: x−3y−11 = 0

Bài 2. Trong mặt phẳng với hệ toạ độ Oxy, lập phương trình tham số của đường thẳng đi

điểm N(−5; 4) và cắt hai đường thẳng d1 : x + 2y + 1 = 0 va d2 : x + 2y − 1 = 0 tại hai

điểm A, B sao cho độ dài đoạn thẳng AB bằng 5 biết rằng hoành độ điểm A lớn hơn 2

Lời giải:

Gọi giao điểm của đường thẳng đó với d1; d2 lần lượt là A(−2a− 1; a);B(1− 2b; b)

Ta có−−→NA (−2a+ 4; a− 4) và

−−→NB(6− 2b; b− 4) cùng phương nên suy ra:

(6−2b)(a−4) = (b−4)(4−2a) ⇐⇒ 6a−24+8b = 4b−16+8a. ⇐⇒ 2a−4b+8 = 0 ⇐⇒ a−2b+4 = 0

Thay vào ta có: A(7− 4b; 2b− 4) với b < 54. Kết hợp điều kiện: AB = 5, ta có phương trình:

(2b− 6)2 + (4− b)2 = 25 ⇐⇒ 5b2 − 32b+ 27 = 0 ⇐⇒

[b = 1

b = 275

Đổi chiếu điều kiện ta có b = 1 thỏa mãn điều kiện bài toán khi đó A(3;−2).

Vậy phương trình đường thẳng cần tìm là: 3x+ 4y − 1 = 0

Bài 3. Trong mặt phẳng tọa độ cho điểm A(2, 2) và các đường thẳng d1 : x + y − 2 = 0 và

d2 : x + y − 8 = 0.Tìm tọa độ các điểm B và C lần lượt thuộc d1, d2 sao cho tam giác ABC

vuông cân tại A. (Trích Đề thi tuyển sinh đại học năm 2007, môn toán khối B)

Lời giải:

Do B ∈ d1 , C ∈ d2 nên ta có B(t ; 2− t) , C(n ; n− 8)

Theo bài 4ABC vuông cân tại A nên ta có hệ phương trình :AB⊥ACAB = AC⇐⇒

AB = AC−→AB.

−→AC = 0

⇐⇒

(t− 1)(n− 4) = 2

(t− 1)2 − (n− 4)2 = 3

Từ đây ta giải ra được:{

t− 1 = 2

n− 4 = 1{t− 1 = −2

n− 4 = −1

⇐⇒

{

t = 3

n = 5{t = −1

n = −3

Vậy: B(3 ; −1) , C(5 ; −3) hoặc B(−1 ; 3) , C(−3 ; −11)

Bài 4. Trong mặt phẳng với hệ tọa độ Oxy, cho 4ABC với C(2; 3), phương trình đường

thẳng (AB) : 3x− 4y+ 1 = 0 phương trình trung tuyến (AM) : 2x− 3y+ 2 = 0 . Viết phương

trình tổng quát của đường thẳng AC và BC.

165

Page 167: Chuyên đề luyện thi đại học môn Toán - VipLam.Net

Lời giải:

Tọa độ điểm A là nghiệm của hệ phương trình :{3x− 4y + 1 = 0

2x− 3y + 2 = 0⇐⇒

{x = 5

y = 4

Suy raA(5; 4) Phương trình đường thẳng AClà: x− 3y + 7 = 0

Ta có: B ∈ AB nên B

(x0;

3x0 + 1

4

)Gọi M là trung điểm của BC, tọa độ M là: M

(x0 + 2

2;3x0 + 13

8

)Do M ∈ AM nên 2 · x0 + 2

2− 3 · 3x0 + 13

8+ 2 = 0 ⇐⇒ x0 = −7.

Từ đó ta có : B(−7;−5)

Phương trình đường thẳng BC là: 8x− 9y + 11 = 0

Bài 5. Trong mặt phẳng tọa độ Oxy, cho hai đường thẳng ∆ : x−y−4 = 0 và d : 2x−y−2 = 0.

Tìm tọa độ điểm N thuộc đường thẳng d sao cho đường thẳng ON cắt đường thẳng ∆ tại

điểm M thỏa mãn OM.ON = 8. (Trích Đề thi tuyển sinh đại học năm 2011, môn toán khối

B)

Lời giải:

Tọa độ M, N có dạng : N(a, 2a− 2), M(b, b− 4).

Từ điều kiện 3 điểm O, M, N thẳng hàng, hay−−→OM(a; 2a− 2),

−−→ON(b; b− 4) cùng phương ta có:

a(b− 4) = (2a− 2)b ⇐⇒ 4a = (2− a)b =⇒ b =4a

2− a(Vì a = 2 không thỏa mãn).

Ta có−−→ON

(4a

2− a;8a− 8

2− a

)Theo bài ra : OM.ON = 8 ⇐⇒ OM2.ON2 = 64 ta thiết lập được phương trình :

(5a2 − 8a+ 4)16(5a2 − 8a+ 4)

(a− 2)2 = 64 ⇐⇒ (5a2 − 8a+ 4)2

= 4(a− 2)2

⇐⇒

[5a2 − 8a+ 4 = 2(a− 2)

5a2 − 8a = 4 = 2(2− a)⇐⇒

[5a2 − 10a+ 8 = 0

5a2 − 6a = 0⇐⇒

a = 0

a =6

5

Vậy N(0;−2) hoặc N

(6

5;2

5

)Bài 6. Trong mặt phẳng hệ tọa độ Oxy cho tam giác ABC cân tại A có đỉnh A(−1; 4) và

các đỉnh B,C thuộc đường thẳng ∆ : x− y− 4 = 0. Xác định tọa độ các điểm B,C biết tam

giác ABC có diện tích bằng 18.

Lời giải:

Gọi H là trung điểm BC thì AH ⊥ BC =⇒ AH : x+ y − 3 = 0 =⇒ H

(7

2,−1

2

).

Gọi B(x, x− 4) ( Vì B ∈ BC) =⇒ C(7− x, 3− x) (Vì H là trung điểm BC)

Vì tam giác ABC có diện tích bằng 18. =⇒ SABC =1

2.d(A,∆).BC = 18 =⇒ BC = 4.

√2

166

Page 168: Chuyên đề luyện thi đại học môn Toán - VipLam.Net

Do đó ta có (x− 7 + x)2 + (x− 4− 3 + x)2 = 32 =⇒

x =11

2

x =3

2

Vậy B

(11

2,3

2

)=⇒ C

(3

2,−5

2

)hoặc B

(3

2,−5

2

)=⇒ C

(11

2,3

2

)Bài 7. Trong mặt phẳng tọa độ vuông góc Oxy, cho hình chữ nhật ABCD có diện tích bằng

12, tâm I là giao điểm của hai đường thẳng d1 : x− y − 2 = 0và d2 : 2x+ 4y − 13 = 0. Trung

điểm M của cạnh AD là giao điểm của d1 với trục hoành. Tìm tọa độ các đỉnh của hình chữ

nhật, biết A có tung độ dương.

Lời giải:

Theo bài ra ta có :

Tọa độ điểm I là nghiệm của hệ

x− y − 2 = 0

2x+ 4y − 13 = 0=⇒ I

(7

2;3

2

)

Tọa độ điểm M là nghiệm của hệ

x− y − 2 = 0

y = 0=⇒ M (2; 0)

Có−−→MI =

(3

2;3

2

)=⇒ MI =

3√

2

2=⇒ CD = 3

√2

Mà SABCD = CD.AB =⇒ AB = 2√

2

Mặt khác : pt (IM) : x− y − 2 = 0 nên (AD) : x+ y − 2 = 0

Gọi A(a; 2− a) Vậy AM =√

2 suy ra a = 0 hoặc a = 4

Theo đề bài suy ra A(0; 2) =⇒ D(4;−2); B(3; 5); C(7; 1)

Bài 8. Trong mặt phẳng với hệ tọa độ vuông góc Oxy, cho hình vuông ABCD có các đỉnh

A (−1; 2) , C (3;−2) . Gọi E là trung điểm của cạnh AD,BM là đường thẳng vuông góc với

CE tại M ; Biết phương trình đường thẳng BM : 2x− y − 4 = 0. Tìm tọa độ điểm B,D

Lời giải:

Tâm I của hình vuông là trung điểm của AC nên I(1; 0) Đường thẳng CE đi qua C(3;−2) và

vuông góc với BM : 2x−y+4 = 0 nên có phương trình: x+2y+1 = 0. Gọi E(−2e−1; e) ∈ CElà trung điểm của AD. Khi đó ta có:

−→EI.−→EA = 0 ⇐⇒ (1+2e+1)(−1+2e+1)+(0−e)(2−e) = 0

167

Page 169: Chuyên đề luyện thi đại học môn Toán - VipLam.Net

⇐⇒ 4e+ 4e2 − 2e+ e2 = 0 ⇐⇒ 2e+ 5e2 = 0 ⇐⇒

[e = 0

e = −25

Mặt khác EI = EA

Bài 9. Trong mặt phẳng toạ độ Oxy cho tam giác ABC, có điểm A(2; 3), trọng tâm G(2; 0).

Hai đỉnh B và C lần lượt nằm trên hai đường thẳng d1 : x+ y + 5 = 0 và d2 : x+ 2y − 7 = 0.

Viết phương trình đường tròn có tâm C và tiếp xúc với đường thẳng BG.

Lời giải:

Ta có−→AG =

2

3

−−→AM . Gọi M(a; b) là trung điểm BC suy ra M

(2;−3

2

)Gọi B(b;−5− b) =⇒ C(4− b; 2 + b) ∈ d2 =⇒ b = −1. Vậy B(−1;−4), C(5; 1)

Phương trình đường thẳng (BG) : 4x− 3y − 8 = 0

Phương trình đường tròn cần tìm là: (C) : (x− 5)2 + (y − 1)2 =81

25

§ 3. BÀI TẬP NÂNG CAO RÈN LUYỆN KĨ NĂNG

Bài 1. Trong mặt phẳng hệ tọa độ Oxy, cho hình chữ nhật ABCD có I(6; 2) là giao điểm

của hai đường chéo AC và BD, điểm M (1; 5) thuộc đường thẳng AB và trung điểm E

của cạnh CD thuộc đường thẳng ∆ : x+ y − 4 = 0.

Viết phương trình đường thẳng AB. (Trích Đề thi ĐH Khối A- Năm 2009)

Lời giải:

Gọi N là điểm đối xứng với M qua I.

Ta có N(11;−1) và N ∈ CD. Gọi E(x; 5− x) ∈ CD là trung điểm của cạnh CD.

Khi đó−→IE = (x− 6; 3− x) và

−−→NE (x− 11; 6− x).

Ta có IE⊥EN ⇐⇒−→IE.−−→EN = 0

⇐⇒ (x− 6) (x− 11) + (3− x) (11− x) = 0 ⇐⇒ x2 − 13x+ 42 = 0 ⇐⇒

[x = 6

x = 7

Với x = 6 =⇒−→IE = (0;−3). Đường thẳng AB đi qua M và có một véc tơ pháp tuyến

−13

−→IE = (0; 1) nên ta có phương trình y − 5 = 0

Với x = 7 =⇒−→IE = (1;−4). Đường thẳng AB đi qua M và có một véc tơ pháp tuyến

−→IE = (1;−4) nên ta có phương trình x− 4y + 19 = 0

168

Page 170: Chuyên đề luyện thi đại học môn Toán - VipLam.Net

Bài 2. Trong mặt phẳng hệ tọa độ Oxy, cho tam giác ABC cân tại A có đỉnh A(6; 6), đường

thẳng đi qua trung điểm của các cạnh AB, AC có phương trình:x+ y − 4 = 0.

Tìm tọa độ các đỉnh B và C biết E(1;−3)nằm trên đường cao đi qua đỉnh C của tam giác

đã cho. (Trích Đề thi ĐH Khối A- Năm 2010)

Lời giải:

Gọi H là trung điểm của BC thì AH vuông góc BC (Vì tam giác ABC cân).

Gọi I, J lần lượt là trung điểm của các cạnh AB,AC, giao điểm AH và IJ là K(k; 4− k).

Khi đó :−−→AK(k − 6;−k − 2) thì ta có (k − 6) − (−k − 2) = 0 ⇐⇒ 2k − 4 = 0 ⇐⇒ k = 2 .

Suy ra K(2; 2)

Ta có K là trung điểm của AH nên H(−2;−2)

Phương trình đường thẳng BC có phương trình: x+ y + 4 = 0

Gọi B(b;−b− 4) suy ra C(−4− b; b).Ta có:

−→AB(6− b; b+ 10) vuông góc với

−−→CE(5 + b;−b− 3)

nên ta có phương trình (5+b)(6−b)+(−b−3)(b+10) = 0 ⇐⇒ 2b2+12b = 0 ⇐⇒

[b = 0

b = −6

Vậy B(0;−4);C(−4; 0) hoặc B(−6; 2), C2;−6)

Bài 3. Cho tam giác ABC có diện tích bằng 24 và phương trình các đường trung tuyến kẻ từ

các đỉnh A, B, C lần lượt là ∆1 : x− y+ 2 = 0, ∆2 : 5x− y− 2 = 0, ∆3 : x+ 3y− 10 = 0.

Tìm toạ độ các đỉnh của tam giác ABC.

Lời giải:

Vì A , B thuộc đường thẳng ∆1 : x− y + 2 = 0, ∆2 : 5x− y − 2 = 0

Gọi tọa độ điểm A(a; a+ 2);B

(b;

10− b3

)Gọi M là trung điểm của AB thì tọa độ M

(a+ b

2;15a− b+ 4

6

)Vì điểm M thuộc trung tuyến qua A, nên thay tọa độ trên và rút gọn ta được: b = 3a− 1.

Thay vào trên ta có: C(3a− 2; 4− a).Ta có:−−→BC = (2a− 2; 6− 6a)

Ta dễ dàng tìm được: S∆ABC = 3S∆GBC = 24 =⇒ S∆GBC = 8

Ta viết được phương trình đường thẳng BC là: (x− a)(6a− 6) + (y − 5a+ 2)(2a− 2) = 0

Từ đây suy ra: a 6= 1, và ta rút gọn lại thành: 6(x− a) + 2(y − 5a+ 2) = 0.

Ta thay vào công thức diện tích là: S∆GBC = 8 ⇐⇒ 1

2d(G,BC).BC = 8,

169

Page 171: Chuyên đề luyện thi đại học môn Toán - VipLam.Net

thì ta dễ dàng suy ra: |a− 1| = 1 ⇐⇒

[a = 0

a = 2

Với: a = 0, suy ra tọa độ các điểm là: B(0;−2);C(−2; 4), A(5; 7)

Với: a = 2, suy ra tọa độ các điểm là: B(2; 8);C(4; 2);A(−3;−1)

Bài 4. Trong mặt phẳng với hệ trục tọa độ Oxy, cho hình vuông ABCD. Gọi M là trung

điểm BC, N là điểm trên cạnh CD sao cho CN = 2ND. Giả sử M(

112

; 12

)và đường thẳng

AN có phương trình : 2x− y − 3 = 0. Tìm tọa độ điểm A. (Trích Đề thi ĐH Khối A năm

2012)

Bình luận: Đây là một bài toán đã làm cho nhiều học sinh mất điểm trong kỳ thi ĐH vừa

qua. Các bạn có thể tham khảo lời giải trong đáp án chính thức của Bộ GD & ĐT.

Nhưng chúng ta có thể khai thác bài toán này theo các hướng khác nhau như sau:

Tham số hóa tọa độ điểm M , tính độ dài đoạn AM theo các cách khác nhau.

Hướng 1: Tính cạnh hình vuông rồi sử dụng định lí Pitago tính AM

Hướng 2: Tính số đo góc MAN rồi sử dụng hệ thức lượng trong tam giác vuông.

Lời giải:

Ta có tan MAN = tan(MAD − NAD

)=

tan MAD − tan NAD

1 + tan MAD. tan NADMà theo bài ra ta có ABCD là hình vuông và M là trung điểm của BC

nên tan MAD = 2 và N là điểm trên cạnh CD sao cho CN = 2ND nên tan NAD =1

3.

Suy ra: tan MAN =2− 1

3

1 + 23

= 1 =⇒ MAN = 45◦

Gọi H là hình chiếu của M lên đường thẳng AN

thì ta có MH = d(M ;AN) =

∣∣2.112− 1

2− 3∣∣

√22 + 12

=15

2√

5=

3√

5

2

Trong tam giác AMH ta có AM = MH. sin MAH =3√

5

2. sin 45◦ =

3√

10

2.

Gọi A(a; 2a− 3) ∈ AN .Ta có

AM =3√

10

2⇐⇒ AM2 =

45

2⇐⇒

(a− 11

2

)2

+

(2a− 7

2

)2

=45

2

170

Page 172: Chuyên đề luyện thi đại học môn Toán - VipLam.Net

⇐⇒ 5a2 − 25a+ 20 = 0 ⇐⇒ a2 − 5a+ 4 = 0 ⇐⇒

[a = 1

a = 4

Vậy A(1;−1) hay A(4; 5)

Bài 5. Trong mặt phẳng với hệ tọa độ vuông góc Oxy cho hình thoi ABCD có A = 60o.Trên

các cạnh AB,BC lấy các điểm M,N sao cho MB + NB = AB. Biết P (√

3; 1) thuộc đường

thẳng DN và đường phân giác trong của góc MDN có phương trình là d : x−y√

3+6 = 0.Tìm

toạ độ đỉnh D của hình thoi ABCD.

Lời giải:

Từ giả thiết A = 60o =⇒ tam giác ABD,CBD là các tam giác đều.

Theo đề bài ta có AM = BN,BM = CN .

Xét hai tam giác ADM và BDN ta có: DAM = DBN = 600, AD = BD,AM = BN

=⇒ hai tam giác bằng nhau =⇒ ADM = BDN (1).

Xét hai tam giác BMD và CND ta có: DBM = DCN = 600,CD = BD,CN = BM

=⇒ hai tam giác bằng nhau =⇒ NDC = MDB (2).

Từ (1) và (2) =⇒ MDN = 60o.

Gọi P ′ là điểm đối xứng của P qua đường phân giác d =⇒ P ′ thuộc đường thẳng DM

=⇒ tam giác PDP ′ là tam giác đều. =⇒ DP = PP ′ = 2d(P/d) = 6.

Gọi D có tọa độ D

(a;a+ 6√

3

)=⇒ PD2 = (a−

√3)2 +

(a+ 6−

√3√

3

)2

= 36

=⇒ a = 3 +√

3, a = −6 +√

3 =⇒ D(3 +√

3; 1 + 3√

3), D(−6 +√

3; 1).

Bài 6. Trong mặt phẳng Oxy, cho ∆ABC vuông tại A, A,B ∈ Ox, đường thẳng BC có

phương trình: 4x + 3y − 16 = 0. Tìm trọng tâm G của ∆ABC biết bán kính đường tròn nội

tiếp r = 1

Lời giải:

Tọa độ của B thỏa mãn hệ:

4x+ 3y − 16 = 0

y = 0⇐⇒

x = 4

y = 0.

Khi đó A(a; 0);B(4; 0); C ∈ CB =⇒ C

(c;

16− 4c

3

)

Khi đó ta được

−→AB = (4− a; 0)

−−→BC =

(c− 4;

16− 4c

3

)−→AC =

(c− a;

16− 4c

3

) ⇐⇒

AB =√

(4− a)2

BC =

√(c− 4)2 +

(16− 4c

3

)2

AC =

√(c− a)2 +

(16− 4c

3

)2

.

Vì tam giác ABC vuông tại A và r = 1 nên ta có

AB⊥ACS = pr = p⇐⇒

−→AB.−→AC = 0

AB.AC

2=AB + AC +BC

2

171

Page 173: Chuyên đề luyện thi đại học môn Toán - VipLam.Net

Bài 7. Trong mặt phẳng Oxy, lập phương trình đường tròn (C) có bán kính R = 2, có tâm I

nằm trên đường thẳng (d1) : x+y−3 = 0 và đường tròn đó cắt đường thẳng (d2) : 3x+4y−6 = 0

tại hai điểm A,B sao cho AIB = 120◦

Lời giải:

Gọi M là trung điểm của AB

Với điều kiện giả thiết =⇒ AIH = 60o =⇒ IH = AI. cos 60o = 1

Goi I(a; 3− a). Khi đó d(I;d2) = AH = 1 ⇐⇒ |3a+ 4(3− a)− 6|5

= 1 ⇐⇒ a = 1; a = 11

Vậy I(1;−2); I(11;−8). Từ đây ta có có 2 phương trình đương tròn .

Bài 8. Cho tam giác ABC có A nằm trên Ox với 0 < xA <5

2. Hai đường cao xuất phát từ

B và C lần lượt có phương trình: d1 : x− y + 1 = 0; d2 : 2x+ y − 4 = 0 . Tìm tọa độ A,B,C

sao cho diện tích tam giác ABC là lớn nhất.

Lời giải:

Gọi hai đường cao là BE và CF . Ta có A(a ; 0), B(b ; b+ 1), C(4− 2c ; c)

Nên

−→AB.−−→nCF = 0−→AC.−−→nBE = 0

⇐⇒

b =

2a− 1

3

c =4− a

3

Do đó ta có: B

(2a− 1

3;

2a+ 2

3

), C

(4 + 2a

3;

4− a3

)Việc còn lại là tính diện tích tam giác ABC và xét hàm số biến a với điều kiện 0 < a <

5

2.

Bài 9. Trong mặt phẳng tọa độ Oxy cho đường tròn (C) : x2 + y2− 2x+ 4y− 4 = 0 và đương

thẳng (d) : x − y + 1 = 0. Tìm điểm M thuộc đường thẳng (d) sao cho qua M kẻ được các

tiếp tuyến MA, MB đến đường tròn (C) và độ dài AB nhỏ nhất (A,B là các tiếp điểm).

Lời giải:

Đường tròn tâm I(1;−2), bán kínhR = 3. Gọi M(a; a+ 1) ∈ d; IM2 = 2(a+ 2)2 + 2.

Gọi H = AB ∩MI. Ta có AB = 2AH = 2HB

Trong tam giác vuông MAH ta có:1

BH2=

1

IA2=

1

MA2

=⇒ AH2 =AI2 +MA2

AI2.MA2≥ 2

IA.MA≥ 4

AI2 +MA2=

4

IM2=⇒ AH ≥

√2 hay AB ≥ 2

√2

Dấu bằng xảy ra khi và chỉ khi a+ 2 = 0 ⇐⇒ a = −2

Vậy M(−2;−1)

Bài 10. Cho elip (E) :x2

16+y2

5= 1 và 2 điểm A(−5;−1) và B(−1; 1). Tìm điểm M trên

(E) sao cho diện tích ∆MAB lớn nhất.

Lời giải:

Viết được phương trình AB : x− 2y + 3 = 0. Ta có S4MAB =1

2dM/AB.AB

Vì AB không đổi nên diện tích ∆MAB lớn nhất khi và chỉ khi d(M ;AB)max

Vì điểm M thuộc Elip nên suy ra M(4 sinα;√

5 cosα) với α ∈ [0; 2π]

172

Page 174: Chuyên đề luyện thi đại học môn Toán - VipLam.Net

Ta có: dM/AB =|4 sinα− 2

√5 cosα + 3|√5

Mà (4 sinα− 2√

5 cosα)2 ≤ (16 + 20)(sin2 α + cos2 α) = 36 =⇒ 4 sinα− 2√

5 cosα ≤ 6

Dấu bằng xảy ra khi

sinα

4=

cosα

−2√

5

4 sinα− 2√

5 cosα = 6⇐⇒

sinα =

2

3

cosα =−√

5

3

=⇒ M

(8

3; −5

3

).

Bài 11. Cho ∆ABC có A ∈ (d) : 2x − y + 6 = 0, đường trung tuyến (BM) : x + y + 3 = 0,

trung điểm cạnh BC là N(1; 2). Tính SABC biết BC‖(d)

Lời giải:

Vì BC//(d) và BC qua N nên BC : 2x− y = 0

Ta có: B là giao điểm của BC và BM =⇒ B(−1,−2) =⇒ C(3, 6) (Vì N là trung điểm

BC).

M ∈ BM =⇒ M(m,−m− 3) =⇒ A(2m− 3,−2m− 12)

Mặt khác A ∈ d =⇒ m = −2 =⇒ A(−7,−8)

Ta có: SABC =d(A,BC).BC

2=

3

20.

Bài 12. Trong mặt phẳng hệ tọa độ Oxy, cho tam giác ABC với AB =√

5, C(−1;−1).

Đường thẳng AB có phương trình:x + 2y − 3 = 0, và trọng tâm tam giác ABC thuộc đường

thẳng:x+ y − 2 = 0. Tìm tọa độ 2 đỉnh A,B.

Lời giải:

Vì A,B thuộc AB : x+ 2y − 3 = 0 nên A(3− 2a; a) và B(3− 2b; b).

Lúc này, bài toán của mình có hai ẩn. Do đó: ta cần thiết lập hai phương trình để giải.

Phương trình thứ nhất: AB =√

5 ⇐⇒ (2a− 2b)2 + (a− b)2 = 5 ⇐⇒ (a− b)2 = 1

Phương trình thứ hai: GọiG là trọng tâm của tam giácABC. Khi đóG

(−2a− 2b+ 5

3,a+ b− 1

3

).

Tới đây, thay tọa độ trọng tâm vào x+ y− 2 = 0. Ta được phương trình thứ hai: a+ b+ 2 = 0

Do đó ta có hệ:

(a− b)2 = 1

a+ b = −2

173

Page 175: Chuyên đề luyện thi đại học môn Toán - VipLam.Net

PHẦN THỨ II

CÁC BÀI TOÁN HAY

174

Page 176: Chuyên đề luyện thi đại học môn Toán - VipLam.Net

CÁC BÀI TOÁN

BẤT PHƯƠNG TRÌNH (hthtb22)

Bài 1: Giải bất phương trình:

8x3 + 76x√x+ 1 ≥ 58x2 + 29x

Hướng dẫn

Ta thấy x = 0 thỏa mãn phương trình đã cho.

Xét x khác 0. Lúc này chia hai vế của bất phương trình cho x√x ta được:

8x√x+

1

x√x

+ 76 ≥ 29

(2√x+

1√x

)t =

(2√x+

1√x

)=⇒ t3 =

(8x√x+

1

x√x

)+ 3.2

√x. 1√

x.

(2√x+

1√x

)=⇒

(8x√x+

1

x√x

)= t3 − 6t

Việc còn lại là t hay trở lại vào bất phương trình để giải!

Bài 2: Giải bất phương trình (GBPT):

9x2 − 4√5x2 − 1

≤ 3x+ 2

Lời giải

Điều kiện 5x2 − 1 > 0 ⇐⇒ x < −1√5∨ x > 1√

5.

Khi đó, bất phương trình tương đương

(3x+ 2)(3x− 2−

√5x2 − 1

)≤ 0

Xét 3x− 2−√

5x2 − 1 = 0 ⇐⇒

{x ≥ 2

3

4x2 − 12x+ 5 = 0⇐⇒ x = 5

2

Lập bảng xét dấu ta nhận được tập nghiệm

S =

[−2

3;−1√

5

)⋃(1√5

;5

2

].

175

Page 177: Chuyên đề luyện thi đại học môn Toán - VipLam.Net

Bài 3: Giải bất phương trình:

√x4 − 9x2 + 14x2 ≥ 2x3 + 24x

Lời giải

ĐKXĐ: x4 − 9x2 ≥ 0 ⇐⇒ x2(x− 3)(x+ 3) ≥ 0

Nếu x = 0 (4) ⇐⇒ 0 ≥ 0x luôn đúng

Nếu x 6= 0 ta có:

ĐK: x ≥ 3 hoặc x ≤ −3

+ Xét x ≥ 3. Ta có:

x√x2 − 9 ≥ x(2x2 − 14x+ 24)

⇐⇒√x2 − 9 ≥ 2x2 − 14x+ 24

Nếu 2x2 − 14x+ 24 ≤ 0 ⇐⇒ (x− 3)(x− 4) ≤ 0 ⇐⇒ 3 ≤ x ≤ 4

Nếu 2x2 − 14x+ 24 > 0 ⇐⇒ x < 3hoặcx > 4

(4.1)⇐⇒ (x− 3)(x− 5)(4x2 − 24x+ 39) ≤ 0 ⇐⇒ 3 ≤ x ≤ 5

Nên 3 ≤ x ≤ 5

+ Xét x ≤ −3. Ta có:√x2 − 9 ≤ 2x2 − 14x+ 24 =⇒ (x− 3)(x− 4) ≥ 0 thoả mãn

⇐⇒ (x− 3)(x− 5)(4x2 − 24x+ 39) ≥ 0- luôn đúng

Vậy 3 ≤ x ≤ 5;x = 0;x ≤ −3

Bài 4: Giải bất phương trình:

√x ≥ x4 − 2x3 + 2x− 1

x3 − 2x2 + 2x

Lời giải

Từ bất phương trình suy ra x > 0 Khi đó bất phương trình đã cho tương đương:√x ≥ (x+ 1)(x− 1)3

x (x2 − 2x+ 2)⇐⇒ x

√x ≥ (x+ 1)(x− 1)3

(x− 1)2 + 1⇐⇒ (

√x)3

(√x)2 + 1

≥ (x− 1)3

(x− 1)2 + 1(1)

* Hàm số f(t) =t3

t2 + 1đồng biến trên R.

* Vậy (1) ⇐⇒

{ √x ≥ x− 1

x > 0⇐⇒

0 < x ≤ 1

1 < x ≤ 3 +√

5

2

⇐⇒ 0 < x ≤ 3+√

52

Bài 5: Giải bất phương trình:

2x2 − 11x+ 20√2x2 − 10x+ 22−

√x+ 2

≤ 3− x

Lời giải

Bất phương trình đã cho tương đương với:√2x2 − 10x+ 22 +

√x+ 2 ≤ 3− x

ĐK: −2 ≤ x < 3

176

Page 178: Chuyên đề luyện thi đại học môn Toán - VipLam.Net

BPT →√

2x2 − 10x+ 22 ≤ 3− x−√x+ 2

⇐⇒ 2x2 − 10x+ 22 ≤ x2 − 5x+ 11− 2 (3− x)√x+ 2

⇐⇒ x2 − 5x+ 11 ≤ −2 (3− x)√x+ 2 (∗)

Ta có: V T (∗) > 0 ;V P (∗) < 0

Nên BPT vô nghiệm.

Bài 6: Giải bất phương trình:

√x2 − 8x+ 15 ≤

√4x2 − 18x+ 18−

√x2 + 2x− 15

Lời giải 1

√x2 − 8x+ 15 ≤

√4x2 − 18x+ 18−

√x2 + 2x− 15

ĐK:

[x ≥ 5

x ≤ −5

Bất phương trình đã cho tuơng đương với:

√(x− 3) (x− 5) ≤

√2 (x− 3) (2x− 3)−

√(x− 3) (x+ 5)

Trường hợp 1:x ≥ 5

BPT ⇐⇒√x− 5 +

√x+ 5 ≤

√2(2x− 3) ⇐⇒

√x2 − 25 ≤ x − 3 ⇐⇒ 6x − 34 ≤ 0 ⇐⇒

x ≤ 17

3

Kết hợp điều kiện của x cho ta: 5 ≤ x ≤ 17

3Trường hợp 2: x ≤ −5

BPT ⇐⇒√

5− x+√−x− 5 ≤

√2 (3− 2x) ⇐⇒

√x2 − 25 ≤ 3− x ⇐⇒ 6x− 34 ≤ 0 ⇐⇒

x ≤ 173

Đối chiếu với điều kiện của x cho ta: x ≤ −5

Lời giải 2

Điều kiện

x2 − 8x+ 15 ≥ 0

4x2 − 18x+ 18 ≥ 0

x2 + 2x− 15 ≥ 0

⇐⇒

[x ≤ −5

x ≥ 5.

Đặt a =√x2 − 8x+ 15 ≥ 0, b =

√x2 + 2x− 15 ≥ 0 thì 4x2 − 18x+ 18 =

13

5a2 +

7

5b2.

Do đó phương trình trở thành

a+ b ≤√

135a2 +

7

5b2 ⇐⇒ 8a2 − 10ab+ 2b2 ≥ 0.

Nếu b = 0 thì a2 ≥ 0,∀a nên nghiệm trong trường hợp này là x = −5.

177

Page 179: Chuyên đề luyện thi đại học môn Toán - VipLam.Net

Xét b > 0, chia hai vế cho b2 thì bất phương trình trở thành

4(ab

)2

− 5a

b+ 1 ≥ 0 ⇐⇒

ab ≤ 1

4a

b≥ 1

Xéta

b≤ 1

4⇐⇒ 4a ≤ b ⇐⇒ 16(x2 − 8x+ 15) ≤ x2 + 2x− 15 ⇐⇒ 5 ≤ x ≤ 17

3.

Xéta

b≥ 1 ⇐⇒ a > b ⇐⇒ x2 − 8x+ 15 ≥ x2 + 2x− 15 ⇐⇒ x < −5.

Kết hợp lại được tập nghiệm là S = (−∞;−5]⋃[

5;17

3

]Bài 7: Giải bất phương trình:

√2x− 1 + x ≤ 4−

√x2 + 3

Lời giải√2x− 1 + x ≤ 4−

√x2 + 3(∗)

Đkxđ: x ≥ 1

2Khi đó: (∗) ⇐⇒

√2x− 1 + x+

√x2 + 3 ≤ 4

Thấy f(x) =√

2x− 1 + x+√x2 + 3 đồng biến trên |R

Mà f(1) = 4

Nên bất phương trình có nghiệm1

2≤ x ≤ 1

Bài 8: Giải bất phương trình:

x3 − 3x2 + 2

√(x+ 2)3 ≤ 6x

Lời giải

Điều kiện: x ≥ −2.

Khi đó, bất phương trình tương đương với:(x−√x+ 2

)2 (x+ 2

√x+ 2

)≤ 0√

x+ 2 = x hoặc 2√x+ 2 ≤ −x

* Với√x+ 2 = x ⇐⇒

x ≥ 0

x+ 2 = x2⇐⇒ x = 2

* Với 2√x+ 2 ≤ −x⇐⇒

x ≤ 0

x ≥ −2

4(x+ 2) ≤ x2

⇐⇒ −2 ≤ x ≤ 2− 2√

3

Kết luận: Tập ghiệm của bất phương trình đã cho là: S =[−2; 2− 2

√3]∪ {2}

Bài 9: Giải bất phương tình:

√x2 − 2x+

√x2 + 3x ≥ 2x

178

Page 180: Chuyên đề luyện thi đại học môn Toán - VipLam.Net

Lời giải

ĐK:x ≥ 2;x = 0;x ≤ −3

TH1. x = 0 là nghiệm của BPT.

TH2. x ≤ −3 cũng là nghiệm của BPT vì V T ≥ 0 > V P .

TH3. x ≥ 2

Bất phương trình đã cho tương đương:√x− 2 +

√x+ 3 ≥ 2

√x

⇐⇒ 2√

(x− 2)(x+ 3) ≥ 2x− 1

⇐⇒ 4(x2 + x− 6) ≥ 4x2 − 4x+ 1 ⇐⇒ 8x ≥ 25 ⇐⇒ x ≥ 25

8Đối chiếu với điều kiện ta có:

Nghiệm của bất phương trình đã cho là x ≥ 25

8; x = 0 hoặc x ≤ −3

Bài 10:GBPT √2x+ 4− 2

√2− x > 12x− 8√

9x2 + 16

Hướng dẫn

ĐK:−2 ≤ x ≤ 2

Ta có:

BPT ⇐⇒√

2x+ 4− 2√

2− x >2(√

2x+ 4− 2√

2− x) (√

2x+ 4 + 2√

2− x)

√9x2 + 16

⇐⇒(√

2x+ 4− 2√

2− x) (−2√

2x+ 4− 4√

2− x+√

9x2 + 16)> 0

⇐⇒

{ √

2x+ 4− 2√

2− x > 0

−2√

2x+ 4− 4√

2− x+√

9x2 + 16 > 0{ √2x+ 4− 2

√2− x < 0

−2√

2x+ 4− 4√

2− x+√

9x2 + 16 < 0

Bây giờ chỉ cần nhân liên hợp là OK

Bài 11: GBPT

x+x√

x2 − 1>

35

12

Hướng dẫn

Điều kiện: x > 1 hoặc x < −1.

Từ bất phương trình ta có:

x+x√

x2 − 1>

35

12=⇒ x > 0.

Do đó, x > 1. Bình phương hai vế thu được bất phương trình tương đương:

x2 +x2

x2 − 1+ 2

x2

√x2 − 1

>1225

144⇐⇒ x4

x2 − 1+ 2

x2

√x2 − 1

>1225

144

179

Page 181: Chuyên đề luyện thi đại học môn Toán - VipLam.Net

Đến đây, đặt ẩn phụ t =x2

√x2 − 1

, t > 0. Thu được:

t2 + 2t− 1225

144> 0 =⇒ t >

25

12

Dễ rồi

Bài 12:Giải bất phương trình sau trên tập số thực :

√8 + x− 9x2

x+ 2≤√

8 + x− 9x2

2x+ 1

Lời giải

Ta nhận thấy rằng bất phương trình đã cho có 1 đặc điểm đó là cả 2 phân số này đều có chung

tử số là √8 + x− 9x2 ≥ 0

Vậy bất phương trình đã cho tương đương với:√

8 + x− 9x2

(1

x+ 2− 1

2x+ 1

)≤ 0

⇐⇒

−8

9≤ x ≤ 1

x− 1

(x+ 2) . (2x+ 1)≤ 0

⇐⇒

−8

9≤ x ≤ 1

1

(2x+ 1)≤ 0

=⇒ −8

9≤ x <

−1

2;x = 1

Vậy nghiệm của bất phương trình đã cho là:−8

9≤ x <

−1

2;x = 1

Bài 13:Giải bất phương trình :

√2x− 11 + x√

2 (x2 − 8x+ 14) + 5≥ 1

Lời giải

Điều kiện x ≥ 11

2.

Với điều kiện đó, bất phương trình tương đương với

√2x− 11 + x−

√2x2 − 16x+ 28− 5 ≥ 0

⇐⇒(√

2x− 11− (x− 5))

+((2x− 10)−

√2x2 − 16x+ 28

)≥ 0

⇐⇒ −(x− 6)2

√2x− 11 + x− 5

+2(x− 6)2

2x− 10 +√

2x2 − 16x+ 28≥ 0

⇐⇒ (x− 6)2

(2

2x− 10 +√

2x2 − 16x+ 28− 1√

2x− 11 + x− 5

)≥ 0

180

Page 182: Chuyên đề luyện thi đại học môn Toán - VipLam.Net

⇐⇒ (x− 6)2 (2√2x− 11−√

2x2 − 16x+ 28)≥ 0

⇐⇒ (x− 6)2

(−2(x− 6)2

2√

2x− 11 + 2x2 − 16x+ 28

)≥ 0⇐⇒ x = 6.

Vậy tập nghiệm bất phương trình S = {6}.

Bài 14: Giải bất phương trình:

(x+ 1)(x− 3)√−x2 + 2x+ 3 < 2− (x− 1)2

Lời giải

Đặt t =√−x2 + 2x+ 3, 0 � t � 2

Khi đó bất phương trình sẽ thành:

t3 + t2 − 2 > 0 ⇐⇒ (t− 1)(t2 + 2t+ 2) > 0 ⇐⇒ t > 1

Với t>1 thì√−x2 + 2x+ 3 > 1 ⇐⇒ −x2 + 2x+ 3 > 1 ⇐⇒ x2 − 2x− 2 > 0

⇐⇒ x > 1 +√

3 hoặc x < 1−√

3

Bài 15: Giải BPT √6(x2 − 3x+ 1) +

√x4 + x2 + 1 ≤ 0

Lời giải

Ta có: x4 + x2 + 1 = (x2 + 1)2 − x2 = (x2 + x+ 1) (x2 − x+ 1)

Đặt a =√x2 + x+ 1; b =

√x2 − x+ 1.

BPT trở thành:

√6(2b2 − a2

)+ ab ≤ 0

Có lẽ dễ rồi.

181

Page 183: Chuyên đề luyện thi đại học môn Toán - VipLam.Net

CÁC BÀI TOÁN

HỆ PHƯƠNG TRÌNH (Lê Nhất Duy)

1 Giải hệ phương trình: x3 − y3 = 35

2x2 + 3y2 = 4x− 9y

Lời giải:

Phân tích: Đây có lẽ là bài quen thuộc đối với nhiều bạn, để giải hệ này ta phải quan sát các

hạng tử của 2 phương trình. Phương trình ban đầu là bậc 3, phương trình 2 là bậc 2 và bậc

một, từ đó ta liên tưởng đến hằng đẳng thức (a+ b)3, vậy ta phải cố gắng tìm 1 hệ số nhân vào

phương trình 1 hoặc phương trình 2 để khi cộng hoặc trừ 2 vế ta sẽ ra hằng đẳng thức đó.

Giải:

Ta nhân phương trình (2) cho 3. Khi đó ta có hệ mới là:{x3 − y3 = 35

6x2 + 9y2 = 12x− 27y

Ta lấy phương trình (1) trừ đi phương trình (2), ta được:

x3 − y3 − 35− 6x2 − 9y2 + 12x− 27y = 0

⇐⇒ (x3 − 6x2 + 12x− 8)− (y3 + 9y2 + 27y + 3) = 0

⇐⇒ (x− 2)3 = (y + 3)3 ⇐⇒ x = y + 5

Thay x = y + 5 vào một trong 2 phương trình ban đầu ta sẽ tìm được nghiệm

Đáp số: (x; y) = (3;−2); (2;−3)

Vậy ý tưởng giải quyết bài dạng này là tìm 1 hệ số α nhân vào phương trình chứa bậc 2 và

bậc 1 để khi cộng trừ 2 vế phương trình ta sẽ thu được hằng đẳng thức (a+b)3. (1)+(2).α ⇐⇒(x+ a)3 = (y + b)3

Sau đây là một số bài tương tự để các bạn rèn luyện thêm:

1)

{x3 + y3 = 91

4x2 + 3y2 = 16x+ 9yĐáp số: (x; y) = (3; 4); (4; 3)

2)

{x3 + y3 = 9

x2 + 2y2 = x+ 4yĐáp số: (x; y) = (2; 1); (1; 2)

3)

{x3 + 3xy2 = −49

x2 − 8xy + y2 = 8y − 17xĐáp số: (x; y) = (−1;−4); (−1; 4)

182

Page 184: Chuyên đề luyện thi đại học môn Toán - VipLam.Net

2 Giải hệ phương trình: −x3 + 3x+ 4 = y

2y3 − 6y − 2 = x

Lời giải:

Phân tích: Thoạt nhìn bài này, có nhiều bạn sẽ cố gắng dùng các phương pháp thế hoặc tìm

hệ số nhân cho 1 phương trình nào đó để biến đổi, nhưng các cách đó sẽ rất phức tạp hoặc khó

khăn trong việc xoay sở và tìm kiếm. Vì thế ta liên tưởng đến việc dùng phương pháp đánh giá

để tìm nghiệm hệ phương trình.( nếu bạn nào nhanh mắt có thể đoán nghiệm của phương trình

rồi cố gắng tách để so sánh với nghiệm đó để biện luận nghiệm duy nhất)

Giải:

Ta có hệ phương trình đã cho tương đương với:−(x3 − 3x− 2) = y − 2

2(y3 − 3y − 2) = x− 2

⇐⇒

−(x+ 1)2(x− 2) = y − 2 (1)

2(y + 1)2(y − 2) = x− 2 (2)

Từ đó, ta xét: Nếu x > 2 thì từ phương trình (1) ta suy ra y < 2, nhưng y < 2 thì sẽ không

thỏa phương trình (2) vì thế ta loại. Tương tự nếu x < 2 ta cũng loại.

Vậy x = 2 ,suy ra y = 2. Thử lại ta thấy đó là nghiệm của hệ.

Đáp số: (x; y) = (2; 2).

3 Giải hệ phương trình: x4 + y2 =698

81x2 + y2 + xy − 3x− 4y + 4 = 0

Lời giải:

Phân tích: Các bạn sẽ rất khó giải nếu cứ chú ý tới phương trình 1, vì nó là một cái bậc 4 và

1 cái bậc 2 không liên quan gì nhau. Hãy quan sát phương trình 2 ta thấy đó là phương trình

bậc cao nhất là bậc 2 đối với các hạng tử.Vì thế ta sẽ phân tích tích nghiệm của phương trình

2 theo ẩn x và theo ẩn y. Một là nếu ∆ là số chính phương thì ta có thể phân tích thành nhân

tử rồi kết hợp với phương trình 1 tìm nghiệm,hai là ta có thể tìm điều kiện của xvà y để biện

luận phương trình.

Giải:

Từ phương trình (2) ta có: x2 + (y − 3)x+ (y − 2)2 = 0

Để phương trình có nghiệm thì: ∆ = (y − 3)2 − 4(y − 2)2 ≥ 0 ⇐⇒ 1 ≤ y ≤ 7

3Tương tự ta viết phương trình (2) thành: y2 + (x− 4)y + x2 − 3x+ 4 = 0

Để phương trình có nghiệm thì: ∆ = (x− 4)2 − 4(x2 − 3x+ 4) ≥ 0 ⇐⇒ 0 ≤ x ≤ 4

3

Từ đó ta suy ra: x4 + y2 ≤ 256

81+

49

9=

687

81<

698

81Vậy hệ phương trình đã cho vô nghiệm.

183

Page 185: Chuyên đề luyện thi đại học môn Toán - VipLam.Net

4 Giải hệ phương trình:4√y2x3 − 6x2y2 + 81 + 3 + 2012

√x2y2 − 9y2x+

18x2

y2= y2

x(x4 + y4) = y6(1 + y4)

Lời giải:

Phân tích: Ta cũng quan sát thấy phương trình (1) có cái căn bậc 2012 rất cồng kềnh và phức

tạp nên ta sẽ nghĩ hướng đi sẽ là ở phương trình (2). Quan sát thấy phương trình (2) có các

hạng tử x,y liên quan nhau nên ta thử chia cho y5.

Giải:

Xét y = 0 không là nghiệm của hệ phương trình,ta chia 2 vế của phương trình (2) cho y5 Từ

phương trìnnh thứ 2,ta có:x5

y5+x

y= y5 + y

Xét hàm f(t) = t5 + t, f ′(t) = 5t4 + 1 > 0∀t. ⇐⇒ x = y2 Thay vào phương trình (1) ta được:4√x4 − 6x3 + 81 + 2012

√x3 − 9x2 + 18x = x− 3

Đặt 4√x4 − 6x3 + 81 = a, 2012

√x3 − 9x2 + 18x = b(a, b > 0)

Ta có:

{a+ b = x− 3

a4 − 6b2012 = (x− 3)4=⇒ (a+ b)4 = a4 − 6b2012 =⇒ b = 0

=⇒ x = 6 hoặc x = 3 (nghiệm x = 0 loại)

Đáp số: (x; y) = (3;±√

3); (6;±√

6)

5 Giải hệ phương trình:x+ 6

√xy − y = 6 (1)

x+6(x3 + y3)

x2 + xy + y2−√

2(x2 + y2) = 3 (2)

Lời giải:

Phân tích: Dùng bất đẳng thức đề đánh giá nghiệm.

Giải:

Điều kiện:

{xy ≥ 0

x2 + xy + y2 6= 0

Nếux = 0 hoặc y = 0 thì hệ phương trình vô nghiệm

Nếu x ≤ 0, y ≤ 0 (x, y không đồng thời bằng 0) thì VT của (2) âm,

PT (2) không thỏa mãn.Do đó x > 0, y > 0.

Vì 2√xy ≤ x+ y nên PT (1) suy ra:

6 = x+√xy − y ≤ x+ 3(x+ y)− y = 4x+ 2y =⇒ 2x+ y ≥ 3 (3).

Mặt khác, ta có:

xy ≤ x2 + y2

2=⇒ x2 + xy + y2 ≤ 3(x2 + y2)

2=⇒ 3(x3 + y3)

x2 + xy + y2≥ 2(x3 + y3)

x2 + y2(4)

Ta chứng minh rằng:2(x3 + y3)

x2 + y2≥√

2(x2 + y2) (5)

184

Page 186: Chuyên đề luyện thi đại học môn Toán - VipLam.Net

Thật vậy BDT (5) tương đương với:

2(x3 + y3)2 ≥ (x2 + y2)3 ⇐⇒ x6 + y6 + 4x3y3 ≥ 3x4y2 + 3x2y4 (6)

Áp dụng BDT Cauchy ta có:

x6 + x3y3 + x3y3 ≥ 3 3√x12y6 = 3x4y2, y6 + x3y3 + x3y3 ≥ 3

√x6y12 = 3x2y4

Cộng vế theo vế ta được BDT (6) , suy ra BDT (5) đúng.

Từ (4) và (5) suy ra3(x3 + y3)

x2 + xy + y2≥√

2(x2 + y2)

Kết hợp với PT (2) và lưu ý rằng:√

2(x2 + y2) ≥ x+ y , ta được :

3 = x+6(x3 + y3)

x2 + xy + y2−√

2(x2 + y2) ≥ x+√

2(x2 + y2) ≥ x+ (x+ y) = 2x+ y (7)

Từ (3) và (7) suy ra 2x+ y = 3 và x = y. Ta được x = y = 1 ( thỏa mản điều kiện).

Đáp số: (x; y) = (1; 1).

6 Giải hệ phương trình: x+ y

1 + xy=

1− 2y

2− yx− y1− xy

=1− 3x

3− xLời giải:

Phân tích: Bài này nhìn vào rất phức tạp, không biết định hướng đi từ đâu, vì thế phãi cố

gắng tìm cách đặt ẩn đề đưa về một phương trình đơn giản hơn.

Giải:

Đặt: x =u− 1

u+ 1, y =

v − 1

v + 1 u− vu+ v

=2− uu+ 2

(1)

uv − 1

uv + 1=

3− v3 + v

(2)

Từ phương trình (1) ta cóu− vu+ v

=2− uu+ 2

=2− v

2 + v + 2u=

2 + v − 2u

2− v=⇒ (2− v)2 = (2 + v)2 − 4u2 =⇒ u2 = 2v

Từ phương trình (2) ta có :uv − 1

uv + 1=

3− v3 + v

=3u− uv3u+ uv

=3u− 1

3u+ 1 + 2uv=

3u+ 1− 2uv

3u− 1=⇒ (3u− 1)2 = (3u+ 1)2 − 4u2v2 ⇐⇒ u2v2 = 3u

Vậy ta có hệ:

{u2 = 2v

u2v2 = 3u(3)

Xét u = 0 =⇒ v = 0 =⇒ x = y = −1 =⇒ xy = 1 (loại do 1− xy 6= 0)

Như vậy (3) tương đương:

{u2 = 2v

uv2 = 3

Từ hệ trên suy ra u > 0 =⇒ u2v4 = 9 =⇒ 2v.v4 = 9 =⇒ v = 5

√92

=⇒ u2 = 5√

144

=⇒ u = 5√

12 (do u > 0)

Đáp số: (x; y) =

(5√12−15√12+1

;5√

92−1

5√

92

+1

)

185

Page 187: Chuyên đề luyện thi đại học môn Toán - VipLam.Net

7 Giải hệ phương trình:√xy + (x− y)(

√xy − 2) +

√x = y +

√y

(x+ 1)[y +√xy + x(1− x)

]= 4

Lời giải:

Phân tích: Ta thử và dự đoán hệ phương trình sẽ có nghiệm x = y. Vì thế ta sẽ tìm cách để

phân tích thành phương trình tích xuất hiện (x − y)(. . .). Từ đó ta quan sát và thấy phương

trình (1) là khả thi nhất.

Giải:

Điều kiện: x ≥ 0; y ≥ 0 Phương trình (1) tương đương:

⇐⇒√xy + (x− y)(

√xy − 2)− y + (

√x−√y) = 0

⇐⇒y(x− y) + (x− y)(

√xy − 2)√

xy + (x− y)(√xy − 2) + y

+x− y√x+√y

= 0

⇐⇒ (x− y)(y +√xy − 2√

xy + (x− y)(√xy − 2) + y

+1√

x+√y

) = 0

Từ PT (2) suy ra :

y +√xy =

4

x+ 1− x(1− x) =

4

x+ 1+ (x+ 1) + (x− 1)2 − 2 ≥ 2.

√4

x+ 1.(x+ 1) + (x− 1)2 − 2 ≥ 2

Từ đó ta suy ra x = y Thay x = y vào phương trình (2). Ta có:

x3 − 2x2 − 3x+ 4 = 0 ⇐⇒ (x− 1)(x2 − x− 1) = 0 ⇐⇒ x = 1 hoặc x2 − x− 1 = 0

Xét x = 1 ⇐⇒ y = 1

Xét x2 − x− 1 = 0 ta có x =1 +√

5

2⇐⇒ y =

1 +√

5

2hay x =

1−√

5

2(loại vì x ≥ 0)

Đáp số: (x; y) = (1; 1),

(1 +√

5

2;1 +√

5

2

)

8 Giải hệ phương trình: x3 + 3xy2 = 49x

x2 − 8xy + y2 = 8y − 17x

Lời giải:

Phân tích: Cũng giống như bài 1, bài này ta thấy các bậc của các phương trình trong hệ lần

lượt là 1,2 và 3 nên ta sẽ tìm 1 hệ số nhận và phương trình (1) hoặc (2) để nhóm lại phân tích

thành nhân tử.

Giải:

Ta thấy x = 0 là 1 nghiệm của hệ, x = 0 ⇐⇒ y = 0 hoặc y = 8

Hệ đã cho tương đương với:

{x3 + 3xy2 = 49x

3(x2 − 8xy + y2) = 3(8y − 17x)

Cộng vế theo vế ta thu được: (x+ 1)(x2 + 3y2 − 24y + 2x) = 0

Xét x = −1 ⇐⇒ y = ±4

Xét x2 + 3y2 − 24y + 2x = 0 ⇐⇒ 49− 24y + 2x = 0 ⇐⇒ x =24y − 49

2

186

Page 188: Chuyên đề luyện thi đại học môn Toán - VipLam.Net

Thay x =24y − 49

2vào phương trình (1) hoặc (2) của hệ ta giải ra tìm nghiệm. Đáp số:

(x; y) = (−1; 4), (−1;−4), (0; 0), (0; 8),(−13

2; 3

2

),(

112

; 52

)Lời bình: Mình đưa ra bài này chủ yếu để các bãn nhận thức trong việc nhân hệ số, có

thể có nhiều cách ngắn gọn hơn.

9 Giải hệ phương trình: x3 − 8x = y3 + 2y

x2 − 3 = 3(1 + y2)

Lời giải:

Phân tích: Nếu bài này ta làm như bình thường là thế thì sẽ là rất khó khăn trong việc xử lý.

Nên ta sẽ tìm 1 phương pháp khác, đó là phương pháp thế vào một hệ số nào đó của 1 phương

trình bằng 1 phương trình trong hệ để tạo sự đồng bậc giữa 2 phương trình.

Giải:

Hệ đã cho tương đương với:x3 − y3 = 2(4x+ y)

x2 − 3y2 = 6⇐⇒

3(x3 − y3) = 6(4x+ y)

x2 − 3y2 = 6

Thế x2 − 3y2 = 6 vào phương trình (1) khi đã nhân 3, ta được:

3(x3 − y3) = (x2 − 3y2)(4x+ y) ⇐⇒ x3 + x2y − 12xy2 = 0 (∗)Xét y = 0 không là nghiệm của hệ, ta chia y2 cho 2 vế của phương trình (∗), ta được:

(∗) ⇐⇒ x3

y3+x2

y2− 12x

y= 0

Đặt t =x

y, suy ra:

(∗) ⇐⇒ t3 + t2 − 12t = 0 ⇐⇒ t(t2 + t− 12) = 0

Xétt = 0 ⇐⇒ x = 0 ( không là nghiệm)

Xét t = 3 ⇐⇒ x = 3y

Xét t = −4 ⇐⇒ x = −4y

Thay lần lượt vào 1 trong hai phương trình ban đầu ta giải ra nghiệm.

Đáp số: (x; y) = (3; 1), (−3;−1),(−4√

613

;√

613

),(

4√

613,−√

613

)Sau đây là một bài tương tự để các bạn rèn luyện thêm:

1) Giải hệ phương trình:

x3 + 4y = y3 + 16x

1 + y2 = 5(1 + x2)Đáp số:

(x; y) = (−1; 3), (1;−3), (0; 2), (0;−2)

10 Giải hệ phương trình:(x− 3y) (6x+ 18y + 5) = 4x

(x2 + 2xy + 4y2) (8x− 16y − 9) + 9x2 + 4x = 78y − 18xy + 26

Lời giải:

Phân tích: Do 2 phương trình của hệ đều có phương trình tích nên ta sẽ phân phối và rút

187

Page 189: Chuyên đề luyện thi đại học môn Toán - VipLam.Net

gọn cho bớt cồng kềnh. Sau đó sẽ dùng các biện pháp để giải.

Giải:

Ta có:

(x− 3y)(6x+ 18y + 5) = 4x ⇐⇒ 6x2 + x = 54y2 + 15y

(x2 + 2xy + 4y2)(8x− 16y − 9) + 9x2 + 4x = 78y − 18xy + 26 ⇐⇒ 8x3 + 4x = 64y3 + 36y2 + 78y + 26

Như vậy ta viết hệ thành: 6x2 + x = 54y2 + 15y

8y3 + 4x = 64y3 + 36y2 + 78y + 26

Ta nhân phương trình thứ nhất với 2 rồi cộng với phương trình thứ hai thì thu được:

(2x+ 1)3 = (4y + 3)3.

Từ đây ta có: x = 2y + 1. Tới đây các bạn thế vào (1) hoặc (2) giải sẽ ra nghiệm.

Đáp số: (x; y) =

(12

5;

7

10

),

(1

3;−1

3

)11 Giải hệ phương trình: (x+

√x2 + 1)(y +

√y2 + 1) = 1

y +1√

5x2 − 1+−3

2= 0

Lời giải:

Phân tích: Ta biến đổi bằng cách dùng biểu thức liên hợp từ phương trình đầu .

Giải:

Từ phương trình đầu ta có :

(x+√x2 + 1)(x−

√x2 + 1)(y +

√y2 + 1) = x−

√x2 + 1 ⇐⇒ y +

√y2 + 1 =

√x2 + 1− x

Tương tự ta cũng có:

x+√x2 + 1 =

√y2 + 1− y

Cộng vế theo vế ta được x+ y = 0 Thay vào phương trình 2 ta được :

y +1√

5y2 − 1− 3

2= 0

Ta chuyển vế sau đó bình phương , ta được:

(y − 1)(2y + 1)(10y2 − 25y + 13) = 0

Ta chỉ nhận các nghiệm : y = 1, y = −1

2, y =

5−√

215

4, Từ đó ta suy ra nghiệm của hệ.

Đáp số: (x; y) = (−1; 1),

(1

2;−1

2

),

−5 +√

215

4;5−

√215

4

.

188

Page 190: Chuyên đề luyện thi đại học môn Toán - VipLam.Net

CÁC BÀI TOÁN

HỆ PHƯƠNG TRÌNH NÂNG CAO

(Lê Trung Tín)

1 Giải hệ phương trình x2 + xy + y2 + x+

y3

x+ 1= 2

2x+ y +y2

x+ 1= 2

Lời giải:

Ta biến đổi hệ trở thành hệ :

(x+ y + 1)

(x+

y2

x+ 1

)= 2

(x+ y + 1) +

(x+

y2

x+ 1

)= 3

2 Giải hệ phương trình (x2 + y2) (x+ y + 1) = 25 (y + 1)

x2 + xy + 2y2 + x− 8y = 9

Lời giải:

Ta biến đổi hệ trở thành hệ: (x2 + y2

y + 1

)(x+ y + 1) = 25(

x2 + y2

y + 1

)+ (x+ y + 1) = 10

3 Giải hệ phương trình {x√y2 + 6 + y

√x2 + 3 = 7xy

x√x2 + 3 + y

√y2 + 6 = 2 + x2 + y2

Lời giải:

189

Page 191: Chuyên đề luyện thi đại học môn Toán - VipLam.Net

Ta biến đổi hệ trở thành hệ: √x2 + 3 + x

x+

√y2 + 6 + y

y= 9

3x√x2 + 3 + x

+6y√

y2 + 6 + y= 2

4 Giải hệ phương trình x3 + y3 + 6xy = 8

x2 + y2 = 2x+ y + 14

Lời giải:

Ta biến đổi phương trình thứ nhất của hệ:

x3+y3+6xy = 8 ⇐⇒ (x+y)3−8−3xy(x+y−2) = 0 ⇐⇒ (x+y−2)

((x+ y + 4)2 + 3(x− y)2

4

)= 0

5 Giải hệ phương trình 8y3 + 6y + 1 +√

3x2 + 16y2 = 19

x2 + 4y2 = 4

Lời giải:

Xét hàm f(y) = 8y3 + 6y + 1, g(x) =√

16− x2 với x ∈ [−2; 2], y ∈ [−1; 1]. Khi đó

f(y) + g(x) ≤ f(1) + g(0) = 19

6 Giải hệ phương trình 4x3 + 4y3 = 3x2y + 2√

3xy + 2x

x2 = y2 + 1

Lời giải:

Đặt y = tan a, x =1

cos a

7 Giải hệ phương trìnhy3 + 3xy − 17x+ 18 = x3 − 3x2 + 13y − 9

x2 + y2 + xy − 6y − 5x+ 10 = 0

Lời giải:

Từ phương trình (2) ta tìm được :5

3≤ y ≤ 3,

2

3≤ x ≤ 2

Nhân phương trình (2) với −3 và cộng với phương trình (1) ta có : (y− 1)3 + 2(y− 1) = x3 + 2x

190

Page 192: Chuyên đề luyện thi đại học môn Toán - VipLam.Net

8 Giải hệ phương trình 1 + x3y3 = 19x3

x+ xy2 = −6x2

Lời giải:

Ta đưa về hệ sau:

(

1

x+ y

)3

− 3y

x

(1

x+ y

)= 19

y

x

(1

x+ y

)= −6

9 Giải hệ phương trình xy2 + y3 + 3x− 6y = 0

x2 + xy − 3 = 0

Lời giải:

Thế từ pt thứ hai vào pt thứ nhất, ta được: x3 + y3 − x2y − xy2 = 0

10 Giải hệ phương trình x(y2 + 1)

x2 + y2=

3

5y(x2 − 1)

x2 + y2=

4

5

Lời giải:

Lần lượt lấy phương trình (1) nhân cả 2 vế với x, phương trình (2) nhân cả 2 vế với y, rồi cộng

2 vế của 2 phương trình trên ta được3x

5− 4y

5= 1 ⇐⇒ 3x− 4y = 5

11 Giải hệ phương trình xy2 + 3x2 = 2y

y2 + x2y = −2x

Lời giải:

Hệ viết lại

3x3 = 2xy − x2y2

y3 = −2xy − x2y2

12 Giải hệ phương trình {(√x2 + 1 + x)(

√y2 + 1 + y) = 1

4√x+ 2 +

√22− 3x = y2 + 8

Lời giải:

Từ PT (1), suy ra:√x2 + 1 + x =

√y2 + 1− y. Đến đây xét hàm số: f (t) =

√t2 + 1 + t

191

Page 193: Chuyên đề luyện thi đại học môn Toán - VipLam.Net

13 Giải hệ phương trình 1

x+

1

2y= (x2 + 3y2)(3x2 + y2)

1

x− 1

2y= 2(y4 − x4)

Lời giải:

Biến đổi thành hệ sau:

2

x= x4 + 5y4 + 10x2y2

1

y= 5x4 + y4 + 10x2y2

⇐⇒

2 = x5 + 5xy4 + 10x3y2

1 = 5x4y + y5 + 10x2y3

14 Giải hệ phương trình x3 + 3xy2 = −49

x2 − 8xy + y2 = 8y − 17x

Lời giải:

Nhân phương trình thứ hai của hệ đã cho với 3 rồi cộng vào phương trình thứ nhất, ta được

(x+ 1)((x+ 1)2 + 3(y − 4)2

)= 0

15 Giải hệ phương trình x4 − y4 = 240

x3 − 2y3 = 3(x2 − 4y2)− 4(x− 8y)

Lời giải:

Nhân phương trình thứ hai với -8 rồi cộng vào với phương trình thứ nhất, ta được (x − 2)4 =

(y − 4)4

16 Giải hệ phương trình √

3x

(1 +

1

x+ y

)= 2

√7x

(1− 1

x+ y

)= 4√

2

Lời giải:

Biến đổi về hệ sau

1 +

1

x+ y=

2√3x

1− 1

x+ y=

4√

2√7x

⇐⇒

1

x+ y=

1√3x− 2√

2√7x

1 =1√3x

+2√

2√7x

17 Giải hệ phương trình 2x3 − 9y3 = (x− y)(2xy + 3)

x2 − xy + y2 = 3

192

Page 194: Chuyên đề luyện thi đại học môn Toán - VipLam.Net

Lời giải:

Thế 3 = x2 − xy + y2 vào phương trình thứ nhất của hệ, ta được

2x3 − 9y3 = (x− y)(2xy + x2 − xy + y2)

18 Giải hệ phương trình 9y3(3x2 − 1) = −125

45x2y + 75x = 6y2

Lời giải:

Biến đổi về hệ sau:

27x3 +

125

y3= 9

45x2

y+

75x2

y2= 6

⇐⇒

(3x)3 +

(5

y

)3

= 9

3x.5

y

(3x+

5

y

)= 6

19 Giải hệ phương trình 8(x2 + y2) + 4xy +

5

(x+ y)2= 13

2x+1

x+ y= 1

Lời giải:

Biến đổi về hệ sau5(x+ y)2 +

5

(x+ y)2+ 3(x− y)2 = 13

2x+1

x+ y= 1

⇐⇒

5

(x+ y +

1

x+ y

)2

+ 3(x− y)2 = 23(x+ y +

1

x+ y

)+ (x− y) = 1

20 Giải hệ phương trình(17− 3x)√

5− x+ (3y − 14)√

4− y = 0

2√

2x+ y + 5 + 3√

3x+ 2y + 11 = x2 + 6x+ 13

Lời giải:

Phương trình thứ nhất của hệ tương đương (3 (5− x) + 2)√

5− x = (3 (4− y) + 2)√

4− y.Xét hàm số f (t) = (3t2 + 2) t với t ∈ R+.

21 Giải hệ phương trình 2x3 + 3x2 − 18 = y3 + y

2y3 + 3y2 − 18 = z3 + z

2z3 + 3z2 − 18 = x3 + x

Lời giải:

Xét hàm số f(t) = 2t3 + 3t2 − 18 và g(t) = t3 + t.

193

Page 195: Chuyên đề luyện thi đại học môn Toán - VipLam.Net

Ta có g′(t) = 3t2 + 1,∀t ∈ R.Suy ra g(t) đồng biến trên R.

Khi đó, hệ phương trình được viết lại dưới dạng:

f(x) = g(y)

f(y) = g(z)

f(z) = g(x)

Không mất tính tổng quát giả sử x = max{x, y, z}. Khi đóx ≥ y

x ≥ z⇐⇒

g(x) ≥ g(y)

g(x) ≥ g(z)⇐⇒

x ≥ y

x ≥ z

⇐⇒

(x− 2)(x2 + 5x+ 9) ≤ 0

(z − 2)(z2 + 5z + 9) ≥ 0⇐⇒

x ≤ 2

x ≥ 2

Từ đây, suy ra 2 ≤ z ≤ x ≤ 2 =⇒ x = z = 2, thế vào hệ phương trình, ta được y = 2.

Vậy hệ có nghiệm là (x; y; z) = (2; 2; 2).

22 Giải hệ phương trình 2x − 2y = (y − x)(xy + 2)

x2 + y2 = 2

Lời giải:

Thế 2 = x2 + y2 vào phương trình thứ nhất, ta được 2x + x3 = 2y + y3.

Xét hàm số f(t) = 2t + t3 với t ∈ R.

23 Giải hệ phương trình

x3 + 3x2 + 2x− 5 = y

y3 + 3y2 + 2y − 5 = z

z3 + 3z2 + 2z − 5 = x

Lời giải:

Biến đổi thành hệ sau:

(x− 1)(x2 + 4x+ 6) = y − 1

(y − 1)(y2 + 4y + 6) = z − 1

(z − 1)(z2 + 4z + 6) = x− 1

24 Giải hệ phương trình

x3 + y = 3x+ 4

2y3 + z = 6y + 6

3z3 + x = 9z + 8

Lời giải:

Biến đổi thành hệ sau:

(x− 2)(x+ 1)2 = 2− y

2(y − 2)(y + 1)2 = 2− z

3(z − 2)(z + 1)2 = 2− x

194

Page 196: Chuyên đề luyện thi đại học môn Toán - VipLam.Net

25 Giải hệ phương trình

2(x− y)(1 + 4xy) =√

3

x2 + y2 = 1;

x√

1− y2 + y√

1− x2

(1− x)(1 + y) = 2

Lời giải:

Đặt x = cosα, y = cos β

26 Giải hệ phương trình

2x+ x2y = y

2y + y2z = z

2z + z2x = x

Lời giải:

Biến đổi thành hệ:

y =

2x

1− x2

z =2y

1− y2

x =2z

1− z2

. Đặt t = tanα

27 Giải hệ phương trình

3

(x+

1

x

)= 4

(y +

1

x

)= 5

(z +

1

z

)xy + yz + zx = 1

Lời giải:

Đặt x = tanα, y = tan β, z = tan γ.

28 Giải hệ phương trình

xy + x− 2 = 0

2x3 − x2y + x2 + y2 − 2xy − y = 0

Lời giải:

Đưa phương trình thứ hai về tích (x2 − y)(2x− y + 1) = 0

29 Giải hệ phương trình

5x2y − 4xy2 + 3y3 − 2(x+ y) = 0

xy(x+ y)2 + 2 = (x+ y)2

Lời giải:

Đưa phương trình thứ hai về tích (xy − 1)(x2 + y2 − 2) = 0

30 Giải hệ phương trình

(x+ y − 3)3 = 4y3(x2y2 + xy + 45

4

)x+ 4y − 3 = 2xy2

Lời giải:

Từ pt thứ hai ta có y3(2xy − 3)3 = (x + y − 3)3. Kết hợp với pt đầu ta được: y3(2xy − 3)3 =

4y3

(x2y2 + xy +

45

4

)

195

Page 197: Chuyên đề luyện thi đại học môn Toán - VipLam.Net

31 Giải hệ phương trình

x4 + 2xy + 6y − (7 + 2y)x2 = −9

2x2y − x3 = 10

Lời giải:

Đưa phương trình thứ hai về dạng (x2−y)2−6(x2−y)+9 = (x−y)2 ⇐⇒ (x2−y−3)2 = (x−y)2

32 Giải hệ phương trình

2x+√

2− x+ y − x2 − y2 = 1

2x3 = 2y3 + 1

Lời giải:

Biến đổi về hệ sau:

5x2 − 3x− 1 = y − y2

2x3 − 2y3 = 1⇐⇒

5x2 − 3x+ y2 − y = 1

2x3 − 2y3 = 1

Do đó:

2x3 − 2y3 = 5x2 − 3x+ y2 − y ⇐⇒ (x− y − 1)(2x2 − 3x+ 2xy + 2y2 − y

)= 0

196

Page 198: Chuyên đề luyện thi đại học môn Toán - VipLam.Net

CÁC BÀI TOÁN

PHƯƠNG TRÌNH & BẤT PHƯƠNG TRÌNH

VÔ TỶ (Đinh Văn Trường)

Bài 1. Giải phương trình:

2√x+ 1√

x+ 1−√

3− x= 2x− 1

Lời giải

Với mục tieu lam đơn giản mẫu số của vế phải nên nhân liên hơp cho biểu thức:

√x+ 1 +

√3− x

Ta thu được phương trình sau:

2x+ 2 + 2√

(x+ 1)(3− x) = (2x− 1)(2x− 2)

Hay: √−x2 + 2x+ 3 = −2(−x2 + 2x)

. Phương trình này là dạng cơ bản.

Bài 2. Giải phương trình:8

x2+√x =

5

2

Lời giải

ĐK: x > 0. Áp dụng bất đẳng thức AM-GM cho năm số dương gồm một số8

x2và bốn số

√x

4ta có

8

x2+ 4

√x

4≥ 5

2

Đẳng thức xảy ra khi8

x2=

√x

4⇐⇒ x = 4. Vậy x = 4 là nghiệm duy nhất của phương trình.

Bài 3. Giải BPT √6(x2 − 3x+ 1) +

√x4 + x2 + 1 ≤ 0

197

Page 199: Chuyên đề luyện thi đại học môn Toán - VipLam.Net

Lời giải

Ta có: x4 + x2 + 1 = (x2 + 1)2 − x2 = (x2 + x+ 1) (x2 − x+ 1) Đặt a =

√x2 + x+ 1; b =√

x2 − x+ 1. BPT trở thành: √6(2b2 − a2

)+ ab ≤ 0

Có lẽ dễ rồi.»>

Bài 4. Giải phương trình:

x =√x2 − 7x+ 12 +

√x2 − 9x+ 20 +

√x2 − 8x+ 15

Lời giải

Điều kiện :

x ≥ 0

x2 − 7x+ 12 ≥ 0

x2 − 9x+ 20 ≥ 0

x2 − 8x+ 15 ≥ 0

⇐⇒

x ≥ 0

(x− 3)(x− 4) ≥ 0

(x− 4)(x− 5) ≥ 0

(x− 3)(x− 5) ≥ 0

⇐⇒

[0 ≤ x ≤ 3

x ≥ 5

Qua bước tìm điều kiện của bài toán ta thấy được các đại lượng x − 3, x − 4, x − 5 luân

phiên xuất hiện chéo nhau nên một điều tự nhiên ta nghỉ đến là để có thể tối ưu hóa phương

trình đã cho phải chăng ta nên tìm đến con đường ẩn phụ hóa bài toán.

Nhưng chúng ta, để ý phía phải phương trình thì cũng nên để ý phía trái phương trình

vì đó là ’một thể thống nhất" của bài toán. Và rõ ràng ta quan sát thấy được các đại lượng

x− 3, x− 4, x− 5 luôn chứa đại lượng x nên cho dù ta chủ yếu ẩn phụ hóa ba đại lượng kia

nhưng ta vẫn ó thể kéo ẩn phụ theo đại lượng x.

Thật vậy ta đặt : a =√x− 3, b =

√x− 4, c =

√x− 5, a, b, c ≥ 0. Khi đó ta có : x =

3 + a2, x = 4 + b2, x = 5 + c2.

Và lúc này theo phương trình đã cho cộng hưởng với việc biểu diễn x qua ba đại lượng ẩn phụ

a, b, c ta sẽ thu được một hệ phương trình sau :3 + a2 = ab+ bc+ ca (1)

4 + b2 = ab+ bc+ ca (2)

5 + a2 = ab+ bc+ ca (3)

Quan sát thấy ở hệ này có cái gì đó làm chúng ta hơi nhát tay không dám biến đổi tiếp nữa.

Bình tỉnh một chút ta quan sát ở vế trái của ba phương trình (1), (2) và (3) ta cảm giác được

có gì đó liên quan đến việc nhân tử chung.

Thật vây ta có :

ab+ bc+ ca = ab+ c(a+ b)

ab+ bc+ ca = a(b+ c) + bc

ab+ bc+ ca = b(a+ c) + ac

Vì sao? ta lại đưa các phép biến đổi về như vậy.

Câu trả lời là do vế trái của (1) chứa a2. vế trái (2) chứa b2 và vế trái (3) chứa c2.

198

Page 200: Chuyên đề luyện thi đại học môn Toán - VipLam.Net

Tuy nhiên đó chỉ là cảm giác còn thực tế thì thật khác xa vì các dấu đứng trước a2, b2, c2 đều

là dấu (+) nên rõ ràng ý đồ bắt nhân tử chung lúc này cho mỗi phương trình trong hệ xem như

"tan thành mây khói".

Nhưng mà giữa cơn tuyệt vọng, con người ta luôn ao ước có thêm niềm hy vọng để có thể chiến

đấu và ta sẽ ước mơ dấu (+) kia sao nó không thành dấu (−) ?

Mặt khác, muốn có dấu trừ thì buộc ta phải thay đổi lại "chiến thuật" để có thể "xoay đổi càn

khôn".

Có nghĩa là bây giờ ta cần phải có : x = 3− a2, x = 4− b2, x = 5− c2.

Điều này cũng có nghĩa là ta cần thay đổi : a =√

3− x, b =√

4− x, c =√

5− x.Một sự thay đổi quá lớn lao phải không? Không, sự thay đổi này không hề lớn lao vì ta đã biết

được : √x2 − 7x+ 12 =

√(x− 3)(x− 4) =

√(3− x)(4− x)

√x2 − 9x+ 20 =

√(x− 5)(x− 4) =

√(5− x)(4− x)

√x2 − 8x+ 15 =

√(x− 3)(x− 45) =

√(3− x)(5− x)

Vậy ta đã hoàn thành được "sự biến chuyển lớn" với sự thay đổi này, kèm với sự thay đổi này

ta đưa phương trình đầu về hệ phương trình :3− a2 = ab+ bc+ ca

4− b2 = ab+ bc+ ca

5− a2 = ab+ bc+ ca

⇐⇒

3 = (a+ b)(a+ c)

4 = (b+ c)(a+ b)

5 = (a+ c)(c+ b)

⇐⇒

a+ b =

2√

15

5

b+ c =2√

15

3

c+ a =

√15

2

=⇒ a =7√

15

60

Từ đó ta có :x = 3−

(7√

15

60

)2

=671

240. Đối chiếu điều kiện và thử lại ta có x =

671

240là nghiệm

của phương trình.

Bài 5. Giải bất phương trình:

5 + 8√

1− x < 3x+ 4(√

1 + x+√

1− x2)

Lời giải

Điều kiện : −1 ≤ x ≤ 1. Bất phương trình đã cho biến đổi thành bất phương trình :

5 + 4(

2√

1− x−√

1 + x)− 3x− 4

√1− x2 < 0

Để ý rằng :(2√

1− x−√

1 + x)2

= 5− 3x− 4√

1− x2 Tới đây hướng giải bài toán đã quá rõ

ràng.

199

Page 201: Chuyên đề luyện thi đại học môn Toán - VipLam.Net

Bài 6. Giải phương trình : √1− xx

=2x+ x2

1 + x2

Lời giải ĐK:0 < x < 1

Dễ thấy rằng phương trình trên có một nghiệm x =1

2bằng cách bấm nhanh cái máy tính. Ta

có PT

⇐⇒ (1 + x2)√

1− x = (2x+ x2)√x

⇐⇒ x2(√

1− x−√x) + (

√1− x− 2x

√x) = 0

⇐⇒ x2(1− 2x)√1 + x+

√x

+1− x− 4x3

√1− x+ 2x

√x

= 0

⇐⇒ x2(1− 2x)√1 + x+

√x

+(1− 2x)(2x2 + x− 1)√

1− x+ 2x√x

= 0

⇐⇒ (1− 2x)

[x2

√1 + x+

√x

+2x2 + x− 1√1− x+ 2x

√x

]= 0

Vìx2

√1 + x+

√x

+2x2 + x− 1√1− x+ 2x

√x> 0 Nên (1) ⇐⇒ 1− 2x = 0 ⇐⇒ x =

1

2

Vậy phương trình đã cho có nghiệm duy nhất x = 12

Tổng quát:

√a− bxcx

=(b+ c)x+ x2

a+ x2với a, b, c > 0

Đk: ....0 < x ≤ a

bCó √

a− bxcx

=(b+ c)x+ x2

a+ x2

⇐⇒√a− bxcx

− 1 =(b+ c)x+ x2

a+ x2− 1

⇐⇒

a− bxcx

− 1√a− bxcx

+ 1

=(b+ c)x+ x2 − a− x2

a+ x2

⇐⇒ a− (b+ c)x

cx

(√a− bxcx

+ 1

) +a− (b+ c)x

a+ x2= 0

Vì1

cx

(√a− bxcx

+ 1

) +1

a+ x2> 0 với a, b, c > 0 và 0 < x ≤ a

b=⇒ x =

a

b+ c

Bài 7. Giải bất phương trình :

√2x− 11 + x√

2 (x2 − 8x+ 14) + 5≥ 1

.

200

Page 202: Chuyên đề luyện thi đại học môn Toán - VipLam.Net

Lời giải

Điều kiện x ≥ 11

2. Với điều kiện đó, bất phương trình tương đương với

√2x− 11 + x−

√2x2 − 16x+ 28− 5 ≥ 0

⇐⇒(√

2x− 11− (x− 5))

+(

(2x− 10)−√

2x2 − 16x+ 28)≥ 0

⇐⇒ −(x− 6)2

√2x− 11 + x− 5

+2(x− 6)2

2x− 10 +√

2x2 − 16x+ 28≥ 0

⇐⇒ (x− 6)2

(2

2x− 10 +√

2x2 − 16x+ 28− 1√

2x− 11 + x− 5

)≥ 0

⇐⇒ (x− 6)2(

2√

2x− 11−√

2x2 − 16x+ 28)≥ 0

⇐⇒ (x− 6)2

(−2(x− 6)2

2√

2x− 11 + 2x2 − 16x+ 28

)≥ 0 ⇐⇒ x = 6.

Vậy tập nghiệm bất phương trình S = {6}.

Bài 8. Giải phương trình :

8x2 − 13x+ 7 = (1 +1

x)

3√

3x2 − 2

Lời giải

PT trên tương đương

(2x− 1)3 − (x2 − x− 1) = (x+ 1) 3√

(x+ 1)(2x− 1) + x2 − x− 1

Đặt

u = 2x− 1

v = 3√

3x2 − 1, đưa bài toán về giải hệ đối xứng rất đẹp sau

u3 − (x2 − x− 1) = (x+ 1)v

v3 − (x2 − x− 1) = (x+ 1)u

Trừ vế theo vế hai PT thu được

(u− v)(u2 + uv + v2 + x+ 1) = 0

Bài 9. Giải phương trình

√x2 + x− 1 +

√−x2 + x+ 1 = x2 − x+ 2

Lời giải

Nhờ vào ông Cauchy ta có

2(x2 − x+ 2) ≤ 2x+ 2 ⇐⇒ (x− 1)2 ≤ 0 =⇒ x = 1.

201

Page 203: Chuyên đề luyện thi đại học môn Toán - VipLam.Net

Thử lại thấy thỏa mãn và chắc chắn PT ban đầu nhận x = 1 là nghiệm duy nhất.

Bài 10. Giải phương trình

7x2 − 13x+ 8 = 2x2 3√x(1 + 3x− 3x2)

Lời giải

+ TH1: x.= 0, không là nghiệm.

+ TH2: x 6= 0, chia cả 2 vế của phương trình cho x3, ta được

8

x3− 13

x2+

7

x= 2

3

√1

x2+

3

x− 3

Đặt t =1

x, phương trình trở thành:

8t3 − 13t2 + 7t = 23√t2 + 3t− 3

⇐⇒ (2t− 1)3 + 2(2t− 1) = t2 + 3t− 3 + 23√t2 + 3t− 3

Xét hàm số: f(u) = u3 + 2u. Ta có f ′(u) = 3u2 + 2 > 0 , suy ra hàm f(u) đồng biến với mọi u.

Suy ra:

2t− 1 =3√t2 + 3t− 3

⇐⇒ t = 1; t =1

16

(5 +√

89)

Bài 11. Giải phương trình: √x+ 1 + 1 = 2

√x+ 9x2

Lời giải

Lùa cho nó sang một vế cho đẹp(2√x−√x+ 1

)+ (9x2 − 1) = 0 Tới đây cứ mạnh tay mà

phang thôi : (2√x−√x+ 1

)+(9x2 − 1

)= 0

⇐⇒ (3x− 1)

2√x+√x+ 1

+ (3x− 1) (3x+ 1) = 0

⇐⇒ (3x− 1)

(1

2√x+√x+ 1

+ 3x+ 1

)︸ ︷︷ ︸

A>0

= 0 ⇐⇒ x =1

3

Bài 12. Giải phương trình:

x√x+√x+ 12 = 12

(√5− x+

√4− x

)Lời giải

ĐK: 0 ≤ x ≤ 4.

Xét f(x) = x√x+√x+ 12, 0 ≤ x ≤ 4. Ta có: f ′(x) =

3

2

√x+

1

2√x+ 12

> 0 , với mọi 0 ≤ x ≤ 4

202

Page 204: Chuyên đề luyện thi đại học môn Toán - VipLam.Net

Suy ra f(x) là hàm đồng biến trên [0; 4]

Xét g(x) = 12(√

5− x+√

4− x), 0 ≤ x ≤ 4. Ta có: g′(x) =

−6√5− x

+−6√4− x

< 0, 0 ≤ x ≤ 4.

Suy ra , g(x) làm hàm nghịch biến trên 0 ≤ x ≤ 4

Do đó f(x) = g(x) có nghiệm duy nhất. Lại có f(4) = g(4). Vậy x = 4 là nghiệm duy nhất của

pt

Bài 13. Giải phương trình :

√2− x2 +

√2− 1

x2= 4−

(x+

1

x

)Lời giải

Điều kiện x ∈[−√

2;− 1√2

]∪[

1√2

;√

2

]. Áp dụng BCS cho VT ta được

V T 2 ≤ 2

[6− (x+

1

x)2

]=⇒ 3(x+

1

x)2 − 8(x+

1

x) + 4 ≤ 0

⇐⇒(x+

1

x− 2

)[3(x+

1

x)− 2

]≤ 0 (∗)

Nếu x ∈[−√

2;− 1√2

]thì (∗) vô lý.

Nếu x ∈[

1√2

;√

2

]thì ta có x+

1

x≥ 2 do đó x+

1

x−2 ≥ 0 dấu bằng khi x = 1 và 3(x+

1

x)−2 > 0.

Do đó (∗) đúng khi và chỉ khi x+1

x= 2 hay x = 1.

Vậy nghiệm của phương trình đã cho là x = 1. Thử lại thấy đúng.

Bài 14. Giải phương trình

√12− x

√15− x+

√15− x

√20− x+

√20− x

√12− x = x

Lời giải

ĐK: x ≤ 12.

Đặt√x− 12 = a =⇒

√15− x =

√a2 + 3;

√20− x =

√a2 + 8;x = 12 − a2. Phương trình

tương đương với

a√a2 + 3 +

√a2 + 3

√a2 + 8 +

√a2 + 8a+ a2 = 12

Dễ thấy f(a) = a√a2 + 3 +

√a2 + 3

√a2 + 8 +

√a2 + 8a+ a2 nên f(a) đồng biến (do a ≥ 0).

Và f(1) = 12. Nên a = 1 là nghiệm duy nhất. Tức là:√

12− x = 1 ⇐⇒ x = 11.

Bài 15. Giải bất phương trình :(4x2 − x− 7

)√x+ 2 > 10 + 4x− 8x2

Lời giải

203

Page 205: Chuyên đề luyện thi đại học môn Toán - VipLam.Net

ĐK:x ≥ −2. Bất phương trình đã cho

⇐⇒ (4x2 − x− 7)√x+ 2 + 8x2 − 2x− 14 > 2(x− 2)

⇐⇒ (4x2 − x− 7)(

2 +√x+ 2

)> 2

(√x+ 2− 2

)(√x+ 2 + 2

)⇐⇒ 4x2 − x− 7 > 2

√x+ 2− 4 ⇐⇒ 4x2 > 2

√x+ 2 + x+ 3

⇐⇒ (2x)2 >(√

x+ 2 + 1)2

⇐⇒(√

x+ 2 + 1− 2x)(√

x+ 2 + 1 + 2x)< 0

Vậy nghiệm của bất phương trình là:[−2; 1) ∪

(5 +√

41

8; +∞

)

Bài 16. Giải phương trình sau:

x+ 2

√3x− 1

5= 4

4

√x4 + 4

20

Lời giải

Ta có:

2

√3x− 1

5.1 ≤ 3x− 1

5+ 1 =

3x+ 4

5

4

√x4 + 4

20=

4

√(x4 + 4)(16 + 4)

400≥ 4

√(4x2 + 4)2

400= 2

√x2 + 1

20.

Do đó: V T ≤ x+3x+ 4

5;V P ≥ 8

√x2 + 1

20Từ đó

x+3x+ 4

5≥ 8

√x2 + 1

20⇐⇒ 8x+ 4 ≥ 40

√x2 + 1

20⇐⇒ 2x+ 1 ≥ 10

√x2 + 1

20

⇐⇒

x >−1

24x2 + 4x+ 1 ≥ 5x2 + 5

⇐⇒

{x > −1

2

(x− 2)2 ≤ 0⇐⇒ x = 2.

Vậy phương trình đã cho có nghiệm x = 2

Bài 17. Giải phương trình sau:

3√

3x− 5 = 8x3 − 36x2 + 53x− 25

Lời giải

PT ⇐⇒ (2x− 3)3 + (2x− 3) = (3x− 5) + 3√

3x− 5

Bài 18. Giải bất phương trình :

4x2 +√

2x+ 3 ≥ 8x+ 1

204

Page 206: Chuyên đề luyện thi đại học môn Toán - VipLam.Net

Lời giải

Cách 1. ĐK : x ≥ −3

2

4x2 − 8x− 1 + 2x+ 3 ≥ 2x+ 3 +√

2x+ 3

⇐⇒ (1− 2x)2 + (1− 2x) ≥ (2x+ 3) +√

2x+ 3

⇐⇒ (1− 2x−√

2x+ 3)(2− 2x+√

2x+ 3) ≥ 0

Cách 2. ĐK : x ≥ −3

2Giải phương trình : 4x2 +

√2x+ 3 = 8x+ 1 =⇒ 4x2 +

√2x+ 3− 8x− 1 = 0.

Giải được 2 nghiệm x1 =5−√

21

4< x2 =

3 +√

17

4

Xét hàm số : f (x) = 4x2 +√

2x+ 3− 8x− 1;x ≥ −3

2.

Ta có: hàm số f (x) liên tục trên

[−3

2; + ∝

)nên f (x) không đổi dấu trên mỗi khoảng

(− ∝;x1) ; (x1;x2) ; (x2; + ∝) .

Khi đó ta có dấu của f (x) Biểu diễn trên trục số. Nhìn vào dấu của f (x) ta thấy

f (x) ≥ 0 ⇐⇒

−3

2≤ x ≤ 5−

√21

4

x ≥ 3 +√

17

4

Bài 19. Giải phương trình sau trên tập số thực :

3x+1√

2x− 7+

1√x− 3

= 2(

4√

2x− 7 + 4√x− 3 + 5

)Lời giải

Điều kiện x >7

2.

Đặt a = 4√

2x− 7, b = 4√x− 3, phương trình trở thành

a4 + b4 +1

a2+

1

b2= 2a+ 2b

Chuyển vế nhận được(a2 − 1

a

)2

+

(b2 − 1

b

)2

= 0 ⇐⇒

{a2 = 1

a

b2 = 1b

⇐⇒ x = 4

Vậy phương trình có nghiệm duy nhất x = 4.

Bài 20. Giải phương trình :

√x+ 1 +

4(√

x+ 1 +√x− 2

)3(√

x− 2 + 1)2 = 3

205

Page 207: Chuyên đề luyện thi đại học môn Toán - VipLam.Net

Lời giải

Cách 1. Điều kiện x ≥ 2. Đặt t =√x− 2 ≥ 0 =⇒ x = t2 + 2 Phương trình đã cho trở thành:

3(t+ 1)2√t2 + 3 + 4(

√t2 + 3 + t) = 9(t+ 1)2

⇐⇒√t2 + 3(3t2 + 6t+ 7) = 9t2 + 14t+ 9 (?)

⇐⇒√t2 + 3 =

9t2 + 14t+ 9

3t2 + 6t+ 7

Dùng máy tính nhẩm được :t = 1 nên ta giải tiếp như sau:

⇐⇒√t2 + 3− 2 =

9t2 + 14t+ 9

3t2 + 6t+ 7− 2

⇐⇒ t2 − 1√t2 + 3 + 2

=3t2 + 2t− 5

3t2 + 6t+ 7

⇐⇒ (t− 1)

(t+ 1√t2 + 3 + 2

− 3t+ 5

3t2 + 6t+ 7

)= 0

Bây giờ ta xử tiếp thằng:

t+ 1√t2 + 3 + 2

− 3t+ 5

3t2 + 6t+ 7= 0

⇐⇒ t+ 1√t2 + 3 + 2

− 1

2+

1

2− 3t+ 5

3t2 + 6t+ 7= 0

⇐⇒ 2t−√t2 + 3

2(√t2 + 3 + 2)

+3t2 − 3

2(3t2 + 6t+ 7)= 0

⇐⇒ (3t2 − 3)

(1

2(√t2 + 3 + 2)(2t+

√t2 + 3)

+1

2(3t2 + 6t+ 7)

)= 0

Đến đây thì ngon rồi nhé.

Cách 2. ĐK: x ≥ 2

PT đã cho tương đương với:

3(√x+ 1− 1) +

4(√x+ 1− 1)

(√x− 2 + 1)2

+4√

x− 2 + 1= 6

Áp dụng BĐT AM-GM:

6 = 2[2√

x− 2 + 1+(√x+ 1−1)]+[(

√x+ 1−1)+

4(√x+ 1− 1)

(√x− 2 + 1)2

] ≥ 4√

2

√√x+ 1− 1√x− 2 + 1

+4

√x+ 1− 1√x− 2 + 1

Nên

√x+ 1− 1√x− 2 + 1

≤ 1

2. Suy ra (x− 3)2 ≤ 0. Vậy x = 3 . Thử lại thấy thỏa mãn.

Cách 3. Điều kiện: x ≥ 2 Đặt a =√x+ 1, b =

√x− 2 =⇒

a, b ≥ 0

a− b > 0

a2 − b2 = 3

. Phương trình đã

206

Page 208: Chuyên đề luyện thi đại học môn Toán - VipLam.Net

cho trở thành:

a+4 (a+ b)

3(b+ 1)2 = 3

⇐⇒ a+4 (a+ b)

(a2 − b2) (b+ 1)2 = 3

⇐⇒ a+1

(a− b)(b+ 1

2

)(b+ 1

2

) = 3 (∗)

Áp dụng bất đẳng thức Cauchy, ta có

a+1

(a− b)(b+ 1

2

)(b+ 1

2

) ≥ a+27(

a− b+b+ 1

2+b+ 1

2

)3

= a+27

(a+ 1)3

=a+ 1

3+

27

(a+ 1)3 +2 (a+ 1)

3− 1

≥ 6

a+ 1+

2 (a+ 1)

3− 1

≥ 3

Do đó dấu “ =′′ trong phương trình (*) xảy ra khi

a− b =b+ 1

2a2 − b2 = 3

6

a+ 1=

2 (a+ 1)

3a+ 1

3=

27

(a+ 1)3

⇐⇒

a = 2

b = 1⇐⇒ x = 3

Vậy x = 3 là nghiệm của phương trình.

Bài 21. Giải phương trình:

√x− 2 +

√4− x = 2x2 − 5x− 1

Lời giải

ĐK: 2 ≤ x ≤ 4. Khi đó

PT ⇐⇒√x− 2− 1 +

√4− x− 1 = 2x2 − 5x− 3

⇐⇒ x− 3√x+ 2 + 1

+3− x√

4− x+ 1= (x− 3)(2x+ 1)

⇐⇒ (x− 3)

[1√

x− 2 + 1− 1√

4− x+ 1− (2x+ 1)

]= 0

⇐⇒

x = 31√

x− 2 + 1− 1√

4− x+ 1= 2x+ 1 (1)

207

Page 209: Chuyên đề luyện thi đại học môn Toán - VipLam.Net

Ta có1√

x− 2 + 1≤ 1;

1√4− x+ 1

≥ 1√2 + 1

=√

2− 1 =⇒ V T(1) ≤ 2−√

2.

Mặt khác: x ≥ 2 =⇒ V P(1) = 2x+ 1 ≥ 5 =⇒ (1) vô nghiệm.

Vậy nghiệm của phương trình đã cho có nghiệm là: x = 3

Bài 22. Giải phương trình :

x (x+ 1) (x+ 4) + 1 =(x2 + 2x+ 3

)√x2 + x+ 1

Lời giải

Phương trình đã cho viết lại thành:

x (x+ 1) (x+ 4) + 1 =(x2 + 2x+ 3

)√x2 + x+ 1

⇐⇒(x2 + x

)(x+ 4) + 1 =

[(x2 + x+ 1

)+ x+ 2

]√x2 + x+ 1

Đặt :√x2 + x+ 1 = a (a > 0) ; x+ 2 = b. Ta có :(

a2 − 1)

(b+ 2) + 1 =(a2 + b

)a ⇐⇒ a3 − a2 (b+ 2) + ab+ b+ 1 = 0

⇐⇒ a3 − 2a2 + 1 + b(1 + a− a2

)= 0 ⇐⇒ (a− 1)

(a2 − a− 1

)− b(a2 − a− 1

)= 0

⇐⇒(a2 − a− 1

)(a− b− 1) = 0 ⇐⇒

[a2 − a− 1 = 0

a = b+ 1

Bài 23. Giải phương trình sau:

(x+ 1)2 = 3(√x+ 2 + 1)

Lời giải

Cách 1. Dễ thấy rằng x = 2 là nghiệm của phương trình, bởi thế ta sẽ phân tích để có nhân

tử x− 2. Ta thấy√

2 + 2 = 2, nên ta có thể "thêm" và "bớt" phương trình đã cho để xuất hiện

nhân tử chung và có lời giải như sau như sau:

- Điều kiện x ≥ −2 Với điều kiện đó, ta có:

(x+ 1)2 = 3(√x+ 2 + 1) ⇐⇒ x2 + 2x− 8 = 3(

√x+ 2− 2)

⇐⇒ (x− 2) (x+ 4) = 3x− 2√x+ 2 + 2

⇐⇒

x = 2

x+ 4 =3√

x+ 2 + 2(∗)

Từ điều kiện x ≥ −2, ta thấy: x + 4 ≥ 2 và3√

x+ 2 + 2≤ 3

2< 2 nên phương trình (∗) vô

nghiệm.

Vậy phương trình có nghiệm duy nhất x = 2.

Cách 2. (Ẩn phụ không hoàn toàn)

Đặt t =√x+ 2 ≥ 0. Khi đó PT ⇐⇒ t2 +3t−x2−3x = 0. Có ∆t = 9+4(x2 +3x) = (2x+9)2.

208

Page 210: Chuyên đề luyện thi đại học môn Toán - VipLam.Net

Bài 24. Giải phương trình :

13.√x− 1 + 9.

√x+ 1 = 16x

Lời giải

Điều kiện: x ≥ 1

Dùng máy tính và chức năng Solve ta thu được một nghiệm x =5

4. Từ đó ta có hướng giải như

sau:

Áp dụng bất đẳng thức Côsi ta có:

13.√x− 1 = 13.2

√(x− 1)

1

4≤ 13

(x− 1 +

1

4

)

9.√x+ 1 = 3.2

√(x+ 1).

9

4≤ 3

(x+ 1 +

9

4

).

Cộng hai bất đẳng thức cùng chiều ta thu được:

13.√x− 1 + 9.

√x+ 1 ≤ 16x

Dấu bằng xảy ra khi và chỉ khi (x− 1) =

1

4

(x+ 1) =9

4

⇐⇒ x =5

4

Bài 25. Giải phương trình: √√2− 1− x+ 4

√x =

14√

2

Lời giải

ĐK: 0 ≤ x ≤√

2− 1

Đặt:√√

2− 1− x = a; 4√x = b =⇒ 0 ≤ a ≤

√√2− 1; 0 ≤ b ≤ 4

√√2− 1

Từ phương trình đã cho ta đưa về hệ sau:a+ b =14√

2

a2 + b4 =√

2− 1

Rút a ở phương trình trên thế vào phương trình dưới ta được:

(b2+1)2−(b+

14√

2

)2

= 0 =⇒(b2 − b+ 1− 1

4√

2

)(b2 + b+ 1 +

14√

2

)= 0 =⇒ b2−b+1− 1

4√

2= 0

Bài 26. Giải phương trình:

4√

(x− 2)(4− x) + 4√x− 2 + 4

√4− x+ 6x

√3x = x3 + 30

209

Page 211: Chuyên đề luyện thi đại học môn Toán - VipLam.Net

Lời giải

Điều kiện : 2 ≤ x ≤ 4. Ta có:

4√

1.1.(x− 2)(4− x) ≤ 1

4√

1.1.1.(x− 2) ≤ 1 + x

44√

1.1.1.(4− x) ≤ 7− x4

2x.3√

3x ≤ x(9 + 3x)

=⇒ V T ≤ 3 + 3x2 + 9x

Cần chứng minh : x3 + 30 ≥ 3x2 + 9x+ 3 ⇐⇒ (x− 3)2(x+ 3) ≥ 0 (đúng)

Vậy dấu bằng xãy ra khi x = 3

Bài 27. Giải bất phương trình sau trên tập số thực :

√8 + x− 9x2

x+ 2≤√

8 + x− 9x2

2x+ 1

Lời giải

Bất phương trình đã cho tương đương với:

√8 + x− 9x2

(1

x+ 2− 1

2x+ 1

)≤ 0

⇐⇒

−8

9≤ x ≤ 1

x− 1

(x+ 2) . (2x+ 1)≤ 0

⇐⇒

−8

9≤ x ≤ 1

1

(2x+ 1)≤ 0

=⇒ −8

9≤ x <

−1

2;x = 1

Vậy nghiệm của bất phương trình đã cho là:−8

9≤ x < −1

2;x = 1

Bài 28. Giải phương trình

√3x+ 3−

√5− 2x− x3 + 3x2 + 10x− 26 = 0, x ∈ R

Lời giải

ĐK : −1 ≤ x ≤ 5

2.

Đưa phương trình đã cho về phương trình :

√3x+ 3− 3−

(√5− 2x− 1

)= x3 − 3x2 − 10x+ 24

⇐⇒ 3(x− 2)√3x+ 3 + 3

+2(x− 2)√5− 2x+ 1

= (x− 2)(x+ 3)(x+ 4)

⇐⇒

x = 23√

3x+ 3 + 3+

2√5− 2x+ 1

= (x+ 3)(x+ 4) (1)

210

Page 212: Chuyên đề luyện thi đại học môn Toán - VipLam.Net

Bây giờ ta quan tâm tới phương trình (1). Không khó ta nhận thấy vế trái của phương trình

có :3√

3x+ 3 + 3+

2√5− 2x+ 1

> 0, ∀x ∈[−1;

5

2

]Mặt khác ta có :

(x+ 3)(x− 4) < 0 ⇐⇒ −3 < x < 4

Mà :

[−1;

5

2

]⊂ (−3; 4) =⇒ (x+ 3)(x+ 4) < 0, ∀x ∈

[−1;

5

2

].

Do đó phương trình (1) vô nghiệm nên phương trình có nghiệm duy nhất x = 2

Bài 29. Giải phương trình sau: √x3

3− 4x− 1

2√x

=√x

Lời giải

Điều kiện : 0 < x <3

4

Phương trình trở thành :x2√

x(3− 4x)=

2x+ 1

2√x⇐⇒ 4x4 + 16x3 + 4x2 − 8x− 3 = 0

⇐⇒ 4x4−1+4x2−2+8x(2x2−1) = 0 ⇐⇒ (2x2−1)(2x2 + 1 + 2 + 8x

)= 0 =⇒ x =

1√2

(Do x > 0)

Bài 30. Giải bất phương trình:

x3 +(3x2 − 4x− 4

)√x+ 1 ≤ 0

.

Lời giải

Bất phương trình tương đương với : x3 + 3x2√x+ 1− 4

(√x+ 1

)3 ≤ 0. Ta đặt t =√x+ 1 thì

ta có

x3 + 3x2√x+ 1− 4

(√x+ 1

)3

= x3 + 3x2t− 4t3 = (x− t)(x+ 2t)2

Bài 31. Giải phương trình:

3(√

2x2 + 1− 1)

= x(

1 + 3x+ 8√

2x2 + 1)

Lời giải

Ta có: V T ≥ 0 =⇒ V P ≥ 0 =⇒ x ≥ 0

PT

⇐⇒ 3x2 + x+ 8x√

2x2 + 1− 3√

2x2 + 1 + 3 = 0

Xét hàm số : f (x) = 3x2 + x+ 8x√

2x2 + 1− 3√

2x2 + 1 + 3 (x ≥ 0).

Ta có f′(x) = 6x + 1 + 8

(√2x2 + 1 +

2x2

√2x2 + 1

)− 6x√

2x2 + 1= 6x + 1 +

32x2 − 6x+ 8√2x2 + 1

> 0

211

Page 213: Chuyên đề luyện thi đại học môn Toán - VipLam.Net

với mọi x ≥ 0.

Khi đó f (x) đồng biến trên khoảng (0; + ∝) Mà f (0) = 0 nên x=0 là nghiệm duy nhất của

phương trình.

Bài 32. Tìm tất cả các nghiệm thực của phương trình sau :

4(√

x+ 1− 3)x2 +

(13√x+ 1− 8

)x− 4

√x− 1− 3 = 0.

Lời giải

Đưa phương trình ban đầu về phương trình:

4(√

x+ 1− 3)x2 +

(13√x+ 1− 8

)x− 4

[(√x+ 1

)2

− 1

]+(2√x− 1− 1

)2= 0 (1)

Điều kiện của bài toán là :x ≥ 1. Mặt khác x = 1 không thỏa phương trình nên ta chỉ cần xét

x > 1.

Đặt a =√x+ 1 ; b =

√x− 1 ; a, b > 0. Suy ra : x = a2 − 1.

Khi đó phương trình (1) trở thành :

4 (a− 3)(a2 − 1

)2+ (13a− 8)

(a2 − 1

)− 4

(a2 − 1

)+ (2b− 1)2 = 0

⇐⇒(a2 − 1

) (4a3 − 12a2 + 9a

)+ (2b− 1)2 = 0 ⇐⇒ a

(a2 − 1

)(2a− 3)2 + (2b− 1)2 = 0

⇐⇒

{(2b− 1)2 = 0

a (a2 − 1) (2a− 3)2 = 0⇐⇒

b =1

2a = 0

(loại)b =1

2a = 1

(loại)b =

1

2

a =3

2

(nhận)

=⇒ x =5

4

Đối chiếu điều kiện ta nhân được x =5

4là nghiệm của phương trình đã cho.

Bài 33. Giải phương trình:

x√x2 + 1 + 11x2 − 1 = 0

Lời giải

Cách 1. Phương trình tương đương: x√x2 + 1 = 1− 11x2

Bình phương 2 vế ta thu được PT hệ quả:

120x4 − 23x2 + 1 = 0

Giải PT trùng phương này thu được 4 nghiệm :

x =1

2√

2;x =

−1

2√

2;x =

1√15

;x =−1√

15

212

Page 214: Chuyên đề luyện thi đại học môn Toán - VipLam.Net

Cách khác. (Đưa về PT đẳng cấp bậc hai):

PT ⇐⇒ (x2 + 1) + x√x2 + 1− 12x2 = 0

(Điều chỉnh biệt thức ∆): Đặt t =√x2 + 1 ≥ 1.

PT ⇐⇒ a.t2 − x.t+ (11− a)x2 − a− 1 = 0

Tìm được a = −1. (Đặt ẩn phụ đưa về hệ PT): Với a = x, b =√x2 + 1 ta có hệab+ 11b2 = 12

b2 − a2 = 1

Bài 34. Giải phương trình:

(x− 3)√x2 + 5x+ 2 = x2 − x− 6

Lời giải

ĐK:x2 + 5x+ 2 ≥ 0

Phân tích phương trình đã cho ta được:

(x− 3)√x2 + 5x+ 2 = (x− 3)(x+ 2)

⇐⇒ (x− 3)[√x2 + 5x+ 2− (x+ 2)] = 0

⇐⇒

[x = 3√x2 + 5x+ 2 = x+ 2

Với√x2 + 5x+ 2 = x+ 2 ĐK:x ≥ −2. Bình phương 2 vế ta được:

x2 + 5x+ 2 = x2 + 4x+ 4 =⇒ x = 2(thỏa mãn)

Đối chiếu với điều kiên đầu bài ta có: Nghiệm của phương trình đã cho là:x = 2 hoặc x = 3

Bài 35. Giải phương trình sau:

√3x3 + 2x2 + 2 +

√−3x3 + x2 + 2x− 1 = 2x2 + 2x+ 2

Lời giải

Áp dụng Cauchy − Schwarz ta được

(2x2 + 2x+ 2)2 ≤ 2(3x2 + 2x+ 1) ⇐⇒ (x+ 1)2(2x2 + 1) ≤ 0 =⇒ x = −1.

Bài 36. Giải phương trình √x+ 1− 2

3√

2x+ 1− 3=

1

x+ 2

Lời giải

Cách 1.

ĐK :

{x ≥ −1

x 6= 13

BPT ⇐⇒ (x+ 1)√x+ 1 +

√x+ 1 = (2x+ 1) + 3

√2x+ 1

213

Page 215: Chuyên đề luyện thi đại học môn Toán - VipLam.Net

Đặt : a =√x+ 1; b = 3

√2x+ 1 (a ≥ 0) . Ta có :

a3 + a = b3 + b ⇐⇒ (a− b)(a2 + ab+ b2 + 1

)= 0 ⇐⇒ a = b

Nghiệm của phương trình: x = 0 hoặc x =1 +√

5

2.

Cách 2.

BPT ⇐⇒ (x+ 2)√x+ 1−2 (x+ 2) = 3

√2x+ 1−3 ⇐⇒ (x+ 1)

√x+ 1+

√x+ 1 = 2x+1+ 3

√2x+ 1

Xét hàm số : f (t) = t3 + t (t ∈ R) f′(t) = 3t2 + 1 > 0 với mọi t. Khi đó f (t) đồng biến trên R.

Mà : f(√

x+ 1)

= f(

3√

2x+ 1)

=⇒√x+ 1 = 3

√2x+ 1 =⇒ x3 + 3x2 + 3x+ 1 = 4x2 + 4x+ 1

=⇒ x (x2 − x− 1) = 0

Bài 37. Giải phương Trình vô tỷ sau:

√4x2 + 21 = 3x− 1 +

√4x2 + 5

Lời giải

Từ phương trình đã cho ta biến đổi thành:

√4x2 + 21− (2x+ 3) = x− 1 +

√4x2 + 5− 3

Đến đây ta dùng cách nhân liên hợp và chuyển vế được:

(x− 1) +4(x− 1)(x+ 1)√

4x2 + 5 + 3+

12(x− 1)√4x2 + 21 + 2x+ 3

= 0

⇐⇒ (x− 1)

[1 +

4(x+ 1)√4x2 + 5 + 3

+12√

4x2 + 21 + 2x+ 3

]= 0

⇐⇒ x = 1 hoặc 1 +4(x+ 1)√4x2 + 5 + 3

+12√

4x2 + 21 + 2x+ 3= 0

Ta có: Khi chuyển vế của phương trình đã cho được:

√4x2 + 21−

√4x2 + 5 = 3x− 1

Mà ta lại thấyđược rằng :

√4x2 + 21 >

√4x2 + 5 ⇐⇒

√4x2 + 21−

√4x2 + 5 > 0

Nên để phương trình đã cho có nghiệm thì: 3x− 1 > 0 ⇐⇒ x >1

3. Nên:

1 +4(x+ 1)√4x2 + 5 + 3

+12√

4x2 + 21 + 2x+ 3> 0 ∀x > 1

3

=⇒ 1 +4(x+ 1)√4x2 + 5 + 3

+12√

4x2 + 21 + 2x+ 3= 0 (vô nghiệm).

Vậy phương trình đã cho có nghiệm là: x = 1

214

Page 216: Chuyên đề luyện thi đại học môn Toán - VipLam.Net

Bài 38. Giải phương trình:

3(2 +√x− 2) = 2x+

√x+ 6

Lời giải

ĐK: x ≥ 2

Từ phương trình đã cho ta được:

6 + 3√x− 2 = 2x+

√x+ 6

Đến đây ta dùng cách nhân liên hợp sẽ được:

8x− 24

3√x− 2 +

√x+ 6

− 2x+ 6 = 0

⇐⇒ 8(x− 3)

3√x− 2 +

√x+ 6

− 2(x− 3) = 0

⇐⇒ (x− 3)

(8

3√x− 2 +

√x+ 6

− 2

)= 0

⇐⇒

x = 38

3√x− 2 +

√x+ 6

= 2 ⇐⇒ 3√x− 2 +

√x+ 6 = 4.

Giải phương trình ta được kết quả là: x =11− 3

√5

2.

Vậy phương trình đã cho có nghiệm: x = 3 hoặc x =11− 3

√5

2

Bài 39. Giải bất phương trình :

√2x2 − 10x+ 16−

√x− 1 ≤ x− 3

Lời giải

Điều kiện: x ≥ 1

Khi đó bất phương trình tương đương với:

√2x2 − 10x+ 16 ≤ x− 3 +

√x− 1

⇐⇒

{x− 3 +

√x− 1 ≥ 0

2x2 − 10x+ 16 ≤ x2 − 6x+ 9 + x− 1 + 2 (x− 3)√x− 1

⇐⇒

{x− 3 +

√x− 1 ≥ 0

x2 − 5x+ 8 ≤ 2 (x− 3)√x− 1

⇐⇒

{x− 3 +

√x− 1 ≥ 0(

x− 3−√x− 1

)2 ≤ 0

⇐⇒

{x− 3 +

√x− 1 ≥ 0

x− 3−√x− 1 = 0

⇐⇒ x = 5 thỏa mãn điều kiện.

Bài 40. Giải phương trình

2x4 − x3 = 1− 3

√(1 + x2)2

215

Page 217: Chuyên đề luyện thi đại học môn Toán - VipLam.Net

Lời giải

V P < 0 =⇒ V T < 0 =⇒ x ∈[0; 1

2

]. Thực hiện phép nhân liên hợp ta được

x2(2x2 − x) =x2(−x2 − 2)

1 + 3√

(1 + x2)2 + 3√

(1 + x2)4

TH1: x = 0 là một nghiệm của PT.

TH2: 2x2 − x =−x2 − 2

1 + 3√

(1 + x2)2 + 3√

(1 + x2)4(1) Ta có minV T(1) = −1

4.

Mặt khác ta có

1 + 3√

(1 + x2)2 + 3√

(1 + x2)4 < 1 +3√

22 +3√

24 < 1 + 2 + 4 < 8 + 4x2

=⇒ −x2 − 2

1 + 3√

(1 + x2)2 + 3√

(1 + x2)4< −1

4.

Do đó PT (1) vô nghiệm. Dĩ nhiên PT ban đầu có nghiệm duy nhất x = 0.

Bài 41. Giải phương trình:

x3 +√

1− 3x2 + 3x4 − x6 = x√

2− 2x2

Lời giải

Thật vậy, ta đưa phương trình đã cho tương đương với phương trình sau với điều kiện của bài

toán là −1 ≤ x ≤ 1. :

x3 +√

(1− x2)3 = x√

2(1− x2)

Đặt x = cos a, a ∈ [0; π] =⇒ sin a > 0. Lúc đó phương trình vừa biến đổi được viết lại :

cos3 a+ sin3 a =√

2 cos a sin a ⇐⇒ (cos a+ sin a)(1− sin a cos a) =√

2 sin a cos a. (1)

Không khó để nhận thấy phương trình (1) là một phương trình đối xứng sin a, cos a quen thuộc.

Tiếp tục đặt t = sin a+ cos a =√

2 sin(a+π

4), t ∈ [−1;

√2] =⇒ sin a cos a =

t2 − 1

2.

Thay vào phương trình (1) và biến đổi thu gọn ta thu được phương trình :

t3 +√

2t− 3t−√

2 = 0 ⇐⇒ (t−√

2)(t2 + 2√

2t+ 1) = 0

Với t =√

2 =⇒ t =π

4+ k2π =⇒ x =

√2

2.

Với t2 + 2√

2t +√

2 = 0 =⇒ t = 1−√

2 ⇐⇒ sin a + cos a = 1−√

2 (2). Để giải (2) ta trả

lại ẩn ban đầu ta thu được một phương trình :

√1− x2 + x = 1−

√2 ⇐⇒

√1− x2 = x+ 1−

√2 (3)

Phương trình (3) là một phương trình cơ bản sau một hồi loay hoay bình phương và kiên trì

tính ta thu được nghiệm của phương trình này là :

x =1−√

2−√

2√

2− 1

2.

216

Page 218: Chuyên đề luyện thi đại học môn Toán - VipLam.Net

Bài 42. Giải bất phương trình:

x ≥ 3√

2− x2 − 2x2√

2− x2

Lời giải

Điều kiện : 2− x2 ≥ 0 ⇐⇒ −√

2 ≤ x ≤√

2 (∗).Bất phương trình đã cho được biến đổi thành bất phương trình tương đương :

x ≥ (3− 2x2)√

2− x2

⇐⇒ x ≥(1 + 2− x2 − x2

)√2− x2

⇐⇒(√

2− x2)3

+(1− x2

)√2− x2 − x ≤ 0

⇐⇒(√

2− x2 − x)(

2− x2 + x√

2− x2 + 1)≤ 0 (1)

Tới đây ta chú ý rằng : 2− x2 + x√

2− x2 + 1 > 0 do điều kiện (∗).Vậy (1) cho

√2− x2 ≤ x. Đây là bài toán cơ bản.

Bài 43. Giải phương trình :

(1 +√

1 + x)(√

2x2 − 2x+ 1 + x− 1) = x√x (1)

Lời giải

Điều kiện : x ≥ 0.

Với sự đối lập của 1 trong căn và −1 ngoài căn ở tổng thứ hai trong tích hai tổng bên vế trái,

đồng thời bên vế phải không chứa bất kì hệ số tự do nào. Nên một điều tự nhiên ta nghỉ ngay

đến thử xem giá trị x = 0 xem thế nào?

Thật vậy : Kiểm tra thấy ngay được x = 0 thỏa phương trình đã cho. Do đó, ta chỉ cần quan

sát bài toán với x > 0 là đủ rồi.

Mặt khác bên vế trái chứa hai tích hai tổng, ta có thể nghỉ ngay đến việc khử bớt tổng bên trái

cho gọn nhẹ.

Để giúp được điều này ta nghỉ đến phép liên hiệp.

Với x > 0 ta luôn có :√

2x2 − 2x+ 1− (x− 1) 6= 0.

Do đó ta dùng phép toán “liên hiệp kiểu ngang” ta thu được phương trình :(1 +√

1 + x)x2 = x

√x(√

2x2 − 2x+ 1− (x− 1))

⇐⇒(

1 +√

1 + x)√

x =√

2x2 − 2x+ 1− (x− 1) (∗)

Lại có với x > 0 thì 1 −√

1 + x 6= 0 nên ta tiếp tục sử dụng phép toán kiểu “liên hiệp ngang”

từ (∗) ta thu được phương trình :

−x√x =

(1−√

1− x) (√

2x2 − 2x+ 1− (x− 1))

(2)

Ta lấy (1) và (2) cộng vế theo vế ta thu được phương trình :

√2x2 − 2x+ 1 = (1− x)

√1 + x =⇒ 2x2 − 2x+ 1 = (1− x)2 (1 + x)

217

Page 219: Chuyên đề luyện thi đại học môn Toán - VipLam.Net

⇐⇒ x(x2 − 3x+ 4

)= 0 ⇐⇒

x = 0

x =3−√

5

2

x =3 +√

5

2

Đối chiếu điều kiện x > 0 và thử lại ta có : x =3−√

5

2.

Vậy phương trình đã cho có hai nghiệm : x = 0 ; x =3−√

5

2

Bài 44. Giải phương trình:

2x2 − x− 1

8=

3

√9

8x2+

1

x− 1

Lời giải

Điều kiện : x 6= 0

Nhân cả 2 vế của phương trình với x ta có :

2x3 − x2 − 1

8x =

3

√9

8x− x2 − x3

⇐⇒ x3 + x = −x3 − x2 +9

8x+

2

√−x3 − x2 +

9

8x

Xét hàm số :f (t) = t3 + t(t ∈ R). Ta có :f′= 3t2 + 1 > 0

Mà f (x) = f

(3

√9

8x− x2 − x3

). Nên ta có : x = 3

√9

8x− x2 − x3 ⇐⇒ 2x3 + x2 − 9

8x = 0

Bài 45. Giải bất phương trình:

x− 7

x− 1− 2√

(x+ 2)≥

2√

(4− x)

x+ 1

Lời giải

ĐK:−2 ≤ x ≤ 4 và x 6= −1

Khi đó

bpt ⇐⇒ x+ 2− 9

x+ 2− 2√x+ 2− 3

≥ 2√

4− xx+ 2− 1

⇐⇒(√

x+ 2 + 3) (√

x+ 2− 3)(√

x+ 2 + 1) (√

x+ 2− 3) ≥ 2

√4− x(√

x+ 2− 1) (√

x+ 2 + 1)

⇐⇒ x− 1 + 2√x+ 2− 2

√4− x ≥ 0

Xét hàm số f (x) = x− 1 + 2√x+ 2− 2

√4− x, x ∈ R.

Ta có f / (x) = 1 +1√x+ 2

+1√

4− x≥ 0, ∀x ∈ R. Mà (1) = 0 =⇒ 1 ≤ x ≤ 4

218

Page 220: Chuyên đề luyện thi đại học môn Toán - VipLam.Net

Bài 46. Giải phương trình:

2 + 3x− x2 =√

(1 + x) (4− x)

Lời giải

pt⇐⇒ 2 + 3x− x2 =√−x2 + 3x+ 4

Đặt t=√−x2 + 3x+ 4. Thay vào phương trình t2 − 2 = t

Bài 47. Giải bất phương trình :

x2 − 3x+ 3 ≥(

4 + 3x− 4

x

)√x− 1

Lời giải

ĐK: x ≥ 1 Chia hai vế Bpt cho x2 ta được:

1− 3(1

x− 1

x2) ≥

(4(

1

x− 1

x2) + 3

)√1

x− 1

x2

Đặt

√1

x− 1

x2= t. Khi đó

Bpt ⇐⇒ 1− 3t2 ≥ (4t2 + 3)t ⇐⇒ (4t− 1)(t2 + t+ 1) ≤ 0 ⇐⇒ t ≤ 1

4

Bài 48. Giải bất phương trình:

5√x+

5

2√x< 2x+

1

2x+ 4

Lời giải

Đặt t = 5√x+

5

2√x

=⇒ t2 = 25x+25

4x+ 25 =⇒ 2x+

1

2x=

2

25(t2 − 25) (1) ⇐⇒ t ≤ 2

25(t2 − 25) + 4

Bất phương trình đã cho ⇐⇒ 2t2 − 25t− 625 ≤ 0

Bài 49. Giải phương trình

(13− 4x)√

2x− 3 + (4x− 3)√

5− 2x = 2 + 8√

16x− 4x2 − 15

Lời giải

ĐK:3

2≤ x ≤ 5

2Đặt√

2x− 3 = a;√

5− 2x = b(a, b ≥ 0).

Phương trình đã cho trở thành:

(a+ b)(3 + 2ab) = 2 + 8ab

a2 + b2 = 2

Đặt a+ b = S; ab = P (S2 ≥ 4P ). Hệ trên trở thành:

S(3 + 2P ) = 2 + 8P

S2 − 2P = 2

219

Page 221: Chuyên đề luyện thi đại học môn Toán - VipLam.Net

Thế P ở PT(2) vào PT(1) ta được: S3 − 4S2 + S + 6 = 0 =⇒ S = −1 hoặc S = 3 hoặc S = 2

Với S=-1 thì P=−1

2

Với S=3 thì P=7

2(không thỏa mãn).

Với S=2 thì P=1

Bài 50. Giải phương trình:4√x =

3

8+ 2x

Lời giải

ĐK x ≥ 0

Ta có:3

8+ 2x =

1

8+

1

8+

1

8+ 2x ≥ 4

√x

Dấu"=" xảy ra⇐⇒ x =1

16

Bài 51. Giải bất phương trình :

x2 +√

1− 2x+√

1 + 2x ≥ 2

Lời giải

Điều kiện : −1

2≤ x ≤ 1

2Đặt t =

√1− 2x+

√1 + 2x, t ≥ 0.

Lúc đó ta có : t2 = 2 + 2√

1− 4x2 ⇐⇒ t2 − 2 = 2√

1− 4x2 =⇒ (t2 − 2)2

= 4 − 16x2 ⇐⇒16x2 = 4− (t2 − 2)

2.

Bất phương trình đã cho tương đương với bất phương trình :

16x2 + 16(√

1− 2x+√

1 + 2x)− 32 ≥ 0

⇐⇒ 4−(t2 − 2

)2+ 16t− 32 ≥ 0 ⇐⇒ t4 − 4t2 − 26t+ 32 ≤ 0

⇐⇒ (t− 2)2 ·((t+ 2)2 + 4

)≤ 0 ⇐⇒ t = 2

⇐⇒√

1− 2x+√

1 + 2x = 2 ⇐⇒ 2√

1− 4x2 = 2 ⇐⇒ 1− 4x2 = 1 ⇐⇒ x = 0

Thử lại ta có x = 0 là nghiệm duy nhất của bất phương trình đã cho.

Bài 52. Giải phương trình:

4x2 + 13x+ 5 +√

1− 3x = 0

Lời giải

Điều kiện xác định: x ≤ 1

3

Đặt√

1− 3x = 2y + 3 với y ≥ −3

2, từ phương trình đã cho ta có hệ:4x2 + 13x+ 2y + 8 = 0(1)

4y2 + 12y + 3x+ 8 = 0

220

Page 222: Chuyên đề luyện thi đại học môn Toán - VipLam.Net

Trừ từng vế của hệ phương trình ta được: (x− y) [4 (x+ y) + 10] = 0 ⇐⇒ x = y hoặc

2 (x+ y) + 5 = 0

Với x = y, từ (1) ta có: 4x2 + 15x+ 8 = 0 ⇐⇒ x =−15−

√97

8hoặc x =

−15 +√

97

8

Với 2 (x+ y)+5 = 0, từ (1) ta có: 4x2+11x+3 = 0 ⇐⇒ x =−11−

√73

8hoặc x =

−11 +√

73

8

Vậy phương trình đã cho có 4 nghiệm x =−11±

√73

8, x =

−15±√

97

8

Bài 53. Giải phương trình :

4x3 − 7x+3√

4x3 − 3x+ 1 = 3√

4x− 2− 3

Lời giải

PT ⇐⇒ 4x3 − 3x+ 1 +3√

4x3 − 3x+ 1 = 4x− 2 + 3√

4x− 2

Xét hàm số f(t) = t3 + t−−−−−−−−−−−−−−− > 4x3 − 3x+ 1 = 4x− 2

Bài 54. Giải phương trình:

√3x2 − 1 +

√x2 − x− x

√x2 + 1 =

1

2√

2(7x2 − x+ 4)

Lời giải

Điều kiện : x ≤ −√

3

3∨ x ≥ 1. Áp dụng bất đẳng thức Bunyakovsky ta có :

√3x2 − 1 +

√x2 − x+ (−x)

√x2 + 1 ≤

√(x2 + 2)(5x2 − x)

Dấu đẳng thức xảy ra khi x = −1

Mặt khác ta để ý : 7x2−x+ 4 = 2(x2 + 2) + (5x2−x). và 5x2−x > 0 với điều kiện ta đang xét.

Khi đó áp dụng bất đẳng thức AM −GM ta có :

1

2√

2

(2(x2 + 2) + (5x2 − x)

)≥ 1

2√

22√

(5x2 − x)(x2 + 2)

Dấu đẳng thức xảy ra khi x = −1 ∨ x =4

3. Từ đó ta có x = −1 là nghiệm của phương trình.

Bài 55. Giải bất phương trình:√(x+ 2)(2x− 1)− 3

√x+ 6 6 4−

√(x+ 6)(2x− 1) + 3

√x+ 2

Lời giải

ĐK: x ≥ 1

2Bất phương trình đã cho trở thành:

(√x+ 2 +

√x+ 6)(

√2x− 1− 3) ≤ 4(1)

Với√

2x− 1− 3 ≤ 0 ⇐⇒ x ≤ 5 Nên (1) luôn đúng.

Với x>5 ta xét hàm số sau: f(x) = (√x+ 2 +

√x+ 6)(

√2x− 1− 3).

221

Page 223: Chuyên đề luyện thi đại học môn Toán - VipLam.Net

Đạo hàm: f ′(x) =

(1

2√x+ 2

+1

2√x+ 6

)(√2x− 1− 3

)+

√x+ 2 +

√x+ 6√

2x− 1> 0. Nên f(x)

đồng biến.

Ta thấy f(7) = 4 nên: (1) ⇐⇒ f(x) ≤ f(7) ⇐⇒ x ≤ 7.

Đối chiếu với ĐK ta có: Nghiệm của bất phương trình đã cho là:1

2≤ x ≤ 7

Bài 56. Giải phương trình :

(x3 + 2x2 − 1) = 15(√x− 1−

√x− 2)3

Lời giải

Phương trình đã cho: ⇐⇒ (x3 + 2x2 − 1)(√x− 1 +

√x− 2)3 = 15.

VT là 1 hàm đồng biến (tích 2 hàm đồng biến và dương) nên pt có nghiệm duy nhất thấy x = 2

là 1 nghiệm pt.

vậy x = 2 là nghiệm pt

Bài 57. Giải bất phương trình :

24x2 − 60x+ 36 ≥ 1√5x− 7

− 1√x− 1

Lời giải

bpt =⇒ (5x− 6)2 − 1√5x− 6

= x2 − 1√x− 1

Xét hàm số f(t)=t2−√t− 1 .....hàm số đồng biến và liên tục nên từ bpt ban đầu ta có đc kquả

bài toán là x ≥ 3

2

Bài 58. Giải phương trình :

x−√

2 +√x+ 2 +

√1− x = (x+ 1)2

Lời giải

ĐK :−2 ≤ x ≤ 1

Khi đó phương trình tương đương với

x2 + x+√

2−√

2− x+ 1−√

1− x = 0

⇐⇒ x

(x+ 1− 1√

2 +√

2 + x+

1

1 +√

1− x

)= 0

⇐⇒

x = 0

x+ 1− 1√2 +√

2 + x+

1

1 +√

1− x= 0 (2)

Từ (2) ta xét hàm số : f (x) = x+ 1− 1√2 +√

2 + x+

1

1 +√

1− x(x ∈ [−2; 1])

f′(x) = 1 +

1

2√

2 + x(2 +√

2 + x)2 +

1

2√

1− x(1 +√

1− x)2 ≥ 0 (x ∈ D) . Khi đó f (x) đồng

biến trên D.

222

Page 224: Chuyên đề luyện thi đại học môn Toán - VipLam.Net

Mà f (−1) =0 nên x=-1 là nghiệm duy nhất của phương trình. Vậy x=0 và x=-1.

Bài 59. Giải phương trình :

9− 2x√4− x

+4x+ 3√4x+ 1

=15

2

Lời giải

1 + 2 (4− x)√4− x

+(4x+ 1) + 2√

4x+ 1=

15

2

⇐⇒ 1√4− x

+ 2√

4− x+√

4x+ 1 +2√

4x+ 1=

15

2

⇐⇒(

2√

4− x+√

4x+ 1)

+

(2√

4− x+√

4x+ 1)

√4− x.

√4x+ 1

=15

2

Đặt : t = 2√

4− x+√

4x+ 1(√

17 < t ≤√

34). Giải được t = 5.

Bài 60. Giải phương trình :

3√

3x+ 5 = 3√

15x+ 12− 3√

2x− 1

Lời giải

Đặt 3√

3x+ 5 = a ; 3√

2x− 1 = b. Ta có: a = 3√

3a3 + 3b3 − b. Phương trình đã cho tương đương

với:

⇐⇒ 2a3 − 3a2b− 3ab2 + 2b3 = 0 ⇐⇒

a = −b

a = 2b

a =1

2b

Thay vào tìm được: x = 1 hoặc x = −4

5hoặc x = −41

22.

Bài 61. Giải phương trình :√7− x2 + x

√x+ 5 =

√3− 2x− x2

Lời giải

ĐK: x = [−1; 3]

Bình phương 2 vế x√x+ 5 = −2x− 4. Vì x=0 không phải là nghiệm nên ta viết phương trình

về dạng√x+ 5 =

−2x− 4

x⇐⇒

√x+ 5− 2 =

−2x− 4

x− 2

⇐⇒ x+ 1√x− 5 + 2

= −4x+ 1

x⇐⇒ (x+ 1)[

1√x+ 5 + 2

+4

x] = 0

⇐⇒ x = −1 hoặc1√

x+ 5 + 2+

4

x= 0 ⇐⇒ x = −4;x = 4 (loại)

Phương trình có nghiệm duy nhất x = −1.

223

Page 225: Chuyên đề luyện thi đại học môn Toán - VipLam.Net

Bài 62. Giải bất phương trình :(√5x− 1 +

√x− 1

) (3x− 1−

√5x2 − 6x+ 1

)≤ 4x

Lời giải

ĐK: x ≥ 1

Đặt√

5x− 1 = a;√x− 1 = b (a, b ≥ 0) . Ta có :(a+ b)(a2 − 2b2 − ab− 2) ≤ a2 − b2

a2 − 5b2 = 4

⇐⇒

(a+ b)(a2 − 2b2 − ab− 2− a+ b) ≤ 0

a2 − 5b2 = 4

=⇒ 3b2 + b+ 2 ≤ (b+ 1)√

5b2 + 4

⇐⇒ 9b4 + b2 + 4 + 6b3 + 4b+ 12b2 ≤ 5b4 + 4b2 + 10b3 + 8b+ 5b2 + 4

⇐⇒ (b− 1)(b3 + b) ≤ 0

⇐⇒ b ≤ 1 ⇐⇒√x− 1 ≤ 1 ⇐⇒ 2 ≥ x ≥ 1

Bài 63. Giải bất phương trình:

x3 − 2x

x2 − 1−√x2 − 1

≥ 2√

2

Lời giải

Điều kiện x2 > 1;x 6= ±√

2

Bất phương trình được viết lại như sau:

x(√

(x2 − 1)2 − 1)√x2 − 1(

√x2 − 1− 1)

≥ 2√

2

Bất phương trình đã cho trở thành:

x(√x2 − 1 + 1)√x2 − 1

≥ 2√

2 ⇐⇒ x(√x2 − 1 + 1) ≥ 2

√2√x2 − 1

Suy ra bất phương trình có nghiệm khi: x > 1;x 6=√

2. Bình phương 2 vế bất phương trình ta

thu được:

x2(x2 + 2√x2 − 1) ≥ 8(x2 − 1)

Bây giờ ta đặt :√x2 − 1 = t > 0 =⇒ x2 = t2 + 1 thay vào bất phương trình ta thu được:

(t2 + 1)(t2 + 2t+ 1) ≥ 8t2 ⇐⇒ (t− 1)2(t2 + 4t+ 1) ≥ 0

Bài 64. Giải phương trình :

(x− 1)2 + 2 (x+ 1)

√x− 3

x+ 1= 12

224

Page 226: Chuyên đề luyện thi đại học môn Toán - VipLam.Net

Lời giải

TH1: Nếu x ≥ 3 thì

PT ⇐⇒ (x+ 1)(x− 3) + 2√

(x+ 1)(x− 3) = 8

TH2: Nếu x < −1 thì

PT ⇐⇒ (x+ 1)(x− 3)− 2√

(x+ 1)(x− 3) = 8

Bài 65. Giải phương trình :

4√

1− x− 6 = x− 3√

1− x2 + 5√

1 + x

Lời giải

Điều kiện:−1 ≤ x ≤ 1 Phương trình được viết lại như sau:

4√

1− x− 5√

1 + x = x+ 6− 3√

1− x2

Bình phương 2 vế và thu gọn ta được phương trình:

8x2 − 3x− 4 + (6x− 4)√

1− x2 = 0

Ta đặt t =√

1− x2 thì phương trình trở thành:

8x2 − 3x− 4 + (6x− 4)t = 0

Bây giờ ta thêm bớt vào một lượng mt2 để tạo ra phương trình bậc 2 theo t là:

mt2 + (6x− 4)t+ (8 +m)x2 − 3x− 4−m = 0

Ta có ∆′ = (3x−2)2−m[(8+m)x2−3x−4−m] = (9−8m−m2)x2 +(3m−12)x+m2 +4m+4.

Ta mong muốn ∆ = (ax+ b)2 để có điều này thì ta cần: ∆ = (3m−12)2−4(9−8m−m2)(m2 +

4m+ 4) = 0

Giải phương trình này ta thu được m = −8. Như vậy phương trình bậc 2 theo t là :

−8t2 + (6x− 4)t− 3x− 4 = 0

Ta có: ∆′ = (3x− 2)2 − 24x+ 32 = (3x− 6)2

Bài 66. Giải phương trình :

x2 +3√x4 − x2 = 2x+ 1

Lời giải

PT ⇐⇒ (x2 − 1) + 3√x4 − x2 − 2x = 0 (1)

∗x = 0, thay vào phương trình không thoả mãn.

∗x 6= 0, chia 2 vế (1) cho x ta được:

x− 1

x+

3

√x− 1

x− 2 = 0

225

Page 227: Chuyên đề luyện thi đại học môn Toán - VipLam.Net

Đặt: t = 3

√x− 1

x. PT trở thành:

t3 + t− 2 = 0 ⇐⇒ t = 1.

Với t = 1, ta có:

x2 − x− 1 = 0 ⇐⇒ x =1±√

5

2

Bài 67. Giải phương trình sau:

√x+ 1√

x+ 1−√

3− x= x− 1

2

Lời giải

Điều kiện :

−1 ≤ x ≤ 3

x 6= 1. Với điều kiện này phương trình đã cho tương đương với phương

trình :√x+ 1(

√x+ 1 +

√3− x)

2(x− 1)= x− 1

2

⇐⇒ 2x+√

3 + 2x− x2

2(x− 1)= x

⇐⇒√

3 + 2x− x2 = 2x2 − 4x

⇐⇒

3 + 2x− x2 = 4x2(x− 2)2

x ≤ 0 ∨ x ≥ 2

⇐⇒

(x− 1)2(x2 − 2x− 13

4) = 0

x ≤ 0 ∨ x ≥ 2

⇐⇒

x = 1 +

√17

2

x = 1−√

17

2

Đối chiếu điều kiện ban đầu ta có hai nghiệm vừa tìm được chính là nghiệm của phương trình

đã cho.

Bài 68. Giải phương trình:

(x2 + 1)2 = 5− x√

2x2 + 4

Lời giải

Với x ≥ 0. P t⇐⇒ (x2 + 1)2 = 5−√

2(x2 + 1)2 − 2(1).

Đặt t = (x2 + 1)2, t > 1. Khi đó

(1) ⇐⇒

{2t− 2 = (5− t)2

t ≤ 5⇐⇒ t = 3 =⇒ (x2 + 1)

2= 3 Vô nghiệm.

226

Page 228: Chuyên đề luyện thi đại học môn Toán - VipLam.Net

Với x ≤ 0. P t⇐⇒ (x2 + 1)2 = 5 +√

2(x2 + 1)2 − 2(1).

Đặt t = (x2 + 1)2, t > 1. Khi đó

(1) ⇐⇒

{2t− 2 = (t− 5)2

t ≥ 5⇐⇒ t = 9 =⇒ (x2 + 1)

2= 9 =⇒ x = −

√2

Bài 69. Giải phương trình:

3√x+ 6 + x2 = 7−

√x− 1

Lời giải

Đk: x ≥ 1

Pt⇐⇒ 3√x+ 6− 2 + x2 − 4 = 1−

√x− 1

⇐⇒ (x− 2)

(1

3√

(x+ 6)2 + 2 3√x+ 6 + 4

+ x+ 2 +1

1 +√x− 1

)= 0⇐⇒ x = 2

Bài 70. Giải phương trình :

(3x+ 1)√

2x2 − 1 = 5x2 +3

2x− 3

Lời giải

PT ⇐⇒ 2(2x2 − 1)− (3x+ 1)√

2x2 − 1 + x2 +3

2x− 1 = 0.

Đặt: t =√

2x2 − 1. PT trở thành:

2t2 − (3x+ 1)t+ x2 +3

2x− 1 = 0 ⇐⇒

(t− x+

1

2

)(2t− x− 2) = 0

Bài 71. Giải phương trình:

√x− 2

(x2 + 6x− 11

)= 5x2 − 10x+ 1

Lời giải

Đặt t =√x− 2. Pt đã cho trở thành

(t− 1)5 = 0

Bài 72. Giải phương trình:

2√x2 − 7x+ 10 = x+

√x2 − 12x+ 20

Lời giải

ĐK: x ≥ 10 hoặc x ≤ 2

Bình phương 2 vế ta được:

x2 − 8x+ 10 = x.√x2 − 12x+ 20

227

Page 229: Chuyên đề luyện thi đại học môn Toán - VipLam.Net

Bình phương 2 vế tiếp ta lại được:

−4x3 + 64x2 − 160x+ 100 = 0 ⇐⇒ (x− 1)(−4x2 + 60x− 100) = 0

Bài 73. Giải phương trình:

4

x+

√x− 1

x= x+

√2x− 5

x

Lời giải

Đặt

√x− 1

x= a và

√2x− 5

x= b , chuyển vế ta có:

a− b = b2 − a2

Bài 74. Giải phương trình:

2x2 − 5x+ 2 = 4√

2(x3 − 21x− 20)

Lời giải

2x2 − 5x+ 2 = 4√

2(x3 − 21x− 20)

⇐⇒ 2(x2 − 4x− 5) + 3(x+ 4) = 4√

2(x2 − 4x− 5)(x+ 4)

Bài 75. Giải phương trình:

x3 + 6x2 − 2x+ 3 = (5x− 1)√x3 + 3

Lời giải

ĐKXĐ: x ≥ −√

3

Với điều kiện đó đặt :√x3 + 3 = t ≥ 0. Phương trình đã cho trở thành:

t2 − (5x− 1)t+ 6x2 − 2x = 0 ⇐⇒ (t− 2x)(t− 3x+ 1) = 0

(*) Với t = 2x ta có:√x3 + 3 = 2x

⇐⇒

x ≥ 0

x3 − 4x2 + 3 = 0⇐⇒

x = 1

x =3 +√

21

2

(*) Với t = 3x− 1 Ta có:√x3 + 3 = 3x− 1

⇐⇒

x ≥1

3x3 − 9x2 + 6x+ 2 = 0

⇐⇒

[x = 1

x = 4 + 3√

2

Vậy phương trình đã cho có nghiệm là: x = 1 ; x = 4 + 3√

2 ; x =3 +√

21

2

228

Page 230: Chuyên đề luyện thi đại học môn Toán - VipLam.Net

Bài 76. Giải phương trình:√x+ 3 =

1

2x− 7

2x+ 5

Lời giải

ĐK: x ≥ 4√

2− 5

Phương trình đã cho

⇐⇒ x(x+ 3)− 2x√x+ 3 + 7(x− 1) = 0

⇐⇒ x√x+ 3(

√x+ 3− 2) + 7(x− 1) = 0

⇐⇒ x√x+ 3(x− 1)√x+ 3 + 2

+ 7(x− 1) = 0

⇐⇒ (x− 1)

(x√x+ 3√

x+ 3 + 2+ 7

)= 0

Bài 77. Giải phương trình:

√4x2 + 2x− 1 +

√2x− 4x2 + 1 = 4x2 − 2x+ 2

Lời giải

Điều kiện: 4x2 + 2x− 1 ≥ 0 và 2x− 4x2 + 1 ≥ 0

Lúc này sử dụng BĐT AM −GM ta có:

√4x2 + 2x− 1 +

√2x− 4x2 + 1 ≤1

2(4x2 + 2x− 1 + 1) +

1

2(2x− 4x2 + 1 + 1)

=2x+ 1

=4x2 − 2x+ 2− (2x− 1)2

≤4x2 − 2x+ 2

Do đó√

4x2 + 2x− 1 +√

2x− 4x2 + 1 = 4x2 − 2x+ 2

⇐⇒

4x2 + 2x− 1 = 1

2x− 4x2 + 1 = 1

2x− 1 = 0

⇐⇒ x =1

2(thỏa)

Vậy x =1

2là nghiệm duy nhất của phương trình.

Bài 78. Giải phương trình:√1 +√

1− x2[√

(1 + x)3 −√

(1− x)3]

=2√3

+

√1− x2

3

Lời giải

Đặt√

1− x = a ;√

1 + x = b (a, b ≥ 0) =⇒ a2 + b2 = 2.

229

Page 231: Chuyên đề luyện thi đại học môn Toán - VipLam.Net

Ta được:

√1 + ab(a3 − b3) =

2 + ab√3

⇐⇒√

3 + 3ab(a− b)(a2 + ab+ b2) = 2 + ab

⇐⇒√

3 + 3ab(a− b)(2 + ab) = ab+ 2

⇐⇒ (ab+ 2)[√

3 + 3ab(a− b)− 1] = 0

Bài 79. Giải phương trình:

√x+ 4 +

√x− 4 = 2x− 12 + 2

√x2 − 16

Lời giải

ĐKXĐ: x ≥ 4

Đặt:√x+ 4 = a ≥ 0 ;

√x− 4 = b ≥ 0

Ta có hệ phương trình:a2 − b2 = 8

(a+ b)2 + (a+ b)− 12 = 0=⇒

[a+ b = 3

a+ b = −4

Bài 80. Giải phương trình:

x− 2√x− 1− (x− 1)

√x+√x2 − x = 0

Lời giải

Phương trình tương đương với:

(√x− 1− 1)(

√x− 1− 1−

√x2 − x) = 0[ √

x− 1− 1 = 0 ⇐⇒ x = 2√x− 1− 1−

√x2 − x = 0 ⇐⇒

√x− 1(1−

√x) = 1 (vô nghiệm )

Bài 81. Giải phương trình:

√2x2 − 1 +

√x2 − 3x− 2 =

√2x2 + 2x+ 3 +

√x2 − x+ 2

Lời giải

Phương trình đã cho:

⇐⇒ 2(x+ 2)

(1√

2x2 + 2x+ 3 +√

2x2 − 1+

1√x2 − x+ 2 +

√x2 − 3x− 2

)= 0

Bài 82. Giải phương trình :

7x2 − 10x+ 14 = 5√x4 + 4

230

Page 232: Chuyên đề luyện thi đại học môn Toán - VipLam.Net

Lời giải

Phương trình đã cho tương đương với : (x2 + 2x+ 2)−5√

(x2 − 2x+ 2) (x2 + 2x+ 2)+6 (x2 − 2x+ 2) =

0

Đặt : a =√x2 + 2x+ 2, b =

√x2 − 2x+ 2 (a, b > 0) ta có : a2 − 5ab + 6b2 = 0 ⇐⇒[

a = 2b

a = 3b

Bài 83. Giải phương trình:

2

√x2 + x+ 1

x+ 4+ x2 − 4 ≤ 2√

x2 + 1

Lời giải

Đk: x > −4

Trừ cả 2 vế của bất phương trình cho 1 :

2

√x2 + x+ 1

x+ 4− 1

+ x2 − 3 ≤ 2−√x2 + 1√

x2 + 1

⇐⇒ 2(x2 − 3)√x+ 4

(√(x2 + x+ 1) +

√x+ 4

) + x2 − 3 +x2 − 3√

x2 + 1(2 +√x2 + 1)

≤ 0

⇐⇒ x2 ≤ 3 ⇐⇒√

3 ≥ x ≥ −√

3

Bài 84. Giải phương trình:

√7x3 − 11x2 + 25x− 12 =

x2

2+ 3x− 1

2

Lời giải

Cách 1.√

7x3 − 11x2 + 25x− 12 =x2

2+ 3x− 1

2

⇐⇒ 2√

(7x− 4)(x2 − x+ 3) = x2 − x+ 3 + 7x− 4

⇐⇒√

7x− 4 =√x2 − x+ 3 ⇐⇒ x2 − 8x+ 7 = 0

Cách 2. Với x ≥ 4

7, ta có:

V T =√

(7x− 4)(x2 − x+ 3) =⇒ V T ≤ (7x− 4) + (x2 − x+ 3)

2=x2 + 6x− 1

2.

Hay: V T ≤ x2

2+ 3x− 1

2= V P

Dấu = xảy ra ⇐⇒ x2 − x+ 3 = 7x− 4⇐⇒ x2 − 8x+ 7 = 0

Bài 85. Giải phương trình:

51√x− 2 = 3x2 − 58x+ 110

231

Page 233: Chuyên đề luyện thi đại học môn Toán - VipLam.Net

Lời giải

ĐK : x ≥ 2.

Đặt : t =√x− 2. Phương trình trở thành :

27t2 + 51t− 3x2 + 31x− 56 = 0.

Ta có : ∆t = (6x− 31)2....

Bài 86. Giải phương trình:

18x2 − 13x+ 2 =√

3(81x4 − 108x3 + 56x2 − 12x+ 1)

Lời giải

Ta có:

81x4 − 108x3 + 56x2 − 12x+ 1 = (3x− 1)4 + 2x2(1)

Đặt (3x− 1)2 = a. Khi đó, (1) ⇐⇒ a2 + 2x2

PT đã cho trở thành:

2a− x =√

3(a2 + 2x2) ⇐⇒

a ≥ 2x

a2 − 4ax− 5x2 = 0⇐⇒ a = −x hoặc a = 5x

Bài 87. Giải phương trình :

4x2 + 14x+ 11 = 4.√

6x+ 10

Lời giải

Dùng cách đặt ẩn phụ không hoàn toàn ta dễ đưa được về phương trình tương đương là:

(√

6x+ 10− 2x− 3)(√

6x+ 10 + 2x+ 7) = 0

Bài 88. Giải phương trình:

(1− x)√

5− 4x2 = 2x

Lời giải

Bình phương hai vế đưa về phương trình hệ quả sau:

4x4 + 3x2 − 8x3 + 10x− 5 = 0

Nhẩm được hai nghiệm của phương trình là x = −1 và x =1

2.

Ta viết phương trình lại như sau:

(x+ 1)

(x− 1

2

)(4x2 − 10x+ 10) = 0

Thử lại nhận x =1

2. Vậy nghiệm phương trình là x =

1

2.

232

Page 234: Chuyên đề luyện thi đại học môn Toán - VipLam.Net

Bài 89. Giải bất phương trình:

√3− x2(x+ 1) ≤ x

Lời giải

Đặt x =√

3.cost, t ∈ [0; π]. Bất phương trình trở thành :

3. sin t. cos t+√

3(sin t+ cos t) ≤ 0

Đặt tiếp : u = sin t+ cos t, |u| ≤√

2. Lúc đó :

3u2 + 2√

3.u− 3 = 0 ⇐⇒ (u+√

3)(3u−√

3) ≤ 0 ⇐⇒ −√

2 ≤ u ≤√

3

3

Thay trở lại ta có :

−√

2 ≤√

1− x2

3+

x√3≤√

3

3⇐⇒ −

√3 ≤ x ≤ 1−

√5

2

Bài 90. Giải phương trình:

1

x2+

x2

1− x2+

5

2

(√1− x2

x+

x√1− x2

)+ 2 = 0

Lời giải

Đặt a =

√1− x2

x+

x√1− x2

=⇒ a2 =1− x2

x2+

x2

1− x2+ 2 =

1

x2+ 1 +

x2

1− x2.

PTđã cho trở thành:

a2 +5

2a+ 1 = 0 ⇐⇒

a =−1

2a = −2

Suy ra a = −2

Bài 91. Giải bất phương trình:

√x− 1 + x− 3 ≥

√2(x− 3)2 + 2x− 2

Lời giải

Đặt√x− 1 = a;x− 3 = b. Bất phương trình đã cho có dạng

a+ b ≥√

2a2 + 2b2(1)

Mà theo Cauchy-schwarz, ta có: a+ b ≤√

2(a2 + b2)

(1) có nghiệm⇐⇒ a = b hay√x− 1 = x− 3

233

Page 235: Chuyên đề luyện thi đại học môn Toán - VipLam.Net

Bài 92. Giải phương trình:

√x2 − x+ 19 +

√7x2 + 8x+ 13 +

√13x2 + 17x+ 7 = 3

√3(x+ 2)

Lời giải

ĐK: x ∈ R. Ta có:

V T =

√(x− 1

2

)2

+75

4+

√(2x− 1)2 + 3 (x+ 2)2 +

√(x− 1

2

)2

+ 3

(x+

3

2

)2

≥√

75

4+

√3 (x+ 2)2 +

√3

(x+

3

2

)2

Bài 93. Giải phương trình :

4x2 − 2x− 3

2x+√

2x+ 3=

20 + 2x− 11√

2x+ 3

2x

Lời giải

Đặt 2x = a;√

2x+ 3 = b Phương trình ban đầu có dạng

a2 − b2

a+ b=

20 + a− 11b

a⇐⇒ a2 − ab+ 11b− a− 20 = 0

Lại có b2 − a = 3. Thay vào ta có

(b2 − 3)2 − b(b2 − 3) + 11b− (b2 − 3)− 20 = 0 ⇐⇒ (b− 2)(b3 + b2 − 5b+ 4) = 0

Bài 94. Giải bất phương trình

(2x+ 1)√x+ 1 ≥ x2 + 2x− 1

Lời giải

Đặt t =√x+ 1 ta có: Bất phương trình viết lại là :

t4 − 2t3 + t− 2 ≤ 0 ⇐⇒ (t+ 1)(t− 2)(t2 − t+ 1) ≤ 0

Do t ≥ 0 nên ta có :

t ≤ 2 ⇐⇒ x+ 1 ≤ 4 ⇐⇒ −1 ≤ x ≤ 3

Bài 95. Giải phương trình :

8x4 + 33x2 − 9 + 13x3√

3 + x2 = 0

Lời giải

Nhận thấy x = 0 không là nghiệm của pt. Khi đó

Pt ⇐⇒ 8 +33

x2− 9

x4+ 13

√3

x2+ 1 ⇐⇒ 8 + 33t2 − 9t4 + 13

√3t2 + 1 = 0; t =

1

x(1)

234

Page 236: Chuyên đề luyện thi đại học môn Toán - VipLam.Net

Đặt u =√

3t2 + 1 ≥ 1←→ t2 =u2 − 1

3. Khi đó pt(1) ⇐⇒ u4 − 13u2 − 13u+ 4 = 0

⇐⇒ (u− 4)(u3 + 4u2 + 3u− 1

)= 0 ⇐⇒ u = 4(u ≥ 1) ⇐⇒ x = ± 1√

5

Bài 96. Giải bất phương trình :

x+ 2√2 (x4 − x2 + 1)− 1

≥ 1

x− 1

Lời giải

Ta có :√

2 (x4 − x2 + 1) =

√2

(x− 1

2

)2

+3

2> 1

x > 1 .Bất phương trình ban đầu trở thành :

x2 + x− 1 ≥√

2 (x4 − x2 + 1) ⇐⇒

{x2 + x− 1 ≥ 0

x4 − 2x3 − x2 + 2x+ 1 ≤ 0

⇐⇒

{x2 + .x− 1 ≥ 0

(x2 + x− 1)2 ≤ 0

=⇒ x2 + x− 1 = 0 =⇒

x =1−√

5

2(L)

x =1 +√

5

2

x < 1.Bất phương trình ban đầu trở thành :

(x+ 2) (x− 1) ≤√

2 (x4 − x2 + 1)− 1 ⇐⇒

−2 ≤ x < 1{x < −2

(x2 + x− 1)2 ≥ 0

⇐⇒ x < 1

Vậy tập nghiệm của nó là : S =

{1 +√

5

2, (−∞, 1)

}

Bài 97. Giải bất phương trình :

x3 − 2x2 − 40

13− 3√x− 1

≤ x

Lời giải

Điều kiện : x ≥ 1. Khi đó bất phương trình đã cho tương đương với bất phương trình :

x3 − 2x2 − 40

13− 3√x− 1

− x ≤ 0 ⇐⇒ x3 − 2x2 − 13x+ 3x√x− 1− 40

13− 3√x− 1

≤ 0

⇐⇒x2 − 2x− 13 + 3

√x− 1− 40

x13− 3

√x− 1

≤ 0 (3) vì x ≥ 1

Rõ ràng với (3) các kỉ thuật đan dấu cơ bản xem như không thể làm gì được.

Vậy ta chuyển hướng như sau, cái khó là cái tử của bên trái bất phương trình.

235

Page 237: Chuyên đề luyện thi đại học môn Toán - VipLam.Net

Ta đặt f(x) = x2 − 2x− 13 + 3x√x− 1− 40

x, ∀x ≥ 1.

Ta có f ′(x) = 2(x− 1) +3

2√x− 1

+40

x2> 0,∀x ≥ 1.

Vậy hàm số f(x) đồng biến ∀x ≥ 1. Mặt khác ta có f(5) = 0. Do đó dấu của f(x) sẽ cùng dấu

với x− 5.

Do đó bất phương trình (3) sẽ đưa về bất phương trình hết sức cơ bản sau :

x− 5

13− 3√x− 1

≤ 0 ⇐⇒

x− 5 ≤ 0

13− 3√x− 1 > 0x− 5 ≥ 0

13− 3√x− 1 < 0

⇐⇒

x > 178

9x ≤ 5

Kết hợp với điều kiện x ≥ 1 ta có tập nghiệm của bất phương trình đã cho là : S = [1; 5] ∪(178

9; +∞

)Bài 98. Giải phương trình :

1 +1− x√x2 − 1

=5

6x

Lời giải

Cách 1. ĐKXĐ:

{x < −1

x > 1

Với ĐK đó phương trình tương đương với:

6x(√

x2 − 1)

+ (1− x) 6x = 5√x2 − 1 ⇐⇒ (6x− 5)

√x2 − 1 = (x− 1) 6x

=⇒ 12x3 − 47x2 + 60x− 25 = 0 ⇐⇒

x = 54

x = 53

x = 1

Thử lại ta thấy chỉ có x =5

3; x =

5

4thoả mãn phương trình đã cho.

Cách 2.

1 +1− x√x2 − 1

=5

6x⇐⇒

√x2 − 1− x+ 1√

x2 − 1=

5

6x

⇐⇒

−1√x2 − 1 + x

+ 1

√x2 − 1

=5

6x⇐⇒

√x2 − 1 + x− 1√

x2 − 1(√x2 − 1 + x)

=5

6x

Đặt√x2 − 1 = a x = b b 6= 1 ta có:b

2 − a2 = 1a+ b− 1

a2 + ab=

5

6b

=⇒ b2 + b√b2 − 1− 6b+ 5 = 0 =⇒ b2(b2 − 1)− (b− 1)2(5− b)2 = 0

236

Page 238: Chuyên đề luyện thi đại học môn Toán - VipLam.Net

⇐⇒ b2(b+ 1)− (b− 1)(5− b)2 ⇐⇒

[b = 5

4

b = 53

Bài 99. Giải phương trình sau trên tập số thực :

2

√x+√

2x− 1 +√

2 ·(x2 + 3x+ 1

)=√

2 (x− 3)

Lời giải

Đk: x ≥ 3.

Khi đó, phương trình đã cho√2x+ 2

√2x− 1 + x2 + 3x+ 1 =

√x− 3 =⇒

√2x− 1−

√x− 3 + x2 + 3x+ 2 = 0

=⇒ x+ 2√2x− 1 +

√x− 3

+ (x+ 1)(x+ 2) = 0 =⇒ (x+ 2)

(1√

2x− 1 +√x− 3

+ x+ 1

)= 0

Bài 100. Giải phương trình:

3√

2x+ 2 + 3√

2x+ 1 =3√

2x2 +3√

2x2 + 1

Lời giải

Xét hàm số f(t) = 3√t+ 3√t+ 1.Ta có f ′(t) =

1

33√t2

+1

3 3√

(t+ 1)2> 0.

Hàm số đã cho luôn đồng biến. Do vậy từ phương trình ta có:

f(2x+ 1) = f(2x2) ⇐⇒ 2x2 − 2x− 1 = 0 ⇐⇒

x =

1 +√

3

2

x =1−√

3

2

Kết luận: Phương trình đã cho có 2 nghiệm

x =

1 +√

3

2

x =1−√

3

2

Bài 101. Giải phương trình sau:

3√x2 + 4 =

√x− 1 + 2x− 3

Lời giải

PT ⇐⇒ (x− 2)(x+ 2)3√

(x2 + 4)2 + 2 3√

(x2 + 4) + 4− x− 2√

x− 1 + 1− 2(x− 2) = 0

⇐⇒

x = 2(x+ 2)

3√

(x2 + 4)2 + 2 3√

(x2 + 4) + 4=

1√x− 1 + 1

+ 2 (∗)

Phương trình (∗) ta thấy: V T < 2 < V P .

237

Page 239: Chuyên đề luyện thi đại học môn Toán - VipLam.Net

Bài 102. Giải phương trình:

(2x+ 1)(2 +√

(2x+ 1)2 + 3) + 3x(2 +√

9x2 + 3) = 0

Lời giải

Xét hàm số: f(t) = 2t+ t√t2 + 3. Có f ′(t) = 2 +

√t2 + 3 +

t2√t2 + 3

> 0.

Phương trình trở thành:

f(2x+ 1) = f(−3x) ⇐⇒ −3x = 2x+ 1

Bài 103. Giải bất phương trình:

√51− 2x− x2

1− x< 1

Lời giải

Xét 1− x > 0 ⇐⇒ x < 1 Từ bất phương trên ta có

√51− 2x− x2 < 1− x ⇐⇒

{x < 1

2x2 − 50 > 0(∗)

Từ (∗) ta được

(x− 5)(x+ 5) > 0

So sánh với điều kiện ta được tập nghiệm trong trương hợp này là x < −5

Tương tự với trường hợp trên với x > 1 ta có:

√51− 2x− x2 > 1− x

Mặt khác do x > 1 nên bất phương trên chỉ đúng khi và chỉ khi{1− x < 0

51− 2x− x2 > 0

Bài 104. Giải phương trình:

3x(2 +√

9x2 + 3) + (4x+ 2)(1 +√

1 + x+ x2) = 0

Lời giải

Ta có

(4x+ 2)(1 +√x2 + x+ 1 = 2(2x+ 1)(1 +

√x2 + x+ 1 = (2x+ 1)(2 +

√(2x+ 1)2 + 3)

Do vậy phương trình trên trở thành

3x(2 +√

9x2 + 3 + (2x+ 1)(2 +√

(2x+ 1)2 + 3) = 0

⇐⇒ (2x+ 1)(2 +√

(2x+ 1)2 + 3) = −3x(2 +√

(−3x)2 + 3

238

Page 240: Chuyên đề luyện thi đại học môn Toán - VipLam.Net

Đến đây quay về xét hàm số: f(t) = t(2 +√t2 + 3)

Bài 105. Giải phương trình:

√1− x2 +

4√x2 + x− 1 + 6

√1− x = 1

Lời giải

Đặt a =√

1− x2,b = 4√x2 + x− 1,c = 6

√1− x. a, b, c ≥ 0. Ta có :a2 + b4 + c6 = 1

a+ b+ c = 1

Từ đó ta có :

a(a− 1) + b(b− 1)(b2 + b+ 1) + c(c5 − 1) = 0

Dể nhận thấy a, b, c ∈ (0; 1] suy ra V T ≥ 0 = V P .Đẳng thức xảy ra khi x = 1.

Bài 106. Giải phương trình:

x2 − 2x+ 3 =√

2x2 − x+√

1 + 3x− 3x2

Lời giải

Sử dụng bất đẳng thức Cauchy − Schwarz ta có :

√2x2 − x+

√1 + 3x− 3x2 ≤

√2(−x2 + 2x+ 1) ≤

√2(2− (x− 1)2) ≤ 2

Hay

x2 − 2x+ 3 ≤ 2 ⇐⇒ (x− 1)2 ≤ 0

Vậy x = 1 là nghiêm phương trình.

Bài 107. Giải bất phương trình

√x2 + x− 2 +

√x2 + 2x− 3 ≤

√x2 + 4x− 5

Lời giải

ĐK:

[x ≥ 1

x ≤ −5

]Bình phương 2 vế của phương trình đã cho

(x− 1) (x+ 2) + (x+ 3) (x− 1) + 2

√(x+ 2) (x+ 3) (x− 1)2 ≤ (x− 1)(x+ 5)

⇐⇒ 2

√(x+ 2) (x+ 3) (x− 1)2 ≤ −x(x− 1) =⇒ 0 ≤ x ≤ 1

Vậy bất phương trình có nghiệm duy nhất x = 1

Bài 108. Giải phương trình:

(2− x)√x+√

3− 2x =√−x3 + 7x2 − 17x+ 15

239

Page 241: Chuyên đề luyện thi đại học môn Toán - VipLam.Net

Lời giải

Điều kiện: 0 ≤ x ≤ 3

2. Khi đó, phương trình ban đầu tương đương với

(2− x)√x+√

3− 2x =√

(3− x)(x2 − 4x+ 5)

Nhận thấy, x = 0 hay x =3

2đều không phải là nghiệm của PT đã cho nên ta chỉ xét trong điều

kiện 0 < x <3

2.

Sử dụng BĐT Cauchy − Schwarz bên vế trái phương trình ta có

V T 2 ≤[(2− x)2 + 1

] [(√x)2 + (

√3− 2x)2

]= (3− x)(x2 − 4x+ 5) = V P 2

Đẳng thức trên xảy ra khi và chỉ khi

2− x√x

=1√

3− 2x⇐⇒ (x− 1)(2x2 − 9x+ 12) = 0 ⇐⇒ x = 1

Thử lại giá trị x = 1 thỏa mãn PT đã cho. Vậy x = 1 là nghiệm duy nhất của PT.

Bài 109. Giải phương trình:

2− 3√x−√x+ 8− 2x = 0

Lời giải

Điều kiện : x ≥ 0. Với điều kiện này bất phương trình đã cho tương đương với bất phương trình

:

2x+ 3√x+ 3

√x+ 8 ≤ 2 (1)

Với x = 0, ta có : 2√

2 ≤ 2 (vô lí). Xét với x > 0 thì hàm số f(x) = 2x + 3√x + 3

√x+ 8 có

f ′(x) = 2 +3

2√x

+3

2√x+ 8

> 0. ∀x > 0

Do đó hàm số f(x) đồng biến với mọi x > 0 nên f(x) > f(0) = 2√

2 > 2

Bài 110. Giải phương trình :

√1− x+

√1 + x = 2− x2

4

Lời giải

Đặt√

1− x = a,√

1 + x = b, a, b ≥ 0.

Suy ra :

{1− x = a2

1 + x = b2=⇒

a2 + b2 = 2

x =b2 − a2

2

Lúc đó ta có hệ :

a2 + b2 = 1 (1)

a+ b = 2− 1

16(b2 − a2)

2(2)

Từ pt (2) suy ra :

(a+ b)2(a− b)2 + 16 (a+ b)− 32 = 0 =⇒ (2 + 2ab) (2− 2ab) + 16 (a+ b)− 32 = 0.

240

Page 242: Chuyên đề luyện thi đại học môn Toán - VipLam.Net

Từ đó ta có hệ mới : {(a+ b)2 − 2ab = 2

16 (a+ b)− 4(ab)2 − 28 = 0

=⇒ 16 (a+ b)−[(a+ b)2 − 2

]2 − 28 = 0 =⇒ (a+ b)4 − 2(a+ b)2 − 16 (a+ b) + 32 = 0

=⇒ a+ b = 2 (Do a+ b ≥ 0)

Bài 111. Giải phươnh trình:

√2x+ 4− 2

√2− x =

12x− 8√9x2 + 16

Lời giải

Điều kiện: −2 ≤ x ≤ 2. Khi đó

PT ⇐⇒ 2(3x− 2)√2x+ 4 + 2

√2− x

=4(3x− 2)√

9x2 + 16

⇐⇒[x =

2

3

√9x2 + 16 = 2

√2x+ 4 + 4

√2− x (1)

Tiếp tục bình phương PT (1) ta được

(1) ⇐⇒ 9x2 − 32 = 16√

8− 2x2 − 8x

=⇒ 9x2 − 32 =64(32− 9x2)

16√

8− 2x2 + 8x(x 6= −4

√2

3)

⇐⇒

9x2 − 32 = 0 (2)8

2√

8− 2x2 + x+ 1 = 0 (3)

Với điều kiện bài toán thì PT (3) vô nghiệm.

Mà (2) ⇐⇒ x =4√

2

3hoặc x = −4

√2

3(loại).

Vậy, phương trình đã cho có 2 nghiệm x =2

3; x =

4√

2

3.

241

Page 243: Chuyên đề luyện thi đại học môn Toán - VipLam.Net

CÁC BÀI TOÁN

SỬ DỤNG KĨ THUẬT ĐÁNH GIÁ ĐƯA VỀ

CÙNG MẪU ĐỂ CHỨNG MINH BẤT ĐẲNG

THỨC

(Hoàng Trung Hiếu)

Phương pháp đánh giá đưa về cùng mẫu để chứng minh bất đẳng thức có nghĩa là sử dụng các

đánh giá quen thuộc để đánh giá đưa các mẫu khác nhau về cùng môt mẫu giúp việc đánh giá

còn lại thuận tiện. Các bất đẳng thức thường áp dụng là các bất đẳng thức có tính đối xứng.

Mong rằng với bài viết này các bạn có thêm một chút kĩ thuật đánh giá bất đẳng thức

Một số bất đẳng thức và kí hiệu trong bài viết:

∗ Bất đẳng thức AM_GM: Cho x1;x2; ...;xn là các số thực dương. Ta có:

x1 + x2 + ..+ xnn

≥ n√x1.x2.....xn

∗ Bất đẳng thức Cauchy - swchwarz

(a21 + a2

2 + ..+ a2n)(b2

1 + b22 + ..+ b2

n) ≥ (a1b1 + a2b2 + ...+ anbn)2

∗ Ký hiệu∑

: Tổng hoán vị

Bài toán 1: Cho a; b; c > 0. Chứng minh

1

(a+ b)2+

1

(a+ c)2≥ 1

a2 + bc

Phân tích Nhận thấy ở đây các biến không có vai trò như nhau và biến a xuất hiện nhiều

nhất nên ta không thể áp dụng trực tiếp bất đẳng thức Cauchy Swcharz dạng phân thức hay

AM-GM ở đây được Nhưng chúng ta có thể đánh giá mẫu của từng phân thức của vế trái theo

mẫu phân thức vế phải nên ta có lời giải sau

Lời giải

Theo bất đẳng thức Cauchy − Swcharz Ta có:

1

(a+ b)2≥ 1

(a2 + bc)(1 +b

c)

=1

a2 + bc.c

b+ c

242

Page 244: Chuyên đề luyện thi đại học môn Toán - VipLam.Net

Tương tự

1

(a+ c)2≥ 1

a2 + bc.b

b+ c=⇒ 1

(a+ b)2+

1

(a+ c)2≥ 1

a2 + bc(b+ c

b+ c) ≥ 1

a2 + bc

Dấu = xảy ra ⇐⇒ a = b = c

Bài toán 2: Cho a; b; c > 0. Chứng minh rằng:∑ ab

a2 + bc+ ca≤ a2 + b2 + c2

ab+ bc+ ca

Lời giải

Sử dụng bất đẳng thưứcCauchy - schwarz ta có:

(a2 + bc+ ca)(b2 + bc+ ca) ≥ (ab+ bc+ ca)2 =⇒ ab

a2 + bc+ ca≤ ab(b2 + ca+ bc)

(ab+ bc+ ca)2

Như vậy cần chứng minh:∑ab(b2 + ca+ bc) ≤ (a2 + b2 + c2)(ab+ bc+ ca)

⇐⇒ a3b+ b3c+ c3a ≥ abc(a+ b+ c) ⇐⇒∑ a2

b≥ a+ b+ c

Đây là đánh giá quen thuộc

Dấu = xảy ra khi a = b = c

Bài toán 3: Cho a; b; c > 0 thoả mãn ab+ bc+ ca = 3. Tìm giá trị lớn nhất của:

A =1

a2 + b2 + 2013+

1

b2 + c2 + 2013+

1

c2 + a2 + 2013

Lời giải

Áp dụng bất đẳng thức Cauchy - Schwarz ta có:

(a2+b2+1+2012)[1+1+c2+2012(a+ b+ c

3)2] ≥ (a+b+c+2012(

a+ b+ c

3)2 = 20152 (a+ b+ c)2

9

=⇒ 1

a2 + b2 + 2013≤ 9(c2 + 2) + 3.2012.(a+ b+ c)2

20152(a+ b+ c)2

Như vậy

A ≤ 9(a2 + b2 + c2 + 6) + 3.2012(a+ b+ c)2

20152.(a+ b+ c)2=

3

2015- vì ab+ bc+ ca = 3

Dấu đẳng thức khi a = b = c = 1

Tổng quát: Cho a; b; c > 0 thoả mãn ab+ bc+ ca = 3 và k ≥ 1. Chứng minh

A =1

a2 + b2 + k+

1

b2 + c2 + k+

1

c2 + a2 + k≤ 3

2 + k

Bài toán 4: Cho a; b; c > 0. Chứng minh rằng:

1

a+ b+ c≥∑ a3

(2a2 + b2)(2a2 + c2)

Phân tích Rõ ràng ta thấy đây là 1 bất đẳng thức khá dài Xét vế trái khiến ta rất khó đánh

243

Page 245: Chuyên đề luyện thi đại học môn Toán - VipLam.Net

giá ngược Còn vế phải thì không thể áp dụng Cauchy-Schwarz Nên ta nghĩ việc đánh giá mẫu

số

Lời giải

Ta có:

(a2 + b2 + a2)(a2 + a2 + c2) ≥ (a2 + ab+ ac)2 = a2(a+ b+ c)2

=⇒ a3

(2a2 + b2)(2a2 + c2)≤ a

(a+ b+ c)2=⇒

∑ a3

(2a2 + b2)(2a2 + c2)≤ 1

a+ b+ c

Dấu = xảy ra khi a = b = c

Bài toán 5: Cho a; b; c > 0. Chứng minh:

∑ 1

a2 + ab+ bc≤(

a+ b+ c

ab+ bc+ ca

)2

Lời giải

Ta có:

(a2 + ab+ bc)(c2 + ab+ bc) ≥ (ab+ bc+ ca)2

=⇒ 1

a2 + ab+ bc≤ c2 + ab+ bc

(ab+ bc+ ca)2

=⇒∑ 1

a2 + ab+ bc≤∑c2 + ab+ bc

(ab+ bc+ ca)2=

(a+ b+ c

ab+ bc+ ca

)2

Dấu = xảy ra khi a = b = c

Bài toán 6: Cho a; b; c > 0; ab+ bc+ ca = 3. Chứng minh rằng:

1

a2 + 2+

1

b2 + 2+

1

c2 + 2≤ 1

Nhận xét: Nếu ta đánh giá trực tiếp theo Cauchy-Schwarz dạng phân thức thì dấu đẳng thức

phải là ≥ nên ta nghĩ đến việc đánh giá mẫu

Lời giải

Ta có:

1

a2 + 2=

1 +(b+ c)2

2

(a2 + 1)[1 +(b+ c)2

2]

≤1 +

(b+ c)2

2(a+ b+ c)2

Nên

∑ 1

a2 + 2≤

3 +∑ (b+ c)2

2(a+ b+ c)2

=(ab+ bc+ ca) + (a2 + b2 + c2 + ab+ bc+ ca)

(a+ b+ c)2= 1

Dấu = xảy ra ⇐⇒ a = b = c = 1

244

Page 246: Chuyên đề luyện thi đại học môn Toán - VipLam.Net

Bài toán 7: Cho x; y; z > 0 và xyz ≥ 1 Chứng minh

a5 − a2

a5 + c2 + b2+

b5 − b2

b5 + c2 + a2+

c5 − c2

c5 + a2 + b2≥ 0

Phân tích Bài toán này khó hơn bởi ta không thấy mẫu số cần để chứng minh Vì vậy chúng

ta phải vận dụng phương pháp thêm bớt vào đây Thấy

a5 + b2 + c2 − (a5 − a2) = a2 + b2 + c2

một đại lượng đối xứng nên ta trừ 1 ở tùng hạng tử và ta có

Lời giải

a5 − a2

a5 + c2 + b2+

b5 − b2

b5 + c2 + a2+

c5 − c2

c5 + a2 + b2≥ 0

⇐⇒∑

(1− a5 − a2

a5 + c2 + b2) ≤ 3 ⇐⇒

∑ 1

a5 + b2 + c2≤ 3

a2 + b2 + c2

Áp dụng bất đẳng thức CauchySchwarz ta có:

1

a5 + b2 + c2=

1

a+ b2 + c2

(1

a+ b2 + c2)(a5 + b2 + c2)

1

a+ b2 + c2

(a2 + b2 + c2)2

Cần chứng minh:1

a+

1

b+

1

c+ 2(a2 + b2 + c2) ≤ a2 + b2 + c2

⇐⇒ 1

a+

1

b+

1

c≤ a2 + b2 + c2

Luôn đúng vì:1

a+

1

b+

1

c=ab+ bc+ ca

abc≤ ab+ bc+ ca ≤ a2 + b2 + c2

Tổng quát: Cho số tự nhiên n ≥ 3 và 3 số thực dương x, y, z thỏa mãn xyz ≥ 1. Chứng

minhxn − x2

xn + z2 + y2+

yn − y2

yn + z2 + x2+

zn − z2

zn + x2 + y2≥ 0

Bài toán 8: Cho a; b; c ≥ 0; abc = 1. Chứng minh rằng

1

1 + a+ b2+

1

1 + b+ c2+

1

1 + c+ a2≤ 1

Lời giải

Đặt a = x3; b = y3; c = z3. Ta chứng minh

1

1 + x3 + y6+

1

1 + y3 + z6+

1

1 + z3 + x6≤ 1

245

Page 247: Chuyên đề luyện thi đại học môn Toán - VipLam.Net

Áp dụng bất đẳng thức CauchySchwarz ta có

1

1 + x3 + y6=

z4 + x+1

y2

(1 + x3 + y6)(z4 + x+1

y2)≤ z4 + x+ z2x2

(x2 + y2 + z2)2

Tương tự với các phân thức còn lại và cộng lại ta có:

1

1 + x3 + y6+

1

1 + y3 + z6+

1

1 + z3 + x6≤ x4 + y4 + z4 + x+ y + z + x2y2 + y2z2 + z2x2

(x2 + y2 + z2)2

Ta cần chứng minh:

x4 + y4 + z4 + x+ y + z + x2y2 + y2z2 + z2x2 ≤ (x2 + y2 + z2)2

⇐⇒ x+ y + z ≤ x2y2 + y2z2 + z2x2

⇐⇒ xyz(x+ y + z) ≤ x2y2 + y2z2 + z2x2 - luôn đúng theo AM −GMDấu = xảy ra khi a = b = c = 1.

Bài toán 9: Cho a; b; c > 0; abc = 1. Chứng minh rằng:∑ a+ b+ 1

a+ b2 + c3≤ (a+ 1)(b+ 1)(c+ 1) + 1

a+ b+ c

Lời giải

Ta có:a+ b+ 1

a+ b2 + c3=

(a+ b+ 1)(a+ 1 + 1c)

(a+ b2 + c3)(a+ 1 + 1c)≤ (a+ b+ 1)(a+ 1 + ab)

(a+ b+ c)2

Cần chứng minh:∑(a+ b+ 1)(a+ 1 + ab) ≤ [(a+ 1)(b+ 1)(c+ 1) + 1](a+ b+ c)

⇐⇒ ab+ bc+ ca ≥ 3. Luôn đúng do ab+ bc+ ca ≥ 3 3√

(abc)2 = 3

Dấu = xảy ra khi a = b = c = 1

Bài toán 10: Cho a; b; c > 0 thoả mãn: ab+ bc+ ca = 3. Chứng minh rằng:∑ a

a3 + b2 + c≤ 1

Lời giải

Sử dụng bất đẳng thức cauchy − schwarz ta có:

a

a3 + b2 + c=

a( 1a

+ 1 + c)

(a3 + b2 + c)( 1a

+ 1 + c)≤ a+ 1 + ca

9

Cần chứng minh; ∑ a+ 1 + ca

9≤ 1

246

Page 248: Chuyên đề luyện thi đại học môn Toán - VipLam.Net

⇐⇒ ab+ bc+ ca ≤ 3. Luôn đúng do ab+ bc+ ca ≤ (a+ b+ c)2

3= 3

Dấu = xảy ra khi a = b = c = 1

Bài toán tương tự : Cho abc = 1; a; b; c > 0. Chứng minh:∑ 1

a3 + b2 + c≤ 1

Bài toán 11: Cho∑ 1

b+ c+ 1≥ 1 Chứng minh rằng:

a+ b+ c ≥ ab+ bc+ ca

Lời giải

Sử dụng bất đẳng thức Cauchy-schwarz ta có:

1

b+ c+ 1=

b+ c+ a2

(b+ c+ 1)(b+ c+ a2)≤ b+ c+ a2

(a+ b+ c)2

=⇒∑ 1

b+ c+ 1≤ 2(a+ b+ c) + a2 + b2 + c2

(a+ b+ c)2

Từ giả thiết suy ra :

2(a+ b+ c) + a2 + b2 + c2

(a+ b+ c)2≥ 1 ⇐⇒ a+ b+ c ≥ ab+ bc+ ca

Dấu = xảu ra khi: a = b = c = 1

Bài toán 12: cho a; b; c > 0.Chứng minh rằng:∑ 1

2a2 + bc≤ (

a+ b+ c

ab+ bc+ ca)2

Lời giải

Ta có:

(2a2 + bc)((b+ c)2

2+ bc) ≥ (ab+ bc+ ca)2

=⇒ 1

2a2 + bc≤

(b+c)2

2+ bc

(ab+ bc+ ca)2

=⇒∑ 1

2a2 + bc≤ (

a+ b+ c

ab+ bc+ ca)2

Bài toán 13: Cho a; b; c > 0 thoả mãn : a+ b+ c = 3 Chứng minh rằng:∑ 1

a2 − a+ 3≤ 1

Lời giải

Ta có:

a2 − a+ 3 = a2 − a+ a+ b+ c = a2 + b+ c

247

Page 249: Chuyên đề luyện thi đại học môn Toán - VipLam.Net

Lại có:

(a2 + b+ c)(1 + b+ c) ≥ (a+ b+ c)2 = 32

=⇒ 1

a2 − a+ 2013≤ 1 + b+ c

32

=⇒∑ 1

a2 − a+ 2013≤ 3 + 2(a+ b+ c)

32= 1

Dấu = xảy ra khi a = b = c = 1

Tổng quát: Cho n số thực dương x1;x2; ...xn thoả mãn : x1 +x2 + ...+xn = n Chứng minh

rằng: ∑ 1

x21 − x1 + n

≤ 1

Bài toán 14: Cho a; b; c > 0. Chứng minh rằng:

(3a− b+ c)2

2a2 + (b+ c)2+

(3b− c+ a)2

2b2 + (c+ a)2+

(3c− a+ b)2

2c2 + (a+ b)2≥ 9

2

Lời giải

Áp dụng bất đẳng thức Cauchy − Schwarz ta có:

(b+ c)2 ≤ 2(b2 + c2) =⇒ 2a2 + (b+ c)2 ≤ 2(a2 + b2 + c2)

=⇒ (3a− b+ c)2

2a2 + (b+ c)2≥ (3a− b+ c)2

2(a2 + b2 + c2)

Nên :

(3a− b+ c)2

2a2 + (b+ c)2+

(3b− c+ a)2

2b2 + (c+ a)2+

(3c− a+ b)2

2c2 + (a+ b)2≥ (3a− b+ c)2 + (3b− c+ a)2 + (3c− a+ b)2

2(a2 + b2 + c2)

Cần chứng minh :

(3a− b+ c)2 + (3b− c+ a)2 + (3c− a+ b)2 ≥ 9(a2 + b2)

⇐⇒ a2 + b2 + c2 ≥ ab+ bc+ ca- luôn đúng

Dấu = xảy ra khi a = b = c.

Bài toán 15: Cho 3 số thực dương a,b,c, chứng minh rằng:

(2a+ b+ c)2

2a2 + (b+ c)2+

(2b+ a+ c)2

2b2 + (a+ c)2+

(2c+ a+ b)2

2c2 + (a+ b)2≤ 8

Phân tích Ta thấy bài 14 khá giống bài 13 nhưng dấu bất đẳng thức lại là ≤ nên ta cần biến

đổi về giống bài 13 và ta sử dụng kĩ thuật thêm bớt.

Lời giải

Để ý rằng:

3− (2a+ b+ c)2

2a2 + (b+ c)2=

2(b+ c− a)2

2a2 + (b+ c)2

248

Page 250: Chuyên đề luyện thi đại học môn Toán - VipLam.Net

(biến đổi bình thường) nên ta có thể viết lại BĐT đã cho:

2(b+ c− a)2

2a2 + (b+ c)2+

2(c+ a− b)2

2b2 + (c+ a)2+

2(a+ b− c)2

2c2 + (a+ b)2≥ 1 (∗)

Dễ dàng c/m:

(b+ c)2 ≤ 2(b2 + c2); (a+ c)2 ≤ 2(a2 + c2); (a+ b)2 ≤ 2(a2 + b2)

nên ta có:

V T (∗) ≥ (b+ c− a)2

a2 + b2 + c2+

(a+ c− b)2

a2 + b2 + c2+

(a+ b− c)2

a2 + b2 + c2

Do đó, ta chỉ cần c/m:

(b+ c− a)2 + (a+ b− c)2 + (c+ a− b)2 ≥ a2 + b2 + c2

Đúng vì:

(b+ c− a)2 + (a+ b− c)2 ≥ (b+ c− a+ a+ b− c)2

2≥ 2b2

... (Xây dựng các BĐT tương tự rồi cộng lại) Vậy (∗) đúng. BĐT ban đầu được c/m. Dấu bằng

xảy ra khi và chỉ khi a = b = c

Bài toán 16: Cho a, b, c > 0. Chứng minh rằng:(1 +

2a

b

)2

+

(1 +

2b

c

)2

+

(1 +

2c

a

)2

≥ 9(a+ b+ c)2

ab+ bc+ ca

Lời giải

Ta có:

A =

(1 +

2a

b

)2

+

(1 +

2b

c

)2

+

(1 +

2c

a

)2

A =(2a+ b)2 b

aab

+(2b+ c)2 c

bbc

+(2c+ a)2 c

aca

Theo bất đẳng thức Cauchy = Schwart ta có:

A ≥[(2a+ b)

√b

a+ (2b+ c)

√c

b+ (2c+ a)

√c

a]2

ab+ bc+ ca

Như vậy ta phải chứng minh:

(2a+ b)

√b

a+ (2b+ c)

√c

b+ (2c+ a)

√c

a≥ 3(a+ b+ c) (1)

Đặt x =√a; y =

√b; z =

√c . (1) trở thành:

2(x3

y+y3

z+z3

x) + xy + yz + zx ≥ 3(x2 + y2 + z2)

249

Page 251: Chuyên đề luyện thi đại học môn Toán - VipLam.Net

Đây là bất đẳng thức đúng vì: Theo AM −GM ta có:x3

y+ xy ≥ 2x2 Nên

x3

y+y3

z+z3

x+ xy + yz + zx ≥ 2(x2 + y2 + z2) (2)

Mặt khácx3

y+x3

y+ y2 ≥ 3x2

Tướng tự cộng lại ta có:

2(x3

y+y3

z+z3

x) + (x2 + y2 + z2) ≥ 3(x2 + y2 + z2)

=⇒ x3

y+y3

z+z3

x≥ x2 + y2 + z2 (3)

Cộng (2) với (3) chính là (1) Chứng minh hoàn tất

Dấu = xảy ra ⇐⇒ a = b = c

250

Page 252: Chuyên đề luyện thi đại học môn Toán - VipLam.Net

CÁC BÀI TOÁN

SỬ DỤNG CÁC BẤT ĐẲNG THỨC CỔ ĐIỂN

ĐƯA BÀI TOÁN TÌM GTLN, GTNN CỦA

HÀM NHIỀU BIẾN VỀ HÀM MỘT BIẾN

(Lê Hoàng Hải)

§ 1. Kiến thức cần nhớ vể các bất đẳng thức cổ điển

thường dùng.

I. Bất đẳng thức AM-GM.

Nếu x1, x2, ..., xn là các số thực không âm thì:

x1 + x2 + ..+ xnn

≥ n√x1x2...xn

Dấu bằng xảy ra khi và chỉ khi x1 = x2 = ... = xn

II. Bất đẳng thức Cauchy-Schwarz

Nếu a1, a2, ..., an ;b1, b2, ..., bn là các số thực tùy ý thì:

(a1b1 + a2b2 + ...+ anbn)2 ≤ (a12 + a2

2 + ...+ an2)(b1

2 + b22 + ...+ bn

2)

Nếu chọn ai =xi√yi

; bi =√yi với xi; yi ∈ R; yi > 0 ; ta thu được bất đẳng thức Cauchy-Schwarz

dạng phân thức:

Nếu x1, x2, ..., xn là các số thực và y1, y2, ...yn là các số thực dương thì:

x12

y1

+x2

2

y2

+ ...xn

2

yn≥ (x1 + x2 + ...+ xn)2

y1 + y2 + ...+ yn

251

Page 253: Chuyên đề luyện thi đại học môn Toán - VipLam.Net

§ 2. Các ví dụ minh họa.

Ví dụ 1: (Đề thi thử Đại Học số 12 năm 2012-boxmath.vn)

Cho a;b;c là các số thực dương thỏa mãn: a(b2 + c2) = b + c.Tìm giá trị nhỏ nhất của biểu

thức:

P =1

(1 + a)2 +1

(1 + b)2 +1

(1 + c)2 +1

(1 + a)(1 + b)(1 + c)

Phân tích:

Ta thấy biểu thức P và giả thiết đã cho là đối xứng với 2 biến b và c; từ đó ta có thể dự đoán

được dấu bằng xảy ra sẽ có b = c. Sử dụng giả thiết đã cho; ta được;

b+ c = a(b2 + c2) ≥ a(b+ c)2

2=⇒ b+ c ≤ 2

a(∗).

Vì vậy; ta sẽ tìm cách đánh giá biểu thức P để đưa về 2 biến a và (b+ c); sau đó sử dụng điều

kiện (∗) để đưa biểu thức về một biến a. Đánh giá này xem ra rất khả quan vì các biểu thức

1

(1 + b)2 +1

(1 + c)2 ;1

(1 + b)(1 + c)

đều có thể đánh giá đưa về f(b+ c) bằng AM-GM không mấy khó khăn.

Lời giải:

Áp dụng bất đẳng thức AM-GM; ta có:

P ≥ 1

(1 + a)2 +2

(1 + b)(1 + c)+

4

(1 + a)(1 + b)(1 + c)

Cũng theo AM-GM; ta có:

(1 + b)(1 + c) ≤ 1

4(2 + b+ c)2 ≤ 1

4

(2 +

2

a

)2

≤ (1 + a)2

a2

Do đó:

P ≥ 2a2 + 1

(1 + a)2 +4a2

(1 + a)3 =2a3 + 6a2 + a+ 1

(a+ 1)3

(1)Thành viên hoanghai1195 trên diễn đàn http://k2pi.net

Xét hàm số f(a) =2a3 + 6a2 + a+ 1

(a+ 1)2 với a > 0 . f′(a) =

2(5a− 1)

(a+ 1)4 = 0 ⇐⇒ a =1

5.

Lập bảng biến thiên; ta thấy P ≥ f(a) ≥ f

(1

5

)=

91

108

Vậy minP =91

108⇐⇒ a =

1

5; b = c = 5

Ví dụ 2:

Cho x, y, z ≥ 1 thỏa mãn:3(x + y + z) = x2 + y2 + z2 + 2xy .Tìm giá trị nhỏ nhất của biểu

thức:

P =x2

(x+ y)2 + x+

x

z2 + x

252

Page 254: Chuyên đề luyện thi đại học môn Toán - VipLam.Net

Phân tích:

Giả thiết của bài toán có thể viết lại thành:

3(x+ y + z) = (x+ y)2 + z2

Để sử dụng giả thiết này; ta sẽ tìm cách để liên kết được 2 biểu thức (x+ y)2và z2 với nhau

trong 2 phân thức. Khi đó bất đẳng thức mà chúng ta nghĩ đến đầu tiên là bất đẳng thức dạng:

u2

a+v2

b≥ (u+ v)2

a+ b.

Nếu sử dụng ngay đánh giá này; ta sẽ thu được

P ≥ (x+√x)

2

3(x+ y + z) + 2x

Vậy ta cần đánh giá thêm cho đại lượng (x+ y + z)

Lời giải:

Từ giả thiết; ta có:

3(x+ y + z) = (x+ y)2 + z2 ≥ 1

2(x+ y + z)2 =⇒ x+ y + z ≤ 6

Ta có :

P ≥ x2

(x+ y)2 + x+

1

z2 + x≥ (x+ 1)2

(x+ y)2 + z2 + 2x≥ (x+ 1)2

3(x+ y + z) + 2x≥ (x+ 1)2

2x+ 18

Xết hàm số f(x) =(x+ 1)2

2x+ 18với x ≥ 1 ;f

′(x) =

x2 + 18x+ 17

2(x+ 9)2 > 0;∀x ≥ 1

Do đó hàm số đồng biến trên[1; +∞] =⇒ f(x) ≥ f(1) =1

5.

Vậy minP =1

5⇐⇒ x = 1; y = 2; z = 3

(Lời giải này đưa P về hàm số f =(x+ 1)2

2x+ 18để quá trình đạo hàm bớt phức tạp ;

bạn hoàn toàn có thể xét hàm số f =(x+

√x)

2

2x+ 18vẫn cho ra kết quả.

Ví dụ 3: (Đề thi thử Đại Học số 4 năm 2013-k2pi.net)

Cho các số thực x, y, z thỏa mãn : x2 + 2y2 + 5z2 ≤ 2. Tìm giá trị lớn nhất của

P = (xy + yz + zx)[1−√

4− (x2 + 2y2 + 5z2)2]

Phân tích:

Đây là một bài toán khá “lộ” về mặt ý tưởng. Rõ ràng ta luôn có thể có một đánh giá

xy + yz + zx ≤ α(x2 + 2y2 + 5z2).

Vấn đề còn lại là chọn được “điểm rơi” của bài toán.Nhưng cái hay và khó lại nằm ở phần

này. Ta hoàn toàn có thể sử dụng kĩ thuật “chọn điểm rơi giả định ” (Bạn đọc có thể tìm hiểu

253

Page 255: Chuyên đề luyện thi đại học môn Toán - VipLam.Net

thêm trong cuốn “Sử dụng AM-GM để chứng minh bất đẳng thức(Võ Quốc Bá Cẩn-Trần Quốc

Anh)”) ; nhưng lời giải như vậy sẽ rất phức tạp về mặt tính toán.

Lời giải sau sử dụng những biến đổi khá tinh tế của bạn “Inspectorgadget”

Lời giải:

Áp dụng bất đẳng thức Cauchy − Schwarzta có:

2x2 + 3y2 + 6z2 =x2

12

+y2

13

+z2

16

≥ (x+ y + z)2

12

+ 13

+ 16

= (x+ y + z)2

Vậy nên:

x2 + 2y2 + 5z2 ≥ 2(xy + yz + zx) =⇒ 2P ≤ t.(1 +√

4− t2)

Với t = x2 + 2y2 + 5z2 ≤ 2

Vậy ta chỉ cần tìm giá trị lớn nhất của : K = t.(1 +√

4− t2)(t ≤ 2)

Dấu bằng xảy ra khi 2x = 3y = 6z.

Ba ví dụ trên là những bài toán mà việc phát hiên ra các bất đẳng thức phụ để quy hàm

nhiều biến về hàm một biến không gặp nhiều khó khăn ; cái khó chính là việc dự đoán được

‘’điểm rơi” của bài toán. Nhưng thực tế ; nhiều bài toán mà các bất đẳng thức phụ cần phải

phán đoán và chứng minh lại để đưa được bài toán về một biến.

Ví dụ 4:

a, b, c > 0 thỏa mãn: a2 + b2 + c2 − 2bc+ ab− 2ca = 0,Tìm giá trị nhỏ nhất của biểu thức:

P =c2

(a+ b− c)2 +c2

a2 + b2+

√ab

a+ b

Phân tích:

Ta viết lại giả thiết cuả bài toán :(a+ b− c)2 = ab nên ta có thể dự đoán dấu bằng xảy ra khi

và chỉ khi a = b = c.

Lại thấy giả thiết của bài toán và biểu thức P đều là những biểu thức đồng bặc ; nên ta sẽ nghĩ

đến việc đưa biểu thức P về biến t =c

a+ bhoặc t =

c2

a2 + b2

Có thể bạn sẽ nghĩ đến việc đưa về biến t =c2

a2 + b2đầu tiên ; vì ta có thể đánh giá :

c

a+ b≥ c√

2(a2 + b2)=

√c2

2(a2 + b2)

Tuy nhiên ; khi đánh giá với biểu thức

√ab

a+ b; ta sẽ thấy không thể đi theo hướng này được,

bởi vì :√ab

a+ b=

ab√ab(a+ b)

≥ 2ab

(a+ b)2 =2(a+ b− c)2

a+ b= 2(1− c

a+ b)2

=2c2

(a+ b)2 −4c

a+ b+ 2

254

Page 256: Chuyên đề luyện thi đại học môn Toán - VipLam.Net

và biểu thức4c

a+ bkhông thể đưa về biến t =

c2

a2 + b2được nữa.

Ta lại quay lại với ý tưởng đưa P về biến t =c

a+ b. Nhưng có lẽ biểu thức

c2

a2 + b2sẽ làm cho

bạn lúng túng bởi nếu đánh giá a2 + b2 ≥ (a+ b)2

2ngay thì sẽ bị ngược dấu.

Tuy nhiên :

P ≥ c2

ab+

c2

a2 + b2+

2ab

(a+ b)2

nên ta có thể liên kết đượcc2

abvà

c2

a2 + b2làm xuất hiện (a + b) ở mẫu bằng bất đằng thức

Cauchy − schwarz dạng phân thức. Đến đây thì hướng giải quyết bài toán đã rõ ràng.

Lời giải:

P ≥ c2

2ab+

c2

a2 + b2+

c2

2ab+

2ab

(a+ b)2 ≥(c+ c)2

(a+ b)2 + 2

√c2.2ab

2ab.(a+ b)2 =4c2

(a+ b)2 +2c

a+ b

Đặt2c

a+ b= t ; vì

(a+ b− c)2 = ab ≤ (a+ b)2

4=⇒

(1− c

a+ b

)2

≤ 1

4=⇒ 1

2≤ c

a+ b≤ 3

2=⇒ 1 ≤ t ≤ 3

Khảo sát hàm số f(t) = t2 + t trên [1; 3] ; ta thấy : mint∈[1;3]

P = 2 ⇐⇒ t = 1

Vậy minP = 2 ⇐⇒ a = b = c

Ví dụ 5:

Cho a, b, c là các số thực dương thỏa mãn: 3b2c2 + a2 = 2(a+ bc) .Tìm giá trị nỏ nhất của biểu

thức:

P =a2

bc+

4

(a+ b)2 +4

(a+ c)2

Phân tích:

Biếu thức P của bài toán này khá giống với biểu thức P ở ví dụ 1. Hai bài toán này có ý tưởng

giống nhau chăng???

Ta thấy giá thiết của bài toán có chứa a2; 2a nên ta sẽ thử nhóm bình phương với a để đánh

giá bc xem sao:

3b2c2 − 2bc = −(a− 1)2 + 1 ≤ 1 =⇒ 3b2c2 − 2bc− 1 ≤ 0 =⇒ bc ≤ 1 (∗)

Vậy ta cần đánh giá biểu thức P sao cho sử dụng được điều kiện (∗) trên. Rõ ràng nếu sử dụng

các đánh giá như ở ví dụ 1 ; ta sẽ bị ngược dấu ngay. Vì vậy ta cần một đánh giá mạnh hơn để

làm xuất hiện một biểu thức có chứa bc ở mẫu.

Lời giải:

Ta có:1

(a+ b)2 ≥1

(a2 + bc)(1 +b

c)

=c

(a2 + bc)(b+ c)

255

Page 257: Chuyên đề luyện thi đại học môn Toán - VipLam.Net

Tương tự1

(a+ c)2 ≥b

(a2 + bc)(b+ c)

=⇒ 1

(a+ b)2 +1

(a+ c)2 ≥c

(a2 + bc)(b+ c)+

b

(a2 + bc)(b+ c)=

1

a2 + bc

(Dấu bằng xảy ra khi và chỉ khi a = b = c )

=⇒ P ≥ a2 +4

a2 + 1

Xét hàm số f(a) = a2 +4

a2 + 1trên [0;+∞]

f ′(a) = 2a− 8a

(a2 + 1)2 =2a(a2 + 3)(a− 1)(a+ 1)

(a2 + 1)2 = 0 ⇐⇒ a = 1

Lập bảng biến thiên của hàm số f(a) trên [0; +∞] ;ta thấy f(a) ≥ f(1) = 3

Hay: mina∈[0;+∞]

f(a) = 3 ⇐⇒ a = 1 .Vậy minP = 3 ⇐⇒ a = b = c = 1

Ví dụ 6:

Cho a,b,c là các số thực dương thỏa mãn a+ b+ c = 3. Tìm giá trị nhỏ nhất của biểu thức:

P = a2 + b2 + c2 +ab+ bc+ ca

a2b+ b2c+ c2a

Phân tích:

Biểu thức a2 + b2 + c2 ta thấy có thể đánh giá quy về (a + b + c). Nhưng nếu đưa về biến

(a+ b+ c) thì khi đánh giá ab+ bc+ ca sẽ bị ngược dấu ngay. Việc đưa về biến a2 + b2 + c2 xem

ra là khả quan hơn vì:

2(ab+ bc+ ca) = (a+ b+ c)2 − (a2 + b2 + c2) = 9− (a2 + b2 + c2)

Điều còn lại là cần một đánh giá

a2b+ b2c+ c2a ≤ f(a2 + b2 + c2)

Lời giải:

Tacó:

3(a2 + b2 + c2) = (a+ b+ c)(a2 + b2 + c2) = a3 + b3 + c3 + a2b+ b2c+ c2a+ ab2 + bc2 + ca2

mà: a3 + ab2 ≥ 2a2b

b3 + bc2 ≥ 2b2c

c3 + ca2 ≥ 2c2a

l =⇒ 3(a2 + b2 + c2) ≥ 3(a2b+ b2c+ c2a) > 0

=⇒ P ≥ a2 + b2 + c2 +ab+ bc+ ca

a2 + b2 + c2≥ a2 + b2 + c2 +

9− (a2 + b2 + c2)

2(a2 + b2 + c2)

256

Page 258: Chuyên đề luyện thi đại học môn Toán - VipLam.Net

Đặt t = a2 + b2 + c2 ≥ (a+ b+ c)2

3= 3 =⇒ P ≥ t+

9− t2t

=t

2+

9

2t+t

2− 1

2≥ 3 +

3

2− 1

2= 4

=⇒ P ≥ 4 (Dấu bẳng xảy ra:⇐⇒ a = b = c = 1 )

Vậy minP = 4 ⇐⇒ a = b = c = 1

Ngoài ra; việc sử dụng các bất đẳng thức cổ điển còn được kết hợp với các kĩ thuật khác

như: đặt ẩn phụ;“’tham số hóa” (Chi coi một biến là ẩn và coi các biến còn lại là tham số);...

Ví dụ 7:

Cho ba số thực dương a; b; c thỏa mãn :√a− c +

√b− c =

√ab

c.Tìm giá trị nhỏ nhất của

biểu thức:

P =a

b+ c+

b

c+ a+

c

a+ b+

c2

a2 + b2

Phân tích:

Quan sát bài toán; ta thấy giả thiết đã cho và biểu thức P là những biểu thức đồng bậc;nên ta

sẽ khử đi một biến bằng phép đặt ẩn phụ.

Đặt a = xc; y = yc(x; y ≥ 1) Dễ thấy x = 1 hoặc y = 1 đều không thỏa mãn bài toán.

Ta phát biểu lại đề bài:

Cho x, y > 1 thỏa mãn:√x− 1 +

√y − 1 =

√xy.Tìm giá trị nhỏ nhất của biểu thức:

P =x

y + 1+

y

x+ 1+

1

x+ y+

1

x2 + y2

Đây là một bài toán đối xứng với 2 biến x, y; vì vậy ta sẽ tìm cách quy về mọt biến cũng có

tính đối xứng với x và y; như t = x+ y hoặc t = xy.

Lời giải sau đưa P về biến t = xy ; bạn hoàn toàn có thể đưa về biến t = x+ y.

Lời giải:

Từ giả thiết; ta có :

√x− 1 +

√y − 1 =

√xy ⇐⇒ x+ y − 2 + 2

√(x− 1)(y − 1) = xy

⇐⇒ xy − x− y + 1− 2√

(x− 1)(y − 1) + 1 = 0

⇐⇒ xy − x− y + 1− 2√

(x− 1)(y − 1) + 1 = 0 ⇐⇒ (√

(x− 1)(y − 1)− 1)2

= 0

=⇒√

(x− 1)(y − 1) = 1 ⇐⇒ xy = x+ y ≥ 2√xy

=⇒ xy ≥ 4

257

Page 259: Chuyên đề luyện thi đại học môn Toán - VipLam.Net

Biểu thức P viết lại như sau :

P =x

y + 1+

y

x+ 1+

1

x+ y+

1

x2 + y2

=x2

xy + x+

y2

xy + y+

1

x+ y+

1

(x+ y)2 − 2xy

≥ (x+ y)2

2xy + x+ y+

1

x+ y+

1

(x+ y)2 − 2xy

=xy

3+

1

xy+

1

x2y2 − xy

=x3y3 − 2x2y2 + 3xy − 3

3(x2y2 − 2xy)

Đặt t = xy ≥ 4 Xét hàm số f(t) =t3 − 2t2 + 3t− 3

t2 − 2t(t ≥ 4)

Ta có :

f ′(t) =t4 − 4t3 + t2 + 6t− 6

(t2 − 2t)2 =t3(t− 4) + t2 + 6(t− 4) + 18

(t2 − 2t)2 > 0,∀t ≥ 4

Lập bẳng biến thiên ta có : mint∈[4;+∞]

f(t) = f(4) =41

8

Vậy minP =41

24⇐⇒ x = y = 2 haya = b = 2c

Ví dụ 8: (Vietnam TST-2001)

Xét các số thực dương a,b,c thỏa mãn điều kiện:21ab + 2bc + 8ca ≤ 12.Tìm giá trị nhỏ nhất

của biểu thức:

P =1

a+

2

b+

3

c

Phân tích:

(Bài toán này xin được dành lại việc phân tích cho bạn đọc).

Lời giải:

Đặt x =1

a; y =

2

b; z =

3

c.

Khi đó x; y; z là các số dương thỏa mãn:2xyz ≥ 2x+ 4y + 7z(1) và E = x+ y + z

Từ (1) =⇒ z(2xy−7) ≥ 2x+4y =⇒ 2xy > 7 và z ≥ 2x+ 4y

2xy − 7=⇒ x+y+z ≥ x+y+

2x+ y

2xy − 7

Áp dụng bất đẳng thức AM-GM ; ta có :

x+ y +2x+ 4y

2xy − 7= z +

11

2x+

2xy − 7

2x+

2x+14

x2xy − 7

≥ x+11

2x+ 2

√1 +

7

x2(2)

Xét hàm số f(x) = x+11

2x+ 2

√1 +

7

x2(x > 0)

f ′(x) = 1− 11

2x2− 14

x2√x2 + 7

; f ′(x) = 0 ⇐⇒ x = 3

258

Page 260: Chuyên đề luyện thi đại học môn Toán - VipLam.Net

Lập bảng biến thiên của hàm số f(x) trên (0; +∞) ; ta thấy : E ≥ 15

2. Dấu bằng xảy ra khi

và chỉ khi x = 3; y =5

2; z = 2 ⇐⇒ a =

1

3; b =

4

5; c =

3

2

Vậy giá trị nhỏ nhất của E là15

2⇐⇒ a =

1

3; b =

4

5; c =

3

2(Biểu thức (2) có thể đánh giá hoàn toàn bằng các bất đẳng thức cổ điển mà không cần sử

dụng đến công cụ đạo hàm :

x+11

2x+ 2

√1 +

7

x2≥ x+

11

2x+ 2

√(32 + 7)(1 +

7

x2)

4≥ x+

11

2x+

3 +7

x2

= x+9

x+

3

2≥ 2

√x.

9

x+

3

2=

15

2

I. Bài tập tự luyện.

1. Cho các số x, y, z thỏa mãn và x ≥ y;x ≥ z .Tìm giá trị nhỏ nhất của biểu thức:

P =x

2x+ 3y+

y

y + z+

z

x+ z

2. Cho a, b, c > 0 thỏa mãn a+ b+ c = 3 .Chứng minh rằng:

(a+ c)(b+ 1) ≥ abc(a2 + b2 + c2 + 1)

3. Cho các số thực dương a, b, c .Tìm giá trị nhỏ nhất của biểu thức:

P =a3 + b3 + c3 + abc+ 8

a+ b+ c

4. Cho a, b, c > 0 thỏa mãn a+ b+ c = 1 .Tìm giá trị lớn nhất của biểu thức:

P =2abc

a+ b+

2bc

b+ c+

2ca

c+ a+ a2 + b2

5. Cho các số thực không âmx, y, z thỏa mãn điều kiện x2 +xy+yz = 3zx và x2 +y2 +z2 > 0.

Tìm giá trị nhỏ nhất của biểu thức:

P =x

y + z+

16y

z + x+

25z

x+ y

6. Cho 3 số thực dương x, y, z thỏa mãn điều kiện x + y + z = 3 .Tìm giá trị lớn nhất của

biểu thức:

P =25(xy + yz + zx)− 3

2x3 + 3y2 + 6z + 1− 4√xyz

7. Cho các số thực dương a, b, c .Tìm giá trị nhỏ nhất của biểu thức:

P =2

a+√ab+ 3

√abc− 3√

a+ b+ c

259

Page 261: Chuyên đề luyện thi đại học môn Toán - VipLam.Net

8. Cho các số thực dương x, y, z thỏa mãn:

{x2 + 8y2 + 9z2 ≤ 4xyz

6(36y − 11√

2z)− 11x > 0.

Tìm giá trị nhỏ nhất của biểu thức;

P =4x+ 2y2 + z3√

6(36y − 11√

2z)− 11x

9. Cho x, y, z là các số thực dương thỏa mãn x + y + 1 = z Tìm giá trị nhỏ nhất của biểu

thức :

P =x3

x+ yz+

y3

y + zx+

z3

z + xy+

14

(z + 1)√

(x+ 1)(y + 1)

10. Cho x, y, z là các số thực dương thỏa mãn:1

x+

1

y+

1

z− 3

(x+ y + z)2 ≤ 9(1− 1

81xyz)

Tìm giá trị nhỏ nhất của biểu thức:

P = 2(x3 + z3) + 2(y3 + z3) +(z + 3)(1− z)

4

11. Cho x, y, z là các số thực dương thỏa mãn xyz + x+ z = y.Tìm giá trị lớn nhất của biểu

thức:

P =2

x2 + 1− 2

y2 + 1− 4z√

z2 + 1+

3z

(z2 + 1)√z2 + 1

12. Cho a, b, c > 0 thỏa mãn:a3 + b3 = c3. Chứng minh rằng:

a2 + b2 − c2 ≥3√

4 + 23√

4− 1(c− a)(c− b)

13. Cho x, y là 2 số thực thỏa mãn: x2 + xy +y2

3= 1 .

Tìm giá trị nhỏ nhất và giá trị lớn nhất của biểu thức:

P = x3 + 3y

14. Cho 3 số thực x, y, z > 1 . Tìm GTNN của biểu thức:

P = x2

(1

y − 1+

1

z − 1

)+ y2

(1

x− 1+

1

z − 1

)+ z2

(1

y − 1+

1

x− 1

).

15. Cho a, b, c > 0 vàa

1 + a+

1

1 + b+

1

1 + c= 1 . Tìm giá trị nhỏ nhất của biểu thức:

P =a+ 2bc

3a+ 4bc

16. Cho x, y là số thực thỏa x+ y − 1 =√

2x− 4 +√y + 1 .

Tìm giá trị lớn nhất và giá trị nhỏ nhất của biểu thức

P = (x+ y)2 −√

9− x− y +1√x+ y

.

260

Page 262: Chuyên đề luyện thi đại học môn Toán - VipLam.Net

17. Cho x, y, z là các số thực thỏa mãn điều kiện x, y, z ∈ [0; 1] Tìm giá trị nhỏ nhất của

P = (xy − y + 1)2 + (yz − z + 1)2 + (zx− x+ 1)2

18. Cho a, b, c là các số thực dương thỏa mãn4

5b ≥ a− c ≥ 3

5b Tìm giá trị lớn nhất của biểu

thức:

P =12(a− b)

c+

12(b− c)a

+25(c− a)

b

19. Cho các số thực x, y, z không âm và không có 2 số nào đồng thời bằng 0 .

Tìm giá trị nhỏ nhất của biểu thức :

P = (xy + yz + zx)

(1

x2 + y2+

1

y2 + z2+

1

z2 + x2

)

20. Cho các số thực dương a, b, c đôi một khác nhau thỏa mãn

{ab+ bc = 2c2

2a ≤ c.

Tìm giá trị lớn nhất của biểu thức

P =a

a− b+

b

b− c+

c

c− a

21. Cho các số thực dương x, y, z thỏa mãn điều kiện 4xy + 2yz − zx = 25

Tìm giá trị nhỏ nhất của biểu thức

P =

√x2 + 4y2

z2 + 4xy+

2

5

√z2 + 4xy

22. Chứng minh rằng với mọi số thực không âm a, b, c sao cho không có hai số nào đồng

thời bằng 0 ta luôn có :

a2 + b2 + c2

ab+ bc+ ca+

√ab

a+ b+

√bc

b+ c≥ 2

23. Cho các số thực không âm a, b, c thỏa mãn đồng thời các điều kiện c > 0 và a3+b3 = c(c−1).

Tìm giá trị lớn nhất và giá trị nhỏ nhất của biểu thức

P =a2 + b2 + c2

(a+ b+ c)2 .

261

Page 263: Chuyên đề luyện thi đại học môn Toán - VipLam.Net

CÁC BÀI TOÁN

SỬ DỤNG PHÉP BIẾN HÌNH TRONG GIẢI

TOÁN HÌNH HỌC GIẢI TÍCH PHẲNG (Lê

Hoàng Hải)

§ 1. Kiến thức cần nhớ.

Quy tắc cho tương ứng mỗi điểm M với một điểm M ′ xác định trong một mặt phẳng được gọi

là phép biến hình trong mặt phẳng đó.

I. Phép dời hình.

Phép biến hình được gọi là phép dời hình nếu nó bảo toàn khoảng cách giữa điểm bất kì; có

nghĩa là với 2 điểm M và N tùy ý; ta có: MN = f(M)f(N)

Các phép dời hình sử dụng trong bài viết:

a, Phép tính tiến theo một vecto ~v;được kí hiệu T~vlà phép biến hình biến mỗi điểmM thành

một điểm M ′ sao cho−−−→MM ′ = ~v

b, Phép đối xứng trục ∆ ; được kí hiệu là D∆ là phép biến hình biến mỗi điểm M thành

điểm M ′ đối xứng với M qua đưởng thẳng ∆ .

c, Phép đối xứng tâm A ; kí hiệu là DAlà phép biến hình biến mỗi điểm M thành điểm M ′

đối xứng với M qua A.

d, Cho điểm O và góc lượng giác α .Phép quay tâm O góc quay α ; kí hiệu là Q(O;α) là phép

biến hình biến điểmM khác O thànhM ′ sao cho OM ′ = OM và góc lượng giác (OM ;OM ′) = α

.

II. Phép đồng dạng.

Phép đồng dạng theo tỉ só k (k > 0) là phép biến hình sao cho nếu M ′;N ′ theo thứ tự là ảnh

của M;N qua phép biến hình đó thì: M ′N ′ = kMN . Số k được gọi là tỉ số đồng dạng.

Phép đồng dạng sử dụng trong bài viết:

Phép vị tự: Cho điểm I cố định và một số k 6= 0 . Phép vị tự tâm I tỉ số k; kí hiệuV(I;k) là

phép biến hình biến mỗi điểm M thành M ′ sao cho−−→IM ′ = k

−−→IM

262

Page 264: Chuyên đề luyện thi đại học môn Toán - VipLam.Net

§ 2. Các ví dụ minh họa.

I. Phép dời hình.

Ví dụ 1:

Cho hai đường tròn (C1) : x2 + y2 − 2x + 6y + 8 = 0 và (C2) : x2 + y2 + 2x − 25 = 0 và

C(1; 2);D(−1;−1) .Tìm các điểm A và B lần lượt thuộc (C1)va (C2) sao cho tứ giác ABCD

là một hình bình hành.

Phân tích:

Vì ABCD là hình bình hành nên ta có:−→AB =

−−→DC mà

−−→DC(2; 3) là vecto cố định nên B là ảnh

của A qua phép tịnh tiến T−−→DC

; từ đó tìm được tọa độ điểm A; đồng thời tìm được tọa độ B.

Lời giải:

Gọi A(x; y) =⇒ x2 + y2 − 2x+ 6y + 8 = 0 (1)

Ta thấy: T−−→DC

: A 7→ B =⇒ B(x+ 2; y + 3)

Mà B ∈ (C2) =⇒ (x+ 2)2 + (y + 3)2 + 2(x+ 2)− 25 = 0 (2)

Từ (1) và (2) ta có tọa độ A thỏa mãn hệ phương trình :{x2 + y2 − 2x+ 6y + 8 = 0

x2 + y2 + 6x+ 6y − 8 = 0

⇐⇒

{8x− 16 = 0

x2 + y2 − 2x+ 6y + 8 = 0⇐⇒

{x = 2

y = −2hoặc

{x = 2

y = −4

Vậy

{A(2;−2)

B(4; 1)hoặc

{A(2;−4)

B(4;−1)

Ví dụ 2:

Cho hai điểm A(−2;−2) và B(4; 0) và hai đường thẳng d1 : x+2y−3 = 0 và d2 : x+y−4 = 0

.Tìm C ∈ d1;D ∈ d2 sao cho ABCD là hình thang cân có AB song song với CD .

Phân tích:

Ta thấy hình thang cân có một trục đối xứng là đường thẳng (d) đi qua trung điểm hai đáy .

Vì A và B là các điểm đã biết nên ta có thể viết được phương trình (d). D là ảnh của C qua

Dd .

Từ đó suy ra D là giao của đường thẳng d2 với đưởng thẳng d′1 là ảnh của d1 qua Dd .

Từ đó ta tìm được điểm C.

Lời giải:

Gọi (d) là đưởng trung trực của AB =⇒ d : 3x+ y − 2 = 0

Ta thấy :Dd :

{C 7→ D

d1 7→ d′1

=⇒ D là giao của d′1 và d2

d;1 là đường thẳng đối xứng với d1 : x+2y−3 = 0 qua d : 3x+y−2 = 0 =⇒ d

′1 : 2x−y+1 = 0

Tọa độ điểm D thỏa mãn hệ :

{2x− y + 1 = 0

x+ y − 4 = 0=⇒

{x = 1

y = 3=⇒ D(1; 3)

263

Page 265: Chuyên đề luyện thi đại học môn Toán - VipLam.Net

Từ đó ta tìm được C(

95; 3

5

)Ví dụ 3:

Cho hai đường tròn

{C1 : x2 + y2 = 25

C2 : x2 + y2 = 9.Viết phương trình đường thẳng (d) đi qua A(3; 0)

cắt (C1) tai B khác A và cắt (C2) tại C và D sao cho CD = 3AB ; biết C có tung độ dương.

Phân tích:

Giả sử A nằm giữa B và C. Ta có : CD = 3AB ⇐⇒ AC = AB. Do đó C đối xứng với B qua

A.

Mặt khác B ∈ (C2) nên C thuộc đường tròn (C′2) đối xứng với (C2) qua A.

Vậy C là giao điểm của (C′2) và (C1) Từ đó viết được phương trình đường thẳng (d)

Lời giải:

Ta thấy :DA :

{B 7→ C

(C2) 7→ (C′2)

=⇒ C là giao của (C′2) và (C1)

Phương trình C′2 : (x− 6)2 + y2 = 9

=⇒ Tọa độ C thỏa mãn hệ :

{x2 + y2 = 25

(x− 6)2 + y2 = 9⇐⇒

x =

13

3

y =±2√

14

3

Vì yC > 0 nên x =13

3và y =

2√

14

3=⇒ Phương trình AC : x

√14− 2y − 3

√14 = 0

Ví dụ 4:

Cho đường tròn (C) : x2 + (y − 1)2 = 8 và đường thẳng (d) : x+ y + 3 = 0 . Tìm B thuộc (d)

và C thuộc (C) sao cho tam giác ABC vuông cân tại A với A(1; 1)

Phân tích:

∆ABC vuông cân tại A⇐⇒ C là ảnh của B qua phép quay Q(A;π2

)orQ(A;−π2

)

Mặt khác B thuộc (d) nên C thuộc (d′) là ảnh của (d) qua Q(A;π2

) hoặc Q(A;−π2

).

Do đó C là giao của (d) và (d′).Từ đó ta tìm được tọa độ điểm B.

Lời giải:

TH1 : C là ảnh của B qua phép quay Q(A;π2

) Lấy M(0;−3) ∈ (d) .

Khi đó đường thẳng (d′) đi qua ảnh của M(−3; 2) qua Q(A;π2

) .

Mặt khác do góc quay là π2nên d′⊥d =⇒ (d′) : 1(x+ 3)− (y − 2) = 0 ⇐⇒ x− y + 5 = 0

Vì C là giao của (C) và (d′) nên tọa độ C thỏa mãn hệ phương trình :{x− y + 5 = 0

x2 + (y − 1)2 = 8⇐⇒

{x = −2

y = 3=⇒ C(−2; 3) Từ đó suy ra B(−1;−2)

TH2 : C là ảnh của B qua phép quay Q(A;−π2

)

Tương tự ; ta suy ra :(d′) : x− y − 5 = 0

Vì C là giao của (C) và (d′) nên tọa độ C thỏa mãn hệ :

{x− y − 5 = 0

x2 + (y − 1)2 = 8(Vô nghiệm)

Vậy B(−1;−2) và C(−2; 3)

264

Page 266: Chuyên đề luyện thi đại học môn Toán - VipLam.Net

II. Phép đồng dạng.

Ví dụ 1:

Cho tam giác ABC có trọng tâm G(1; 2) .Phương trình đường thẳng đi qua trung điểm 3 cạnh

của tam giác là (C1) : x2 + y2− 2x+ 4y+ 4 = 0. Viết phương trình đường tròn ngoại tiếp tam

giác ABC.

Phân tích:

Gọi M ;N ;P lân lượt là trung điểm AB;BC;CA. Dễ dàng chứng minh trọng tâm G tam giác

ABC cũng là trọng tâm tam giác MNP ; phép vị tự tâm G tỉ số k = −2 biến ∆MNP thành

∆ABC nên đường tròn ngoại tiếp tam giác ABC cũng là ảnh của đường tròn ngoại tiếp tam

giác MNP

Lời giải:

Gọi I;R là tâm và bán kính đường tròn ngoại tiếp ∆ABC .

Gọi I ′;R′ là tâm và bán kính đường tròn ngoại tiếp tam giác MNP =⇒ I ′(1;−2);R′ = 1 .

Do−→GI = −2

−→GI ′ nên dễ dàng tìm được I(1; 10) .Do R = 2R′ =⇒ R = 2

Vậy phương trình đường ròn ngoại tiếp ∆ABC là:(x− 1)2 + (y − 10)2 = 4 .

Ví dụ 2:

Cho đường tròn (C) : x2 + y2 = 4 và I(√

11; 5) .Giả sử điểm M thuộc đường tròn (C): phân

giác IOM cắt IM tại M ′. Tìm tọa độ điểm M ′ biết M ′ thuộc đường tròn (T ) : (x−√

112

)2

+

(y − 12)2

= 4

Phân tích:

Nhận thấy OI = 6 = 3R . Theo tính chất đường phân giác trong; ta có:

M ′I

MM ′ =OI

OM=

3R

R= 3 =⇒ IM ′

IM ′ +MM ′ =3

3 + 1=

3

4⇐⇒ IM ′ =

3

4IM ⇐⇒

−−→IM ′ =

3

4

−−→IM

Hay M ′ chính là ảnh của M qua V(I; 34

)

Lời giải:

Vì M thuộc (C) nên M ′ thuộc (C ′) là ảnh của (C) qua V(I; 34

)

Từ đó suy ra phương trình (C ′) :(C ′) : (x−√

114

)2

+ (y − 54)2

= 94

Vậy tọa độ điểm M thỏa mãn hệ phương trình :{(x−

√114

)2

+ (y − 54)2

= 94

(x−√

112

)2

+ (y − 12)2

= 4⇐⇒

{x =

√112

y = 52

hoặc :

{x = −

√11

10

y = 310

Ví dụ 3:

Cho tam giác ABC cân tại A ; nội tiếp đường tròn (C1) và ngoại tiếp đường tròn (C2) có

tâm I(1; 2).Gọi (C3) là đường tròn tiếp xúc với AB tại D ; AC tại E và tiếp xúc trong với

(C1).Viết phương trình đường thẳng DE ; biết (C2) tiếp xúc với BC tại H(3; 5)

Phân tích:

Giả sử (C3) tiếp xúc với (C1) tại K ; J là tâm đường tròn (C3)

265

Page 267: Chuyên đề luyện thi đại học môn Toán - VipLam.Net

Xét V(A:k=AHAK

) : (C3) 7→ (C2). Gọi M là trung điểm DE ;

ta có : ∆ABK đồng dạng với ∆ADI(g.g) =⇒ AHAK

= AMAJ

=⇒ V(A;k) : J 7→M

Lời giải:

Vì J là tâm đường tròn (C3) nên M là tâm đường tròn (C2)

Suy ra : DE đi qua I(1; 2) và nhận−→IH(2; 3) là vtpt.

Vậy phương trình DE : 2(x− 1) + 3(y − 2) = 0 ⇐⇒ 2x+ 3y − 8 = 0

§ 3. Bài tập tự luyện.

1. Cho đường tròn (C) : x2 + y2 = 4 và 2 điểm A(−2; 0) và B(2; 0) . Điểm M thay đổi trên

đoạn AB; đường thẳng qua M tọa với AB góc 45o cắt (C) tại C và D.Tìm tọa độ 2 điểm

C và D sao cho MC +MD đạt giá trị lớn nhất.

2. Cho đường tròn (C) : x2 + (y − 2)2 = 13 .Đường thẳng (d) : x − 5y − 3 = 0 cắt đường

tròn (C) tại 2 điểm phân biệt B và C. Điểm A thay đổi trên (C) (A khác B và C).Tìm

tọa độ trực tâm tam giác ABC biết H thuộc đường tròn (C) : (x− 2)2 + (y + 1)2 = 4 .

3. Viết phương trình bốn cạnh của hình vuông ABCD biết các đường thẳng AB;BC;CD :

DA lần lượt đi qua các điểm M(12; 3

2);N(1

3; 13

3);P (−1;−3);Q(−2;−4)

4. Cho A(4; 3) .Tìm điểm B thuộc đường tròn (C1) : x2 + y2 = 5 và C thuộc đường tròn

(C2) : (x− 1)2 + (y − 1)2 = 39 sao cho tam giác ABC là tam giác đều.

5. Cho 2 đường thẳng (d1) : x − y + 1 = 0 và (d2) : 2x + y − 4 = 0 và A(1; 1) .Tìm tọa độ

điểm B thuộc (d1) và C thuộc (d2)sao cho tam giác ABC có chu vi nhỏ nhất.

6. Tìm tọa độ 4 đỉnh của một hình bình hành ABCD: biết M(0; 2);N(−52

; 12);P (−1;−1) lần

lượt là trung điểm 3 cạnh AB;BC;CD của hình bình hành.

7. Cho 2 đường thẳng (d1) : 3x− 5y+ 12 = 0 (d2) : 4x+ y− 8 = 0cắt nhau tại A và M(1; 2)

.Viết phương trình đường thẳng qua M ; cắt (d1) và (d2) lần lượt tại B và C sao cho

S∆ABC nhỏ nhất.

8. Cho đường thẳng (d1) : 3x − y = 0 và hai đường tròn (C1) : (x+ 3)2 + (y − 3)2 = 5và

(C2) : (x− 5)2 + (y − 3)2 = 13.Tìm A thuộc (C1) ;C thuộc (C2) và B;D thuộc (d) sao

cho ABCD là hình vuông.

9. Cho A(1; 2) và đường tròn (C) : (x− 3)2 + (y − 4)2 = 4 .Tìm điểm M thuộc tia Ox sao

cho nếu vẽ tiếp tuyến MB đến đường tròn (C) (B là tiếp điểm) thì AMO = BMx

10. Cho 2 đường tròn (C1) : (x− 4)2 + (y − 5)2 = 34 và (C2) : x2 + (y + 11)2 = 170 cắt nhau

tại A và B; trong đó A là điểm có hoành độ dương. Viết phương trình đường thẳng đi

qua A cắt 2 đường tròn tại C và D thỏa mãn A nằm giữa C và D và AD−AC = 16√

2 .

266

Page 268: Chuyên đề luyện thi đại học môn Toán - VipLam.Net

11. Cho hai đường tròn (C) : (x− 4)2 + (y − 6)2 = 13 và (C ′) : (x− 6)2 + y2 = 25 cắt nhau

tại A, B với điểm A có hoành độ bé hơn 3. Một đường thẳng qua A cắt (C), (C ′) lần lượt

tại M, N. Gọi d1 là tiếp tuyến tại M của (C), d2 là tiếp tuyến tại N của (C ′). Biết d1 và

d2 cắt nhau tại I. Tìm tọa điểm I khi bán kính đường tròn ngoại tiếp tam giác IMN lớn

nhất.

(Trích đề thi thử Đại Học lần 2 năm 2013-toangphothong.vn)

12. Cho 2 đường tròn (C1) : x2 + y2 = 9 và (C2) : (x− 3)2 + (y − 2)2 = 4 cắt nhau tại A và

B.Từ M chạy trên (C2) ;kẻ MA cắt (C1) tại C; MB cắt (C1) tại D.Viết phương trình

quỹ tích trọng tâm G của tam giác ACD

13. Cho tam giác ABC có tọa độ 3 đỉnh là: A(−12

; 4) ;B(−134

; 32) và C(2; 3

2) .Tìm tọa độ 4 đỉnh

của hình vuông MNPQ biết M ;N thuộc BC; P thuộc AC và Q thuộc AB.

14. Cho 2 đường tròn (C1) : (x− 1)2 + (y − 2)2 = 16 và (C2) : (x+ 3)2 + (y + 4)2 = 8.Đường

tròn (C3) tiếp xúc ngoài với (C1) tại A và tiếp xúc ngoài với (C2) tại B. Viết phương trình

đường tròn AB sao cho khoảng cách từ điểm M(−2;−10) đến AB là lớn nhất.

15. Cho hai đường tròn (C1) : (x− 1)2 + (y − 2)2 = 4 và (C2) : (x+ 2)2 + (y − 6)2 = 9 tiếp

xúc ngoài với nhau tại A. Tìm B thuộc (C1) và C thuộc (C2) sao cho BAC = 90o và BC

đi qua M(−12

; 32)

16. Cho đường tròn (C) : x2 + y2 = 10 có đường kính AB với A(1; 3) .Gọi C là điểm đối xứng

với A qua B và PQ là đường kính thay đổi của (C) khác với AB. Đường thẳng CQ cắt

PA, PB lần lượt tại M và N . Biết M và N thuộc đường tròn (T ) : (x− 1)2 + y2 = 25.

Tìm tọa độ 2 điểm M và N .

267

Page 269: Chuyên đề luyện thi đại học môn Toán - VipLam.Net

CÁC BÀI TOÁN

PHƯƠNG PHÁP TỌA ĐỘ HOÁ ĐỂ CHỨNG

MINH HÌNH HỌC PHẲNG

Lưu Giang Nam - Hoàng Trung Hiếu

Như chúng ta đã biết hình học giải tích được xây dựng trên cơ sở của hình học phẳng và lồng

ghép với các hệ trục toạ độ. Nhưng đã bao giờ bạn tự hỏi một phần của hình họck phẳng là

hình giải tích chưa? Với bài viết này mình sẽ giới thiệu với các bạn phương pháp toạ đọ hoá

hình học phẳng. Ứng dụng của phương pháp khá hiệu quả; đây là công cụ để chứng minh các

bài toán thi quốc gia quốc tế mà rất ít học sinh ngghĩ tới . Đồng thời phương pháp còn luyện

tập kĩ năng tính toán với tham số ; các kĩ năng giải hình giải tích Mong bài viết này sẽ hữu ích

với các bạn

§ 1. Các ví dụ

BÀI 1: (Câu 3, VMO 2007)

Cho tam giác ABC có hai đỉnh B,C cố định và đỉnh A thay đổi. Gọi H,G lần luợt là trọng

tâm và trực tâm của tam giác ABC. Tìm quỹ tích điểm A, biết rằng trung điểm K của HG

thuộc BC.

Bài làm:

Chon hệ trục tọa độ Oxy với O là trung điểm BC và trục Ox là đường thẳng BC.

Đặt BC = 2a > 0.

Khi đó các đỉnh B,C có tọa độ là B(-a;0); C(a;0).

Giả sử A(x0; y0) y0 6= 0.

Khi đó tọa độ trực tâm H là H(x0;a2 − b2

y0

).

Tọa độ trọng tâm là G(x0

3;y0

3).

268

Page 270: Chuyên đề luyện thi đại học môn Toán - VipLam.Net

=⇒ K(2x0

3;3a2 − 3x2

0 + y20

6y0

)

K thuộc BC khi và chỉ khi :3a2 − 3x20 + y2

0 = 0 ⇐⇒ x20

a2− y2

0

3a2= 1

Vậy A có quỹ tích là hypebolx2

a2− y2

3a2= 1 , trừ hai điểm B,C.

BÀI 2:

Trong mp (P) cho tam giác đều ABC.

Tìm quỹ tích I và hằng số k sao cho với mọi đường thẳng (d) đi qua I thì tổng bình phương

N các khoảng cách từ A ,B,C đến (d) bằng k.

Bài giải:

Chọn hệ trục Oxy sao cho : B(-1;0), C(1;0) =⇒ A(0;√

3).

Giả sử I(x0; y0) =⇒ (d) : a(x− x0) + b(y − y0) = 0

Ta có N =[−ax0 + b(

√3− y0)]2 + [a(−1− x0)− by0]2 + [a(1− x0)− by0]2

a2 + b2= k

⇐⇒ a2[x20 + (x0 + 1)2 + (x0− 1)2− k] + 2ab[x0(y0−

√3) + (x0 + 1)y0 + y0(x0− 1)] + b2[(y0−√

3)2 + 2y20 − k] = 0 (1)

Để (1) đúng với mọi a, b thì :x2

0 + (x0 + 1)2 + (x0 − 1)2 = k

(y0 −√

3)2 + 2y20 = k

x0(y0 −√

3) + y0(x0 + 1) + y0(x0 − 1) = 0(2)

Từ (2) ⇐⇒

x0 =⇒ y0 =

√3

3

y0 =

√3

3=⇒ x0 = 0

Vậy điểm I cần tìm là I(0; y0 =

√3

3) hay là tâm của ta giác ABC, và k=2

BÀI 3:

Cho tam giác ABC vuông cân tại A. Các điểm D, E,F lần lượt trên các cạnh AB,AC,BC sao

cho DADB

= EBEC

= FCFA

. Chứng minh rằng AE ⊥ FD.

269

Page 271: Chuyên đề luyện thi đại học môn Toán - VipLam.Net

Giải:

Chọn hệ trục tọa độ Oxy sao cho O ≡ A. Ox đi qua B, Oy đi qua C sao cho B(1;0), C(0;1).

ĐặtDA

DB=EB

EC=FC

FA,= m(m > 0)

=⇒ DA

DA+DB=

m

m+ 1,

FA

FA+ FC=

1

m+ 1

EH

AC=EB

BC=

EB

EB + EC=

m

m+ 1

KE

AB=EC

BC=

EC

EB + EC=

1

m+ 1

Vậy DA =m

m+ 1;FA =

1

m+ 1;EH =

m

m+ 1;KE =

1

m+ 1

nên D(m

m+ 1; 0);E(

1

m+ 1;

m

m+ 1);F (0;

1

m+ 1

Ta có ~AE = (1

m+ 1;

m

m+ 1); ~FD = (

m

m+ 1;−1

m+ 1)

Do đó ~AE. ~FD =1

m+ 1.m

m+ 1+

m

m+ 1.(−1

m+ 1) = 0

=⇒ ĐPCM

BÀI 4:

Cho đoạn thẳng AD cố định, dựng hình bình hành ABCD sao choAC

AD=BD

AB.

Chứng tỏ rằng B thuộc 1 đường tròn cố định.

Giải:

Chọn hệ trục tọa đọ Oxy sao cho O ≡ A, tia Ox đi qua D, hình bình hành ABCD nằm

trong góc xOy , A(0; 0);D(1; 0);B(x; y), C(x+ 1; y)

Ta có AC =√x+ 12 + y2, AB =

√x2 + y2, AD = 1, BD =

√x− 12 + y2

Do đóAC

AD=BD

AB

⇐⇒ AC.AB = AD.BD

⇐⇒√x+ 12 + y2.

√x2 + y2 = 1.

√x− 12 + y2

⇐⇒ (x2 + y2 + 1)(x2 + y2 + 2x− 1) = 0

270

Page 272: Chuyên đề luyện thi đại học môn Toán - VipLam.Net

⇐⇒ (x2 + y2 + 2x− 1) = 0

Vậy các điểm B thuộc đường tròn cố định tâm I(-1;0) bán kính R =√

2

BÀI 5 ( APMO 1998):

Cho tam giác ABC với AD là đường cao, d là 1 đường thẳng qua D. Lấy E,F ∈ d, khác D sao

cho AE ⊥ BE,AF ⊥ CF .

Gọi M,N theo thứ tự là trung điểm các đoạn BC,EF. CMR AN ⊥MN

Giải:

Chọn hệ trục tọa đọ sao cho O ≡ A và Ox song song với d.

Vì D, E,F thẳng hàng nên D(d;a),E(e;a),F(f;a), N(e+ f

2; a).

Khi đó đt AE:ax− ey = 0

AD: ax-dy=0 , AF: ax-fy=0

Theo đề ta có BE ⊥ EAv =⇒ BE: ex+ ay − e2 − a2 = 0

CF ⊥ AFv =⇒ CF: fx+ ay − f 2 − a2 = 0

BC ⊥ ADv =⇒ BC: dx+ ay − d2 − a2 = 0

Từ đó B(d+ e; a− de

a), C(d+ f ; a− df

a)

=⇒M(d+e+ f

2; a− d(e− f

)2a)

=⇒ ~MN = (−d;d(e+ f)

2a)

=⇒ ~MN. ~AN = 0

=⇒ ĐPCM

271

Page 273: Chuyên đề luyện thi đại học môn Toán - VipLam.Net

BÀI 6:

Cho tam giác ABC cân tại A , trung tuyến BN. Trên tia đối của CA, lấy đoạn CD=CA.

CMR:BD=2BN.Giải:

Chọn hệ trục Oxy sao cho Ox ≡ BC,A(0; a), B(−1; 0), C(1; 0), a > 0.

=⇒ N(1

2;a

2).

Theo đề ta có D đối xứng với A qua C

=⇒ D(2;-a)

=⇒ BN =

√1

4+a2

4

và BD =√

9 + a2 = 2

√1

4+a2

4= 2BN

=⇒ ĐPCM

BÀI 7:

Cho tam giác ABC vuông cân tại A. M,N là 2 điểm di động lần lượt trên các cạnh AB,AC sao

cho (AM +AN)2 =BC2

2. P là điểm đối xứng với A qua trung điểm O của MN. CMR đt qua

P vuông góc với MN luôn đi qua 1 điểm cố định.

Hướng dẫn:

Chon hệ trục Oxy sao choOx ≡ AC;Oy ≡ AB,A(0; 0), B(0; 1), C(1; 0) tìm được AN+AM=1.

Gọi lại trung điểm O của MN là I.

Từ đó tìm được M, N (qua biến mới), I rồi tới P.

Cuối cùng là viết PTĐT và tìm điểm cố định trên đt đó ( cái này lớp 9 đã học).

*) Nhận xét:

Bài này là 1 bài cơ bản, có thể giải bằng hình học cổ điển. Tuy nhiên với các bạn yếu môn hình

thì các này phù hợp hơn vì 2 lí do:

1) Viết PTĐT là 1 phần của đại số, ko đòi hỏi tư duy hình học .

2) Chúng ta ko phải ngồi dự đoán điểm cố định, chỉ cần viết mấy PTĐT ra thì điểm cố định

272

Page 274: Chuyên đề luyện thi đại học môn Toán - VipLam.Net

tự khắc xuất hiện.

3) Đây còn là 1 PP để các bạn "kiếm điểm" vào cuối giờ nếu giải ko ra.

BÀI 8

Trên các cạnh AB,AC của tam giác vuông ABC tại A, dựng các hình vuông ABEF, ACGI.

Gọi O là giao điểm của GB và AH( H là chân đường cao). CMF C,E,O thẳng hàng.

Hướng dẫn:

Chọn hệ trục Oxy sao cho Ox ≡ AB,Oy ≡ AC,A(0; 0), B(b; 0), C(0; c), b, c > 0.

Từ đó ta tìm được tọa độ tất cả các đỉnh của 2 hv theo b và c.

Đến đây viết PTĐT của GB, AH =⇒ O.

CM thẳng hàng bằng 2 cách:

1) THCS: Viết PTĐT CE ra rồi thay tọa độ O vào, nếu bằng 0 thì thẳng hàng.

2) THPT: C,E,O thẳng hàng ⇐⇒ ~CE và ~CO cùng phương ( hoặc các cặp khác).

BÀI 9:

Trong mặt phẳng cho 2 đường thẳng d1, d2 cắt nhau tại O và điểm A không thuộc d1, d2. Một

đường tròn c© đi qua A,O, cắt lại d1, d2 tại M1,M2 theo thứ tự đó.

Chứng minh rằng: trung điểm của đoạn thẳng M1M2 luôn nằm trên 1 đường thẳng cố định

khi c© thay đổi nhưng luôn đi qua A và O

Lời giải

Xét hệ trục tọa độ Oxy vơi Ox là tia OA.

Gọi A(0;2) thì tâm I củađường tròn đi qua A và O thuộc đường trung trực của OA là (∆) : x = 1.

Gọi tọa độ của I là (1;t).

Vì 2 đường thẳng (d1); (d2) đi qua O nên có phương trình đường thẳng là:

(d1) : y = ax và (d2) : y = bx với a; b là hằng số

Xét đường tròn c© tâm I đi qua O và A có phương trình là:

(x− 1)2 + (y − t)2 = t2 + 1 ⇐⇒ x2 − 2x+ y2 − 2ty = 0

Phương trình hoành độ giao điểm của c© và (d1) là:

x2 − 2x+ (ax)2 − 2atx = 0

⇐⇒ x = 0 hoặc x =2 + 2at

a2 + 1⇒ Hoành độ điểm M1 là giao điểm thứ 2 khác O của c© và (d1) là

xM1 =2 + 2at

a2 + 1

mà M1 thuộc (d1) nên M1(2 + 2at

a2 + 1;2a+ 2a2t

a2 + 1)

273

Page 275: Chuyên đề luyện thi đại học môn Toán - VipLam.Net

Hoàn toàn tương tự ta có : M2(2 + 2bt

b2 + 1;2b+ 2b2t

b2 + 1)

⇒ Tọa độ trung điểm N của M1M2 làxN =

1 + 1at

a2 + 1+

1 + 1bt

b2 + 1

yN =a+ a2t

a2 + 1+b+ b2t

b2 + 1Từ hệ trên rút t theo x,a,b từ phương trình thứ 1 rồi thay t vào phương trình thứ 2 ta được:

yN = αxN + β với α, β là các hằng số theo a và b

Vậy điểm N di động trên đường thẳng y = αx+ β cố định

BÀI 10:

Cho tam giác ABC vuông tại A không cân, trên AB và AC lấy M, N sao cho BM=CN.

Chứng minh rằng đường trung trực của MN luôn đi qua một điểm cố định.

Giải:

Chọn hệ trục Oxy sao cho O ≡ A, Ox ≡ AC,Oy ≡ AB, B(0; b), C(1; 0).

Giả sử M(0;m), 0 < m < b± 1 =⇒ N(1 +m− b; 0).

=⇒ P (1 +m− b

2;m

2), ~MN = (1 +m− b;−m)

PT đường trung trực của MN là :

(1 +m− b)(x− 1 +m− b2

)−m(y − m

2) = 0

⇐⇒ m(x− y − 1 + b) + (1− b)x− 1

2(1− b)2 = 0

=⇒ điểm cố định cần tìm là K(1− b

2;b− 1

2).

BÀI 11:

Trong mặt phẳng cho 2 điểm A,B.

Tìm quỹ tích tất cả những điểm M sao cho[

ˆMAB − ˆMBA]

= 90o.

Giải:

Chọn hệ trục Oxy Ox ≡ AB sao A(-1;0),B(1;0).

Đặt MAB = x,MBA = y ( x;y là số đo góc không tù).

274

Page 276: Chuyên đề luyện thi đại học môn Toán - VipLam.Net

Nếu x = 0o thì y = 90o và M ≡ A, ngược là thì M ≡ B.

Xét x, y ≡ 0o, 90o

Do [x− y] = 90o nên x = y + 90o, x = y − 90o

⇐⇒ tanx = −coty hay tích hệ số góc của các đt MA,MB bằng 1. Vậy tọa độ của M là

nghiệm của hệ{y = k(x+ 1)

ky = x− 1

Điều này cho ta x2 − y2 = 1

Vậy quỹ tích M là Hypebol x2 − y2 = 1.

BÀI 12: (IMO 2000)

Cho hai đường tròn (O1), (O2) cắt nhau tại hai điểm phân biệt M,N. Tiếp tuyến chung ( gần

M hơn) tiếp xúc với (O1) tại A1, (O2) tại A2.

Đường thẳng qua M song song với A1A2 cắt lại đường tròn (O1), (O2) tại B1;B2. Các đường

thẳng A1B1, A2B2 tại C., các đường thẳng A1N,A2N cắt đường thẳng B1B2 ở D,E.

Chứng minh rằng CD=CE.

Giải:

Chọn Oxy sao cho A1(0; 0), A2(a; 0), O1(0, c), O2(a; d)

Giả sử trong hệ trục tọa độ M(s;t), khi đó B1(−s; t), B2(2a−s; t). Từ đó B1B2−2a = 2A1A2.

Để ý rằng A1A2//B1B2 =⇒ A1, A2 theo thứ tự là trung điểm B1C,B2C, do đó C(s;-t).

Do đó ~CM = (0; 2t), ~B1B2 = (2a; 0) =⇒ CM ⊥ B1B2 hay CM ⊥ DE.(1)

Gọi K là giao điểm của MN với A1A2. Ta có :PK/(O1) = ¯KA12

= ¯KM.KN = PK/(O2) = ¯KA22

=⇒ K là trung điểm của A1A2. Từ đó , do A1A2//B1B2 nên M là trung điểm DE. (2)

Từ (1) và (2) suy ra CM là đường trung trực của DE (ĐPCM).

Vậy CM là đường trung trực của DE.

275

Page 277: Chuyên đề luyện thi đại học môn Toán - VipLam.Net

BÀI 13:

Đường phân giác trong và ngoài góc C của tam giác ABC cắt AB tại L và M. CMR: nếu

CL=CM thì AC2 +BC2 = 4R2.

Cách 1:

Nếu CL=CM thì tam giác CML vuông cân . Chọn hệ trục tọa độ sao cho O là trung điểm

của ML, A(0;0), B(b;0), C(0;c) =⇒ L(c;0), M(-c;0)

Theo tính chất đường phân giác ta cóAL

LB=AC

CB

⇐⇒ AL2

LB2=AC2

CB2

⇐⇒ (a− c)2

(b− c)2=a2 + c2

b2 + c2

⇐⇒ b =c2

a=⇒ B(

c2

a; 0)

Khi đó AB2 +BC2 = (a2 + c2

a)2

Gọi I là tâm đường tròn ngoại tiếp tam giác ABC, ta có :{AI2 = CI2

AI2 = BI2

⇐⇒

2ax− 2cy = a2 − c2

x = (a2 + c2

2a; c)

=⇒ I(a2 + c2

2a; c)

=⇒ 4R2 = 4CI2 = (a2 + c2

a)2 đpcm

Cách 2:

Nếu CL=CM thì tam giác CML vuông cân . Chon hệ trục Oxy sao cho O ≡ C,Ox ≡CL,Oy ≡ CM

Giả sử M(a;0), L(0;a)

=⇒ ML:x+y-a=0

Tiếp tục giả sử AC: y=-2x

=⇒ BC: y=2x

Vì B,C thuộc ML nên A(−a; 2a), B(a

3;2a

3)

=⇒ AC =√

5a,BC =

√5a

3

=⇒ AC2 +BC2 =50a2

9

276

Page 278: Chuyên đề luyện thi đại học môn Toán - VipLam.Net

Lại có AB =4√

2a

3

=⇒ cosC =3

5=⇒ sinC =

4

5

MàAB2

sin2C= 4R2

=⇒ 4R2 =50a2

9Vậy ta có ĐPCM.

Nhận xét:

1) Hai bài giải trên cho thấy với 1 bài tọa độ, giải cách nào cũng ra, nhưng tùy theo cách

chọn trục thì sẽ có cách ngắn hơn ( cách 2 ngắn hơn cách 1).

2) Tuy nhiên ở cách 2, ta không dễ để chon AC: y=-2x rồi BC: y=2x. Phải qua nhiều lần

thử ( mình cũng chon thử đến 2 lần :grinder:)

Bài này còn 1 cách chon hệ trục nữa, các bạn hãy thử chọn

I. Bài tập tự luyện

Bài 1: (VMO 2008)

Cho tam giác ABC, trung tuyến AD. Cho đt (d) vuông góc với AD.Xét M trên (d). Gọi E, F

lần lượt là trung điểm của MB và MC. Đt đi qua E , vuông góc với d cắt đt AB tại P, Đt đi

qua F , vuông góc với d cắt đt AC tại Q.CMR đt đi qua M vuông góc với với PQ luôn đi qua

1 điểm cố định khi điểm M di đọng trên đt d.

Bài 2:

Cho tam giác ABC cân tạ A. Gọi O,I lấn lượt là tâm đường tròn ngoại và nội tiếp của tam giác

ABC. Gọi D thuộc AC sao cho ID song song với AB. CM: CI vuông góc với OD.

Bài 3:

Cho tam giác ABC cân tại A. Gọi M là trung điểm của AB, G là trọng tâm của tam giác ACM.

Gọi I là tâm đường tròn ngoại tiếp tam giác ABC. Chứng minh rằng GI vuông góc với CM.

Bài 4:

Cho tam giác ABC cân tại A. Gọi H là trung điểm của BC , D là hình chiếu của H lên AC, M

là trung điểm của HD. Chứng minh rằng AM vuông góc với BD.

Bài 5:

Cho tam giác ABC cân tại A. Gọi O,I lần lượt là tâm đường tròn ngoại tiếp và nội tiếp tam

giác ABC. Điểm D thuộc AC sao cho ID song song với AB. CMR CI vuông góc với OD.

Bài 6:

Cho hcn ABCD có M là trung điểm của AB, N thuộc tia phân giác của góc BCD. Gọi P là hình

chiếu của N trên BC. CMR nếu MN vuông góc DP thì tam giác AND cân.

Bài 7:

Cho tam giác ABC và AD là đường cao.ĐT EF qua D sao cho AE vuông BE, AF vuông CF.

Gọi M,N lần lượt là trung điểm các đoạnBC, È. CMR: AN vuông góc với NM.

Bài 8:

Cho hình vuông ABCD. Trên BD lấy điểm M không trùng với B,D. Gọi E,F lần lượt là hình

277

Page 279: Chuyên đề luyện thi đại học môn Toán - VipLam.Net

chiếu của M trên các cạnh AB,AC. CMR:

a) CM ⊥ EF

b) Ba đường thẳng CM,BF,DE đồng quy.

Bài 9:

Cho hình thoi ABCD có góc BAD=40, O là giao điểm của 2 đt . Gọi H là hc của O trên cạnh

AB. Trên tia đối của tia BC, tia đối của CD lấy M,N sao cho HM || AN. Tính góc MON.

Bài 10:

Cho tam giác ABC cân tại A nội tiếp (O). Gọi I là trung điểm của AB. G là trọng tâm của tam

giác ACI. CMR OG vuông góc với CI.

Bài 11:

Cho tam giác ABC cân tại A, đường cao AH. Từ đỉnh M trên BC kẽ MP vuông góc với AB,

MQ vuông góc với AC sao cho P,Q thuộc AB,AC. CMR đường trung trực của PQ luôn đi qua

điểm cố định khi M di động trên BC.

Bài 12:

Cho ∆ABC đều ; cạnh bằng 2a.(d) là đường thẳng tuỳ ý cắt các cạnh BC;CA;AB. Gọi x; y; z

tương ứng là các góc giữa đường thẳng (d) và các đường thẳng BC,CA,AB. Chứng minh:

sin2 x sin2 y sin2 z + cos2 x cos2 y cos2 z =1

16

Bài 13:

Cho tam giác ABC có trọng tâm G và H không trùng nhau. Chứng minh rằng:

GH//BC ⇐⇒ tanB + tanC = 2 tanA

Bài 14:Trên mặp phẳng xét hình vuông ABCD và một tam giác EFG đều cắt nhau tạo thành

một thất giác MBNPQRS. Chứng minh rằng:

SM = NP = QR ⇐⇒ (MB − PQ)2 + (BN −RS)2 = 0

Bài 15:

Trên cung AB của đường tròn ngoại tiếp hình chữ nhật ABCD, lấy M khác A và B. Gọi P,Q,R,S

là hình chiếu của M trên AD, AB,BC,CD.

a)Chứng minh PQ ⊥ RS

278

Page 280: Chuyên đề luyện thi đại học môn Toán - VipLam.Net

PHẦN THỨ III

CÁC ĐỀ THI TỰ LUYỆN

279

Page 281: Chuyên đề luyện thi đại học môn Toán - VipLam.Net

Đề 1 ngày 12.10.2012

PHẦN CHUNG CHO TẤT CẢ THÍ SINH (7 điểm)

Câu 1. (2 điểm) Cho hàm số y =2x+ 1

x− 1(C)

a) Khảo sát sự biến thiên và vẽ đồ thị của hàm số (c).

b) Gọi I là giao điểm của hai đường tiệm cận. Xác định tọa độ điểm M có hoành độ dương nằm trên đồ thị (C)

sao cho tiếp tuyến tại M cắt hai đường tiệm cận của (C) tại A,B đồng thời hai điểm này cùng với điểm I tạo

thành một tam giác nội tiếp đường tròn có bán kính bằng√

10.

Câu 2. (2 điểm)

a) Giải phương trìnhcos 2x

cosx+(1 + cos2 x

)tanx = 1 + sin2 x.

b) Giải hệ phương trình

(x+ y) (25− 4xy) =105

4+ 4x2 + 17y2

4x2 + 4y2 + 4x− 4y = 7

Câu 3. (1 điểm) Tính tích phân I =

∫ π4

0

(1 + tan2 x

)x− (x− tanx) cos2 x

3 + cos 2xdx.

Câu 4. (1 điểm) Cho hình chóp S.ABCD có đáy ABCD là hình bình hành, mặt phẳng (SBD) vuông góc với đáy,

các đường thẳng SA, SD hợp với đáy một góc 30o. Biết AD = a√

6, BD = 2a và góc ADB = 45o. Tính thể tích

khối chóp S.ABCD và khoảng cách từ đỉnh C đến mặt phẳng (SAD) theo a.

Câu 5. (1 điểm) Cho các số thực không âm x, y thỏa mãn : x (2x+ 2y − 5) + y (y − 3) + 3 = 0. Tìm giá trị lớn nhất

và nhỏ nhất của biểu thức : P = (xy − x+ 1)2 + (xy − y + 1)2

PHẦN RIÊNG (3 điểm): Thí sinh chỉ làm một trong hai phần A hoặc B

A. Theo chương trình chuẩn

Câu 6a. (2 điểm)

a) Trong mặt phẳng với hệ tọa độ Đề-các vuông góc Oxy, cho hình vuông ABCD có các đỉnh A (−1; 2) , C (3;−2)

. Gọi E là trung điểm của cạnh AD,BM là đường thẳng vuông góc với CE tại M ; N là trung điểm của của BM

và P là giao điểm của AN với DM . Biết phương trình đường thẳng BM : 2x− y − 4 = 0 .Tìm tọa độ điểm P .

b) Trong không gian với hệ tọa độ Đề-các vuông góc Oxyz , cho mặt cầu (S) : x2 +y2 +z2−2x−4y+6z−13 = 0

và đường thẳng d :x+ 1

1=y + 2

1=z − 1

1. Xác định tọa độ điểm M trên đường thẳng d sao cho từ M có thể

kẻ được 3 tiếp tuyến MA,MB,MC đến mặt cầu (S) ( A,B,C là các tiếp điểm ). Sao cho AMB = 60o; BMC =

90o; CMA = 120o .

Câu 7a. (1 điểm) Cho các số phức z1; z2 đồng thời thỏa mãn các điều kiện : z1 + 3z1z2 = (−1 + i) z2 và 2z1 − z2 = −3 + 2i

. Tìm mô-đun của số phức w =z1z2

+ z1 + z2 .

B. Theo chương trình nâng cao

Câu 6b. (2 điểm)

a) Trong mặt phẳng với hệ tọa độ Đề-các vuông góc Oxy cho tam giác ABCvuông tại A ngoại tiếp hình chữ nhật

MNPQ . Biết các điểm M (−3; −1) và N (2;−1) thuộc cạnh BC , Q thuộc cạnh AB , P thuộc cạnh AC , đường

thẳng AB có phương trình : x− y + 5 = 0 . Xác định tọa độ các đỉnh của tam giác ABC .

b) Trong không gian với hệ tọa độ Đề-các vuông góc Oxyz , cho mặt cầu (S) : (x− 2)2 + (y − 2)2 + (z − 2)2 = 12

và điểm A (4; 4; 0) . Xác định tọa độ điểm B thuộc mặt cầu (S) biết tam giác BOA cân tại B và có diện tích bằng

4√

3

Câu 7b. (1 điểm) Từ các chữ số 0, 1, 2, 3, 4, 5, 6 có thể lập được bao nhiêu số chẵn có 4 chữ số khác nhau nhỏ hơn 4321

đồng thời các chữ số 1 và 3 luôn có mặt và đứng cạnh nhau.

———————————————–Hết—————————————————-

280

Page 282: Chuyên đề luyện thi đại học môn Toán - VipLam.Net

Đề 2 ngày 12.10.2012

PHẦN CHUNG CHO TẤT CẢ THÍ SINH (7 điểm)

Câu 1. (2 điểm) Cho hàm số y =2x+ 2

x− 1, có đồ thị (C) .

a) Khảo sát sự biến thiên và vẽ đồ thị hàm số (C) .

b) Xác định tất cả những điểm M trên đồ thị (C) sao cho đồ thị hàm số (C) tiếp xúc với đường tròn tâm I (1; 2)

tại M.

Câu 2. (2 điểm)

a) Giải phương trìnhcos 3x

cos 5x− cosx

cos 3x= 2 sin 5x. sin 3x.

b) Giải bất phương trình (√x+ 6)

√x (2x2 + 26x+ 8)− 4 ≥ x (2x+ 3

√x+ 33) .

Câu 3. (1 điểm) Tính tích phân I =

∫ e

1

x2 − 2 lnx+ 1

x2.√x+ lnx

dx.

Câu 4. (1 điểm) Cho lăng trụ đứng ABC.A1B1C1 có BC = 2AB và AB ⊥ BC. Gọi M,N lần lượt là trung điểm của

A1B1 và BC. Khoảng cách giữa hai đường thẳng AM và B1C bằng2a√

7. Góc giữa hai mặt phẳng (AB1C) và

(BCC1B1) bằng 60o. Tính thể tích khối chóp MABC và bán kính mặt cầu ngoại tiếp khối chóp B1ANC theo a.

Câu 5. (1 điểm) Cho các số thực x, y, z không âm sao cho không có 2 số nào đồng thời bằng 0.

Tìm giá trị nhỏ nhất của biểu thức: P = (xy + yz + zx)

(1

x2 + y2+

1

y2 + z2+

1

z2 + x2

).

PHẦN RIÊNG (3 điểm): Thí sinh chỉ làm một trong hai phần A hoặc B

A. Theo chương trình chuẩn

Câu 6A. (2 điểm)

a) Trong mặt phẳng với hệ tọa độ vuông góc Oxy, cho đường tròn (I) : x2 + y2 − 4x + 2y − 11 = 0 và đường

thẳng d : 4x− 3y+ 9 = 0. Gọi A,B là hai điểm thuộc đường thẳng d, C là điểm thuộc đường tròn (C). Biết điểm

H

(22

5;

11

5

)là một giao điểm của AC với đường tròn (I), điểm K

(−6

5;

7

5

)là trung điểm của cạnh AB. Xác

định tọa độ các điểm A,B,C biết diện tích tứ giác AHIK bằng 24 và hoành độ điểm A dương.

b) Trong không gian với hệ tọa độ vuông góc Oxyz, cho hai điểm A (−1;−3;−2) ; B (0;−2; 2) và mặt cầu

(S) : (x+ 1)2 + (y + 2)2 + (z + 3)2 = 14. Gọi (P ) là mặt phẳng đi qua A đồng thời cắt mặt cầu (S) theo giao

tuyến là đường tròn có bán kính nhỏ nhất. Tìm điểm M thuộc mặt phẳng (P ) sao cho tam giác ABM vuông cân

tại A.

Câu 7A. (1 điểm) Tìm n ∈ N∗ thỏa mãn: 3.C0n + 4C1

n + 5C2n + ...+ (n+ 3)Cnn = (n+ 6)

(35

12n+ 2013

)B. Theo chương trình nâng cao

Câu 6B. (2 điểm)

a) Trong mặt phẳng với hệ tọa độ vuông góc Oxy cho điểm A (1; 0) và các đường tròn (C1) : x2 + y2 = 2; (C2) :

x2 + y2 = 5 . Tìm tọa độ các điểm B và C lần lượt nằm trên (C1) và (C2) để tam giác ABC có diện tích lớn nhất.

b) Trong không gian với hệ tọa độ vuông góc Oxyz , cho đường tròn (C) :

{x2 + y2 + z2 + 4x− 6y + 4z + 4 = 0

x+ 2y − 2z − 2 = 0

có tâm I và đường thẳng d :x− 3

2=y + 2

1=z + 1

−1. Lập phương trình mặt cầu có tâm thuộc đường thẳng AI,

bán kính R =√

26 và tiếp xúc với đường thẳng ∆ :x− 1

1=y − 3

2=z + 2

−2, biết rằng A thuộc đường tròn (C) và

đường thẳng d vuông góc với đường thẳng AI.

Câu 7B. (1 điểm) Cho các số phức z1; z2 đồng thời thỏa mãn các điều kiện : z1 + 2z2 là số thực, 2z1 − z2 là số ảo và

3z1 + z2 = 5− 5i. Tìm Mô đun của số phức w = z21 + 3z1.z22 .

———————————————–Hết—————————————————-

281

Page 283: Chuyên đề luyện thi đại học môn Toán - VipLam.Net

Đề 3 ngày 10.11.2012

PHẦN CHUNG CHO TẤT CẢ THÍ SINH (7 điểm)

Câu 1. (2 điểm) Cho hàm số y =x− 2

x− 1(H).

a) Khảo sát sự biến thiên và vẽ đồ thị của hàm số (H).

b) Tìm tất cả các giá trị thực của m để đường thẳng d : y = 2x + m − 2 cắt đồ thị (H) tại hai điểm phân biệt

A,B sao cho tứ giác AMBN có diện tích bằng5√

17

4, biết M (1;−2) ;N (3;−3).

Câu 2. (2 điểm)

a) Giải phương trình: cosx+ cos(x+

π

3

)=√

3. cos2(x+

π

6

)+

1√3

cos2(π

3− x)

b) Giải hệ phương trình:

(2√x+√y)√

xy = 4y√y − x

√x

8y

(x+

√1− x+ (1− y)2

)− 1 = 12x+ 7

(√x− 1 +

√y − 2

)Câu 3. (1 điểm) Tính tích phân I =

∫ π2

0

3e2x + sinx(4ex + 3 sinx)− 1

(ex + sinx)2dx.

Câu 4. (1 điểm) Cho lăng trụ tam giác ABC.A′B′C′ có đáy là tam giác đều tâm O. Đỉnh C′ có hình chiếu trên mặt

phẳng (ABC) trùng với tâm O của đáy. Biết rằng khoảng cách từ O đến cạnh CC′ bằng a. Chứng minh rằng qua

AB ta có thể dựng được mặt phẳng (P ) vuông góc với CC′. Gọi K là giao điểm của CC′ và mặt phẳng (P ), biết

góc AKB = 120o. Tính thể tích hình lăng trụ ABC.A′B′C′ theo a và góc hợp bởi CC′ và mặt phẳng (ABC).

Câu 5. (1 điểm) Cho các số thực a, b, c ∈ [1; 2] Tìm giá trị lớn nhất của biểu thức : P =10a

bc+

11b

ac+

2012c

ab

PHẦN RIÊNG (3 điểm): Thí sinh chỉ làm một trong hai phần A hoặc B

A. Theo chương trình chuẩn

Câu 6A. (2 điểm)

a) Trong mặt phẳng với hệ tọa độ vuông góc Oxy cho hình thoi ABCD có A = 60o. Trên các cạnh AB,BC lấy

các điểm M,N sao cho MB + NB = AB. Biết P (√

3; 1) thuộc đường thẳng DN và đường phân giác trong của

góc MDN có phương trình là d : x− y√

3 + 6 = 0. Tìm toạ độ đỉnh D của hình thoi ABCD.

b) Trong không gian với hệ tọa độ vuông góc Oxyz, cho hình chóp tứ giác đều S.ABCD có các đỉnh S(−3; 2; 1),

A(0; 2; 2) và C(−2; 2;−2).Gọi I là tâm mặt cầu ngoại tiếp hình chóp S.ABCD. Viết phương trình mặt phẳng (α)

đi qua I và vuông góc với BD.

Câu 7A. (1 điểm) Giải bất phương trình sau :

√2 + 32x

√2 + 32x −

√2− 32x

+34x +

√4− 34x − 7

32x≥ 32x − 2√

4− 34x − 2 + 32x.

B. Theo chương trình nâng cao

Câu 6B. (2 điểm)

a) Trong mặt phẳng với hệ tọa độ vuông góc Oxy, cho hình chữ nhật ABCD, đỉnh B thuộc đường thẳng

d1 : 2x− y + 2 = 0, đỉnh C thuộc đường thẳng d2 : x− y − 5 = 0. Gọi H là hình chiếu của B xuống đường chéo

AC. Biết M

(9

5;

2

5

); K (9; 2) lần lượt là trung điểm của AH và CD. Tìm toạ độ các đỉnh của hình chữ nhật

ABCD biết hoành độ đỉnh C lớn hơn 4.

b) Trong không gian với hệ tọa độ vuông góc Oxyz , cho mặt phẳng (P ) : x+ 2y + 2z − 6 = 0 và mặt cầu (S) có

phương trình x2 + y2 + z2 − 2x− 2y + 4z − 10 = 0. Chứng minh rằng mặt cầu (S) cắt mặt phẳng (P ) theo giao

tuyến là một đường tròn (C). Lập phương trình mặt cầu (S1) chứa đường tròn (C) biết tâm của mặt cầu (S1)

cách điểm A(4; 1; 4) một khoảng bằng 3√

5 đồng thời hoành độ tâm mặt cầu (S1) lớn hơn 2.

Câu 7B. (1 điểm) Có ba bình: Bình A đựng 4 viên bi xanh và 5 bi đỏ, bình B đựng 8 viên bi xanh và 7 viên bi đỏ và

bình C đựng 6 viên bi xanh và 9 viên bi đỏ. Người ta chọn ngẫu nhiên ra một bình từ ba bình đã cho, rồi từ bình

đã chọn lấy ngẫu nhiên ra một viên bi. Tính xác suất để viên bi lấy ra là viên bi đỏ.

———————————————–Hết—————————————————-

282

Page 284: Chuyên đề luyện thi đại học môn Toán - VipLam.Net

Đề 4 ngày 24.11.2012

PHẦN CHUNG CHO TẤT CẢ THÍ SINH (7 điểm)

Câu 1. (2 điểm) Cho hàm số y = x4 − 2mx2 + 1 có đồ thị (Cm)

a) Khảo sát sự biến thiên và vẽ đồ thị hàm số đã cho khi m = −1.

b) Xác định các giá trị của tham số m để đồ thị hàm số (Cm) có 3 điểm cực trị tạo thành một tam giác có độ lớn

của diện tích và chu vi bằng nhau.

Câu 2. (2 điểm)

a) Giải phương trình: 2 (1 + sinx) + (2 cosx+ 1) (2 cosx− 1)2 = 4 cosx+ tanx

b) Giải phương trình:1 + 2

√x− x

√x

3− x−√

2− x= 2

(1 + x

√x

1 + x

)

Câu 3. (1 điểm) Tính tích phân I =

∫ π4

0

sinx− 2x. cosx

ex (1 + sin 2x)dx.

Câu 4. (1 điểm) Cho hình chóp S.ABCD , đáy ABCD là hình chữ nhật có AB = 2a,AD = 2√

2a . Cạnh bên SA

vuông góc với mặt phẳng đáy, các điểm M,N lần lượt là trung điểm của DA và DS. Đường thẳng SC cắt mặt

phẳng (BMN) tại P . Tính thể tích khối chóp S.BMNP và khoảng cách giữa hai đường thẳng SB và PN , biết

rằng cô-sin góc giữa đường thẳng CN và mặt phẳng (BMN) bằng

√33

9.

Câu 5. (1 điểm) Cho các số thực x, y, z thỏa mãn : x2 + 2y2 + 5z2 ≤ 2. Tìm giá trị lớn nhất của biểu thức:

P = (xy + yz + zx)

[1 +

√4− (x2 + 2y2 + 5z2)2

]

PHẦN RIÊNG (3 điểm): Thí sinh chỉ làm một trong hai phần A hoặc B

A. Theo chương trình chuẩn

Câu 6A. (2 điểm)

a) Trong mặt phẳng với hệ trục tọa độ Đề-các vuông góc Oxy, cho hình thoi ABCD ngoại tiếp đường tròn

(I) : (x− 5)2 + (y− 6)2 =32

5. Biết rằng các đường thẳng AC và AB lần lượt đi qua các điểm M(7; 8) và N(6; 9).

Tìm tọa độ các đỉnh của hình thoi ABCD.

b) Trong không gian với hệ tọa độ vuông góc Oxyz cho các điểm B(0; 1; 0) và N(2;−1; 2). Viết phương trình mặt

phẳng (P ) đi qua các điểm B,N đồng thời cắt các tia Ox,Oz tại A,C sao cho diện tích tam giác ABC đạt giá

trị nhỏ nhất.

Câu 7A. (1 điểm) Giải hệ phương trình:

{log5 (5x − 4) = 1− 2y

x3 − 2y =(x2 − x

)(2y + 1)

B. Theo chương trình nâng cao

Câu 6B. (2 điểm)

a) Trong mặt phẳng với hệ trục tọa độ Đề-các vuông góc Oxy, cho hai đường tròn (O1) và (O2) có bán kính bằng

nhau và cắt nhau tại A(4; 2) và B. Một đường thẳng đi qua A và N(7; 3) cắt các đường tròn (O1) và (O2) lần

lượt tại D và C . Tìm tọa độ các đỉnh của tam giác BCD biết rằng đường thẳng nối tâm O1, O2 có phương trình

x− y − 3 = 0 và diện tích tam giác BCD bằng24

5.

b) Trong không gian với hệ tọa độ vuông góc Oxyz cho các mặt phẳng (P ) : −mx+ (1−m)z − 2m+ 3 = 0,

(Q) : my + z + 3 = 0 và (R) : x − y = 0 ( m là tham số thực khác 0 ). Viết phương trình mặt phẳng (α) đi qua

giao tuyến của hai mặt phẳng (P ) và (Q) đồng thời vuông góc với mặt phẳng (R).

Câu 7B. (1 điểm) Tính xác suất để có thể lập được một số tự nhiên gồm 7 chữ số mà trong đó chữ số 3 có mặt đúng

2 lần,chữ số 0 có mặt đúng 3 lần và các chữ số còn lại có mặt không quá 1 lần.

———————————————–Hết—————————————————-

283

Page 285: Chuyên đề luyện thi đại học môn Toán - VipLam.Net

Đề 5 ngày 08.12.2012

PHẦN CHUNG CHO TẤT CẢ THÍ SINH (7 điểm)

Câu 1. (2 điểm) Cho hàm số y =x+ 1

x− 1có đồ thị là (C)

a) Khảo sát sự biến thiên và vẽ đồ thị của hàm số (C).

b) Viết phương trình các tiếp tuyến tại điểm M thuộc đồ thị (C), biết tiếp tuyến đó cắt trục hoành tại điểm N sao cho

∆OMN vuông.

Câu 2. (2 điểm)

a) Giải phương trình: 5 sin(

2x+π

4

)− 3 sin

(6x−

π

4

)=√

2 (4 cos 4x− sin 4x)

b) Giải hệ phương trình:

2(x3 − y6

)= 3

(y4 − xy2

)4x3

y4+ 5

x

y− 8y2 + 7 = 0

(x, y ∈ R)

Câu 3. (1 điểm) Tính tích phân I =

∫ π4

−π4

sin 2x+ cos 2x

sinx+ cosx+ 1dx

Câu 4. (1 điểm) Cho hình lăng trụ ABC.A′B′C′ có đáy ABC là tam giác vuông tại A, AB = a, AC = a√

3. Gọi H, M lần

lượt là trung điểm của BC,CC′. Biết A′ cách đều các đỉnh A, B, C. Góc tạo bởi đường thẳng A′B và mặt phẳng (A′AH)

bằng 300. Tính thể tích lăng trụ ABCA′B′C′ và khoảng cách giữa hai đường thẳng A′B và AM .

Câu 5. (1 điểm) Cho a, b, c các số dương thoả mãn : 2a2 + 3b2 + 5ab+ 3bc+ 2ac+ c ≤ 3 + 5a+ 8b. Chứng minh rằng:1

√8a + 1

+1√

8b + 1+

1√

8c + 1≥ 1

PHẦN RIÊNG (3 điểm): Thí sinh chỉ làm một trong hai phần A hoặc B

A. Theo chương trình chuẩn

Câu 6A. (2 điểm)

a) Trong mặt phẳng với hệ trục tọa độ Đề-các vuông góc Oxy, cho đường tròn (C) :

(x−

5

4

)2

+ (y − 1)2 = 2 .Xác định

tọa độ các đỉnh của hình vuông ABCD biết các đỉnh B và C thuộc đường tròn (C), các đỉnh A và D thuộc trục Ox.

b) Trong không gian với hệ tọa độ vuông góc Oxyz cho mặt cầu (S) : x2 + y2 + z2 − 2x− 4y + 6z + 13 = 0 và các đường

thẳng d1 :

x = 1 + t

y = 2− t

z = 3

(t ∈ R), d2 :x− 1

−1=y − 1

2=z + 1

1. Gọi M là điểm thuộc mặt cầu (S) sao cho khoảng cách từ M

đến đường thẳng d1 đạt giá trị nhỏ nhất. Viết phương trình đường thẳng đi qua M , vuông góc với d1 và cắt d2.

Câu 7A. (1 điểm) Giải phương trình:√

3x − x−√x+ 1 + 2x.3x + 2x+ 1 = 9x

B. Theo chương trình nâng cao

Câu 6B. (2 điểm)

a) Trong mặt phẳng tọa độ Oxy cho Elip có phương trình:x2

8+y2

4= 1 và điểm I(1;−1). Một đường thẳng ∆ qua I cắt

Elip tại hai điểm phân biệt A,B .Tìm tọa độ các điểm A,B sao cho độ lớn của tích IA.IB đạt giá trị nhỏ nhất.

b) Trong không gian với hệ tọa độ vuông góc Oxyz cho hai đường thẳng ∆1 :x+ 1

2=y − 1

3=z

1, ∆2 :

x

3=

y

−1=z − 1

2và hai điểm A(−1; 3; 0), B(1; 1; 1). Viết phương trình đường thẳng ∆ cắt các đường thẳng ∆1 và ∆2 tại M và N sao cho

tam giác ANB vuông tại B và thể tích khối tứ diện ABMN bằng1

3.

Câu 7B. (1 điểm) Giải hệ phương trình :

x2 − y2 = 2

8 log16 (x+ y) =1

4log3 (x− y) + 2

———————————————–Hết—————————————————-

284

Page 286: Chuyên đề luyện thi đại học môn Toán - VipLam.Net

Đề 6 ngày 29.12.2012

PHẦN CHUNG CHO TẤT CẢ THÍ SINH (7 điểm)

Câu 1. (2 điểm) Cho hàm số y = x4 − 2 (m+ 1)x2 + 2m− 1 có đồ thị (Cm), ;m là tham số thực.

a) Khảo sát sự biến thiên và vẽ đồ thị của hàm số (C2) khi m = 2.

b) Tìm tất cả các giá trị của m để đường thẳng d : y = −1 cắt đồ thị (Cm) tại đúng hai điểm phân biệt A,B , sao cho tam

giác IAB có diện tích bằng 4

√2(

2−√

2)với I (2; 3).

Câu 2. (2 điểm)

a) Giải phương trình: cosx (cos 2x− 19)− (1 + sinx) (7− cos 2x) = −3 (8 + sin 2x)

b) Giải hệ phương trình:

{2y − 3x+

√y (x− 2) = 4

(√x− 2−√y

)− 6

√y + 2

√y (xy − x+ 5) = 2 (y + 2)−

√5x+ 6

(x, y ∈ R)

Câu 3. (1 điểm) Tính tích phân I =

∫ e

1

x3 lnx+ x2 ln2 x+ 3(x+ 1)

x(x+ lnx)dx.

Câu 4. (1 điểm) Cho hình chóp tứ giác đều S.ABCD. Biết cạnh bên hợp với mặt đáy (ABCD) một góc 60o và mặt cầu ngoại

tiếp hình chóp S.ABCD có bán kính bằnga√

6

3. Gọi E là điểm đối xứng của D qua trung điểm của SA, M là trung điểm

của AE, N là trung điểm của BC. Chứng minh rằng MN vuông góc với BD. Tính thể tích khối chóp S.ABCD và khoảng

cách giữa hai đường thẳng MN và AC theo a.

Câu 5. (1 điểm) Cho các số thực x, y, z thuộc khoảng (1;√

2) . Tìm giá trị nhỏ nhất của biểu thức :

P =xy2

4y2z − z2x+

yz2

4z2x− x2y+

zx2

4x2y − y2z

PHẦN RIÊNG (3 điểm): Thí sinh chỉ làm một trong hai phần A hoặc B

A. Theo chương trình chuẩn

Câu 6A. (2 điểm)

a) Trong mặt phẳng với hệ trục tọa độ Oxy cho tam giác ABC với A (3; 5), B (1; 2), C (6; 3). Gọi ∆ là đường thẳng đi qua

A cắt BC sao cho tổng khoảng cách từ hai điểm B,C đến ∆ là lớn nhất. Hãy lập phương trình đường thẳng d đi qua điểm

E (−1; 1) đồng thời cắt cả hai đường thẳng ∆ và d1 : x− y+ 14 = 0 lần lượt tại hai điểm H,K sao cho 3HK = IH√

10 với

I là giao điểm của ∆ và d1 .

b) Trong không gian với hệ tọa độ Oxyz cho hai điểm A (3; 0; 0) , M (−3; 2; 1) .Gọi (α) là mặt phẳng chứa AM và cắt hai

trục tọa độ Oy,Oz lần lượt tại hai điểm B,C đồng thời tạo với mặt phẳng (β) : x + 2y + 2z − 8 = 0 một góc ϕ có giá

trị cosϕ =20

21. Lập phương trình đường thẳng ∆ đối xứng với đường thẳng d :

x

−3=

y

−2=z

2qua mặt phẳng (α) biết

zC <3

2.

Câu 7A. (1 điểm) Cho số phức có phần thực âm thỏa điều kiện z3 + 2z − 16i = 8z. Hãy tính mô-đun của số phức:

ω = z2 +1

z2− 8

(z +

1

z

)+ 17

B. Theo chương trình nâng cao

Câu 6B. (2 điểm)

a) Trong mặt phẳng với hệ tọa độ Oxy cho cho đường tròn (C) : x2 + y2 − 2x− 6y − 6 = 0 và hai điểm B (5; 3) , C (1;−1)

. Tìm tọa các đỉnh A,D của hình bình hành ABCD biết A thuộc đường tròn (C) và trực tâm H của tam giác ABC thuộc

đường thẳng d : x+ 2y + 1 = 0 và hoành độ điểm H bé hơn hơn 2.

b) Trong không gian với hệ tọa độ Oxyz cho hai điểm A (1; 2; 3) , B (4;−1; 3) và đường tròn (C) là đường tròn lớn nằm

trong mặt cầu (S) có tâm I (1;−1;−2) và đường thẳng ∆ :x

3=y

2=z − 6

−2cắt đường tròn (C) tại hai điểm M,N sao cho

MN = 8√

2 . Lập phương trình mặt cầu (S) , tìm tọa độ điểm C thuộc mặt cầu (S) và mặt phẳng (P ) : 2x+y+3z−22 = 0

sao cho tam giác ABC cân tại C .

Câu 7B. (1 điểm) Cho hàm số y =x2 − 2x+m

x+ 1có đồ thị là (Hm) . Tìm m để tiếp tuyến tại điểm M có hoành độ bằng −2

thuộc (Hm) cắt hai trục tọa độ Ox,Oy lần lượt tại hai điểm A,B sao cho tam giác IAB có IA = 4IB với I là giao điểm

của hai đường tiệm cận của đồ thị (Hm) .

———————————————–Hết—————————————————-

285

Page 287: Chuyên đề luyện thi đại học môn Toán - VipLam.Net

Đề 7 ngày 12.01.2013

PHẦN CHUNG CHO TẤT CẢ THÍ SINH (7 điểm)

Câu 1. (2 điểm) Cho hàm số y = x3 − 3 (m+ 1)x2 + 12mx+m+ 4, (Cm)

a) Khảo sát sự biến thiên và vẽ đồ thị của hàm số (C) khi m = 0.

b) Gọi A và B lần lượt là các điểm cực đại và cực tiểu của đồ thị hàm số (Cm). Tìm tất cả các giá trị của m để

khoảng cách giữa giữa hai đường thẳng tiếp tuyến tại A và B của đồ thị Cm) bằng 4.

Câu 2. (2 điểm)

a) Giải phương trình :cos3x (cosx− 2 sinx)− cos 2x− cos2x

tan(x+ π

4

). tan

(x− π

4

) = 0.

b) Giải phương trình 4√

3 (x+ 5)− 4√x+ 13 = 4

√11− x− 4

√3 (3− x)

Câu 3. (1 điểm) Tính tích phân I =

∫ e

2

1 + (x− 1) lnx− ln2 x

(1 + x lnx)2dx

Câu 4. (1 điểm) Cho hình chóp S.ABCD có đáy ABCD là hình chữ nhật cạnh AB = a,BC = a√

2 . Cạnh bên SA

vuông góc với đáy. Gọi M,N lần lượt là trung điểm của SD và AD. Mặt phẳng (P ) chứa BM cắt mặt phẳng

(SAC) theo một đường thẳng vuông góc với BM . Giả sử BN cắt AC tại I, gọi J là trung điểm của IC. Biết

khoảng cách từ đỉnh S đến mặt phẳng (P ) bằng2√

2a

3. Tính thể tích khối chóp BMDJ và khoảng cách giữa hai

đường thẳng DM và BJ theo a.

Câu 5. (1 điểm) Cho các số thực dương x, y, z thỏa mãn điều kiện 2(9z2 + 16y2

)= (3z + 4y)xyz . Tìm giá trị nhỏ

nhất của biểu thức :

P =x2

x2 + 2+

y2

y2 + 3+

z2

z2 + 4+

5xyz

(x+ 2) (y + 3) (z + 4)

PHẦN RIÊNG (3 điểm): Thí sinh chỉ làm một trong hai phần A hoặc B

A. Theo chương trình chuẩn

Câu 6A. (2 điểm)

a) Trong mặt phẳng với hệ trục tọa độ Oxy, cho tam giác ABC vuông tại A . Gọi H là hình chiếu của A trên

BC . Tam giác ABH ngoại tiếp đường tròn (C) :

(x− 16

5

)2

+

(y − 33

5

)2

=36

25. Tâm đường tròn nội tiếp tam

giác ACH là I

(26

5;

23

5

). Tìm tọa độ trọng tâm G của tam giác ABC.

b) Trong không gian với hệ tọa độ Oxyz, cho hai điểm A (2; 2; 5) và B (1; 1; 7) . Tìm tọa độ điểm M thuộc đường

tròn (C) :

{(x− 1)2 + (y − 1)2 + (z − 2)2 = 9

x+ y + z − 7 = 0để tam giác MAB có diện tích nhỏ nhất.

Câu 7A. (1 điểm) Giải bất phương trình : 3 +4√

6− 2x +√

2 + 2x≥ 2√

8−√

(6− 2x) (2x + 2)

B. Theo chương trình nâng cao

Câu 6B. (2 điểm)

a) Trong mặt phẳng với hệ tọa độ Oxy, cho hình chữ nhật ABCD nội tiếp đường tròn (C) có tâm I(1; 2) . Tiếp

tuyến của (C) tại B,C,D cắt nhau tại M,N . Giả sử H(1;−1) là trực tâm tam giác AMN . Tìm tọa độ các điểm

A,B,M,N biết rằng chu vi tam giác AMN bằng 28 + 4√

10.

b) Trong không gian với hệ tọa độ Oxyz, cho mặt cầu (S) : (x− 1)2 + (y − 2)2 + (z − 3)2 =14

3và đường thẳng

d :x− 4

3=y − 4

2=z − 4

1. Tìm trên đường thẳng d các điểm A sao cho từ A có thể kẻ được 3 tiếp tuyến đến

mặt cầu (S) sao cho tứ diện ABCD là tứ diện đều ( trong đó B,C,D là các tiếp điểm ) .

Câu 7B. (1 điểm) Giải hệ phương trình:

{log2

(x+√x2 + 1

)+ log2

(y +

√y2 + 4

)= 1

2log2

(√2x− y + 2 +

√3y − 2x+ 4

)= 2log4

(5x2 + y2 + 1

)+ 1

———————————————–Hết—————————————————-

286

Page 288: Chuyên đề luyện thi đại học môn Toán - VipLam.Net

Đề 8 ngày 26.01.2013

PHẦN CHUNG CHO TẤT CẢ THÍ SINH (7 điểm)

Câu 1. (2 điểm) Cho hàm số y = x3 − 3x2 + 2 có đồ thị (C).

a) Khảo sát sự biến thiên và vẽ đồ thị của hàm số (C).

b) Tìm trên (C) hai điểm phân biệt M,N biết tiếp tuyến tại M,N song song với nhau, đồng thời đường thẳng

MN cắt các trục Ox,Oy tại hai điểm phân biệt A,B (khác O) sao cho AB =√

37.

Câu 2. (2 điểm)

a) Giải phương trình: 4(sinx+ cosx)(1 + cosx)2 = 6 cos2 x2

+ sinx.

b) Giải hệ phương trình

x− 3√

3 + x = 3√y − 5− y√

x2 + 16(y − x) + y = 2√xy

Câu 3. (1 điểm) Tính tích phân I =

∫ π2

0

2(x+ sinx)sin2x+ sin 2x(1 + sin2x)

(1 + cosx)2dx

Câu 4. (1 điểm) Cho lăng trụ ABCD.A′B′C′D′, có đáy ABCD là hình vuông. Gọi M,N lần lượt là trung điểm của

BC và CD. Hình chiếu vuông góc của A′ lên mặt phẳng (ABCD) trùng với tâm O của hình vuông ABCD. Biết

rằng khoảng cách giữa AB′ và DM bằng a√15

5và mặt phẳng (AA′D′D) hợp với đáy một góc bằng 60o. Tính thể

tích khối lăng trụ và khoảng cách giữa hai đường thẳng A′D và AN theo a.

Câu 5. (1 điểm) Cho các số thực dương x, y thỏa điều kiện :x

(1− 1

y

)+ y

(1− 1

x

)= 4 .

Tìm giá trị nhỏ nhất của biểu thức :

P = xy +√

1 + x2 +√

1 + y2

PHẦN RIÊNG (3 điểm): Thí sinh chỉ làm một trong hai phần A hoặc B

A. Theo chương trình chuẩn

Câu 6A. (2 điểm)

a) Trong mặt phẳng với hệ trục tọa độ Oxy cho đường tròn (C) : (x − 4)2 +

(y − 9

2

)2

=25

4và hai điểm

A(2; 3), B(6; 6). Gọi M,N là hai điểm khác nhau nằm trên đường tròn (C) sao cho các đường thẳng AM và BN

cắt nhau tại H, AN và BM cắt nhau tại C. Tìm tọa độ điểm C, biết tọa độ điểm H(4; 5

2

).

b) Trong không gian với hệ tọa độ Oxyz cho hai điểm M (4; 4; 0) , N (−1; 1;−1) và hai đường thẳng

d1 :x− 3

1=y − 2

−3=z − 1

4, d2 :

x− 2

1=y − 5

2=z + 3

1. Gọi A,B lần lượt là hai điểm thuộc d1, d2.

Hãy lập phương trình mặt phẳng (P ) đi qua M,N sao cho (P ) là mặt phẳng trung trực của AB

Câu 7A. (1 điểm) Giải bất phương trình sau trên tập số thực :

log2

∣∣x2 − 4∣∣+ log√2 x ≤ 2 log4

[|x|(x4 − 10x2 + 16

)]B. Theo chương trình nâng cao

Câu 6B. (2 điểm)

a) Trong mặt phẳng với hệ tọa độ Oxy cho hình vuông ABCD có điểm M(3; 2) nằm trên đường chéo BD. Từ

M kẻ các đường thẳng ME,MF lần lượt vuông góc với AB tại E(3; 4) và AD tại F (−1; 2). Hãy xác định tọa độ

điểm C của hình vuông ABCD

b) Trong không gian với hệ tọa độ Oxyz cho hai điểm A(1; 2; 1), B(−3; 3; 3) và mặt phẳng (P ) : x+2y+2z−16 = 0.

Tìm tọa độ điểm M trên (P ) sao cho MA+MB = 7.

Câu 7B. (1 điểm) Hãy xác định hệ số của số hạng chứa x6y−3 trong khai triển P (x, y) =

(7x+

1

8y

)3n

.

Biết n là số nguyên dương thỏa mãn :24n

n+ 1{0n +

24(n−1)

n{1n +

24(n−2)

n− 1{2n + ....+ {nn = 1305

———————————————–Hết—————————————————-

287

Page 289: Chuyên đề luyện thi đại học môn Toán - VipLam.Net

Đề 9 ngày 23.02.2013

PHẦN CHUNG CHO TẤT CẢ THÍ SINH (7 điểm)

Câu 1. (2 điểm) Cho hàm số y =x− 2

x+ 1(C).

a) Khảo sát sự biến thiên và vẽ đồ thị hàm số (C).

b) Tìm trên (C) những điểm M mà tiếp tuyến của đồ thị tại đó cắt các đường tiệm cận của (C) tại hai điểm

phân biệt A,B sao cho tam giác IAB có bán kính đường tròn ngoại tiếp bằng5

2bán kính đường tròn nội tiếp (I

là giao điểm hai đường tiệm cận).

Câu 2. (2 điểm)

a) Giải phương trình :sin 2x. sinx+ (cosx+ 1) (cosx+ 2)

2 sin 2x+ cos 2x+ 2 sinx+ cosx+ 1= 1

b) Giải hệ phương trình :

(x+ y + 1)xy = x2 + y2

x3 + y3)xy − y2 = 4xy2 4x3y2 + x− 1

)Câu 3. (1 điểm) Tính tích phân : I =

∫ π2

0

x (7− cos 2x) + 3

cosx+ 2dx

Câu 4. (1 điểm) Cho hình chóp tứ giác S.ABCD có đáy ABCD là hình thoi tâm O , OA = 2OB = 2a. Cạnh SO

vuông góc với mặt phẳng đáy. Một mặt phẳng (α) đi qua A và vuông góc với SC cắt các cạnh SB, SC, SD lần

lượt tại B′, C′, D′ . Gọi M là trung điểm của AB′. Tính thể tích khối chóp S.ABCD và góc giữa đường thẳng

SM với mặt phẳng (α), biết 4B′C′D′ đều.

Câu 5. (1 điểm) Cho các số thực x, y, z thuộc đoạn [1; 3] . Tìm giá trị nhỏ nhất của biểu thức :

T =25(y + z)2

12x2 + 2012 (xy + yz + zx)

PHẦN RIÊNG (3 điểm): Thí sinh chỉ làm một trong hai phần A hoặc B

A. Theo chương trình chuẩn

Câu 6A. (2 điểm)

a) Trong mặt phẳng với hệ tọa độ Đề-các Oxy cho tam giác ABC nội tiếp đường tròn (C1) : (x−2)2+(y−3)2 = 45.

Đường tròn (C2) có tâm K(−1;−3) cắt đường tròn (C1) theo một dây cung song song với AC. Biết diện tích tứ

giác AICK = 30√

2, chu vi tam giác ABC bằng 10√

10 trong đó I là tâm đường tròn (C1). Hãy tìm những điểm

B có hoành độ âm.

b) Trong không gian với hệ tọa độ Đề-các Oxyz cho tam giác ABC có C(3; 2; 3) . Phương trình đường cao

AH :x− 2

1=y − 3

1=z − 3

−2, phương trình đường phân giác trong BD :

x− 1

1=y − 4

−2=z − 3

1. Tính chu vi

tam giác ABC.

Câu 7A. (1 điểm) Giải phương trình : 9 2x2 + 4x+ 5).3x

2+3x = x2 + x+ 3

B. Theo chương trình nâng cao

Câu 6B. (2 điểm)

a) Trong mặt phẳng với hệ tọa độ Đề-các Oxy cho tam giác ABC cân tại A, phương trình cạnh bên AC :

x + y − 3 = 0. Trên tia đối của tia CA lấy điểm E. Phân giác trong góc BAC cắt BE tại D. Đường thẳng d đi

qua D song song với AB cắt BC tại F . Tìm tọa độ giao điểm M của AF và BE biết phương trình đường thẳng

AF : 2x+ y − 5 = 0 và I(−1;−3) là trung điểm của DF .

b) Trong không gian với hệ tọa độ Đề-các Oxyz cho tứ diện ABCD có A(2; 0; 0), B(0; 2; 0), C(0; 0; 2), D(2; 2; 2)

và mặt cầu (S) : (x − 1)2 + (y − 2)2 + (z − 3)2 = 16 . Gọi M là điểm có tọa độ nguyên nằm trong tứ diện. Viết

phương trình mặt phẳng (α) qua M và cắt (S) theo một đường tròn có chu vi bé nhất.

Câu 7B. (1 điểm) Cho các số phức z1, z2 thỏa mãn : z1 không phải là số ảo và z1 − z1.|z2|2)là số ảo ; z2 là số thực

và z2 + z2.|z1|2)là số thực. Tính |z1|2012 + |z2|2013

———————————————–Hết—————————————————-

288

VVViiipppLLLaaammm...NNNeeettt

Page 290: Chuyên đề luyện thi đại học môn Toán - VipLam.Net

Đề 10 ngày 16.03.2013

PHẦN CHUNG CHO TẤT CẢ THÍ SINH (7 điểm)

Câu 1. (2 điểm) Cho hàm số y = x3 − (m+ 2)x2 + 3m có đồ thị (Cm) và hai điểm C(5; 2), D(−1;−7).

a) Khảo sát và vẽ đồ thị của hàm số đã cho khi m = 2.

b) Tìm các giá trị của m để (Cm) có hai điểm cực trị A, B sao cho diện tích tam giác ABC bằng3

7lần diện tích

tứ giác ABCD.

Câu 2. (2 điểm)

a) Giải phương trình√

3(sin4 x+ cos4 x

)= sin

(2x+

π

3

)+

1

4sin 4x

b) Giải hệ phương trình

y (2y + 1) (y + 3x) = x3 (x− 1)

(x+ 1)(√

xy4 +√x− y3 − y + 3

)= 3

(y2 + 1

) (y > 0)

Câu 3. (1 điểm) Tính tích phân I =

∫ π4

−π4

(1− x1 + x2

)2

(sinx+ cosx) dx

Câu 4. (1 điểm) Cho hình lăng trụ ABCD.A′B′C′D′ có đáy ABCD là hình thoi cạnh a. Gọi H là hình chiếu của điểm

A trên nữa mặt phẳng (A′B′C′D′) bờ B′D′ chứa A′. Biết rằng, khoảng cách từ H đến trung điểm của B′D′ bằng

a; khoảng cách giữa hai đường thẳng C′D và A′B′ bằng2a√

3

5. Tính thể tích khối lăng trụ ABCD.A′B′C′D′ và

bán kính mặt cầu ngoại tiếp khối chóp DA′C′D′.

Câu 5. (1 điểm) Cho a, b, c là các số thực thỏa mãn 2c(a2 + b2) + b(ab+ c) + c(ac+ b) > b(b2 + c2) + 2ac(1 + 2b) với

3 < ac, bc < 6, c ∈ [2; 3]. Tìm giá trị nhỏ nhất của biểu thức:

P =2a− 2b

b− 1− 2b− 2

a− 3− 2a− 6

a− b + 9 3√

(a− 3)(1− b)(a− b)

PHẦN RIÊNG (3 điểm): Thí sinh chỉ làm một trong hai phần A hoặc B

A. Theo chương trình chuẩn

Câu 6A. (2 điểm)

a) Trong hệ trục tọa độ Đề-các vuông Oxy cho tam giác ABC có B(2; 3) và C(2; 7) . Tìm điểm A sao cho đường

cao AH của tam giác ABC có độ dài bằng 3 lần bán kính đường tròn nội tiếp tam giác ABC.

b) Trong không gian tọa độ Oxyz cho hình thoi ABCD có A(−1;−4; 1), B(3; 6;−5), C thuộc mặt phẳng

(S) : x+ 2y+ z+ 8 = 0. Tìm điểm D và viết phương trình đường chéo BD biết hình thoi có diện tích S = 2√

133.

Câu 7A. (1 điểm) Giải phương trình 3x2−1 + (x2 − 1)3x+1 = 1

B. Theo chương trình nâng cao

Câu 6B. (2 điểm)

a) Trong hệ trục tọa độ Đề-các vuông Oxy ,cho các điểm A (4; 5) ;B (3; 0) ;C (2; 2) .Gọi H; I lần lượt là trực tâm

và tâm đường tròn ngoại tiếp của tam giác ABC , trung trực của AH cắt AB và AC lần lượt tại M và N . Phân

giác góc trong của IMN cắt AC tại P .Tính diện tích tam giác MNP . Biết điểm B có tung độ dương.

b) Trong không gian tọa độ Oxyz cho hình chóp S.ABCD, với S(3√

2; 0; 3√

2). Đáy ABCD là hình vuông, các

điểm M(5;√

2; 1), N(1;−√

2; 5), P (4;−2√

2; 2) lần lượt thuộc ba cạnh AB, CD, AD. Viết phương trình mặt

cầu ngoại tiếp hình chóp S.ABCD. Biết AP = 2DP.

Câu 7B. (1 điểm) Giải bất phương trình (3− x)3x−3 ≥ 3x2−5x+6−1x−2

———————————————–Hết—————————————————

289

Page 291: Chuyên đề luyện thi đại học môn Toán - VipLam.Net

VVViiipppLLLaaammm...NNNeeettt

Page 292: Chuyên đề luyện thi đại học môn Toán - VipLam.Net